All Related Questions of Corporate Finance

Q: The shareholders of the Stackhouse Company need to elect seven new directors

The shareholders of the Stackhouse Company need to elect seven new directors. There are 850,000 shares outstanding currently trading at $43 per share. You would like to serve on the board of directors...

See Answer

Q: Do you agree or disagree with the following statement? A firm’s

Do you agree or disagree with the following statement? A firm’s stockholders will never want the firm to invest in projects with negative net present values. Why?

See Answer

Q: Tom Scott is the owner, president, and primary salesperson for

Tom Scott is the owner, president, and primary salesperson for Scott Manufacturing. Because of this, the company’s profits are driven by the amount of work Tom does. If he works 40 hours each week, th...

See Answer

Q: What are the differences between preferred stock and debt?

What are the differences between preferred stock and debt?

See Answer

Q: The shareholders of Motive Power Corp. need to elect three new

The shareholders of Motive Power Corp. need to elect three new directors to the board. There are 13,000,000 shares of common stock outstanding, and the current share price is $10.50. If the company us...

See Answer

Q: In a world with no taxes, no transaction costs, and

In a world with no taxes, no transaction costs, and no costs of financial distress, is the following statement true, false, or uncertain? Moderate borrowing will not increase the required return on a...

See Answer

Q: Suppose the company in Problem 1 has a market-to-

Suppose the company in Problem 1 has a market-to-book ratio of 1.0. a. Calculate return on equity, ROE, under each of the three economic scenarios efore any debt is issued. Also calculate the percent...

See Answer

Q: Due to large losses incurred in the past several years, a

Due to large losses incurred in the past several years, a firm has $2 billion in tax loss carryforwards. This means that the next $2 billion of the firm’s income will be free from corporate income tax...

See Answer

Q: Dream, Inc., has debt outstanding with a face value of

Dream, Inc., has debt outstanding with a face value of $6 million. The value of the firm if it were entirely financed by equity would be $17.85 million. The company also has 350,000 shares of stock ou...

See Answer

Q: Preferred stock doesn’t offer a corporate tax shield on the dividends paid

Preferred stock doesn’t offer a corporate tax shield on the dividends paid. Why do we still observe some firms issuing preferred stock?

See Answer

Q: Candle box Inc. is going to elect six board members next

Candle box Inc. is going to elect six board members next month. Betty Brown owns 17.4 percent of the total shares outstanding. How confident can she be of having one of her candidate friends elected u...

See Answer

Q: List the three assumptions that lie behind the Modigliani–Miller theory

List the three assumptions that lie behind the Modigliani–Miller theory in a world without taxes. Are these assumptions reasonable in the real world? Explain.

See Answer

Q: Rolston Corporation is comparing two different capital structures, an all-

Rolston Corporation is comparing two different capital structures, an all-equity plan (Plan I) and a levered plan (Plan II). Under Plan I, Rolston would have 265,000 shares of stock outstanding. Under...

See Answer

Q: What steps can stockholders take to reduce the costs of debt?

What steps can stockholders take to reduce the costs of debt?

See Answer

Q: Dream, Inc., has debt outstanding with a face value of

Dream, Inc., has debt outstanding with a face value of $6 million. The value of the firm if it were entirely financed by equity would be $17.85 million. The company also has 350,000 shares of stock ou...

See Answer

Q: The yields on nonconvertible preferred stock are lower than the yields on

The yields on nonconvertible preferred stock are lower than the yields on corporate bonds. Why is there a difference? Which investors are the primary holders of preferred stock? Why?

See Answer

Q: KIC, Inc., plans to issue $5 million of bonds

KIC, Inc., plans to issue $5 million of bonds with a coupon rate of 8 percent and 30 years to maturity. The current market interest rates on these bonds are 7 percent. In one year, the interest rate o...

See Answer

Q: Explain what is meant by business and financial risk. Suppose Firm

Explain what is meant by business and financial risk. Suppose Firm A has greater business risk than Firm B. Is it true that Firm A also has a higher cost of equity capital? Explain.

See Answer

Q: Rolston Corporation is comparing two different capital structures, an all-

Rolston Corporation is comparing two different capital structures, an all-equity plan (Plan I) and a levered plan (Plan II). Under Plan I, Rolston would have 265,000 shares of stock outstanding. Under...

See Answer

Q: How does the existence of financial distress costs and agency costs affect

How does the existence of financial distress costs and agency costs affect Modigliani and Miller’s theory in a world where corporations pay taxes?

See Answer

Q: Edwards Construction currently has debt outstanding with a market value of $

Edwards Construction currently has debt outstanding with a market value of $85,000 and a cost of 9 percent. The company has EBIT of $7,650 that is expected to continue in perpetuity. Assume there are...

See Answer

Q: What are the main differences between corporate debt and equity? Why

What are the main differences between corporate debt and equity? Why do some firms try to issue equity in the guise of debt?

See Answer

Q: Money, Inc., has no debt outstanding and a total market

Money, Inc., has no debt outstanding and a total market value of $275,000. Earnings before interest and taxes, EBIT, are projected to be $21,000 if economic conditions are normal. If there is strong e...

See Answer

Q: New Business Ventures, Inc., has an outstanding perpetual bond with

New Business Ventures, Inc., has an outstanding perpetual bond with a 10 percent coupon rate that can be called in one year. The bond makes annual coupon payments. The call premium is set at $150 over...

See Answer

Q: How would you answer in the following debate? Q :

How would you answer in the following debate? Q : Isn’t it true that the riskiness of a firm’s equity will rise if the firm increases its use of debt financing? A : Yes, that’s the essence of MM Propo...

See Answer

Q: Kolby Corp. is comparing two different capital structures. Plan I

Kolby Corp. is comparing two different capital structures. Plan I would result in 900 shares of stock and $65,700 in debt. Plan II would result in 1,900 shares of stock and $29,200 in debt. The intere...

See Answer

Q: What are the sources of agency costs of equity?

What are the sources of agency costs of equity?

See Answer

Q: Steinberg Corporation and Dietrich Corporation are identical firms except that Dietrich is

Steinberg Corporation and Dietrich Corporation are identical firms except that Dietrich is more levered. Both companies will remain in business for one more year. The companies’ economists agree that...

See Answer

Q: A company is contemplating a long-term bond issue. It

A company is contemplating a long-term bond issue. It is debating whether to include a call provision. What are the benefits to the company from including a call provision? What are the costs? How do...

See Answer

Q: Is there an easily identifiable debt–equity ratio that will maximize

Is there an easily identifiable debt–equity ratio that will maximize the value of a firm? Why or why not?

See Answer

Q: Kolby Corp. is comparing two different capital structures. Plan I

Kolby Corp. is comparing two different capital structures. Plan I would result in 900 shares of stock and $65,700 in debt. Plan II would result in 1,900 shares of stock and $29,200 in debt. The intere...

See Answer

Q: Refer to the observed capital structures given in Table 17.3

Refer to the observed capital structures given in Table 17.3 of the text. What do you notice about the types of industries with respect to their average debt–equity ratios? Are certa...

See Answer

Q: Fountain Corporation’s economists estimate that a good business environment and a bad

Fountain Corporation’s economists estimate that a good business environment and a bad business environment are equally likely for the coming year. The managers of Fountain must choos...

See Answer

Q: What are the direct and indirect costs of bankruptcy? Briefly explain

What are the direct and indirect costs of bankruptcy? Briefly explain each.

See Answer

Q: Bowdeen Manufacturing intends to issue callable, perpetual bonds with annual coupon

Bowdeen Manufacturing intends to issue callable, perpetual bonds with annual coupon payments. The bonds are callable at $1,175. One-year interest rates are 9 percent. There is a 60 percent probability...

See Answer

Q: Illinois Industries has decided to borrow money by issuing perpetual bonds with

Illinois Industries has decided to borrow money by issuing perpetual bonds with a coupon rate of 7 percent, payable annually. The one-year interest rate is 7 percent. Next year, there is a 35 percent...

See Answer

Q: Star, Inc., a prominent consumer products firm, is debating

Star, Inc., a prominent consumer products firm, is debating whether or not to convert its all-equity capital structure to one that is 35 percent debt. Currently there are 6,000 shares outstanding and...

See Answer

Q: As mentioned in the text, some firms have filed for bankruptcy

As mentioned in the text, some firms have filed for bankruptcy because of actual or likely litigation-related losses. Is this a proper use of the bankruptcy process?

See Answer

Q: Good Time Company is a regional chain department store. It will

Good Time Company is a regional chain department store. It will remain in business for one more year. The probability of a boom year is 60 percent and the probability of a recession is 40 percent. It...

See Answer

Q: Do you think preferred stock is more like debt or equity?

Do you think preferred stock is more like debt or equity? Why?

See Answer

Q: An outstanding issue of Public Express Airlines debentures has a call provision

An outstanding issue of Public Express Airlines debentures has a call provision attached. The total principal value of the bonds is $250 million, and the bonds have an annual coupon rate of 9 percent....

See Answer

Q: ABC Co. and XYZ Co. are identical firms in all

ABC Co. and XYZ Co. are identical firms in all respects except for their capital structure. ABC is all equity financed with $750,000 in stock. XYZ uses both stock and perpetual debt; its stock is wort...

See Answer

Q: When personal taxes on interest income and bankruptcy costs are considered,

When personal taxes on interest income and bankruptcy costs are considered, the general expression for the value of a levered firm in a world in which the tax rate on equity distributions equals zero...

See Answer

Q: New equity issues are generally only a small portion of all new

New equity issues are generally only a small portion of all new issues. At the same time, companies continue to issue new debt. Why do companies tend to issue little new equity but continue to issue n...

See Answer

Q: Janetta Corp. has an EBIT rate of $975,000

Janetta Corp. has an EBIT rate of $975,000 per year that is expected to continue in perpetuity. The unlevered cost of equity for the company is 14 percent, and the corporate tax rate is 35 percent. Th...

See Answer

Q: Continental Airlines once filed for bankruptcy, at least in part,

Continental Airlines once filed for bankruptcy, at least in part, as a means of reducing labor costs. Whether this move was ethical or proper was hotly debated. Give both sides of the argument.

See Answer

Q: Overnight Publishing Company (OPC) has $2.5 million

Overnight Publishing Company (OPC) has $2.5 million in excess cash. The firm plans to use this cash either to retire all of its outstanding debt or to repurchase equity. The firm’s debt is held by one...

See Answer

Q: Charles River Associates is considering whether to call either of the two

Charles River Associates is considering whether to call either of the two perpetual bond issues the company currently has outstanding. If the bond is called, it will be refunded, that is, a new bond i...

See Answer

Q: Nina Corp. uses no debt. The weighted average cost of

Nina Corp. uses no debt. The weighted average cost of capital is 9 percent. If the current market value of the equity is $37 million and there are no taxes, what is EBIT? Suppose the corporate tax ra...

See Answer

Q: Consider the prices of the following three Treasury issues as of February

Consider the prices of the following three Treasury issues as of February 24, 2012: The bond in the middle is callable in February 2013. What is the implied value of the call feature? (there a way t...

See Answer

Q: The following Treasury bond quote appeared in The Wall Street Journal on

The following Treasury bond quote appeared in The Wall Street Journal on May 11, 2004: Why would anyone buy this Treasury bond with a negative yield to maturity? How is this possible?

See Answer

Q: Weston Industries has a debt–equity ratio of 1.5

Weston Industries has a debt–equity ratio of 1.5. Its WACC is 11 percent, and its cost of debt is 7 percent. The corporate tax rate is 35 percent. a. What is Weston’s cost of equity capital? b. What i...

See Answer

Q: Several publicly traded companies have issued more than one class of stock

Several publicly traded companies have issued more than one class of stock. Why might a company issue more than one class of stock?

See Answer

Q: Shadow Corp. has no debt but can borrow at 8 percent

Shadow Corp. has no debt but can borrow at 8 percent. The firm’s WACC is currently 11 percent, and the tax rate is 35 percent. a. What is Shadow’s cost of equity? b. If the firm converts to 25 percent...

See Answer

Q: Do you agree or disagree with the following statement: In an

Do you agree or disagree with the following statement: In an efficient market, callable and noncallable bonds will be priced in such a way that there will be no advantage or disadvantage to the call p...

See Answer

Q: What are the main features of a corporate bond that would be

What are the main features of a corporate bond that would be listed in the indenture?

See Answer

Q: Bruce & Co. expects its EBIT to be $185,

Bruce & Co. expects its EBIT to be $185,000 every year forever. The firm can borrow at 9 percent. Bruce currently has no debt, and its cost of equity is 16 percent. If the tax rate is 35 percent, what...

See Answer

Q: If interest rates fall, will the price of noncallable bonds move

If interest rates fall, will the price of noncallable bonds move up higher than that of callable bonds? Why or why not?

See Answer

Q: Bruce & Co. expects its EBIT to be $185,

Bruce & Co. expects its EBIT to be $185,000 every year forever. The firm can borrow at 9 percent. Bruce currently has no debt, and its cost of equity is 16 percent. If the tax rate is 35 percent, what...

See Answer

Q: Sinking funds have both positive and negative characteristics for bondholders. Why

Sinking funds have both positive and negative characteristics for bondholders. Why?

See Answer

Q: Levered, Inc., and Unlevered, Inc., are identical in

Levered, Inc., and Unlevered, Inc., are identical in every way except their capital structures. Each company expects to earn $29 million before interest per year in perpetuity, with each company distr...

See Answer

Q: Tool Manufacturing has an expected EBIT of $57,000 in

Tool Manufacturing has an expected EBIT of $57,000 in perpetuity and a tax rate of 35 percent. The firm has $90,000 in outstanding debt at an interest rate of 8 percent, and its unlevered cost of capi...

See Answer

Q: Cavo Corporation expects an EBIT of $19,750 every year

Cavo Corporation expects an EBIT of $19,750 every year forever. The company currently has no debt, and its cost of equity is 15 percent. a. What is the current value of the company? b. Suppose the com...

See Answer

Q: The Maxwell Company is financed entirely with equity. The company is

The Maxwell Company is financed entirely with equity. The company is considering a loan of $1.8 million. The loan will be repaid in equal installments over the next two years, and it has an interest r...

See Answer

Q: Alpha Corporation and Beta Corporation are identical in every way except

Alpha Corporation and Beta Corporation are identical in every way except their capital structures. Alpha Corporation, an allequity firm, has 15,000 shares of stock outstanding, currently worth $30 pe...

See Answer

Q: Acetate, Inc., has equity with a market value of $

Acetate, Inc., has equity with a market value of $23 million and debt with a market value of $7 million. Treasury bills that mature in one year yield 5 percent per year, and the expected return on the...

See Answer

Q: An election is being held to fill three seats on the board

An election is being held to fill three seats on the board of directors of a firm in which you hold stock. The company has 7,600 shares outstanding. If the election is conducted under cumulative votin...

See Answer

Q: The Veblen Company and the Knight Company are identical in every respect

The Veblen Company and the Knight Company are identical in every respect except that Veblen is not levered. The market value of Knight Company’s 6 percent bonds is $1.4 million. Fina...

See Answer

Q: Locomotive Corporation is planning to repurchase part of its common stock by

Locomotive Corporation is planning to repurchase part of its common stock by issuing corporate debt. As a result, the firm’s debt–equity ratio is expected to rise from 35 percent to 50 percent. The fi...

See Answer

Q: Green Manufacturing, Inc., plans to announce that it will issue

Green Manufacturing, Inc., plans to announce that it will issue $2 million of perpetual debt and use the proceeds to repurchase common stock. The bonds will sell at par with a coupon rate of 6 percent...

See Answer

Q: Williamson, Inc., has a debt–equity ratio of 2

Williamson, Inc., has a debt–equity ratio of 2.5. The firm’s weighted average cost of capital is 10 percent, and its pretax cost of debt is 6 percent. Williamson is subject to a corporate tax rate of...

See Answer

Q: In a world of corporate taxes only, show that the R

In a world of corporate taxes only, show that the R WACC can be written as R WACC = R0 × [1 – tC ( B/V )].

See Answer

Q: Assuming a world of corporate taxes only, show that the cost

Assuming a world of corporate taxes only, show that the cost of equity, R S, is as given in the chapter by MM Proposition II with corporate taxes.

See Answer

Q: Assume a firm’s debt is risk-free, so that the

Assume a firm’s debt is risk-free, so that the cost of debt equals the risk-free rate, R f . Define βA as the firm’s asset beta—that is, the systematic risk of the firm’s assets. Define βS to be the b...

See Answer

Q: Suppose a firm’s business operations mirror movements in the economy as a

Suppose a firm’s business operations mirror movements in the economy as a whole very closely—that is, the firm’s asset beta is 1.0. Use the result of previous problem to find the equity beta for this...

See Answer

Q: Beginning with the cost of capital equation—that is:

Beginning with the cost of capital equation—that is: show that the cost of equity capital for a levered firm can be written as follows:

See Answer

Q: In a world with no taxes, no transaction costs, and

In a world with no taxes, no transaction costs, and no costs of financial distress, is the following statement true, false, or uncertain? If a firm issues equity to repurchase some of its debt, the pr...

See Answer

Q: Money, Inc., has no debt outstanding and a total market

Money, Inc., has no debt outstanding and a total market value of $275,000. Earnings before interest and taxes, EBIT, are projected to be $21,000 if economic conditions are normal. If there is strong e...

See Answer

Q: In broad terms, why is some risk diversifiable? Why are

In broad terms, why is some risk diversifiable? Why are some risks nondiversifiable? Does it follow that an investor can control the level of unsystematic risk in a portfolio, but not the level of sys...

See Answer

Q: For each of the following scenarios, discuss whether profit opportunities exist

For each of the following scenarios, discuss whether profit opportunities exist from trading in the stock of the firm under the conditions that (1) the market is not weak form efficient, (2) the marke...

See Answer

Q: Suppose Tom O’Bedlam, president of Bedlam Products, Inc., has

Suppose Tom O’Bedlam, president of Bedlam Products, Inc., has hired you to determine the firm’s cost of debt and cost of equity capital. a. The stock currently sells for $50 per share, and the dividen...

See Answer

Q: Security F has an expected return of 10 percent and a standard

Security F has an expected return of 10 percent and a standard deviation of 43 percent per year. Security G has an expected return of 15 percent and a standard deviation of 62 percent per year. a. Wha...

See Answer

Q: Prospectors, Inc., is a publicly traded gold prospecting company in

Prospectors, Inc., is a publicly traded gold prospecting company in Alaska. Although the firm’s searches for gold usually fail, the prospectors occasionally find a rich vein of ore. What pattern would...

See Answer

Q: Suppose the returns on large-company stocks are normally distributed.

Suppose the returns on large-company stocks are normally distributed. Based on the historical record, use the NORMDIST function in Excel ® to determine the probability that in any given year you will...

See Answer

Q: Suppose the expected returns and standard deviations of Stocks A and B

Suppose the expected returns and standard deviations of Stocks A and B are E( R A ) = .09, E( RB ) = .15, αA = .36, and αB 5 .62. a. Calculate the expected return and standard deviation of a portfolio...

See Answer

Q: You have been provided the following data about the securities of three

You have been provided the following data about the securities of three firms, the market portfolio, and the risk-free asset: a. Fill in the missing values in the table. b. Is the stock of Firm A co...

See Answer

Q: David McClemore, the CFO of Ultra Bread, has decided to

David McClemore, the CFO of Ultra Bread, has decided to use an APT model to estimate the required return on the company’s stock. The risk factors he plans to use are the risk premium on the stock mark...

See Answer

Q: Suppose a factor model is appropriate to describe the returns on a

Suppose a factor model is appropriate to describe the returns on a stock. The current expected return on the stock is 10.5 percent. Information about those factors is presented in the following chart:...

See Answer

Q: Shanken Corp. issued a 30-year, 6.2

Shanken Corp. issued a 30-year, 6.2 percent semiannual bond 7 years ago. The bond currently sells for 108 percent of its face value. The company’s tax rate is 35 percent. a. What is the pretax cost of...

See Answer

Q: Which of the following statements are true about the efficient market hypothesis

Which of the following statements are true about the efficient market hypothesis? a. It implies perfect forecasting ability. b. It implies that prices reflect all available information. c. It implies...

See Answer

Q: Assume that the returns on individual securities are generated by the following

Assume that the returns on individual securities are generated by the following two-factor model: R it = E ( R it ) + β ij F 1 t + β i 2 F 2 t Here: Rit is the return on Securi...

See Answer

Q: Is it possible for the risk premium to be negative before an

Is it possible for the risk premium to be negative before an investment is undertaken? Can the risk premium be negative after the fact? Explain.

See Answer

Q: The following figures present the results of four cumulative abnormal returns (

The following figures present the results of four cumulative abnormal returns (CAR) studies. Indicate whether the results of each study support, reject, or are inconclusive about the semi-strong form...

See Answer

Q: Suppose a stock had an initial price of $75 per share

Suppose a stock had an initial price of $75 per share, paid a dividend of $1.20 per share during the year, and had an ending share price of $86. Assuming the ending share price is $67. Compute the pe...

See Answer

Q: The market portfolio has an expected return of 12 percent and a

The market portfolio has an expected return of 12 percent and a standard deviation of 22 percent. The risk-free rate is 5 percent. a. What is the expected return on a well-diversified portfolio with a...

See Answer

Q: A portfolio that combines the risk-free asset and the market

A portfolio that combines the risk-free asset and the market portfolio has an expected return of 7 percent and a standard deviation of 10 percent. The risk-free rate is 4 percent, and the expected ret...

See Answer

Q: Suppose the risk-free rate is 4.2 percent and

Suppose the risk-free rate is 4.2 percent and the market portfolio has an expected return of 10.9 percent. The market portfolio has a variance of .0382. Portfolio Z has a correlation coefficient with...

See Answer

Q: Consider the following information about Stocks I and II:

Consider the following information about Stocks I and II: The market risk premium is 7.5 percent, and the risk-free rate is 4 percent. Which stock has the most systematic risk? Which one has the mos...

See Answer

Q: Suppose you observe the following situation: /

Suppose you observe the following situation: Assume these securities are correctly priced. Based on the CAPM, what is the expected return on the market? What is the risk-free rate?

See Answer

Q: There are three securities in the market. The following chart shows

There are three securities in the market. The following chart shows their possible payoffs: a. What are the expected return and standard deviation of each security? b. What are the covariances and c...

See Answer

Q: Suppose you observe the following situation: /

Suppose you observe the following situation: a. Calculate the expected return on each stock. b. Assuming the capital asset pricing model holds and Stock A’s beta is greater than St...

See Answer

Q: There are two stocks in the market, Stock A and Stock

There are two stocks in the market, Stock A and Stock B . The price of Stock A today is $75. The price of Stock A next year will be $64 if the economy is in a recession, $87 if the economy is normal,...

See Answer

Q: The historical asset class returns presented in the chapter are not adjusted

The historical asset class returns presented in the chapter are not adjusted for inflation. What would happen to the estimated risk premium if we did account for inflation? The returns are also not ad...

See Answer

Q: What factors determine the beta of a stock? Define and describe

What factors determine the beta of a stock? Define and describe each.

See Answer

Q: Assume Stocks A and B have the following characteristics:

Assume Stocks A and B have the following characteristics: The covariance between the returns on the two stocks is .001. a. Suppose an investor holds a portfolio consisting of only Stock A and Stock...

See Answer

Q: You have $10,000 to invest in a stock portfolio

You have $10,000 to invest in a stock portfolio. Your choices are Stock X with an expected return of 14 percent and Stock Y with an expected return of 9 percent. If your goal is to create a portfolio...

See Answer

Q: You own stock in the Lewis-Striden Drug Company. Suppose

You own stock in the Lewis-Striden Drug Company. Suppose you had expected the following events to occur last month: a. The government would announce that real GNP had grown 1.2 percent during the prev...

See Answer

Q: Suppose stock returns can be explained by the following three factor model

Suppose stock returns can be explained by the following three factor model: R i = R F + β 1 F 1 + β 2 F 2 β 3 F 3 Assume there is no firm-specific risk. The inform...

See Answer

Q: What are the advantages of using the SML approach to finding the

What are the advantages of using the SML approach to finding the cost of equity capital? What are the disadvantages? What are the specific pieces of information needed to use this method? Are all of t...

See Answer

Q: For the firm in the previous problem, suppose the book value

For the firm in the previous problem, suppose the book value of the debt issue is $70 million. In addition, the company has a second debt issue on the market, a zero coupon bond with 12 years left to...

See Answer

Q: Explain why a characteristic of an efficient market is that investments in

Explain why a characteristic of an efficient market is that investments in that market have zero NPVs.

See Answer

Q: A study analyzed the behavior of the stock prices of firms that

A study analyzed the behavior of the stock prices of firms that had lost antitrust cases. Included in the diagram are all firms that lost the initial court decision, even if the decision was later ove...

See Answer

Q: Suppose you bought a 6 percent coupon bond one year ago for

Suppose you bought a 6 percent coupon bond one year ago for $1,040. The bond sells for $1,063 today. a. Assuming a $1,000 face value, what was your total dollar return on this investment over the past...

See Answer

Q: Using the following returns, calculate the average returns, the variances

Using the following returns, calculate the average returns, the variances, and the standard deviations for X and Y:

See Answer

Q: Based on the following information, calculate the expected return and standard

Based on the following information, calculate the expected return and standard deviation for the two stocks:

See Answer

Q: Briefly explain why the covariance of a security with the rest of

Briefly explain why the covariance of a security with the rest of a well-diversified portfolio is a more appropriate measure of the risk of the security than the security’s variance.

See Answer

Q: A portfolio is invested 10 percent in Stock G, 65 percent

A portfolio is invested 10 percent in Stock G, 65 percent in Stock J, and 25 percent in Stock K. The expected returns on these stocks are 9 percent, 11 percent, and 14 percent, respectively. What is t...

See Answer

Q: You are forming an equally weighted portfolio of stocks. Many stocks

You are forming an equally weighted portfolio of stocks. Many stocks have the same beta of .84 for Factor 1 and the same beta of 1.69 for Factor 2. All stocks also have the same expected return of 11...

See Answer

Q: Both Dow Chemical Company, a large natural gas user, and

Both Dow Chemical Company, a large natural gas user, and Superior Oil, a major natural gas producer, are thinking of investing in natural gas wells near Houston. Both are all-equity financed companies...

See Answer

Q: Fama’s Llamas has a weighted average cost of capital of 9.

Fama’s Llamas has a weighted average cost of capital of 9.8 percent. The company’s cost of equity is 13 percent, and its cost of debt is 6.5 percent. The tax rate is 35 percent. What is Fama’s debt–eq...

See Answer

Q: Two years ago, General Materials’ and Standard Fixtures’ stock prices were

Two years ago, General Materials’ and Standard Fixtures’ stock prices were the same. During the first year, General Materials’ stock price increased by 10 percent while Standard Fixtures’ stock price...

See Answer

Q: Suppose stock returns can be explained by a two-factor model

Suppose stock returns can be explained by a two-factor model. The firm-specific risks for all stocks are independent. The following table shows the information for two diversified portfolios: If the...

See Answer

Q: Refer to Table 10.1 in the text and look at

Refer to Table 10.1 in the text and look at the period from 1973 through 1978. a. Calculate the arithmetic average returns for large-company stocks and T-bills over this period. b. Calculate the s...

See Answer

Q: Consider the following quotation from a leading investment manager: “The

Consider the following quotation from a leading investment manager: “The shares of Southern Co. have traded close to $12 for most of the past three years. Since Southern’s stock has demonstrated very...

See Answer

Q: Consider the following information: / a.

Consider the following information: a. What is the expected return on an equally weighted portfolio of these three stocks? b. What is the variance of a portfolio invested 20 percent each in A and B,...

See Answer

Q: You own a portfolio equally invested in a risk-free asset

You own a portfolio equally invested in a risk-free asset and two stocks. If one of the stocks has a beta of 1.65 and the total portfolio is equally as risky as the market, what must the beta be for t...

See Answer

Q: There are two stock markets, each driven by the same common

There are two stock markets, each driven by the same common force, F, with an expected value of zero and standard deviation of 10 percent. There are many securities in each market; thus, you can inves...

See Answer

Q: Under what circumstances would it be appropriate for a firm to use

Under what circumstances would it be appropriate for a firm to use different costs of capital for its different operating divisions? If the overall firm WACC was used as the hurdle rate for all divisi...

See Answer

Q: Two years ago, the Lake Minerals and Small Town Furniture stock

Two years ago, the Lake Minerals and Small Town Furniture stock prices were the same. The average annual return for both stocks over the past two years was 10 percent. Lake Minerals’ stock price incre...

See Answer

Q: You’ve observed the following returns on Mary Ann Data Corporation’s stock over

You’ve observed the following returns on Mary Ann Data Corporation’s stock over the past five years: 27 percent, 13 percent, 18 percent, 214 percent, and 9 percent. a. What was the arithmetic average...

See Answer

Q: A broker has advised you not to invest in oil industry stocks

A broker has advised you not to invest in oil industry stocks because they have high standard deviations. Is the broker’s advice sound for a risk-averse investor like yourself? Why or why not?

See Answer

Q: Consider the following information: / a.

Consider the following information: a. Your portfolio is invested 30 percent each in A and C, and 40 percent in B. What is the expected return of the portfolio? b. What is the variance of this portf...

See Answer

Q: How do you determine the appropriate cost of debt for a company

How do you determine the appropriate cost of debt for a company? Does it make a difference if the company’s debt is privately placed as opposed to being publicly traded? How would you estimate the cos...

See Answer

Q: What is data mining? Why might it overstate the relation between

What is data mining? Why might it overstate the relation between some stock attribute and returns?

See Answer

Q: Assume that the following market model adequately describes the return generating behavior

Assume that the following market model adequately describes the return generating behavior of risky assets: R it = α i + β i R M t + € it Here: R it = The re...

See Answer

Q: Consider a levered firm’s projects that have similar risks to the firm

Consider a levered firm’s projects that have similar risks to the firm as a whole. Is the discount rate for the projects higher or lower than the rate computed using the security market line? Why?

See Answer

Q: Filer Manufacturing has 8.3 million shares of common stock outstanding

Filer Manufacturing has 8.3 million shares of common stock outstanding. The current share price is $53, and the book value per share is $4. Filer Manufacturing also has two bond issues outstanding. Th...

See Answer

Q: Given the following information for Huntington Power Co., find the WACC

Given the following information for Huntington Power Co., find the WACC. Assume the company’s tax rate is 35 percent. Debt: 5,000 6 percent coupon bonds outstanding, $1,000 par value, 25 years to ma...

See Answer

Q: Several celebrated investors and stock pickers frequently mentioned in the financial press

Several celebrated investors and stock pickers frequently mentioned in the financial press have recorded huge returns on their investments over the past two decades. Does the success of these particul...

See Answer

Q: What is the difference between arithmetic and geometric returns? Suppose you

What is the difference between arithmetic and geometric returns? Suppose you have invested in a stock for the last 10 years. Which number is more important to you, the arithmetic or geometric return?...

See Answer

Q: What would a technical analyst say about market efficiency?

What would a technical analyst say about market efficiency?

See Answer

Q: Suppose the average inflation rate over this period was 4.2

Suppose the average inflation rate over this period was 4.2 percent, and the average T-bill rate over the period was 5.1 percent, what was the average real risk-free rate over this time period? What w...

See Answer

Q: Titan Mining Corporation has 9.3 million shares of common stock

Titan Mining Corporation has 9.3 million shares of common stock outstanding and 260,000 6.8 percent semiannual bonds outstanding, par value $1,000 each. The common stock currently sells for $34 per sh...

See Answer

Q: What was the arithmetic average annual return on large-company stocks

What was the arithmetic average annual return on large-company stocks from 1926 through 2011? a. In nominal terms? b. In real terms?

See Answer

Q: A technical analysis tool that is sometimes used to predict market movements

A technical analysis tool that is sometimes used to predict market movements is an investor sentiment index. AAII, the American Association of Individual Investors, publishes an investor sentiment ind...

See Answer

Q: A stock has had returns of 16.12 percent, 12

A stock has had returns of 16.12 percent, 12.11 percent, 5.83 percent, 26.14 percent, and −13.19 percent over the past five years, respectively. What was the holding period return for the stock?

See Answer

Q: A stock has an expected return of 10.2 percent,

A stock has an expected return of 10.2 percent, the risk-free rate is 4 percent, and the market risk premium is 7 percent. What must the beta of this stock be?

See Answer

Q: Describe the difference between systematic risk and unsystematic risk.

Describe the difference between systematic risk and unsystematic risk.

See Answer

Q: An all-equity firm is considering the following projects:

An all-equity firm is considering the following projects: The T-bill rate is 3.5 percent, and the expected return on the market is 11 percent. a. Which projects have a higher expected return than th...

See Answer

Q: In the middle to late 1990s, the performance of the pros

In the middle to late 1990s, the performance of the pros was unusually poor—on the order of 90 percent of all equity mutual funds underperformed a passively managed index fund. How does this bear on t...

See Answer

Q: You purchased a zero coupon bond one year ago for $109

You purchased a zero coupon bond one year ago for $109.83. The market interest rate is now 9 percent. If the bond had 25 years to maturity when you originally purchased it, what was your total return...

See Answer

Q: Suppose your company needs $20 million to build a new assembly

Suppose your company needs $20 million to build a new assembly line. Your target debt–equity ratio is .75. The flotation cost for new equity is 7 percent, but the flotation cost for debt is only 3 per...

See Answer

Q: A hundred years ago or so, companies did not compile annual

A hundred years ago or so, companies did not compile annual reports. Even if you owned stock in a particular company, you were unlikely to be allowed to see the balance sheet and income statement for...

See Answer

Q: You bought a share of 4 percent preferred stock for $94

You bought a share of 4 percent preferred stock for $94.89 last year. The market price for your stock is now $96.12. What was your total return for last year?

See Answer

Q: What is the historical real return on long-term government bonds

What is the historical real return on long-term government bonds? On long-term corporate bonds?

See Answer

Q: Southern Alliance Company needs to raise $55 million to start a

Southern Alliance Company needs to raise $55 million to start a new project and will raise the money by selling new bonds. The company will generate no internal equity for the foreseeable future. The...

See Answer

Q: Aerotech, an aerospace technology research firm, announced this morning that

Aerotech, an aerospace technology research firm, announced this morning that it has hired the world’s most knowledgeable and prolific space researchers. Before today Aerotech’s stock had been selling...

See Answer

Q: You bought a stock three months ago for $43.18

You bought a stock three months ago for $43.18 per share. The stock paid no dividends. The current share price is $46.21. What is the APR of your investment? The EAR?

See Answer

Q: A stock has a beta of 1.13 and an expected

A stock has a beta of 1.13 and an expected return of 12.1 percent. A risk-free asset currently earns 5 percent. a. What is the expected return on a portfolio that is equally invested in the two assets...

See Answer

Q: What rule should a firm follow when making financing decisions? How

What rule should a firm follow when making financing decisions? How can firms create valuable financing opportunities?

See Answer

Q: Och, Inc., is considering a project that will result in

Och, Inc., is considering a project that will result in initial aftertax cash savings of $3.5 million at the end of the first year, and these savings will grow at a rate of 4 percent per year indefini...

See Answer

Q: When the 56-year-old founder of Gulf & Western

When the 56-year-old founder of Gulf & Western, Inc., died of a heart attack, the stock price immediately jumped from $18.00 a share to $20.25, a 12.5 percent increase. This is evidence of market inef...

See Answer

Q: Refer to Table 10.1. What was the average real

Refer to Table 10.1. What was the average real return for Treasury bills from 1926 through 1932? Table 10.1

See Answer

Q: Asset W has an expected return of 12.3 percent and

Asset W has an expected return of 12.3 percent and a beta of 1.3. If the risk-free rate is 4 percent, complete the following table for portfolios of Asset W and a risk-free asset. Illustrate the relat...

See Answer

Q: The Saunders Investment Bank has the following financing outstanding. What is

The Saunders Investment Bank has the following financing outstanding. What is the WACC for the company? Debt: 60,000 bonds with a coupon rate of 6 percent and a current price quote of 109.5; the bo...

See Answer

Q: Is it possible that a risky asset could have a beta of

Is it possible that a risky asset could have a beta of zero? Explain. Based on the CAPM, what is the expected return on such an asset? Is it possible that a risky asset could have a negative beta? Wha...

See Answer

Q: Today, the following announcement was made: “Early today the

Today, the following announcement was made: “Early today the Justice Department reached a decision in the Universal Product Care (UPC) case. UPC has been found guilty of discriminatory practices in hi...

See Answer

Q: Refer back to Table 10.2. What range of returns

Refer back to Table 10.2. What range of returns would you expect to see 68 percent of the time for long-term corporate bonds? What about 95 percent of the time? Table 10.2

See Answer

Q: Stock Y has a beta of 1.35 and an expected

Stock Y has a beta of 1.35 and an expected return of 14 percent. Stock Z has a beta of .80 and an expected return of 11.5 percent. If the risk-free rate is 4.5 percent and the market risk premium is 7...

See Answer

Q: Goodbye, Inc., recently issued new securities to finance a new

Goodbye, Inc., recently issued new securities to finance a new TV show. The project cost $19 million, and the company paid $1,150,000 in flotation costs. In addition, the equity issued had a flotation...

See Answer

Q: Newtech Corp. is going to adopt a new chip-testing

Newtech Corp. is going to adopt a new chip-testing device that can greatly improve its production efficiency. Do you think the lead engineer can profit from purchasing the firm’s stock before the news...

See Answer

Q: Delta, United, and American Airlines announced purchases of planes on

Delta, United, and American Airlines announced purchases of planes on July 18 (7/18), February 12 (2/12), and October 7 (10/7), respectively. Given the following information, calculate the cumulative...

See Answer

Q: Refer back to Table 10.2. What range of returns

Refer back to Table 10.2. What range of returns would you expect to see 68 percent of the time for large-company stocks? What about 95 percent of the time? Table 10.2

See Answer

Q: In the previous problem, what would the risk-free rate

In the previous problem, what would the risk-free rate have to be for the two stocks to be correctly priced?

See Answer

Q: Floyd Industries stock has a beta of 1.3. The

Floyd Industries stock has a beta of 1.3. The company just paid a dividend of $.95, and the dividends are expected to grow at 4.5 percent per year. The expected return on the market is 11 percent, and...

See Answer

Q: TransTrust Corp. has changed how it accounts for inventory. Taxes

TransTrust Corp. has changed how it accounts for inventory. Taxes are unaffected, although the resulting earnings report released this quarter is 20 percent higher than what it would have been under t...

See Answer

Q: Based on the following information, calculate the expected return and standard

Based on the following information, calculate the expected return and standard deviation:

See Answer

Q: You find a certain stock that had returns of 12 percent,

You find a certain stock that had returns of 12 percent, 221 percent, 9 percent, and 32 percent for four of the last five years. If the average return of the stock over this period was 11 percent, wha...

See Answer

Q: You own a portfolio that has $2,100 invested in

You own a portfolio that has $2,100 invested in Stock A and $3,200 invested in Stock B. If the expected returns on these stocks are 11 percent and 14 percent, respectively, what is the expected return...

See Answer

Q: Advance, Inc., is trying to determine its cost of debt

Advance, Inc., is trying to determine its cost of debt. The firm has a debt issue outstanding with 17 years to maturity that is quoted at 95 percent of face value. The issue makes semiannual payments...

See Answer

Q: Using the CAPM, show that the ratio of the risk premiums

Using the CAPM, show that the ratio of the risk premiums on two assets is equal to the ratio of their betas.

See Answer

Q: Happy Times, Inc., wants to expand its party stores into

Happy Times, Inc., wants to expand its party stores into the Southeast. In order to establish an immediate presence in the area, the company is considering the purchase of the privately held Joe’s Par...

See Answer

Q: A stock has had the following year-end prices and dividends

A stock has had the following year-end prices and dividends: What are the arithmetic and geometric returns for the stock?

See Answer

Q: Consider the following information about three stocks: /

Consider the following information about three stocks: a. If your portfolio is invested 40 percent each in A and B and 20 percent in C, what is the portfolio expected return? The variance? The stand...

See Answer

Q: Photochronograph Corporation (PC) manufactures time series photographic equipment. It

Photochronograph Corporation (PC) manufactures time series photographic equipment. It is currently at its target debt–equity ratio of .55. It’s considering building a new $50 million manufacturing fac...

See Answer

Q: Refer to T able 10.1 in the text and look

Refer to T able 10.1 in the text and look at the period from 1973 through 1980. a. Calculate the average return for Treasury bills and the average annual inflation rate (consumer price index) for this...

See Answer

Q: You’ve observed the following returns on Mary Ann Data Corporation’s stock over

You’ve observed the following returns on Mary Ann Data Corporation’s stock over the past five years: 27 percent, 13 percent, 18 percent, 214 percent, and 9 percent. Suppose the average inflation rate...

See Answer

Q: You want to create a portfolio equally as risky as the market

You want to create a portfolio equally as risky as the market, and you have $1,000,000 to invest. Given this information, fill in the rest of the following table:

See Answer

Q: Trower Corp. has a debt–equity ratio of .85

Trower Corp. has a debt–equity ratio of .85. The company is considering a new plant that will cost $145 million to build. When the company issues new equity, it incurs a flotation cost of 8 percent. T...

See Answer

Q: The following three stocks are available in the market:

The following three stocks are available in the market: Assume the market model is valid. a. Write the market model equation for each stock. b. What is the return on a portfolio with weights of 30 p...

See Answer

Q: Define the three forms of market efficiency.

Define the three forms of market efficiency.

See Answer

Q: Suppose the market is semistrong form efficient. Can you expect to

Suppose the market is semistrong form efficient. Can you expect to earn excess returns if you make trades based on: a. Your broker’s information about record earnings for a stock? b. Rumors about a me...

See Answer

Q: You bought one of Bergen Manufacturing Co.’s 7 percent coupon

You bought one of Bergen Manufacturing Co.’s 7 percent coupon bonds one year ago for $1,080.50. These bonds make annual payments and mature six years from now. Suppose you decide to sell your bonds to...

See Answer

Q: You have $100,000 to invest in a portfolio containing

You have $100,000 to invest in a portfolio containing Stock X, Stock Y, and a risk-free asset. You must invest all of your money. Your goal is to create a portfolio that has an expected return of 11.2...

See Answer

Q: Suppose you have been hired as a financial consultant to Defense Electronics

Suppose you have been hired as a financial consultant to Defense Electronics, Inc. (DEI), a large, publicly traded firm that is the market share leader in radar detection systems (RDSs). The company i...

See Answer

Q: Imagine that a particular macroeconomic variable that influences your firm’s net earnings

Imagine that a particular macroeconomic variable that influences your firm’s net earnings is positively serially correlated. Assume market efficiency. Would you expect price changes in your stock to b...

See Answer

Q: Suppose the returns on long-term government bonds are normally distributed

Suppose the returns on long-term government bonds are normally distributed. Based on the historical record, what is the approximate probability that your return on these bonds will be less than 23.7 p...

See Answer

Q: Based on the following information, calculate the expected return and standard

Based on the following information, calculate the expected return and standard deviation of each of the following stocks. Assume each state of the economy is equally likely to happen. What are the cov...

See Answer

Q: Assuming that the returns from holding small company stocks are normally distributed

Assuming that the returns from holding small company stocks are normally distributed, what is the approximate probability that your money will double in value in a single year? Triple in value?

See Answer

Q: Is the following statement true or false? A risky security cannot

Is the following statement true or false? A risky security cannot have an expected return that is less than the risk-free rate because no risk-averse investor would be willing to hold this asset in eq...

See Answer

Q: Based on the following information, calculate the expected return and standard

Based on the following information, calculate the expected return and standard deviation for each of the following stocks. What are the covariance and correlation between the returns of the two stocks...

See Answer

Q: The following diagram shows the cumulative abnormal returns (CAR) for

The following diagram shows the cumulative abnormal returns (CAR) for 386 oil exploration companies announcing oil discoveries between 1950 and 1980. Month 0 in the diagram is the announcement month....

See Answer

Q: Suppose a stock had an initial price of $75 per share

Suppose a stock had an initial price of $75 per share, paid a dividend of $1.20 per share during the year, and had an ending share price of $86. Compute the percentage total return.

See Answer

Q: Suppose a stock had an initial price of $75 per share

Suppose a stock had an initial price of $75 per share, paid a dividend of $1.20 per share during the year, and had an ending share price of $86. What was the dividend yield? The capital gains yield?

See Answer

Q: Schultz Industries is considering the purchase of Arras Manufacturing. Arras is

Schultz Industries is considering the purchase of Arras Manufacturing. Arras is currently a supplier for Schultz, and the acquisition would allow Schultz to better control its material supply. The cur...

See Answer

Q: The Durkin Investing Agency has been the best stock picker in the

The Durkin Investing Agency has been the best stock picker in the country for the past two years. Before this rise to fame occurred, the Durkin newsletter had 200 subscribers. Those subscribers beat t...

See Answer

Q: In contrast to the CAPM, the APT does not indicate which

In contrast to the CAPM, the APT does not indicate which factors are expected to determine the risk premium of an asset. How can we determine which factors should be included? For example, one risk fa...

See Answer

Q: A stock has had returns of 27 percent, 12 percent,

A stock has had returns of 27 percent, 12 percent, 32 percent, 212 percent, 19 percent, and 231 percent over the last six years. What are the arithmetic and geometric returns for the stock?

See Answer

Q: Suppose a three-factor model is appropriate to describe the returns

Suppose a three-factor model is appropriate to describe the returns of a stock. Information about those three factors is presented in the following chart: a. What is the systematic risk of the stock...

See Answer

Q: Why might a firm choose to engage in a sale and

Why might a firm choose to engage in a sale and leaseback transaction? Give two reasons.

See Answer

Q: In the previous problem, assume that the exercise style on the

In the previous problem, assume that the exercise style on the option is American rather than European. What is the price of the option now? Previous problem The stock price is $73, and the standard d...

See Answer

Q: What is a call option? A put option? Under what

What is a call option? A put option? Under what circumstances might you want to buy each? Which one has greater potential profit? Why?

See Answer

Q: Gary Levin is the chief executive officer of Mountainbrook Trading Company.

Gary Levin is the chief executive officer of Mountainbrook Trading Company. The board of directors has just granted Mr. Levin 30,000 at-the-money European call options on the company’s stock, which is...

See Answer

Q: If a firm is selling futures contracts on lumber as a

If a firm is selling futures contracts on lumber as a hedging strategy, what must be true about the firm’s exposure to lumber prices?

See Answer

Q: Refer to Table 25.2 in the text to answer this

Refer to Table 25.2 in the text to answer this question. Suppose you purchase a March 2012 cocoa futures contract on November 22, 2011, at the last price of the day. What will your profit or loss be i...

See Answer

Q: Indicate the impact of the following corporate actions on cash, using

Indicate the impact of the following corporate actions on cash, using the letter I for an increase, D for a decrease, or N when no change occurs. a. A dividend is paid with funds received from a sale...

See Answer

Q: Use the option quote information shown here to answer the questions that

Use the option quote information shown here to answer the questions that follow. The stock is currently selling for $114. a. Suppose you buy 10 contracts of the February 110 call option. How much wi...

See Answer

Q: Zoso is a rental car company that is trying to determine whether

Zoso is a rental car company that is trying to determine whether to add 25 cars to its fleet. The company fully depreciates all its rental cars over five years using the straight-line method. The new...

See Answer

Q: Complete the following sentence for each of these investors: a

Complete the following sentence for each of these investors: a. A buyer of call options. b. A buyer of put options. c. A seller (writer) of call options. d. A seller (writer) of put options. “The (buy...

See Answer

Q: Jared Lazarus has just been named the new chief executive officer of

Jared Lazarus has just been named the new chief executive officer of BluBell Fitness Centers, Inc. In addition to an annual salary of $410,000, his three-year contract states that his compensation wil...

See Answer

Q: Explain the following limits on the prices of warrants: a

Explain the following limits on the prices of warrants: a. If the stock price is below the exercise price of the warrant, the lower bound on the price of a warrant is zero. b. If the stock price is ab...

See Answer

Q: In the previous problem, suppose you wanted the option to sell

In the previous problem, suppose you wanted the option to sell the land to the buyer in one year. Assuming all the facts are the same, describe the transaction that would occur today. What is the pric...

See Answer

Q: If a firm is buying call options on pork belly futures as

If a firm is buying call options on pork belly futures as a hedging strategy, what must be true about the firm’s exposure to pork belly prices?

See Answer

Q: Refer to Table 25.2 in the text to answer this

Refer to Table 25.2 in the text to answer this question. Suppose you sell five March 2012 silver futures contracts on November 22, 2011, at the last price of the day. What will your profit or loss be...

See Answer

Q: McConnell Corp. has a book value of equity of $13

McConnell Corp. has a book value of equity of $13,205. Long-term debt is $8,200. Net working capital, other than cash, is $2,205. Fixed assets are $18,380. How much cash does the company have? If curr...

See Answer

Q: Gemini, Inc., an all-equity firm, is considering

Gemini, Inc., an all-equity firm, is considering a $1.7 million investment that will be depreciated according to the straight-line method over its four-year life. The project is expected to generate e...

See Answer

Q: What are some of the potential problems with looking at IRRs when

What are some of the potential problems with looking at IRRs when evaluating a leasing decision?

See Answer

Q: What would the lease payment have to be for both the lessor

What would the lease payment have to be for both the lessor and the lessee to be indifferent about the lease? You work for a nuclear research laboratory that is contemplating leasing a diagnostic scan...

See Answer

Q: The price of Ervin Corp. stock will be either $74

The price of Ervin Corp. stock will be either $74 or $96 at the end of the year. Call options are available with one year to expiration. T-bills currently yield 5 percent. a. Suppose the current price...

See Answer

Q: What is the difference between an American option and a European option

What is the difference between an American option and a European option?

See Answer

Q: Gasworks, Inc., has been approached to sell up to 5

Gasworks, Inc., has been approached to sell up to 5 million gallons of gasoline in three months at a price of $3.65 per gallon. Gasoline is currently selling on the wholesale market at $3.30 per gallo...

See Answer

Q: Eckely, Inc., recently issued bonds with a conversion ratio of

Eckely, Inc., recently issued bonds with a conversion ratio of 17.5. If the stock price at the time of the bond issue was $48.53, what was the conversion premium?

See Answer

Q: When should a firm force conversion of convertibles? Why?

When should a firm force conversion of convertibles? Why?

See Answer

Q: What is the difference between a forward contract and a futures contract

What is the difference between a forward contract and a futures contract? Why do you think that futures contracts are much more common? Are there any circumstances under which you might prefer to use...

See Answer

Q: Suppose a financial manager buys call options on 50,000 barrels

Suppose a financial manager buys call options on 50,000 barrels of oil with an exercise price of $95 per barrel. She simultaneously sells a put option on 50,000 barrels of oil with the same exercise p...

See Answer

Q: For the year just ended, you have gathered the following information

For the year just ended, you have gathered the following information about the Holly Corporation: a. A $200 dividend was paid. b. Accounts payable increased by $500. c. Fixed asset purchases were $900...

See Answer

Q: Indicate the effect that the following will have on the operating cycle

Indicate the effect that the following will have on the operating cycle. Use the letter I to indicate an increase, the letter D for a decrease, and the letter N for no change. a. Receivables average g...

See Answer

Q: Milano Pizza Club owns three identical restaurants popular for their specialty pizzas

Milano Pizza Club owns three identical restaurants popular for their specialty pizzas. Each restaurant has a debt–equity ratio of 40 percent and makes interest payments of $41,000 at the end of each y...

See Answer

Q: Comment on the following remarks: a. Leasing reduces risk

Comment on the following remarks: a. Leasing reduces risk and can reduce a firm’s cost of capital. b. Leasing provides 100 percent financing. c. If the tax advantages of leasing were eliminated, leasi...

See Answer

Q: Assume that your company does not contemplate paying taxes for the next

Assume that your company does not contemplate paying taxes for the next several years. What are the cash flows from leasing in this case? You work for a nuclear research laboratory that is contemplati...

See Answer

Q: The price of Tara, Inc., stock will be either $

The price of Tara, Inc., stock will be either $50 or $70 at the end of the year. Call options are available with one year to expiration. T-bills currently yield 5 percent. a. Suppose the current price...

See Answer

Q: Utility companies often face a decision to build new plants that burn

Utility companies often face a decision to build new plants that burn coal, oil, or both. If the prices of both coal and gas are highly volatile, how valuable is the decision to build a plant that can...

See Answer

Q: The Webber Company is an international conglomerate with a real estate division

The Webber Company is an international conglomerate with a real estate division that owns the right to erect an office building on a parcel of land in downtown Sacramento over the next year. This buil...

See Answer

Q: General Modems has five-year warrants that currently trade in the

General Modems has five-year warrants that currently trade in the open market. Each warrant gives its owner the right to purchase one share of common stock for an exercise price of $55. a. Suppose the...

See Answer

Q: What happens to the price of a convertible bond if interest rates

What happens to the price of a convertible bond if interest rates increase?

See Answer

Q: Hannon Home Products, Inc., recently issued $2 million worth

Hannon Home Products, Inc., recently issued $2 million worth of 8 percent convertible debentures. Each convertible bond has a face value of $1,000. Each convertible bond can be converted into 21.50 sh...

See Answer

Q: Bubbling Crude Corporation, a large Texas oil producer, would like

Bubbling Crude Corporation, a large Texas oil producer, would like to hedge against adverse movements in the price of oil because this is the firm’s primary source of revenue. What should the firm do?...

See Answer

Q: You are long 10 gold futures contracts, established at an initial

You are long 10 gold futures contracts, established at an initial settle price of $1,580 per ounce, where each contract represents 100 ounces. Over the subsequent four trading days, gold settles at $1...

See Answer

Q: Indicate the impact of the following on the cash and operating cycles

Indicate the impact of the following on the cash and operating cycles, respectively. Use the letter I to indicate an increase, the letter D for a decrease, and the letter N for no change. a. The terms...

See Answer

Q: If Wild Widgets, Inc., were an all-equity company

If Wild Widgets, Inc., were an all-equity company, it would have a beta of .85. The company has a target debt–equity ratio of .40. The expected return on the market portfolio is 11 percent, and Treasu...

See Answer

Q: North Pole Fishing Equipment Corporation and South Pole Fishing Equipment Corporation would

North Pole Fishing Equipment Corporation and South Pole Fishing Equipment Corporation would have identical equity betas of 1.10 if both were all equity financed. The market value information for each...

See Answer

Q: Discuss the accounting criteria for determining whether a lease must be reported

Discuss the accounting criteria for determining whether a lease must be reported on the balance sheet. In each case give a rationale for the criterion.

See Answer

Q: What are the prices of a call option and a put option

What are the prices of a call option and a put option with the following characteristics? Stock price = $57 Exercise price = $60 Risk-free rate = 6% per year, compounded continuously Maturity...

See Answer

Q: In the previous question, over what range of lease payments will

In the previous question, over what range of lease payments will the lease be profitable for both parties? Required information You work for a nuclear research laboratory that is contemplating leasing...

See Answer

Q: Explain why a swap is effectively a series of forward contracts.

Explain why a swap is effectively a series of forward contracts. Suppose a firm enters a swap agreement with a swap dealer. Describe the nature of the default risk faced by both parties.

See Answer

Q: Jet Black is an international conglomerate with a petroleum division and is

Jet Black is an international conglomerate with a petroleum division and is currently competing in an auction to win the right to drill for crude oil on a large piece of land in one year. The current...

See Answer

Q: What is dilution, and why does it occur when warrants are

What is dilution, and why does it occur when warrants are exercised?

See Answer

Q: A warrant gives its owner the right to purchase three shares of

A warrant gives its owner the right to purchase three shares of common stock at an exercise price of $53 per share. The current market price of the stock is $58. What is the minimum value of the warra...

See Answer

Q: A company produces an energy-intensive product and uses natural gas

A company produces an energy-intensive product and uses natural gas as the energy source. The competition primarily uses oil. Explain why this company is exposed to fluctuations in both oil and natura...

See Answer

Q: You are short 25 gasoline futures contracts, established at an initial

You are short 25 gasoline futures contracts, established at an initial settle price of $2.46 per gallon, where each contract represents 42,000 gallons. Over the subsequent four trading days, gasoline...

See Answer

Q: Is it possible for a firm’s cash cycle to be longer than

Is it possible for a firm’s cash cycle to be longer than its operating cycle? Explain why or why not.

See Answer

Q: The Litzenberger Company has projected the following quarterly sales amounts for the

The Litzenberger Company has projected the following quarterly sales amounts for the coming year: a. Accounts receivable at the beginning of the year are $310. Litzenberger has a 45-day collection pe...

See Answer

Q: What are the two types of risk that are measured by a

What are the two types of risk that are measured by a levered beta?

See Answer

Q: Discuss the IRS criteria for determining whether a lease is tax deductible

Discuss the IRS criteria for determining whether a lease is tax deductible. In each case give a rationale for the criterion.

See Answer

Q: What are the prices of a call option and a put option

What are the prices of a call option and a put option with the following characteristics? Stock price = $93 Exercise price = $90 Risk-free rate = 4% per year, compounded continuously Maturity...

See Answer

Q: Refer to Table 25.2 given below to answer this question

Refer to Table 25.2 given below to answer this question. Suppose today is November 22, 2011, and your firm produces breakfast cereal and needs 140,000 bushels of corn in March 2012 for an upcoming pro...

See Answer

Q: Rework Problem 1 assuming that the scanner will be depreciated as three

Rework Problem 1 assuming that the scanner will be depreciated as three-year property under MACRS Problem 1 Assume that the tax rate is 35 percent. You can borrow at 8 percent before taxes. Should yo...

See Answer

Q: Star Mining buys a gold mine, but the cost of extraction

Star Mining buys a gold mine, but the cost of extraction is currently too high to make the mine profitable. In option terminology, what type of option(s) does the company have on this mine?

See Answer

Q: An analyst has recently informed you that at the issuance of a

An analyst has recently informed you that at the issuance of a company’s convertible bonds, one of the two following sets of relationships existed: Assume the bonds are available for...

See Answer

Q: An analyst has recently informed you that at the issuance of a company’s

An analyst has recently informed you that at the issuance of a company’s convertible bonds, one of the two following sets of relationships existed: Assume the bonds are available f...

See Answer

Q: If a textile manufacturer wanted to hedge against adverse movements in cotton

If a textile manufacturer wanted to hedge against adverse movements in cotton prices, it could buy cotton futures contracts or buy call options on cotton futures contracts. What would be the pros and...

See Answer

Q: What is the duration of a bond with three years to maturity

What is the duration of a bond with three years to maturity and a coupon of 7 percent paid annually if the bond sells at par?

See Answer

Q: What are the costs of shortages? Describe them.

What are the costs of shortages? Describe them.

See Answer

Q: Consider the following financial statement information for the Bulldog Icers Corporation:

Consider the following financial statement information for the Bulldog Icers Corporation: Calculate the operating and cash cycles. How do you interpret your answer?

See Answer

Q: Daniel Kaffe, CFO of Kendrick Enterprises, is evaluating a 10

Daniel Kaffe, CFO of Kendrick Enterprises, is evaluating a 10-year, 8 percent loan with gross proceeds of $5,850,000. The interest payments on the loan will be made annually. Flotation costs are estim...

See Answer

Q: What is meant by the term off–balance sheet financing?

What is meant by the term off–balance sheet financing? When do leases provide such financing, and what are the accounting and economic consequences of such activity?

See Answer

Q: The following is the sales budget for Shleifer, Inc., for

The following is the sales budget for Shleifer, Inc., for the first quarter of 2013: Credit sales are collected as follows: 65 percent in the month of the sale. 20 percent in the month after the sale...

See Answer

Q: What are the deltas of a call option and a put option

What are the deltas of a call option and a put option with the following characteristics? What does the delta of the option tell you? Stock price = $67 Exercise price = $70 Risk-free rate = 5%...

See Answer

Q: Super Sonics Entertainment is considering buying a machine that costs $540

Super Sonics Entertainment is considering buying a machine that costs $540,000. The machine will be depreciated over five years by the straight-line method and will be worthless at that time. The comp...

See Answer

Q: You are discussing real options with a colleague. During the discussion

You are discussing real options with a colleague. During the discussion, the colleague states, “Real option analysis makes no sense because it says that a real option on a risky venture is worth more...

See Answer

Q: Wet for the Summer, Inc., manufactures filters for swimming pools

Wet for the Summer, Inc., manufactures filters for swimming pools. The company is deciding whether to implement a new technology in its pool filters. One year from now the company will know whether th...

See Answer

Q: Why do firms issue convertible bonds and bonds with warrants?

Why do firms issue convertible bonds and bonds with warrants?

See Answer

Q: Sportime Fitness Center, Inc., issued convertible bonds with a conversion

Sportime Fitness Center, Inc., issued convertible bonds with a conversion price of $51. The bonds are available for immediate conversion. The current price of the company’s common stock is $44 per sha...

See Answer

Q: Explain why a put option on a bond is conceptually the same

Explain why a put option on a bond is conceptually the same as a call option on interest rates.

See Answer

Q: What is the duration of a bond with four years to maturity

What is the duration of a bond with four years to maturity and a coupon of 8 percent paid annually if the bond sells at par?

See Answer

Q: In an ideal economy, net working capital is always zero.

In an ideal economy, net working capital is always zero. Why might net working capital be positive in a real economy?

See Answer

Q: Lewellen Products has projected the following sales for the coming year:

Lewellen Products has projected the following sales for the coming year: Sales in the year following this one are projected to be 15 percent greater in each quarter. a. Calculate payments to supplier...

See Answer

Q: Bolero, Inc., has compiled the following information on its financing

Bolero, Inc., has compiled the following information on its financing costs: The company is in the 35 percent tax bracket and has a target debt–equity ratio of 60 percent. The targ...

See Answer

Q: Shattered Glass, Inc., is an all-equity firm.

Shattered Glass, Inc., is an all-equity firm. The cost of the company’s equity is currently 11 percent, and the risk-free rate is 3.5 percent. The company is currently considering a project that will...

See Answer

Q: Why wouldn’t China Eastern Airlines purchase the planes if they were obviously

Why wouldn’t China Eastern Airlines purchase the planes if they were obviously needed for the company’s operations?

See Answer

Q: Suppose it is estimated that the equipment will have an after tax

Suppose it is estimated that the equipment will have an after tax residual value of $700,000 at the end of the lease. What is the maximum lease payment acceptable to Wildcat now? Required information...

See Answer

Q: Quartz Corporation is a relatively new firm. Quartz has experienced enough

Quartz Corporation is a relatively new firm. Quartz has experienced enough losses during its early years to provide it with at least eight years of tax loss carry forwards. Thus, Quartz’s effective ta...

See Answer

Q: When you take out an ordinary student loan, it is usually

When you take out an ordinary student loan, it is usually the case that whoever holds that loan is given a guarantee by the U.S. government, meaning that the government will make up any payments you s...

See Answer

Q: How would the analysis of real options change if a company has

How would the analysis of real options change if a company has competitors?

See Answer

Q: You are given the following information concerning options on a particular stock

You are given the following information concerning options on a particular stock: Stock price = $83 Exercise price = $80 Risk-free rate = 6% per year, compounded continuously Maturity = 6 mont...

See Answer

Q: You are in discussions to purchase an option on an office building

You are in discussions to purchase an option on an office building with a strike price of $63 million. The building is currently valued at $60 million. The option will allow you to purchase the buildi...

See Answer

Q: A warrant with six months until expiration entitles its owner to buy

A warrant with six months until expiration entitles its owner to buy 10 shares of the issuing firm ’ s common stock for an exercise price of $31 per share. If the current market price of the stock is...

See Answer

Q: Bernanke Corp. has just issued a 30-year callable,

Bernanke Corp. has just issued a 30-year callable, convertible bond with a coupon rate of 6 percent annual coupon payments. The bond has a conversion price of $93. The company’s stock is selling for $...

See Answer

Q: ABC Company and XYZ Company need to raise funds to pay for

ABC Company and XYZ Company need to raise funds to pay for capital improvements at their manufacturing plants. ABC Company is a well-established firm with an excellent credit rating in the debt market...

See Answer

Q: Here are some important figures from the budget of Cornell, Inc

Here are some important figures from the budget of Cornell, Inc., for the second quarter of 2013: The company predicts that 5 percent of its credit sales will never be collected, 35 percent of its sa...

See Answer

Q: Triad Corporation has established a joint venture with Tobacco Road Construction,

Triad Corporation has established a joint venture with Tobacco Road Construction, Inc., to build a toll road in North Carolina. The initial investment in paving equipment is $80 million. The equipment...

See Answer

Q: Why would ALC be willing to buy planes from Boeing and Airbus

Why would ALC be willing to buy planes from Boeing and Airbus and then lease the planes to China Eastern Airlines? How is this different from just lending money to China Eastern Airlines to buy the pl...

See Answer

Q: There is a European put option on a stock that expires in

There is a European put option on a stock that expires in two months. The stock price is $73, and the standard deviation of the stock returns is 70 percent. The option has a strike price of $80, and t...

See Answer

Q: Many lessors require a security deposit in the form of a cash

Many lessors require a security deposit in the form of a cash payment or other pledged collateral. Suppose Lambert requires Wildcat to pay a $500,000 security deposit at the inception of the lease. If...

See Answer

Q: What is the impact of lengthening the time to expiration on an

What is the impact of lengthening the time to expiration on an option’s value? Explain.

See Answer

Q: Rob Stevens is the chief executive officer of Isner Construction, Inc

Rob Stevens is the chief executive officer of Isner Construction, Inc., and owns 950,000 shares of stock. The company currently has 6 million shares of stock and convertible bonds with a face value of...

See Answer

Q: A stock is currently priced at $73. The stock will

A stock is currently priced at $73. The stock will either increase or decrease by 15 percent over the next year. There is a call option on the stock with a strike price of $70 and one year until expir...

See Answer

Q: What is the difference between transactions and economic exposure? Which can

What is the difference between transactions and economic exposure? Which can be hedged more easily? Why?

See Answer

Q: Ted and Alice Hansel have a son who will begin college 10

Ted and Alice Hansel have a son who will begin college 10 years from today. School expenses of $30,000 will need to be paid at the beginning of each of the four years that their son plans to attend co...

See Answer

Q: Here are the most recent balance sheets for Country Kettles, Inc

Here are the most recent balance sheets for Country Kettles, Inc. Excluding accumulated depreciation, determine whether each item is a source or a use of cash, and the amount:

See Answer

Q: MVP, Inc., has produced rodeo supplies for over 20 years

MVP, Inc., has produced rodeo supplies for over 20 years. The company currently has a debt–equity ratio of 50 percent and is in the 40 percent tax bracket. The required return on the firm’s levered eq...

See Answer

Q: An asset costs $550,000 and will be depreciated in

An asset costs $550,000 and will be depreciated in a straight-line manner over its three-year life. It will have no salvage value. The corporate tax rate is 34 percent, and the appropriate interest ra...

See Answer

Q: An insurance policy is considered analogous to an option. From the

An insurance policy is considered analogous to an option. From the policyholder’s point of view, what type of option is an insurance policy? Why?

See Answer

Q: The capital structure of Ricketti Enterprises, Inc., consists of 20

The capital structure of Ricketti Enterprises, Inc., consists of 20 million shares of common stock and 1.5 million warrants. Each warrant gives its owner the right to purchase one share of common stoc...

See Answer

Q: For the following scenarios, describe a hedging strategy using futures contracts

For the following scenarios, describe a hedging strategy using futures contracts that might be considered. a. A public utility is concerned about rising costs. b. A candy manufacturer is concerned abo...

See Answer

Q: Blue Steel Community Bank has the following market value balance sheet:

Blue Steel Community Bank has the following market value balance sheet: a. What is the duration of the assets? b. What is the duration of the liabilities? c. Is the bank immune to interest rate risk?...

See Answer

Q: The forward price (F) of a contract on an asset

The forward price (F) of a contract on an asset with neither carrying costs nor convenience yield is the current spot price of the asset (S0) multiplied by 1, plus the appropriate interest rate betwee...

See Answer

Q: Wildcat, Inc., has estimated sales (in millions) for

Wildcat, Inc., has estimated sales (in millions) for the next four quarters as follows: Sales for the first quarter of the year after this one are projected at $120 million. Accounts receivable at th...

See Answer

Q: Neon Corporation’s stock returns have a covariance with the market portfolio of

Neon Corporation’s stock returns have a covariance with the market portfolio of .0415. The standard deviation of the returns on the market portfolio is 20 percent, and the expected market risk premium...

See Answer

Q: Wolfson Corporation has decided to purchase a new machine that costs $

Wolfson Corporation has decided to purchase a new machine that costs $3.2 million. The machine will be depreciated on a straight-line basis and will be worthless after four years. The corporate tax ra...

See Answer

Q: For the company in the previous problem, what is the value

For the company in the previous problem, what is the value of being able to issue subsidized debt instead of having to issue debt at the terms it would normally receive? Assume the face amount and mat...

See Answer

Q: It is said that the equityholders of a levered firm can be

It is said that the equityholders of a levered firm can be thought of as holding a call option on the firm’s assets. Explain what is meant by this statement.

See Answer

Q: Sunburn Sunscreen has a zero coupon bond issue outstanding with a $

Sunburn Sunscreen has a zero coupon bond issue outstanding with a $25,000 face value that matures in one year. The current market value of the firm’s assets is $26,300. The standard deviation of the r...

See Answer

Q: You have been hired to value a new 20-year callable

You have been hired to value a new 20-year callable, convertible bond. The bond has a 5.8 percent coupon rate, payable annually. The conversion price is $150, and the stock currently sells for $32.20....

See Answer

Q: You enter into a forward contract to buy a 10-year

You enter into a forward contract to buy a 10-year, zero coupon bond that will be issued in one year. The face value of the bond is $1,000, and the 1-year and 11-year spot interest rates are 5 percent...

See Answer

Q: Rework Problem 13 assuming the following: a. Wildcat maintains

Rework Problem 13 assuming the following: a. Wildcat maintains a minimum cash balance of $20 million. b. Wildcat maintains a minimum cash balance of $10 million. Based on your answers in (a) and (b),...

See Answer

Q: Stackhouse Industries has a new project available that requires an initial investment

Stackhouse Industries has a new project available that requires an initial investment of $4.5 million. The project will provide unlevered cash flows of $675,000 per year for the next 20 years. The com...

See Answer

Q: The Thakor Corporation’s purchases from suppliers in a quarter are equal to

The Thakor Corporation’s purchases from suppliers in a quarter are equal to 75 percent of the next quarter’s forecast sales. The payables period is 60 days. Wages,...

See Answer

Q: An asset costs $620,000 and will be depreciated in

An asset costs $620,000 and will be depreciated in a straight-line manner over its three-year life. It will have no salvage value. The lessor can borrow at 7 percent and the lessee can borrow at 9 per...

See Answer

Q: You are the CEO of Titan Industries and have just been awarded

You are the CEO of Titan Industries and have just been awarded a large number of employee stock options. The company has two mutually exclusive projects available. The first project has a large NPV an...

See Answer

Q: Superior Clamps, Inc., has a capital structure consisting of 7

Superior Clamps, Inc., has a capital structure consisting of 7 million shares of common stock and 900,000 warrants. Each warrant gives its owner the right to purchase one share of newly issued common...

See Answer

Q: Monster Magnet Manufacturing is considering leasing some equipment. The annual lease

Monster Magnet Manufacturing is considering leasing some equipment. The annual lease payment would be $375,000 per year for six years. The appropriate interest rate is 8 percent and the company is in...

See Answer

Q: William Santiago is interested in entering the import/export business.

William Santiago is interested in entering the import/export business. During a recent visit with his financial advisers, he said, “If we play the game right, this is the safest business in the world....

See Answer

Q: This morning you agreed to buy a one-year Treasury bond

This morning you agreed to buy a one-year Treasury bond in six months. The bond has a face value of $1,000. Use the spot interest rates listed here to answer the following questions: Time EAR...

See Answer

Q: Cleveland Compressor and Pnew York Pneumatic are competing manufacturing firms. Their

Cleveland Compressor and Pnew York Pneumatic are competing manufacturing firms. Their financial statements are printed here. a. How are the current assets of each firm financed? b. Which firm has the...

See Answer

Q: Suppose the firm in the previous problem is considering two mutually exclusive

Suppose the firm in the previous problem is considering two mutually exclusive investments. Project A has an NPV of $1,200, and Project B has an NPV of $1,600. As a result of taking Project A, the sta...

See Answer

Q: Seger, Inc., is an unlevered firm with expected annual earnings

Seger, Inc., is an unlevered firm with expected annual earnings before taxes of $21 million in perpetuity. The current required return on the firm’s equity is 16 percent, and the firm distributes all...

See Answer

Q: Automobiles are often leased, and there are several terms unique to

Automobiles are often leased, and there are several terms unique to auto leases. Suppose you are considering leasing a car. The price you and the dealer agree on for the car is $36,000. This is the ba...

See Answer

Q: You find a put and a call with the same exercise price

You find a put and a call with the same exercise price and maturity. What do you know about the relative prices of the put and call? Prove your answer and provide an intuitive explanation.

See Answer

Q: National Electric Company (NEC) is considering a $45 million

National Electric Company (NEC) is considering a $45 million project in its power systems division. Tom Edison, the company’s chief financial officer, has evaluated the project and determined that the...

See Answer

Q: Omega Airline ’ s capital structure consists of 2.7 million

Omega Airline ’ s capital structure consists of 2.7 million shares of common stock and zero coupon bonds with a face value of $18 million that mature in six months. The firm just announced that it wil...

See Answer

Q: Kevin Nomura is a Japanese student who is planning a one year

Kevin Nomura is a Japanese student who is planning a one year stay in the United States. He expects to arrive in the United States in eight months. He is worried about depreciation of the yen relative...

See Answer

Q: A company has a large bond issue maturing in one year.

A company has a large bond issue maturing in one year. When it matures, the company will float a new issue. Current interest rates are attractive, and the company is concerned that rates next year wil...

See Answer

Q: Suppose there were call options and forward contracts available on coal,

Suppose there were call options and forward contracts available on coal, but no put options. Show how a financial engineer could synthesize a put option using the available contracts. What does your a...

See Answer

Q: Mojito Mint Company has a debt–equity ratio of .35

Mojito Mint Company has a debt–equity ratio of .35. The required return on the company’s unlevered equity is 13 percent, and the pretax cost of the firm’s debt is 7 percent. Sales revenue for the comp...

See Answer

Q: Return to the case of the diagnostic scanner discussed in Problems 1

Return to the case of the diagnostic scanner discussed in Problems 1 through 6. Suppose the entire $5,200,000 purchase price of the scanner is borrowed. The rate on the loan is 8 percent, and the loan...

See Answer

Q: A put and a call have the same maturity and strike price

A put and a call have the same maturity and strike price. If they have the same price, which one is in the money? Prove your answer and provide an intuitive explanation.

See Answer

Q: Lone Star Industries just issued $235,000 of perpetual 8

Lone Star Industries just issued $235,000 of perpetual 8 percent debt and used the proceeds to repurchase stock. The company expects to generate $118,000 of earnings before interest and taxes in perpe...

See Answer

Q: Frostbite Thermalwear has a zero coupon bond issue outstanding with a face

Frostbite Thermalwear has a zero coupon bond issue outstanding with a face value of $30,000 that matures in one year. The current market value of the firm’s assets is $36,400. The standard deviation o...

See Answer

Q: High electricity costs have made Farmer Corporation’s chicken plucking machine economically worthless

High electricity costs have made Farmer Corporation’s chicken plucking machine economically worthless. Only two machines are available to replace it. The International Plucking Machine (IPM) model is...

See Answer

Q: Blue Angel, Inc., a private firm in the holiday gift

Blue Angel, Inc., a private firm in the holiday gift industry, is considering a new project. The company currently has a target debt–equity ratio of .40, but the industry target debt–equity ratio is ....

See Answer

Q: What is the NAL for Wildcat? What is the maximum lease

What is the NAL for Wildcat? What is the maximum lease payment that would be acceptable to the company? Required information The Wildcat Oil Company is trying to decide whether to lease or buy a new c...

See Answer

Q: Survivor, Inc., an all-equity firm, has eight

Survivor, Inc., an all-equity firm, has eight shares of stock outstanding. Yesterday, the firm ’ s assets consisted of nine ounces of platinum, currently worth $1,750 per ounce. Today, the company iss...

See Answer

Q: Brozik Corp. has a zero coupon bond that matures in five

Brozik Corp. has a zero coupon bond that matures in five years with a face value of $60,000. The current value of the company’s assets is $57,000, and the standard deviation of its return on assets is...

See Answer

Q: Insurance, whether purchased by a corporation or an individual, is

Insurance, whether purchased by a corporation or an individual, is in essence an option. What type of option is an insurance policy?

See Answer

Q: If a U.S. company exports its goods to Japan

If a U.S. company exports its goods to Japan, how would it use a futures contract on Japanese yen to hedge its exchange rate risk? Would it buy or sell yen futures? Does the way the exchange rate is q...

See Answer

Q: Strudler Real Estate, Inc., a construction firm financed by both

Strudler Real Estate, Inc., a construction firm financed by both debt and equity, is undertaking a new project. If the project is successful, the value of the firm in one year will be $280 million, bu...

See Answer

Q: In addition to the five factors discussed in the chapter, dividends

In addition to the five factors discussed in the chapter, dividends also affect the price of an option. The Black–Scholes option pricing model with dividends is: C = S × e–dt × N(d1) – E × e–Rt × N(d2...

See Answer

Q: The put–call parity condition is altered when dividends are paid

The put–call parity condition is altered when dividends are paid. The dividend-adjusted put–call parity formula is: S × e−dt − P = E × e−Rt + C where d is again the continuously compounded dividend yi...

See Answer

Q: In the chapter we noted that the delta for a put option

In the chapter we noted that the delta for a put option is N( d 1 ) − 1. Is this the same thing as −N(− d 1 )?

See Answer

Q: Use the option quote information shown here to answer the questions that

Use the option quote information shown here to answer the questions that follow. The stock is currently selling for $83. a. Are the call options in the money? What is the intrinsic value of an RWJ C...

See Answer

Q: Use the Black–Scholes model for pricing a call, put

Use the Black–Scholes model for pricing a call, put–call parity, and the previous question to show that the Black–Scholes model for directly pricing a put can be written as follows: P = E × e–Rt ×N(–d...

See Answer

Q: A stock is currently priced at $50. The stock will

A stock is currently priced at $50. The stock will never pay a dividend. The risk-free rate is 12 percent per year, compounded continuously, and the standard deviation of the stock’s return is 60 perc...

See Answer

Q: Suppose Sunburn Sunscreen and Frostbite Thermalwear in the previous problems have decided

Suppose Sunburn Sunscreen and Frostbite Thermalwear in the previous problems have decided to merge. Because the two companies have seasonal sales, the combined firm’s return on assets will have a stan...

See Answer

Q: You purchase one call and sell one put with the same strike

You purchase one call and sell one put with the same strike price and expiration date. What is the delta of your portfolio? Why?

See Answer

Q: What are the key differences between leasing and borrowing? Are they

What are the key differences between leasing and borrowing? Are they perfect substitutes?

See Answer

Q: Assume that the tax rate is 35 percent. You can borrow

Assume that the tax rate is 35 percent. You can borrow at 8 percent before taxes. Should you lease or buy? You work for a nuclear research laboratory that is contemplating leasing a diagnostic scanne...

See Answer

Q: What is the duration of a bond with two years to maturity

What is the duration of a bond with two years to maturity if the bond has a coupon rate of 7 percent paid semiannually, and the market interest rate is 5 percent?

See Answer

Q: The sales budget for your company in the coming year is based

The sales budget for your company in the coming year is based on a quarterly growth rate of 10 percent with the first-quarter sales projection at $225 million. In addition to this basic trend, the sea...

See Answer

Q: In the previous problem, assume the risk-free rate is

In the previous problem, assume the risk-free rate is only 5 percent. What is the risk-neutral value of the option now? What happens to the risk-neutral probabilities of a stock price increase and a s...

See Answer

Q: A company has a single zero coupon bond outstanding that matures in

A company has a single zero coupon bond outstanding that matures in 10 years with a face value of $15 million. The current value of the company’s assets is $13.4 million, and the standard deviation of...

See Answer

Q: T-bills currently yield 4.8 percent. Stock in

T-bills currently yield 4.8 percent. Stock in Nina Manufacturing is currently selling for $63 per share. There is no possibility that the stock will be worth less than $61 per share in one year. a. Wh...

See Answer

Q: Ken is interested in buying a European call option written on Southeastern

Ken is interested in buying a European call option written on Southeastern Airlines, Inc., a non-dividend–paying common stock, with a strike price of $75 and one year until expiration. Currently, Sout...

See Answer

Q: Rob wishes to buy a European put option on BioLabs, Inc

Rob wishes to buy a European put option on BioLabs, Inc., a non-dividend–paying common stock, with a strike price of $40 and six months until expiration. BioLabs’ common stock is currently selling for...

See Answer

Q: Maverick Manufacturing, Inc., must purchase gold in three months for

Maverick Manufacturing, Inc., must purchase gold in three months for use in its operations. Maverick’s management has estimated that if the price of gold were to rise above $1,530 per ounce, the firm...

See Answer

Q: An investor is said to take a position in a “collar

An investor is said to take a position in a “collar” if she buys the asset, buys an out-of-the-money put option on the asset, and sells an out-of-the-money call option on the asset. The two options sh...

See Answer

Q: McLemore Industries has a zero coupon bond issue that matures in two

McLemore Industries has a zero coupon bond issue that matures in two years with a face value of $50,000. The current value of the company’s assets is $29,000, and the standard deviation of the return...

See Answer

Q: Bishop, Inc., has current assets of $5,700

Bishop, Inc., has current assets of $5,700, net fixed assets of $27,000, current liabilities of $4,400, and long-term debt of $12,900. What is the value of the shareholders’ equity account for this fi...

See Answer

Q: In a typical month, the Jeremy Corporation receives 140 checks totaling

In a typical month, the Jeremy Corporation receives 140 checks totaling $124,000. These are delayed four days on average. What is the average daily float?

See Answer

Q: Are there any circumstances under which an investor might be more concerned

Are there any circumstances under which an investor might be more concerned about the nominal return on an investment than the real return?

See Answer

Q: A Japanese company has a bond outstanding that sells for 92 percent

A Japanese company has a bond outstanding that sells for 92 percent of its ¥100,000 par value. The bond has a coupon rate of 2.8 percent paid annually and matures in 21 years. What is the yield to mat...

See Answer

Q: Based on the dividend growth model, what are the two components

Based on the dividend growth model, what are the two components of the total return on a share of stock? Which do you think is typically larger?

See Answer

Q: Suppose you know that a company’s stock currently sells for $72

Suppose you know that a company’s stock currently sells for $72 per share and the required return on the stock is 11.5 percent. You also know that the total return on the stock is evenly divided betwe...

See Answer

Q: A mail-order firm processes 5,700 checks per month

A mail-order firm processes 5,700 checks per month. Of these, 60 percent are for $55 and 40 percent are for $80. The $55 checks are delayed two days on average; the $80 checks are delayed three days o...

See Answer

Q: In each of the following pairings, indicate which firm would probably

In each of the following pairings, indicate which firm would probably have a longer credit period and explain your reasoning. a. Firm A sells a miracle cure for baldness; Firm B sells toupees. b. Firm...

See Answer

Q: Describe the advantages and disadvantages of a taxable merger as opposed to

Describe the advantages and disadvantages of a taxable merger as opposed to a tax-free exchange. What is the basic determinant of tax status in a merger? Would an LBO be taxable or nontaxable? Explain...

See Answer

Q: In the previous problem, construct the balance sheet for the new

In the previous problem, construct the balance sheet for the new corporation assuming that the transaction is treated as a purchase for accounting purposes. The market value of All Gold Miningâ&...

See Answer

Q: What is the absolute priority rule?

What is the absolute priority rule?

See Answer

Q: Given that many multinationals based in many countries have much greater sales

Given that many multinationals based in many countries have much greater sales outside their domestic markets than within them, what is the particular relevance of their domestic currency?

See Answer

Q: Describe each of the following: a. Sight draft.

Describe each of the following: a. Sight draft. b. Time draft. c. Banker’s acceptance. d. Promissory note. e. Trade acceptance.

See Answer

Q: Use Figure 31.1 to answer the following questions. Suppose

Use Figure 31.1 to answer the following questions. Suppose interest rate parity holds, and the current six-month risk-free rate in the United States is 1.9 percent. What must the six-month risk-free r...

See Answer

Q: Suppose you own stock in a company. The current price per

Suppose you own stock in a company. The current price per share is $25. Another company has just announced that it wants to buy your company and will pay $35 per share to acquire all the outstanding s...

See Answer

Q: The following table presents the long-term liabilities and stockholders’ equity

The following table presents the long-term liabilities and stockholders’ equity of Information Control Corp. one year ago: Long-term debt………………………….………….……………… $ 65,000,000 Preferred stock……………………….…...

See Answer

Q: Assuming the following ratios are constant, what is the sustainable growth

Assuming the following ratios are constant, what is the sustainable growth rate? Total asset turnover 5 2.20 Profit margin 5 7.4% Equity multiplier 5 1.40 Payout ratio 5 40%

See Answer

Q: TMCC has the right to buy back the securities on the anniversary

TMCC has the right to buy back the securities on the anniversary date at a price established when the securities were issued (this feature is a term of this particular deal). What impact does this fea...

See Answer

Q: Imprudential, Inc., has an unfunded pension liability of $630

Imprudential, Inc., has an unfunded pension liability of $630 million that must be paid in 20 years. To assess the value of the firm’s stock, financial analysts want to discount this liability back to...

See Answer

Q: Are the capital budgeting criteria we discussed applicable to not-for

Are the capital budgeting criteria we discussed applicable to not-for-profit corporations? How should such entities make capital budgeting decisions? What about the U.S. government? Should it evaluate...

See Answer

Q: Bill plans to open a self-serve grooming center in a

Bill plans to open a self-serve grooming center in a storefront. The grooming equipment will cost $385,000, to be paid immediately. Bill expects aftertax cash inflows of $84,000 annually for seven yea...

See Answer

Q: When is EAC analysis appropriate for comparing two or more projects?

When is EAC analysis appropriate for comparing two or more projects? Why is this method used? Are there any implicit assumptions required by this method that you find troubling? Explain.

See Answer

Q: Dog Up! Franks is looking at a new sausage system with

Dog Up! Franks is looking at a new sausage system with an installed cost of $375,000. This cost will be depreciated straight-line to zero over the project’s five-year life, at the end of which the sau...

See Answer

Q: You place an order for 500 units of inventory at a unit

You place an order for 500 units of inventory at a unit price of $135. The supplier offers terms of 1/10, net 30. a. How long do you have to pay before the account is overdue? If you take the full per...

See Answer

Q: The Mango Republic has just liberalized its markets and is now permitting

The Mango Republic has just liberalized its markets and is now permitting foreign investors. Tesla Manufacturing has analyzed starting a project in the country and has determined that the project has...

See Answer

Q: The manager for a growing firm is considering the launch of a

The manager for a growing firm is considering the launch of a new product. If the product goes directly to market, there is a 50 percent chance of success. For $175,000 the manager can conduct a focus...

See Answer

Q: If Treasury bills are currently paying 4.5 percent and the

If Treasury bills are currently paying 4.5 percent and the inflation rate is 2.1 percent, what is the approximate real rate of interest? The exact real rate?

See Answer

Q: Gruber Corp. pays a constant $9 dividend on its stock

Gruber Corp. pays a constant $9 dividend on its stock. The company will maintain this dividend for the next 12 years and will then cease paying dividends forever. If the required return on this stock...

See Answer

Q: Paper Submarine Manufacturing is investigating a lockbox system to reduce its collection

Paper Submarine Manufacturing is investigating a lockbox system to reduce its collection time. It has determined the following: The total collection time will be reduced by three days if the lockbox...

See Answer

Q: What are the different inventory types? How do the types differ

What are the different inventory types? How do the types differ? Why are some types said to have dependent demand, whereas other types are said to have independent demand?

See Answer

Q: Essence of Skunk Fragrances, Ltd., sells 4,900 units

Essence of Skunk Fragrances, Ltd., sells 4,900 units of its perfume collection each year at a price per unit of $495. All sales are on credit with terms of 1/10, net 40. The discount is taken by 40 pe...

See Answer

Q: Penn Corp. is analyzing the possible acquisition of Teller Company.

Penn Corp. is analyzing the possible acquisition of Teller Company. Both firms have no debt. Penn believes the acquisition will increase its total after tax annual cash flow by $1.1 million indefinite...

See Answer

Q: What are DIP loans? Where do DIP loans fall in the

What are DIP loans? Where do DIP loans fall in the APR?

See Answer

Q: Are the following statements true or false? Explain why.

Are the following statements true or false? Explain why. a. If the general price index in Great Britain rises faster than that in the United States, we would expect the pound to appreciate relative to...

See Answer

Q: Explain the purchase accounting method for mergers. What is the effect

Explain the purchase accounting method for mergers. What is the effect on cash flows? On EPS?

See Answer

Q: The treasurer of a major U.S. firm has $

The treasurer of a major U.S. firm has $30 million to invest for three months. The annual interest rate in the United States is .21 percent per month. The interest rate in Great Britain is .57 percent...

See Answer

Q: Corporate ownership varies around the world. Historically, individuals have owned

Corporate ownership varies around the world. Historically, individuals have owned the majority of shares in public corporations in the United States. In Germany and Japan, however, banks, other large...

See Answer

Q: Could a company’s change in net working capital be negative in a

Could a company’s change in net working capital be negative in a given year? Explain how this might come about. What about net capital spending?

See Answer

Q: The 2011 balance sheet of Anna’s Tennis Shop, Inc., showed

The 2011 balance sheet of Anna’s Tennis Shop, Inc., showed long-term debt of $1.45 million, and the 2012 balance sheet showed longterm debt of $1.52 million. The 2012 income statement showed an intere...

See Answer

Q: The most recent financial statements for Bradley, Inc., are shown

The most recent financial statements for Bradley, Inc., are shown here (assuming no income taxes): Assets and costs are proportional to sales. Debt and equity are not. No dividends are paid. Next ye...

See Answer

Q: Would you be willing to pay $24,099 today in

Would you be willing to pay $24,099 today in exchange for $100,000 in 30 years? What would be the key considerations in answering yes or no? Would your answer depend on who is making the promise to re...

See Answer

Q: Although appealing to more refined tastes, art as a collectible has

Although appealing to more refined tastes, art as a collectible has not always performed so profitably. During 2010, Deutscher-Menzies sold Arkies under the Shower, a painting by renowned Australian p...

See Answer

Q: The investment in Project A is $1 million, and the

The investment in Project A is $1 million, and the investment in Project B is $2 million. Both projects have a unique internal rate of return of 20 percent. Is the following statement true or false? F...

See Answer

Q: Suppose the following two independent investment opportunities are available to Greenplain,

Suppose the following two independent investment opportunities are available to Greenplain, Inc. The appropriate discount rate is 10 percent. a. Compute the profitability index for each of the two p...

See Answer

Q: “When evaluating projects, we’re only concerned with the relevant incremental

“When evaluating projects, we’re only concerned with the relevant incremental aftertax cash flows. Therefore, because depreciation is a noncash expense, we should ignore its effects when evaluating pr...

See Answer

Q: When the Beacon Computer Company filed for bankruptcy under Chapter 7 of

When the Beacon Computer Company filed for bankruptcy under Chapter 7 of the U.S. bankruptcy code, it had the following balance sheet information: Assuming there are no legal fees associated with t...

See Answer

Q: An asset used in a four-year project falls in the

An asset used in a four-year project falls in the five-year MACRS class for tax purposes. The asset has an acquisition cost of $7,100,000 and will be sold for $1,400,000 at the end of the project. If...

See Answer

Q: B&B has a new baby powder ready to market.

B&B has a new baby powder ready to market. If the firm goes directly to the market with the product, there is only a 55 percent chance of success. However, the firm can conduct customer segment resear...

See Answer

Q: U.S. Treasury bonds are not rated. Why?

U.S. Treasury bonds are not rated. Why? Often, junk bonds are not rated. Why?

See Answer

Q: Suppose the real rate is 2.4 percent and the inflation

Suppose the real rate is 2.4 percent and the inflation rate is 3.1 percent. What rate would you expect to see on a Treasury bill?

See Answer

Q: For each of the short-term marketable securities given here,

For each of the short-term marketable securities given here, provide an example of the potential disadvantages the investment has for meeting a corporation’s cash management goals: a. U.S. Treasury bi...

See Answer

Q: It takes Cookie Cutter Modular Homes, Inc., about five days

It takes Cookie Cutter Modular Homes, Inc., about five days to receive and deposit checks from customers. Cookie Cutter’s management is considering a lockbox system to reduce the firm’s collection tim...

See Answer

Q: The Arizona Bay Corporation sells on credit terms of net 30.

The Arizona Bay Corporation sells on credit terms of net 30. Its accounts are, on average, 6 days past due. If annual credit sales are $9.3 million, what is the company’s balance sheet amount in accou...

See Answer

Q: What types of actions might the management of a firm take to

What types of actions might the management of a firm take to fight a hostile acquisition bid from an unwanted suitor? How do the target firm shareholders benefit from the defensive tactics of their ma...

See Answer

Q: The shareholders of Flannery Company have voted in favor of a buyout

The shareholders of Flannery Company have voted in favor of a buyout offer from Stultz Corporation. Information about each firm is given here: Flannery’s shareholders will receive o...

See Answer

Q: Firms sometimes use the threat of a bankruptcy filing to force creditors

Firms sometimes use the threat of a bankruptcy filing to force creditors to renegotiate terms. Critics argue that in such cases the firm is using bankruptcy laws “as a sword rather than a shield.” Is...

See Answer

Q: Suppose the exchange rate for the Swiss franc is quoted as SF

Suppose the exchange rate for the Swiss franc is quoted as SF 1.09 in the spot market and SF 1.11 in the 90-day forward market. a. Is the dollar selling at a premium or a discount relative to the fran...

See Answer

Q: Some countries encourage movements in their exchange rate relative to those of

Some countries encourage movements in their exchange rate relative to those of some other country as a short-term means of addressing foreign trade imbalances. For each of the following scenarios, eva...

See Answer

Q: In recent years, large financial institutions such as mutual funds and

In recent years, large financial institutions such as mutual funds and pension funds have become the dominant owners of stock in the United States, and these institutions are becoming more active in c...

See Answer

Q: The 2011 balance sheet of Anna’s Tennis Shop, Inc., showed

The 2011 balance sheet of Anna’s Tennis Shop, Inc., showed $490,000 in the common stock account and $3.4 million in the additional paid-in surplus account. The 2012 balance sheet showed $525,000 and $...

See Answer

Q: Consider the ratio EBITD/Assets. What does this ratio tell

Consider the ratio EBITD/Assets. What does this ratio tell us? Why might it be more useful than ROA in comparing two companies?

See Answer

Q: Cheryl Colby, CFO of Charming Florist Ltd., has created the

Cheryl Colby, CFO of Charming Florist Ltd., has created the firm’s pro forma balance sheet for the next fiscal year. Sales are projected to grow by 10 percent to $420 million. Current assets, fixed as...

See Answer

Q: An investor purchasing a British consol is entitled to receive annual payments

An investor purchasing a British consol is entitled to receive annual payments from the British government forever. What is the price of a consol that pays $150 annually if the next payment occurs one...

See Answer

Q: Consider the following two mutually exclusive projects available to Global Investments,

Consider the following two mutually exclusive projects available to Global Investments, Inc.: The appropriate discount rate for the projects is 10 percent. Global Investments chose to undertake Proj...

See Answer

Q: A project has an initial cost of I, has a required

A project has an initial cost of I, has a required return of R, and pays C annually for N years. a. Find C in terms of I and N such that the project has a payback period just equal to its life. b. Fin...

See Answer

Q: A major college textbook publisher has an existing finance textbook. The

A major college textbook publisher has an existing finance textbook. The publisher is debating whether to produce an “essentialized” version, meaning a shorter (and lower-priced) book. What are some o...

See Answer

Q: Howell Petroleum is considering a new project that complements its existing business

Howell Petroleum is considering a new project that complements its existing business. The machine required for the project costs $3.8 million. The marketing department predicts that sales related to t...

See Answer

Q: Take a look back at Figure 31.1 to answer the

Take a look back at Figure 31.1 to answer the following questions: a. If you have $100, how many euros can you get? b. How much is one euro worth in dollars? c. If you have 5 million euros, how many d...

See Answer

Q: An option can often have more than one source of value.

An option can often have more than one source of value. Consider a logging company. The company can log the timber today or wait another year (or more) to log the timber. What advantages would waiting...

See Answer

Q: You are considering investing in a company that cultivates abalone for sale

You are considering investing in a company that cultivates abalone for sale to local restaurants. Use the following information: Sales price per abalone…………………………… =$35 Variable costs per abalone…………...

See Answer

Q: An investment offers a 14 percent total return over the coming year

An investment offers a 14 percent total return over the coming year. Alan Wingspan thinks the total real return on this investment will be only 10 percent. What does Alan believe the inflation rate wi...

See Answer

Q: The newspaper reported last week that Bennington Enterprises earned $34 million

The newspaper reported last week that Bennington Enterprises earned $34 million this year. The report also stated that the firm’s return on equity is 16 percent. Bennington retains 80 percent of its e...

See Answer

Q: Air Spares is a wholesaler that stocks engine components and test equipment

Air Spares is a wholesaler that stocks engine components and test equipment for the commercial aircraft industry. A new customer has placed an order for eight high-bypass turbine engines, which increa...

See Answer

Q: Cholern Electric Company (CEC) is a public utility that provides

Cholern Electric Company (CEC) is a public utility that provides electricity to the central Colorado area. Recent events at its Mile-High Nuclear Station have been discouraging. Several shareholders h...

See Answer

Q: Several firms have entered bankruptcy, or threatened to enter bankruptcy,

Several firms have entered bankruptcy, or threatened to enter bankruptcy, at least in part as a means of reducing labor costs. Whether this move is ethical, or proper, is hotly debated. Is this an eth...

See Answer

Q: Suppose your company imports computer motherboards from Singapore. The exchange rate

Suppose your company imports computer motherboards from Singapore. The exchange rate is given in Figure 31.1. You have just placed an order for 30,000 motherboards at a cost to you of 141.30 Singapore...

See Answer

Q: Critics have charged that compensation to top managers in the United States

Critics have charged that compensation to top managers in the United States is simply too high and should be cut back. For example, focusing on large corporations, Larry Ellison of Oracle has been one...

See Answer

Q: Given the information for Anna’s Tennis Shop, Inc., in the

Given the information for Anna’s Tennis Shop, Inc., in the previous two problems, suppose you also know that the firm’s net capital spending for 2012 was $945,000 and that the firm reduced its net wor...

See Answer

Q: Who owns a corporation? Describe the process whereby the owners control

Who owns a corporation? Describe the process whereby the owners control the firm’s management. What is the main reason that an agency relationship exists in the corporate form of organization? In this...

See Answer

Q: The Steiben Company has an ROE of 13.1 percent and

The Steiben Company has an ROE of 13.1 percent and a payout ratio of 40 percent. a. What is the company’s sustainable growth rate? b. Can the company’s actual growth rate be different from its sustain...

See Answer

Q: The TMCC security is bought and sold on the New York Stock

The TMCC security is bought and sold on the New York Stock Exchange. If you looked at the price today, do you think the price would exceed the $24,099 original price? Why? If you looked in the year 20...

See Answer

Q: Compute the future value of $1,900 continuously compounded for

Compute the future value of $1,900 continuously compounded for a. 7 years at a stated annual interest rate of 12 percent. b. 5 years at a stated annual interest rate of 10 percent. c. 12 years at a st...

See Answer

Q: Projects A and B have the following cash flows:

Projects A and B have the following cash flows: a. If the cash flows from the projects are identical, which of the two projects would have a higher IRR? Why? b. If C1B = 2C1A, C2B = 2C2A, and C3B =...

See Answer

Q: Suppose you are offered $7,000 today but must make

Suppose you are offered $7,000 today but must make the following payments: Year ………………………Cash Flows ($) 0 ……………………………………………….$ 7,000 1 …………………………………………………−3,700 2 ………………………………………………..−2,400 3 …………………...

See Answer

Q: You are evaluating two different silicon wafer milling machines. The Techron

You are evaluating two different silicon wafer milling machines. The Techron I costs $215,000, has a three-year life, and has pretax operating costs of $35,000 per year. The Techron II costs $270,000,...

See Answer

Q: You are discussing a project analysis with a coworker. The project

You are discussing a project analysis with a coworker. The project involves real options, such as expanding the project if successful, or abandoning the project if it fails. Your coworker makes the fo...

See Answer

Q: Niko has purchased a brand new machine to produce its High Flight

Niko has purchased a brand new machine to produce its High Flight line of shoes. The machine has an economic life of five years. The depreciation schedule for the machine is straight-line with no salv...

See Answer

Q: Say you own an asset that had a total return last year

Say you own an asset that had a total return last year of 12.5 percent. If the inflation rate last year was 5.3 percent, what was your real return?

See Answer

Q: The Germinating Flower Co. has earnings of $1.75

The Germinating Flower Co. has earnings of $1.75 per share. The benchmark PE for the company is 18. What stock price would you consider appropriate? What if the benchmark PE were 21?

See Answer

Q: Why might the revenue and cost figures shown on a standard income

Why might the revenue and cost figures shown on a standard income statement not represent the actual cash inflows and outflows that occurred during a period?

See Answer

Q: No More Books Corporation has an agreement with Floyd Bank, whereby

No More Books Corporation has an agreement with Floyd Bank, whereby the bank handles $3.2 million in collections a day and requires a $350,000 compensating balance. No More Books is contemplating canc...

See Answer

Q: At least part of Dell’s corporate profits can be traced to its

At least part of Dell’s corporate profits can be traced to its inventory management. Using just-in-time inventory, Dell typically maintains an inventory of three to four days’ sales. Competitors such...

See Answer

Q: Lealos, Inc., is considering a change in its cash-

Lealos, Inc., is considering a change in its cash-only sales policy. The new terms of sale would be net one month. Based on the following information, determine if Lealos should proceed or not. Descri...

See Answer

Q: Acquiring firm stockholders seem to benefit little from takeovers. Why is

Acquiring firm stockholders seem to benefit little from takeovers. Why is this finding a puzzle? What are some of the reasons offered for it?

See Answer

Q: Consider the following premerger information about a bidding firm (Firm B

Consider the following premerger information about a bidding firm (Firm B) and a target firm (Firm T). Assume that both firms have no debt outstanding. Firm B has estimated that the value of the syne...

See Answer

Q: Why do so many firms file for legal bankruptcy when private workouts

Why do so many firms file for legal bankruptcy when private workouts are so much less expensive?

See Answer

Q: Suppose the spot and six-month forward rates on the Norwegian

Suppose the spot and six-month forward rates on the Norwegian krone are Kr 5.61 and Kr 5.72, respectively. The annual risk-free rate in the United States is 3 percent, and the annual risk-free rate in...

See Answer

Q: Why is the goal of financial management to maximize the current share

Why is the goal of financial management to maximize the current share price of the company’s stock? In other words, why isn’t the goal to maximize the future share price?

See Answer

Q: Firm A and Firm B have debt–total asset ratios of

Firm A and Firm B have debt–total asset ratios of 35 percent and 55 percent and returns on total assets of 9 percent and 7 percent, respectively. Which firm has a greater return on equity?

See Answer

Q: Conoly Co. has identified an investment project with the following cash

Conoly Co. has identified an investment project with the following cash flows. If the discount rate is 10 percent, what is the present value of these cash flows? What is the present value at 18 percen...

See Answer

Q: You are evaluating Project A and Project B. Project A has

You are evaluating Project A and Project B. Project A has a short period of future cash flows, while Project B has relatively long future cash flows. Which project will be more sensitive to changes in...

See Answer

Q: In Problem 10, are the shareholders of Firm T better off

In Problem 10, are the shareholders of Firm T better off with the cash offer or the stock offer? At what exchange ratio of B shares to T shares would the shareholders in T be indifferent between the t...

See Answer

Q: The next dividend payment by ECY, Inc., will be $

The next dividend payment by ECY, Inc., will be $3.20 per share. The dividends are anticipated to maintain a growth rate of 6 percent, forever. If ECY stock currently sells for $63.50 per share, what...

See Answer

Q: Mau Corporation stock currently sells for $58.32 per share

Mau Corporation stock currently sells for $58.32 per share. The market requires a return of 11.5 percent on the firm’s stock. If the company maintains a constant 5 percent growth rate in dividends, wh...

See Answer

Q: In Problem 14, what is the break-even price per

In Problem 14, what is the break-even price per unit that should be charged under the new credit policy? Assume that the sales figure under the new policy is 3,400 units and all other values remain th...

See Answer

Q: The Chocolate Ice Cream Company and the Vanilla Ice Cream Company have

The Chocolate Ice Cream Company and the Vanilla Ice Cream Company have agreed to merge and form Fudge Swirl Consolidated. Both companies are exactly alike except that they are located in different tow...

See Answer

Q: We know that the actual relationship between a nominal rate, R

We know that the actual relationship between a nominal rate, R, a real rate, r, and an inflation rate, h, can be written as follows: 1 + r = (1 + R)y(1 + h) This is the domestic Fisher effect. a. Wha...

See Answer

Q: During 2012, Raines Umbrella Corp. had sales of $630

During 2012, Raines Umbrella Corp. had sales of $630,000. Cost of goods sold, administrative and selling expenses, and depreciation expenses were $470,000, $95,000, and $140,000, respectively. In addi...

See Answer

Q: Thorpe Mfg., Inc., is currently operating at only 90 percent

Thorpe Mfg., Inc., is currently operating at only 90 percent of fixed asset capacity. Current sales are $725,000. How much can sales increase before any new fixed assets are needed?

See Answer

Q: One of your customers is delinquent on his accounts payable balance.

One of your customers is delinquent on his accounts payable balance. You’ve mutually agreed to a repayment schedule of $700 per month. You will charge 1.3 percent per month interest on the overdue bal...

See Answer

Q: Consider two mutually exclusive new product launch projects that Nagano Golf is

Consider two mutually exclusive new product launch projects that Nagano Golf is considering. Assume the discount rate for Nagano Golf is 15 percent. Project A: Nagano NP-30. Professional clubs that...

See Answer

Q: Bridgton Golf Academy is evaluating different golf practice equipment. The “

Bridgton Golf Academy is evaluating different golf practice equipment. The “Dimple-Max” equipment costs $94,000, has a three-year life, and costs $8,600 per year to operate. The relevant discount rate...

See Answer

Q: A financial ratio by itself tells us little about a company because

A financial ratio by itself tells us little about a company because financial ratios vary a great deal across industries. There are two basic methods for analyzing financial ratios for a company: Time...

See Answer

Q: You purchase a bond with an invoice price of $950.

You purchase a bond with an invoice price of $950. The bond has a coupon rate of 6.8 percent, and there are 2 months to the next semiannual coupon date. What is the clean price of the bond?

See Answer

Q: Each business day, on average, a company writes checks totaling

Each business day, on average, a company writes checks totaling $17,000 to pay its suppliers. The usual clearing time for the checks is four days. Meanwhile, the company is receiving payments from its...

See Answer

Q: You purchase a bond with an invoice price of $950.

You purchase a bond with an invoice price of $950. The bond has a coupon rate of 6.8 percent, and there are 2 months to the next semiannual coupon date. What is the clean price of the bond?

See Answer

Q: Fifth National Bank just issued some new preferred stock. The issue

Fifth National Bank just issued some new preferred stock. The issue will pay an annual dividend of $8 in perpetuity, beginning five years from now. If the market requires a return of 5.6 percent on th...

See Answer

Q: In Problem 15, what is the break-even price per

In Problem 15, what is the break-even price per unit under the new credit policy? Assume all other values remain the same. Problem 15

See Answer

Q: In Problem 18, suppose Raines Umbrella Corp. paid out $

In Problem 18, suppose Raines Umbrella Corp. paid out $34,000 in cash dividends. Is this possible? If spending on net fixed assets and net working capital was zero, and if no new stock was issued duri...

See Answer

Q: For the company in the previous problem, suppose fixed assets are

For the company in the previous problem, suppose fixed assets are $690,000 and sales are projected to grow to $830,000. How much in new fixed assets are required to support this growth in sales?

See Answer

Q: Friendly’s Quick Loans, Inc., offers you “three for four

Friendly’s Quick Loans, Inc., offers you “three for four or I knock on your door.” This means you get $3 today and repay $4 when you get your paycheck in one week (or else). What’s the effective annua...

See Answer

Q: You are evaluating a project that costs $75,000 today

You are evaluating a project that costs $75,000 today. The project has an inflow of $155,000 in one year and an outflow of $65,000 in two years. What are the IRRs for the project? What discount rate r...

See Answer

Q: Scott Investors, Inc., is considering the purchase of a $

Scott Investors, Inc., is considering the purchase of a $360,000 computer with an economic life of five years. The computer will be fully depreciated over five years using the straight-line method. Th...

See Answer

Q: Nuber Company has a debt–equity ratio of .80.

Nuber Company has a debt–equity ratio of .80. Return on assets is 9.7 percent, and total equity is $735,000. What is the equity multiplier? Return on equity? Net income?

See Answer

Q: Your buddy comes to you with a sure-fire way to

Your buddy comes to you with a sure-fire way to make some quick money and help pay off your student loans. His idea is to sell T-shirts with the words “I get” on them. “You get it?” He says, “You see...

See Answer

Q: You purchase a bond with a coupon rate of 5.9

You purchase a bond with a coupon rate of 5.9 percent and a clean price of $1,053. If the next semiannual coupon payment is due in four months, what is the invoice price?

See Answer

Q: If Jares, Inc., has an equity multiplier of 1.

If Jares, Inc., has an equity multiplier of 1.55, total asset turnover of 1.75, and a profit margin of 4.3 percent, what is its ROE?

See Answer

Q: Indicate whether you think the following claims regarding takeovers are true or

Indicate whether you think the following claims regarding takeovers are true or false. In each case, provide a brief explanation for your answer. a. By merging competitors, takeovers have created mono...

See Answer

Q: You have found the following stock quote for RJW Enterprises, Inc

You have found the following stock quote for RJW Enterprises, Inc., in the financial pages of today’s newspaper. What is the annual dividend? What was the closing price for this stoc...

See Answer

Q: Saché, Inc., expects to sell 700 of its designer suits

Saché, Inc., expects to sell 700 of its designer suits every week. The store is open seven days a week and expects to sell the same number of suits every day. The company has an EOQ of 500 suits and a...

See Answer

Q: Cusic Industries had the following operating results for 2012: sales =

Cusic Industries had the following operating results for 2012: sales = $19,900; cost of goods sold = $14,200; depreciation expense = $2,700; interest expense = $670; dividends paid = $650. At the begi...

See Answer

Q: The most recent financial statements for Moose Tours, Inc., appear

The most recent financial statements for Moose Tours, Inc., appear below. Sales for 2012 are projected to grow by 20 percent. Interest expense will remain constant; the tax rate and the dividend payou...

See Answer

Q: What is the future value in six years of $1,

What is the future value in six years of $1,000 invested in an account with a stated annual interest rate of 9 percent, a. Compounded annually? b. Compounded semiannually? c. Compounded monthly? d. Co...

See Answer

Q: An investment under consideration has a payback of six years and a

An investment under consideration has a payback of six years and a cost of $434,000. If the required return is 12 percent, what is the worst-case NPV? The best-case NPV? Explain. Assume the cash flows...

See Answer

Q: Suppose it is your task to evaluate two different investments in new

Suppose it is your task to evaluate two different investments in new subsidiaries for your company, one in your own country and the other in a foreign country. You calculate the cash flows of both pro...

See Answer

Q: A firm is considering an investment in a new machine with a

A firm is considering an investment in a new machine with a price of $18 million to replace its existing machine. The current machine has a book value of $6 million and a market value of $4.5 million....

See Answer

Q: Young screenwriter Carl Draper has just finished his first script. It

Young screenwriter Carl Draper has just finished his first script. It has action, drama, and humor, and he thinks it will be a blockbuster. He takes the script to every motion picture studio in town a...

See Answer

Q: Argos Corp. has 9 percent coupon bonds making annual payments with

Argos Corp. has 9 percent coupon bonds making annual payments with a YTM of 7.81 percent. The current yield on these bonds is 8.42 percent. How many years do these bonds have left until they mature?

See Answer

Q: You own $100,000 worth of Smart Money stock.

You own $100,000 worth of Smart Money stock. One year from now, you will receive a dividend of $2.25 per share. You will receive a $2.40 dividend two years from now. You will sell the stock for $65 pe...

See Answer

Q: Consider the following premerger information about firm X and firm Y :

Consider the following premerger information about firm X and firm Y : Assume that Firm X acquires Firm Y by paying cash for all the shares outstanding at a merger premium of $5 per share. Assuming t...

See Answer

Q: Solar Engines manufactures solar engines for tractor trailers. Given the fuel

Solar Engines manufactures solar engines for tractor trailers. Given the fuel savings available, new orders for 125 units have been made by customers requesting credit. The variable cost is $11,400 pe...

See Answer

Q: Consider the following abbreviated financial statements for Weston Enterprises:

Consider the following abbreviated financial statements for Weston Enterprises: a. What is owners’ equity for 2011 and 2012? b. What is the change in net working capital for 2012?...

See Answer

Q: This problem is useful for testing the ability of financial calculators and

This problem is useful for testing the ability of financial calculators and computer software. Consider the following cash flows. How many different IRRs are there? When should we take this project?...

See Answer

Q: Sanders Enterprises, Inc., has been considering the purchase of a

Sanders Enterprises, Inc., has been considering the purchase of a new manufacturing facility for $270,000. The facility is to be fully depreciated on a straight-line basis over seven years. It is expe...

See Answer

Q: Hickock Mining is evaluating when to open a gold mine. The

Hickock Mining is evaluating when to open a gold mine. The mine has 48,000 ounces of gold left that can be mined, and mining operations will produce 6,000 ounces per year. The required return on the g...

See Answer

Q: You observe that the inflation rate in the United States is 1

You observe that the inflation rate in the United States is 1.8 percent per year and that T-bills currently yield 2.3 percent annually. What do you estimate the inflation rate to be in: a. Australia i...

See Answer

Q: Suppose the following bond quote for IOU Corporation appears in the financial

Suppose the following bond quote for IOU Corporation appears in the financial page of today’s newspaper. Assume the bond has a face value of $1,000 and the current date is April 15,...

See Answer

Q: Pasqually Mineral Water, Inc., will pay a quarterly dividend per

Pasqually Mineral Water, Inc., will pay a quarterly dividend per share of $.80 at the end of each of the next 12 quarters. Thereafter, the dividend will grow at a quarterly rate of 1 percent, forever....

See Answer

Q: In the previous problem, assume that the probability of default is

In the previous problem, assume that the probability of default is 15 percent. Should the orders be filled now? Assume the number of repeat customers is affected by the defaults. In other words, 30 pe...

See Answer

Q: First Simple Bank pays 5 percent simple interest on its investment accounts

First Simple Bank pays 5 percent simple interest on its investment accounts. If First Complex Bank pays interest on its accounts compounded annually, what rate should the bank set if it wants to match...

See Answer

Q: The Yurdone Corporation wants to set up a private cemetery business.

The Yurdone Corporation wants to set up a private cemetery business. According to the CFO, Barry M. Deep, business is “looking up.” As a result, the cemetery project will provide a net cash inflow of...

See Answer

Q: When the Master Printing Company filed for bankruptcy, it filed under

When the Master Printing Company filed for bankruptcy, it filed under Chapter 11 of the U.S. bankruptcy code. Key information is shown here: As a trustee, what reorganization plan would you accept?...

See Answer

Q: With the growing popularity of casual surf print clothing, two recent

With the growing popularity of casual surf print clothing, two recent MBA graduates decided to broaden this casual surf concept to encompass a “surf lifestyle for the home.” With limited capital, they...

See Answer

Q: Allied Products, Inc., is considering a new product launch.

Allied Products, Inc., is considering a new product launch. The firm expects to have an annual operating cash flow of $10.5 million for the next 10 years. Allied Products uses a discount rate of 13 pe...

See Answer

Q: You’ve just found a 10 percent coupon bond on the market that

You’ve just found a 10 percent coupon bond on the market that sells for par value. What is the maturity on this bond?

See Answer

Q: Briley, Inc., is expected to pay equal dividends at the

Briley, Inc., is expected to pay equal dividends at the end of each of the next two years. Thereafter, the dividend will grow at a constant annual rate of 4 percent, forever. The current stock price i...

See Answer

Q: Ritter Corporation’s accountants prepared the following financial statements for year-end

Ritter Corporation’s accountants prepared the following financial statements for year-end 2012: a. Explain the change in cash during 2012. b. Determine the change in net working capi...

See Answer

Q: You are planning to save for retirement over the next 30 years

You are planning to save for retirement over the next 30 years. To do this, you will invest $800 a month in a stock account and $350 a month in a bond account. The return of the stock account is expec...

See Answer

Q: Suppose an investment offers to quadruple your money in 12 months (

Suppose an investment offers to quadruple your money in 12 months (don’t believe it). What rate of return per quarter are you being offered?

See Answer

Q: The Utah Mining Corporation is set to open a gold mine near

The Utah Mining Corporation is set to open a gold mine near Provo, Utah. According to the treasurer, Monty Goldstein, “This is a golden opportunity.” The mine will cost $2,400,000 to open and will hav...

See Answer

Q: You have been hired as a consultant for Pristine Urban-Tech

You have been hired as a consultant for Pristine Urban-Tech Zither, Inc. (PUTZ), manufacturers of fine zithers. The market for zithers is growing quickly. The company bought some land three years ago...

See Answer

Q: Applied Nanotech is thinking about introducing a new surface cleaning machine.

Applied Nanotech is thinking about introducing a new surface cleaning machine. The marketing department has come up with the estimate that Applied Nanotech can sell 15 units per year at $305,000 net c...

See Answer

Q: Tesla Corporation needs to raise funds to finance a plant expansion,

Tesla Corporation needs to raise funds to finance a plant expansion, and it has decided to issue 25-year zero coupon bonds to raise the money. The required return on the bonds will be 7 percent. a. W...

See Answer

Q: Use the information in Figure 31.1 to answer the following

Use the information in Figure 31.1 to answer the following questions: a. Which would you rather have, $100 or £100? Why? b. Which would you rather have, 100 Swiss francs (SF) or Â&p...

See Answer

Q: Juggernaut Satellite Corporation earned $18 million for the fiscal year ending

Juggernaut Satellite Corporation earned $18 million for the fiscal year ending yesterday. The firm also paid out 30 percent of its earnings as dividends yesterday. The firm will continue to pay out 30...

See Answer

Q: You are researching Time Manufacturing and have found the following accounting statement

You are researching Time Manufacturing and have found the following accounting statement of cash flows for the most recent year. You also know that the company paid $98 million in current taxes and ha...

See Answer

Q: You’re trying to choose between two different investments, both of which

You’re trying to choose between two different investments, both of which have up-front costs of $65,000. Investment G returns $125,000 in six years. Investment H returns $185,000 in 10 years. Which of...

See Answer

Q: : The Grandmother Calendar Company clearly had a cash flow problem.

The Grandmother Calendar Company clearly had a cash flow problem. In the context of the cash flow analysis we developed, what was the impact of customers not paying until orders were shipped?

See Answer

Q: Butler International Limited is evaluating a project in Erewhon. The project

Butler International Limited is evaluating a project in Erewhon. The project will create the following cash flows: Year ………………..Cash Flow 0…………………..……….−$950,000 1……………..………..……….285,000 2………………….….…...

See Answer

Q: Pilot Plus Pens is deciding when to replace its old machine.

Pilot Plus Pens is deciding when to replace its old machine. The machine’s current salvage value is $2.2 million. Its current book value is $1.4 million. If not sold, the old machine will require main...

See Answer

Q: You are the financial analyst for a tennis racket manufacturer. The

You are the financial analyst for a tennis racket manufacturer. The company is considering using a graphite like material in its tennis rackets. The company has estimated the information in the follow...

See Answer

Q: Suppose your company needs to raise $45 million and you want

Suppose your company needs to raise $45 million and you want to issue 30-year bonds for this purpose. Assume the required return on your bond issue will be 6 percent, and you’re evaluating two issue a...

See Answer

Q: Four years ago, Bling Diamond, Inc., paid a dividend

Four years ago, Bling Diamond, Inc., paid a dividend of $1.35 per share. Bling paid a dividend of $1.77 per share yesterday. Dividends will grow over the next five years at the same rate they grew ove...

See Answer

Q: On the balance sheet, the net fixed assets (NFA)

On the balance sheet, the net fixed assets (NFA) account is equal to the gross fixed assets (FA) account, which records the acquisition cost of fixed assets, minus the accumulated depreciation (AD) ac...

See Answer

Q: Bulla Recording, Inc., wishes to maintain a growth rate of

Bulla Recording, Inc., wishes to maintain a growth rate of 12 percent per year and a debt–equity ratio of .40. Profit margin is 5.3 percent, and the ratio of total assets to sales is constant at .75....

See Answer

Q: Suppose you were the financial manager of a not-for-

Suppose you were the financial manager of a not-for- profit business (a not-for-profit hospital, perhaps). What kinds of goals do you think would be appropriate?

See Answer

Q: Consider two streams of cash flows, A and B. Stream

Consider two streams of cash flows, A and B. Stream A’s first cash flow is $8,900 and is received three years from today. Future cash flows in Stream A grow by 4 percent in perpetuity. Stream B’s firs...

See Answer

Q: Office Automation, Inc., must choose between two copiers, the

Office Automation, Inc., must choose between two copiers, the XX40 or the RH45. The XX40 costs $900 and will last for three years. The copier will require a real aftertax cost of $120 per year after a...

See Answer

Q: Prince Albert Canning PLC had a net loss of £37,

Prince Albert Canning PLC had a net loss of £37,543 on sales of £345,182. What was the company’s profit margin? Does the fact that these figures are quoted in a foreign currency make any difference? W...

See Answer

Q: Consider a project to supply Detroit with 35,000 tons of

Consider a project to supply Detroit with 35,000 tons of machine screws annually for automobile production. You will need an initial $2,900,000 investment in threading equipment to get the project sta...

See Answer

Q: Bond P is a premium bond with a 9 percent coupon.

Bond P is a premium bond with a 9 percent coupon. Bond D is a 5 percent coupon bond currently selling at a discount. Both bonds make annual payments, have a YTM of 7 percent, and have 10 years to matu...

See Answer

Q: Consider Pacific Energy Company and U.S. Blue chips,

Consider Pacific Energy Company and U.S. Blue chips, Inc., both of which reported earnings of $950,000. Without new projects, both firms will continue to generate earnings of $950,000 in perpetuity. A...

See Answer

Q: Refer to the corporate marginal tax rate information in Table 2.

Refer to the corporate marginal tax rate information in Table 2.3. a. Why do you think the marginal tax rate jumps up from 34 percent to 39 percent at a taxable income of $100,001, and then falls back...

See Answer

Q: A prestigious investment bank designed a new security that pays a quarterly

A prestigious investment bank designed a new security that pays a quarterly dividend of $4.50 in perpetuity. The first dividend occurs one quarter from today. What is the price of the security if the...

See Answer

Q: Darin Clay, the CFO of MakeMoney.com, has to

Darin Clay, the CFO of MakeMoney.com, has to decide between the following two projects: The expected rate of return for either of the two projects is 12 percent. What is the range of initial investm...

See Answer

Q: Dickinson Brothers, Inc., is considering investing in a machine to

Dickinson Brothers, Inc., is considering investing in a machine to produce computer keyboards. The price of the machine will be $975,000, and its economic life is five years. The machine will be fully...

See Answer

Q: Consider a project to supply Detroit with 35,000 tons

Consider a project to supply Detroit with 35,000 tons of machine screws annually for automobile production. You will need an initial $2,900,000 investment in threading equipment to get the project st...

See Answer

Q: Why do you think most long-term financial planning begins with

Why do you think most long-term financial planning begins with sales forecasts? Put differently, why are future sales the key input?

See Answer

Q: The YTM on a bond is the interest rate you earn on

The YTM on a bond is the interest rate you earn on your investment if interest rates don’t change. If you actually sell the bond before it matures, your realized return is known as the holding period...

See Answer

Q: Investment X offers to pay you $4,500 per year

Investment X offers to pay you $4,500 per year for nine years, whereas Investment Y offers to pay you $7,000 per year for five years. Which of these cash flow streams has the higher present value if t...

See Answer

Q: The Stambaugh Corporation currently has earnings per share of $9.

The Stambaugh Corporation currently has earnings per share of $9.40. The company has no growth and pays out all earnings as dividends. It has a new project that will require an investment of $1.95 per...

See Answer

Q: Define the following: S = Previous year’s sales

Define the following: S = Previous year’s sales A = Total assets E = Total equity g = Projected growth in sales PM = Profit margin b = Retention (plowback) ratio Assuming that all debt is constant, sh...

See Answer

Q: McKeekin Corp. has a project with the following cash flows:

McKeekin Corp. has a project with the following cash flows: Year …………………Cash Flow 0 ……………………………….$20,000 1 ………………………………..−26,000 2 …………………………………..13,000 What is the IRR of the project? What is happe...

See Answer

Q: Aguilera Acoustics, Inc. (AAI), projects unit sales for

Aguilera Acoustics, Inc. (AAI), projects unit sales for a new seven-octave voice emulation implant as follows: Year Unit Sales 1 ……………………….……….. 83,000 2 ………….…………………….. 92,000 3 …………….………………… 104,00...

See Answer

Q: Consider the following project for Hand Clapper, Inc. The company

Consider the following project for Hand Clapper, Inc. The company is considering a four-year project to manufacture clap-command garage door openers. This project requires an initial investment of $8...

See Answer

Q: The Morgan Corporation has two different bonds currently outstanding. Bond M

The Morgan Corporation has two different bonds currently outstanding. Bond M has a face value of $30,000 and matures in 20 years. The bond makes no payments for the first six years, then pays $800 eve...

See Answer

Q: Rite Bite Enterprises sells toothpicks. Gross revenues last year were $

Rite Bite Enterprises sells toothpicks. Gross revenues last year were $7.5 million, and total costs were $3.4 million. Rite Bite has 1 million shares of common stock outstanding. Gross revenues and co...

See Answer

Q: What is the present value of an annuity of $6,

What is the present value of an annuity of $6,500 per year, with the first cash flow received three years from today and the last one received 25 years from today? Use a discount rate of 7 percent.

See Answer

Q: What is the value today of a 15-year annuity that

What is the value today of a 15-year annuity that pays $650 a year? The annuity’s first payment occurs six years from today. The annual interest rate is 11 percent for Years 1 through 5, and 13 percen...

See Answer

Q: Y3K, Inc., has sales of $2,700,

Y3K, Inc., has sales of $2,700, total assets of $1,310, and a debt–equity ratio of 1.20. If its return on equity is 15 percent, what is its net income?

See Answer

Q: A proposed cost-saving device has an installed cost of $

A proposed cost-saving device has an installed cost of $640,000. The device will be used in a five-year project but is classified as three year MACRS property for tax purposes. The required initial ne...

See Answer

Q: Ramsay Corp. currently has an EPS of $2.35

Ramsay Corp. currently has an EPS of $2.35, and the benchmark PE for the company is 21. Earnings are expected to grow at 7 percent per year. a. What is your estimate of the current stock price? b. Wha...

See Answer

Q: The following premerger information about Firm A and Firm B:

The following premerger information about Firm A and Firm B: Assume that Firm A acquires Firm B via an exchange of stock at a price of $18 for each share of B ’s stock. Both A and B...

See Answer

Q: In the chapter, we used Rosengarten Corporation to demonstrate how to

In the chapter, we used Rosengarten Corporation to demonstrate how to calculate EFN. The ROE for Rosengarten is about 7.3 percent, and the plowback ratio is about 67 percent. If you calculate the sust...

See Answer

Q: The most recent financial statements for Martin, Inc., are shown

The most recent financial statements for Martin, Inc., are shown here: Assets and costs are proportional to sales. Debt and equity are not. A dividend of $2,500 was paid, and Martin wishes to mainta...

See Answer

Q: Solve for the unknown interest rate in each of the following:

Solve for the unknown interest rate in each of the following:

See Answer

Q: A project has perpetual cash flows of C per period, a

A project has perpetual cash flows of C per period, a cost of I , and a required return of R . What is the relationship between the project’s payback and its IRR? What implications does your answer ha...

See Answer

Q: An investment project costs $15,000 and has annual cash

An investment project costs $15,000 and has annual cash flows of $3,800 for six years. What is the discounted payback period if the discount rate is 0 percent? What if the discount rate is 10 percent?...

See Answer

Q: Fuji Software, Inc., has the following mutually exclusive projects.

Fuji Software, Inc., has the following mutually exclusive projects. Year Project a. Suppose Fuji’s payback period cutoff is two years. Which of these two projects should be chosen?...

See Answer

Q: Given the choice, would a firm prefer to use MACRS depreciation

Given the choice, would a firm prefer to use MACRS depreciation or straight-line depreciation? Why?

See Answer

Q: Toys Inc. just purchased a $390,000 machine to

Toys Inc. just purchased a $390,000 machine to produce toy cars. The machine will be fully depreciated by the straight-line method over its five-year economic life. Each toy sells for $25. The variabl...

See Answer

Q: Rhiannon Corporation has bonds on the market with 11.5 years

Rhiannon Corporation has bonds on the market with 11.5 years to maturity, a YTM of 7.6 percent, and a current price of $1,060. The bonds make semiannual payments. What must the coupon rate be on these...

See Answer

Q: Consider the following cash flows on two mutually exclusive projects for the

Consider the following cash flows on two mutually exclusive projects for the Bahamas Recreation Corporation (BRC). Both projects require an annual return of 14 percent. As a financial analyst for BR...

See Answer

Q: Under what two assumptions can we use the dividend growth model presented

Under what two assumptions can we use the dividend growth model presented in the chapter to determine the value of a share of stock? Comment on the reasonableness of these assumptions.

See Answer

Q: An investment in a foreign subsidiary is estimated to have a positive

An investment in a foreign subsidiary is estimated to have a positive NPV after the discount rate used in the calculations is adjusted for political risk and any advantages from diversification. Does...

See Answer

Q: White Wedding Corporation will pay a $2.65 per share

White Wedding Corporation will pay a $2.65 per share dividend next year. The company pledges to increase its dividend by 4.75 percent per year, indefinitely. If you require a return of 11 percent on y...

See Answer

Q: Your neighbor goes to the post office once a month and picks

Your neighbor goes to the post office once a month and picks up two checks, one for $11,000 and one for $3,400. The larger check takes four days to clear after it is deposited; the smaller one takes f...

See Answer

Q: What are the five Cs of credit? Explain why each is

What are the five Cs of credit? Explain why each is important.

See Answer

Q: In July 2011, fast food restaurant chain Wendy’s/Arby’s announced

In July 2011, fast food restaurant chain Wendy’s/Arby’s announced that it had sold its Arby’s restaurants and would change its name back to Wendy’s. Arby’s was purchased by the private equity firm Roa...

See Answer

Q: In the previous problem, suppose the fair market value of James’s

In the previous problem, suppose the fair market value of James’s fixed assets is $15,000 versus the $8,900 book value shown. Jurion pays $23,000 for James and raises the needed fund...

See Answer

Q: Jon Fulkerson has also received a credit application from Seether, LLC

Jon Fulkerson has also received a credit application from Seether, LLC, a private company. An abbreviated portion of the financial information provided by the company is shown below: Total assets ………...

See Answer

Q: We are evaluating a project that costs $644,000,

We are evaluating a project that costs $644,000, has an eight-year life, and has no salvage value. Assume that depreciation is straight-line to zero over the life of the project. Sales are projected a...

See Answer

Q: Suppose the spot exchange rate for the Canadian dollar is Can$

Suppose the spot exchange rate for the Canadian dollar is Can$1.05 and the six-month forward rate is Can$1.03. a. Which is worth more, a U.S. dollar or a Canadian dollar? b. Assuming absolute PPP hold...

See Answer

Q: The Robb Computer Corporation is trying to choose between the following two

The Robb Computer Corporation is trying to choose between the following two mutually exclusive design projects: a. If the required return is 10 percent and Robb Computer applies the profitability in...

See Answer

Q: Can the goal of maximizing the value of the stock conflict with

Can the goal of maximizing the value of the stock conflict with other goals, such as avoiding unethical or illegal behavior? In particular, do you think subjects like customer and employee safety, the...

See Answer

Q: Ranney, Inc., has sales of $18,700,

Ranney, Inc., has sales of $18,700, costs of $10,300, depreciation expense of $1,900, and interest expense of $1,250. If the tax rate is 40 percent, what is the operating cash flow, or OCF?

See Answer

Q: Massey Machine Shop is considering a four-year project to improve

Massey Machine Shop is considering a four-year project to improve its production efficiency. Buying a new machine press for $640,000 is estimated to result in $270,000 in annual pretax cost savings. T...

See Answer

Q: Suppose the spot and three-month forward rates for the yen

Suppose the spot and three-month forward rates for the yen are ¥80.13 and ¥78.96, respectively. a. Is the yen expected to get stronger or weaker? b. What would you estimate is the difference between t...

See Answer

Q: Broslofski Co. maintains a positive retention ratio and keeps its debt

Broslofski Co. maintains a positive retention ratio and keeps its debt–equity ratio constant every year. When sales grow by 20 percent, the firm has a negative projected EFN. What does this tell you a...

See Answer

Q: The most recent financial statements for Fontenot Co. are shown here

The most recent financial statements for Fontenot Co. are shown here: Assets and costs are proportional to sales. The company maintains a constant 30 percent dividend payout ratio and a constant deb...

See Answer

Q: Solve for the unknown number of years in each of the following

Solve for the unknown number of years in each of the following:

See Answer

Q: In October 2010, BMW announced plans to spend $1 billion

In October 2010, BMW announced plans to spend $1 billion to expand production at its plant in South Carolina. The plant produced the second generation BMW X3 as well as the company’s X5 and X6 models....

See Answer

Q: Stone Sour, Inc., has a project with the following cash

Stone Sour, Inc., has a project with the following cash flows: Year ………………………..Cash Flows ($) 0 ………………………………….………….−$20,000 1 …………………………………………………….8,500 2 …………………………………………………..10,200 3 ……………………………………...

See Answer

Q: In our capital budgeting examples, we assumed that a firm would

In our capital budgeting examples, we assumed that a firm would recover all of the working capital it invested in a project. Is this a reasonable assumption? When might it not be valid?

See Answer

Q: For each of the following, compute the present value:

For each of the following, compute the present value:

See Answer

Q: Assume a firm is considering a new project that requires an initial

Assume a firm is considering a new project that requires an initial investment and has equal sales and costs over its life. Will the project reach the accounting, cash, or financial break-even point f...

See Answer

Q: What is the price of a 15-year, zero coupon

What is the price of a 15-year, zero coupon bond paying $1,000 at maturity if the YTM is: a. 5 percent? b. 10 percent? c. 15 percent?

See Answer

Q: Your company is deciding whether to invest in a new machine.

Your company is deciding whether to invest in a new machine. The new machine will increase cash flow by $475,000 per year. You believe the technology used in the machine has a 10-year life; in other w...

See Answer

Q: How does a bond issuer decide on the appropriate coupon rate to

How does a bond issuer decide on the appropriate coupon rate to set on its bonds? Explain the difference between the coupon rate and the required return on a bond.

See Answer

Q: The Stancil Corporation provided the following current information: Proceeds

The Stancil Corporation provided the following current information: Proceeds from long-term borrowing……………….………… $17,000 Proceeds from the sale of common stock…………..…………. 4,000 Purchases of fixed ass...

See Answer

Q: Even though most corporate bonds in the United States make coupon payments

Even though most corporate bonds in the United States make coupon payments semiannually, bonds issued elsewhere often have annual coupon payments. Suppose a German company issues a bond with a par val...

See Answer

Q: Siblings, Inc., is expected to maintain a constant 6.

Siblings, Inc., is expected to maintain a constant 6.4 percent growth rate in its dividends, indefinitely. If the company has a dividend yield of 4.3 percent, what is the required return on the compan...

See Answer

Q: Your firm has an average receipt size of $117. A

Your firm has an average receipt size of $117. A bank has approached you concerning a lockbox service that will decrease your total collection time by two days. You typically receive 6,500 checks per...

See Answer

Q: What are some of the factors that determine the length of the

What are some of the factors that determine the length of the credit period? Why is the length of the buyer’s operating cycle often considered an upper bound on the length of the credit period?

See Answer

Q: A firm offers terms of 1/10, net 30.

A firm offers terms of 1/10, net 30. What effective annual interest rate does the firm earn when a customer does not take the discount? Without doing any calculations, explain what will happen to this...

See Answer

Q: Use the sustainable growth rate equations from the previous problem to answer

Use the sustainable growth rate equations from the previous problem to answer the following questions. No Return, Inc., had total assets of $285,000 and equity of $176,000 at the beginning of the year...

See Answer

Q: Silver Enterprises has acquired All Gold Mining in a merger transaction.

Silver Enterprises has acquired All Gold Mining in a merger transaction. Construct the balance sheet for the new corporation if the merger is treated as a purchase for accounting purposes. The market...

See Answer

Q: Suppose the Japanese yen exchange rate is ¥85 = $1

Suppose the Japanese yen exchange rate is ¥85 = $1, and the British pound exchange rate is £ 1 = $1.53. a. What is the cross-rate in terms of yen per pound? b. Suppose the cross-rate is ¥131.4 = £ 1....

See Answer

Q: Under standard accounting rules, it is possible for a company’s liabilities

Under standard accounting rules, it is possible for a company’s liabilities to exceed its assets. When this occurs, the owners’ equity is negative. Can this happen with market values? Why or why not?...

See Answer

Q: The Starr Co. just paid a dividend of $2.

The Starr Co. just paid a dividend of $2.15 per share on its stock. The dividends are expected to grow at a constant rate of 5 percent per year, indefinitely. If investors require a return of 11 perce...

See Answer

Q: Gordon Driving School’s 2011 balance sheet showed net fixed assets of $

Gordon Driving School’s 2011 balance sheet showed net fixed assets of $1.42 million, and the 2012 balance sheet showed net fixed assets of $1.69 million. The company’s 2012 income statement showed a d...

See Answer

Q: The Optical Scam Company has forecast a 15 percent sales growth rate

The Optical Scam Company has forecast a 15 percent sales growth rate for next year. The current financial statements are shown here: a. Using the equation from the chapter, calculate the external fu...

See Answer

Q: One tool of financial analysis is common-size financial statements.

One tool of financial analysis is common-size financial statements. Why do you think common-size income statements and balance sheets are used? Note that the accounting statement of cash flows is not...

See Answer

Q: If the Layla Corp. has a 13 percent ROE and a

If the Layla Corp. has a 13 percent ROE and a 20 percent payout ratio, what is its sustainable growth rate?

See Answer

Q: At 8 percent interest, how long does it take to double

At 8 percent interest, how long does it take to double your money? To quadruple it?

See Answer

Q: What are some of the difficulties that might come up in actual

What are some of the difficulties that might come up in actual applications of the various criteria we discussed in this chapter? Which one would be the easiest to implement in actual applications? Th...

See Answer

Q: The technique for calculating a bid price can be extended to many

The technique for calculating a bid price can be extended to many other types of problems. Answer the following questions using the same technique as setting a bid price; that is, set the project NPV...

See Answer

Q: Compute the internal rate of return for the cash flows of the

Compute the internal rate of return for the cash flows of the following two projects:

See Answer

Q: Suppose a financial manager is quoted as saying, “Our firm

Suppose a financial manager is quoted as saying, “Our firm uses the stand-alone principle. Because we treat projects like minifirms in our evaluation process, we include financing costs because they a...

See Answer

Q: Your firm is contemplating the purchase of a new $670,

Your firm is contemplating the purchase of a new $670,000 computer-based order entry system. The system will be depreciated straight-line to zero over its five-year life. It will be worth $50,000 at t...

See Answer

Q: Why does traditional NPV analysis tend to underestimate the true value of

Why does traditional NPV analysis tend to underestimate the true value of a capital budgeting project?

See Answer

Q: Ang Electronics, Inc., has developed a new DVDR. If

Ang Electronics, Inc., has developed a new DVDR. If the DVDR is successful, the present value of the payoff (when the product is brought to market) is $34 million. If the DVDR fails, the present value...

See Answer

Q: An investment offers $4,900 per year for 15 years

An investment offers $4,900 per year for 15 years, with the first payment occurring one year from now. If the required return is 8 percent, what is the value of the investment? What would the value be...

See Answer

Q: Suppose a project has conventional cash flows and a positive NPV.

Suppose a project has conventional cash flows and a positive NPV. What do you know about its payback? Its discounted payback? Its profitability index? Its IRR? Explain.

See Answer

Q: Cutler Petroleum, Inc., is trying to evaluate a generation project

Cutler Petroleum, Inc., is trying to evaluate a generation project with the following cash flows: Year …………………….Cash Flow 0 ………………………….−$ 85,000,000 1 …………..………………….125, 000,000 2 ……………………………..−15,00...

See Answer

Q: You are considering a new product launch. The project will cost

You are considering a new product launch. The project will cost $820,000, have a four-year life, and have no salvage value; depreciation is straight-line to zero. Sales are projected at 450 units per...

See Answer

Q: You buy a zero coupon bond at the beginning of the year

You buy a zero coupon bond at the beginning of the year that has a face value of $1,000, a YTM of 7 percent, and 25 years to maturity. If you hold the bond for the entire year, how much in interest in...

See Answer

Q: You are planning to save for retirement over the next 30 years

You are planning to save for retirement over the next 30 years. To save for retirement, you will invest $900 a month in a stock account in real dollars and $300 a month in a bond account in real dolla...

See Answer

Q: Bucksnort, Inc., has an odd dividend policy. The company

Bucksnort, Inc., has an odd dividend policy. The company has just paid a dividend of $12 per share and has announced that it will increase the dividend by $3 per share for each of the next five years,...

See Answer

Q: Prove that when carrying costs and restocking costs are as described in

Prove that when carrying costs and restocking costs are as described in the chapter, the EOQ must occur at the point where the carrying costs and restocking costs are equal.

See Answer

Q: Consider a project with a required return of R percent that costs

Consider a project with a required return of R percent that costs $ I and will last for N years. The project uses straight-line depreciation to zero over the N -year life; there are neither salvage va...

See Answer

Q: If a U.S. firm raises funds for a foreign

If a U.S. firm raises funds for a foreign subsidiary, what are the disadvantages to borrowing in the United States? How would you overcome them?

See Answer

Q: Suppose the spot exchange rate for the Hungarian forint is HUF 206

Suppose the spot exchange rate for the Hungarian forint is HUF 206. The inflation rate in the United States is 2.8 percent per year and is 3.7 percent in Hungary. What do you predict the exchange rate...

See Answer

Q: During the year, the Senbet Discount Tire Company had gross sales

During the year, the Senbet Discount Tire Company had gross sales of $1.06 million. The firm’s cost of goods sold and selling expenses were $525,000 and $215,000, respectively. Senbet also had notes p...

See Answer

Q: A company has net income of $265,000, a

A company has net income of $265,000, a profit margin of 9.3 percent, and an accounts receivable balance of $145,300. Assuming 80 percent of sales are on credit, what is the company’s days’ sales in r...

See Answer

Q: The Perpetual Life Insurance Co. is trying to sell you an

The Perpetual Life Insurance Co. is trying to sell you an investment policy that will pay you and your heirs $15,000 per year forever. If the required return on this investment is 5.2 percent, how muc...

See Answer

Q: An investment project provides cash inflows of $840 per year for

An investment project provides cash inflows of $840 per year for eight years. What is the project payback period if the initial cost is $3,200? What if the initial cost is $4,800? What if it is $7,300...

See Answer

Q: It is sometimes stated that “the internal rate of return approach

It is sometimes stated that “the internal rate of return approach assumes reinvestment of the intermediate cash flows at the internal rate of return.” Is this claim correct? To answer, suppose you cal...

See Answer

Q: FFDP Corp. has yearly sales of $28 million and costs

FFDP Corp. has yearly sales of $28 million and costs of $12 million. The company’s balance sheet shows debt of $54 million and cash of $18 million. There are 950,000 shares outstanding and the industr...

See Answer

Q: Mario Brothers, a game manufacturer, has a new idea for

Mario Brothers, a game manufacturer, has a new idea for an adventure game. It can market the game either as a traditional board game or as an interactive DVD, but not both. Consider the following cash...

See Answer

Q: Vandalay Industries is considering the purchase of a new machine for the

Vandalay Industries is considering the purchase of a new machine for the production of latex. Machine A costs $2,900,000 and will last for six years. Variable costs are 35 percent of sales, and fixed...

See Answer

Q: McGilla Golf has decided to sell a new line of golf clubs

McGilla Golf has decided to sell a new line of golf clubs. The clubs will sell for $875 per set and have a variable cost of $430 per set. The company has spent $150,000 for a marketing study that dete...

See Answer

Q: Why it that municipal bonds are not is taxed at the federal

Why it that municipal bonds are not is taxed at the federal level, but are taxable across state lines? Why it is that U.S. Treasury bonds are not taxable at the state level?

See Answer

Q: Miller Corporation has a premium bond making semiannual payments. The bond

Miller Corporation has a premium bond making semiannual payments. The bond pays a coupon of 8 percent, has a YTM of 6 percent, and has 13 years to maturity. The Modigliani Company has a discount bond...

See Answer

Q: Lohn Corporation is expected to pay the following dividends over the next

Lohn Corporation is expected to pay the following dividends over the next four years: $10, $7, $6, and $2.75. Afterwards, the company pledges to maintain a constant 5 percent growth rate in dividends...

See Answer

Q: The Harrington Corporation is considering a change in its cash-only

The Harrington Corporation is considering a change in its cash-only policy. The new terms would be net one period. Based on the following information, determine if Harrington should proceed or not. Th...

See Answer

Q: Fly-By-Night Couriers is analyzing the possible acquisition of

Fly-By-Night Couriers is analyzing the possible acquisition of Flash-in-the-Pan Restaurants. Neither firm has debt. The forecasts of Fly-By-Night show that the purchase would increase its annual after...

See Answer

Q: Lakonishok Equipment has an investment opportunity in Europe. The project costs

Lakonishok Equipment has an investment opportunity in Europe. The project costs €19 million and is expected to produce cash flows of €3.6 million in Year 1,€4.1 million in Year 2, and €5.1 million in...

See Answer

Q: Schwert Corp. shows the following information on its 2012 income statement

Schwert Corp. shows the following information on its 2012 income statement: sales = $185,000; costs = $98,000; other expenses = $6,700; depreciation expense = $16,500; interest expense = $9,000; taxes...

See Answer

Q: You receive a credit card application from Shady Banks Savings and Loan

You receive a credit card application from Shady Banks Savings and Loan offering an introductory rate of 2.40 percent per year, compounded monthly for the first six months, increasing thereafter to 18...

See Answer

Q: Which of the following should be treated as an incremental cash flow

Which of the following should be treated as an incremental cash flow when computing the NPV of an investment? a. A reduction in the sales of a company’s other products caused by the investment. b. An...

See Answer

Q: The Le Bleu Company has a ratio of long-term debt

The Le Bleu Company has a ratio of long-term debt to total assets of .35 and a current ratio of 1.25. Current liabilities are $950, sales are $5,780, profit margin is 9.4 percent, and ROE is 18.2 perc...

See Answer

Q: Find the EAR in each of the following cases:

Find the EAR in each of the following cases:

See Answer

Q: Hanmi Group, a consumer electronics conglomerate, is reviewing its annual

Hanmi Group, a consumer electronics conglomerate, is reviewing its annual budget in wireless technology. It is considering investments in three different technologies to develop wireless communication...

See Answer

Q: Locate the Treasury bond in Figure 8.4 maturing in February

Locate the Treasury bond in Figure 8.4 maturing in February 2037. What is its coupon rate? What is its bid price? What was the previous day’s asked price?

See Answer

Q: Consider the following cash flows on two mutually exclusive projects:

Consider the following cash flows on two mutually exclusive projects: The cash flows of project A are expressed in real terms, whereas those of project B are expressed in nominal terms. The appropri...

See Answer

Q: McGilla Golf has decided to sell a new line of golf

McGilla Golf has decided to sell a new line of golf clubs. The clubs will sell for $875 per set and have a variable cost of $430 per set. The company has spent $150,000 for a marketing study that det...

See Answer

Q: The 100-year bonds we discussed in the chapter have something

The 100-year bonds we discussed in the chapter have something in common with junk bonds. Critics charge that, in both cases, the issuers are really selling equity in disguise. What are the issues here...

See Answer

Q: Laurel, Inc., and Hardy Corp. both have 7 percent

Laurel, Inc., and Hardy Corp. both have 7 percent coupon bonds outstanding, with semiannual interest payments, and both are priced at par value. The Laurel, Inc., bond has 2 years to maturity, whereas...

See Answer

Q: Hughes Co. is growing quickly. Dividends are expected to grow

Hughes Co. is growing quickly. Dividends are expected to grow at a rate of 20 percent for the next three years, with the growth rate falling off to a constant 5 percent thereafter. If the required ret...

See Answer

Q: Happy Times currently has an all-cash credit policy. It

Happy Times currently has an all-cash credit policy. It is considering making a change in the credit policy by going to terms of net 30 days. Based on the following information, what do you recommend?...

See Answer

Q: Sparkling Water, Inc., expects to sell 2.8 million

Sparkling Water, Inc., expects to sell 2.8 million bottles of drinking water each year in perpetuity. This year each bottle will sell for $1.25 in real terms and will cost $.90 in real terms. Sales in...

See Answer

Q: A check-cashing store is in the business of making personal

A check-cashing store is in the business of making personal loans to walk-up customers. The store makes only one-week loans at 7 percent interest per week. a. What APR must the store report to its cus...

See Answer

Q: A 5-year annuity of ten $5,300 semiannual

A 5-year annuity of ten $5,300 semiannual payments will begin 9 years from now, with the first payment coming 9.5 years from now. If the discount rate is 12 percent compounded monthly, what is the val...

See Answer

Q: Suppose you are going to receive $20,000 per year

Suppose you are going to receive $20,000 per year for five years. The appropriate interest rate is 7 percent. a. What is the present value of the payments if they are in the form of an ordinary annuit...

See Answer

Q: You want to buy a new sports car from Muscle Motors for

You want to buy a new sports car from Muscle Motors for $73,000. The contract is in the form of a 60-month annuity due at a 6.45 percent APR. What will your monthly payment be?

See Answer

Q: Locate the Treasury bond in Figure 8.4 maturing in November

Locate the Treasury bond in Figure 8.4 maturing in November 2039. Is this a premium or a discount bond? What is its current yield? What is its yield to maturity? What is the bid-ask spread? Figure 8.4...

See Answer

Q: Find the APR, or stated rate, in each of the

Find the APR, or stated rate, in each of the following cases:

See Answer

Q: You want to lease a set of golf clubs from Pings Ltd

You want to lease a set of golf clubs from Pings Ltd. The lease contract is in the form of 24 equal monthly payments at a 10.4 percent stated annual interest rate, compounded monthly. Because the club...

See Answer

Q: You are saving for the college education of your two children.

You are saving for the college education of your two children. They are two years apart in age; one will begin college 15 years from today and the other will begin 17 years from today. You estimate yo...

See Answer

Q: Tom Adams has received a job offer from a large investment bank

Tom Adams has received a job offer from a large investment bank as a clerk to an associate banker. His base salary will be $55,000. He will receive his first annual salary payment one year from the da...

See Answer

Q: Audrey Sanborn has just arranged to purchase a $550,000

Audrey Sanborn has just arranged to purchase a $550,000 vacation home in the Bahamas with a 20 percent down payment. The mortgage has a 6.1 percent stated annual interest rate, compounded monthly, and...

See Answer

Q: You have recently won the super jackpot in the Washington State Lottery

You have recently won the super jackpot in the Washington State Lottery. On reading the fine print, you discover that you have the following two options: a. You will receive 31 annual payments of $250...

See Answer

Q: What is the equation for the present value of a growing perpetuity

What is the equation for the present value of a growing perpetuity with a payment of C one period from today if the payments grow by C each period?

See Answer

Q: You have 30 years left until retirement and want to retire with

You have 30 years left until retirement and want to retire with $2 million. Your salary is paid annually, and you will receive $70,000 at the end of the current year. Your salary will increase at 3 pe...

See Answer

Q: Down Under Boomerang, Inc., is considering a new three-

Down Under Boomerang, Inc., is considering a new three-year expansion project that requires an initial fixed asset investment of $1.4 million. The fixed asset will be depreciated straight-line to zero...

See Answer

Q: On September 1, 2009, Susan Chao bought a motorcycle for

On September 1, 2009, Susan Chao bought a motorcycle for $30,000. She paid $1,000 down and financed the balance with a five-year loan at a stated annual interest rate of 7.2 percent, compounded monthl...

See Answer

Q: Bilbo Baggins wants to save money to meet three objectives. First

Bilbo Baggins wants to save money to meet three objectives. First, he would like to be able to retire 30 years from now with a retirement income of $23,000 per month for 20 years, with the first payme...

See Answer

Q: After deciding to buy a new car, you can either lease

After deciding to buy a new car, you can either lease the car or purchase it with a three-year loan. The car you wish to buy costs $31,000. The dealer has a special leasing arrangement where you pay $...

See Answer

Q: Consider the following cash flows of two mutually exclusive projects for AZ

Consider the following cash flows of two mutually exclusive projects for AZ-Motorcars. Assume the discount rate for AZ-Motorcars is 10 percent. a. Based on the payback period, which project should b...

See Answer

Q: An All-Pro defensive lineman is in contract negotiations. The

An All-Pro defensive lineman is in contract negotiations. The team has offered the following salary structure: Time………….………….……... Salary 0………..…….….….…. $8, 500,000 1…………….….………….3, 900,000 2………………....

See Answer

Q: Metallica Bearings, Inc., is a young start-up company

Metallica Bearings, Inc., is a young start-up company. No dividends will be paid on the stock over the next nine years, because the firm needs to plow back its earnings to fuel growth. The company wil...

See Answer

Q: This question illustrates what is known as discount interest. Imagine you

This question illustrates what is known as discount interest. Imagine you are discussing a loan with a somewhat unscrupulous lender. You want to borrow $20,000 for one year. The interest rate is 15 pe...

See Answer

Q: You are serving on a jury. A plaintiff is suing the

You are serving on a jury. A plaintiff is suing the city for injuries sustained after a freak street sweeper accident. In the trial, doctors testified that it will be five years before the plaintiff i...

See Answer

Q: M.V.P. Games, Inc., has hired

M.V.P. Games, Inc., has hired you to perform a feasibility study of a new video game that requires a $7 million initial investment. M.V.P. expects a total annual operating cash flow of $1.3 million fo...

See Answer

Q: Another utilization of cash flow analysis is setting the bid price on

Another utilization of cash flow analysis is setting the bid price on a project. To calculate the bid price, we set the project NPV equal to zero and find the required price. Thus the bid price repres...

See Answer

Q: You are looking at a one-year loan of $10

You are looking at a one-year loan of $10,000. The interest rate is quoted as 8 percent plus three points. A point on a loan is simply 1 percent (one percentage point) of the loan amount. Quotes simil...

See Answer

Q: Two banks in the area offer 30-year, $200

Two banks in the area offer 30-year, $200,000 mortgages at 5.3 percent and charge a $2,400 loan application fee. However, the application fee charged by Insecurity Bank and Trust is refundable if the...

See Answer

Q: This problem illustrates a deceptive way of quoting interest rates called add

This problem illustrates a deceptive way of quoting interest rates called add-on interest. Imagine that you see an advertisement for Crazy Judy’s Stereo City that reads something like this: “$1,000 In...

See Answer

Q: Your Christmas ski vacation was great, but it unfortunately ran a

Your Christmas ski vacation was great, but it unfortunately ran a bit over budget. All is not lost: You just received an offer in the mail to transfer your $10,000 balance from your current credit car...

See Answer

Q: In each of the following cases, find the unknown variable.

In each of the following cases, find the unknown variable. Ignore taxes.

See Answer

Q: A useful rule of thumb for the time it takes an investment

A useful rule of thumb for the time it takes an investment to double with discrete compounding is the “Rule of 72.” To use the Rule of 72, you simply divide 72 by the interest rate to determine the nu...

See Answer

Q: A corollary to the Rule of 72 is the Rule of 69

A corollary to the Rule of 72 is the Rule of 69.3. The Rule of 69.3 is exactly correct except for rounding when interest rates are compounded continuously. Prove the Rule of 69.3 for continuously comp...

See Answer

Q: Watters Umbrella Corp. issued 15-year bonds 2 years ago

Watters Umbrella Corp. issued 15-year bonds 2 years ago at a coupon rate of 6.4 percent. The bonds make semiannual payments. If these bonds currently sell for 105 percent of par value, what is the YTM...

See Answer

Q: For the company in the previous problem, what is the dividend

For the company in the previous problem, what is the dividend yield? What is the expected capital gains yield?

See Answer

Q: Purple Feet Wine, Inc., receives an average of $16

Purple Feet Wine, Inc., receives an average of $16,000 in checks per day. The delay in clearing is typically three days. The current interest rate is .018 percent per day. a. What is the company’s flo...

See Answer

Q: The Trektronics store begins each week with 750 phasers in stock.

The Trektronics store begins each week with 750 phasers in stock. This stock is depleted each week and reordered. If the carrying cost per phaser is $65 per year and the fixed order cost is $395, what...

See Answer

Q: When Marilyn Monroe died, ex-husband Joe DiMaggio vowed to

When Marilyn Monroe died, ex-husband Joe DiMaggio vowed to place fresh flowers on her grave every Sunday as long as he lived. The week after she died in 1962, a bunch of fresh flowers that the former...

See Answer

Q: An insurance company is offering a new policy to its customers.

An insurance company is offering a new policy to its customers. Typically the policy is bought by a parent or grandparent for a child at the child’s birth. The details of the policy are as follows: Th...

See Answer

Q: Kyoto Joe, Inc., sells earnings forecasts for Japanese securities.

Kyoto Joe, Inc., sells earnings forecasts for Japanese securities. Its credit terms are 2/15, net 30. Based on experience, 65 percent of all customers will take the discount. a. What is the average co...

See Answer

Q: Explain why diversification per se is probably not a good reason for

Explain why diversification per se is probably not a good reason for merger.

See Answer

Q: Assume that the following balance sheets are stated at book value.

Assume that the following balance sheets are stated at book value. The fair market value of James’ fixed assets is equal to the book value. Jurion pays $15,000 for James and raises t...

See Answer

Q: Paul Adams owns a health club in downtown Los Angeles. He

Paul Adams owns a health club in downtown Los Angeles. He charges his customers an annual fee of $500 and has an existing customer base of 600. Paul plans to raise the annual fee by 6 percent every ye...

See Answer

Q: Fair-to-Midland Manufacturing, Inc., (FMM)

Fair-to-Midland Manufacturing, Inc., (FMM) has applied for a loan at True Credit Bank. Jon Fulkerson, the credit analyst at the bank, has gathered the following information from the company’s financia...

See Answer

Q: If a project with conventional cash flows has a payback period less

If a project with conventional cash flows has a payback period less than the project’s life, can you definitively state the algebraic sign of the NPV? Why or why not? If you know that the discounted p...

See Answer

Q: The exchange rate for the Australian dollar is currently A$1

The exchange rate for the Australian dollar is currently A$1.40. This exchange rate is expected to rise by 10 percent over the next year. a. Is the Australian dollar expected to get stronger or weaker...

See Answer

Q: Use the information in Figure 31.1 to answer the following

Use the information in Figure 31.1 to answer the following questions: a. What is the six-month forward rate for the Japanese yen in yen per U.S. dollar? Is the yen selling at a premium or a discount?...

See Answer

Q: Klingon Cruisers, Inc., purchased new cloaking machinery three years ago

Klingon Cruisers, Inc., purchased new cloaking machinery three years ago for $9.5 million. The machinery can be sold to the Romulans today for $6.5 million. Klingon’s current balance sheet shows net f...

See Answer

Q: How do financial cash flows and the accounting statement of cash flows

How do financial cash flows and the accounting statement of cash flows differ? Which is more useful for analyzing a company?

See Answer

Q: You have just won the lottery. You will receive $2

You have just won the lottery. You will receive $2,500,000 today, and then receive 40 payments of $1,250,000. These payments will start one year from now and will be paid every six months. A represent...

See Answer

Q: The Locker Co. had $273,000 in taxable income

The Locker Co. had $273,000 in taxable income. Using the rates from Table 2.3 in the chapter, calculate the company’s income taxes. What is the average tax rate? What is the marginal...

See Answer

Q: McGilla Golf would like to know the sensitivity of NPV to changes

McGilla Golf would like to know the sensitivity of NPV to changes in the price of the new clubs and the quantity of new clubs sold. What is the sensitivity of the NPV to each of these variables?

See Answer

Q: a. What is the relationship between the price of a bond

a. What is the relationship between the price of a bond and its YTM? b. Explain why some bonds sell at a premium over par value while other bonds sell at a discount. What do you know about the relati...

See Answer

Q: The Faulk Corp. has a 6 percent coupon bond outstanding.

The Faulk Corp. has a 6 percent coupon bond outstanding. The Gonas Company has a 14 percent bond outstanding. Both bonds have 12 years to maturity, make semiannual payments, and have a YTM of 10 perce...

See Answer

Q: Janicek Corp. is experiencing rapid growth. Dividends are expected to

Janicek Corp. is experiencing rapid growth. Dividends are expected to grow at 30 percent per year during the next three years, 18 percent over the following year, and then 8 percent per year indefinit...

See Answer

Q: We are evaluating a project that costs $644,000,

We are evaluating a project that costs $644,000, has an eight-year life, and has no salvage value. Assume that depreciation is straight-line to zero over the life of the project. Sales are projected a...

See Answer

Q: The Silver Spokes Bicycle Shop has decided to offer credit to its

The Silver Spokes Bicycle Shop has decided to offer credit to its customers during the spring selling season. Sales are expected to be 600 bicycles. The average cost to the shop of a bicycle is $525....

See Answer

Q: Bird’s Eye Tree houses, Inc., a Kentucky company, has

Bird’s Eye Tree houses, Inc., a Kentucky company, has determined that a majority of its customers are located in the Pennsylvania area. It therefore is considering using a lockbox sy...

See Answer

Q: Harrods PLC has a market value of £400 million and 30

Harrods PLC has a market value of £400 million and 30 million shares outstanding. Selfridge Department Store has a market value of £160 million and 18 million shares outstanding. Harrods is contemplat...

See Answer

Q: Bentley Corp. and Rolls Manufacturing are considering a merger. The

Bentley Corp. and Rolls Manufacturing are considering a merger. The possible states of the economy and each company’s value in that state are shown here: Bentley currently has a bon...

See Answer

Q: A financial planning service offers a college savings program. The plan

A financial planning service offers a college savings program. The plan calls for you to make six annual payments of $11,000 each, with the first payment occurring today, your child’s 12th birthday. B...

See Answer

Q: Atreides International has operations in Arrakis. The balance sheet for this

Atreides International has operations in Arrakis. The balance sheet for this division in Arrakeen solaris shows assets of 34,000 solaris, debt in the amount of 12,000 solaris, and equity of 22,000 sol...

See Answer

Q: Huang, Inc., is obligated to pay its creditors $10

Huang, Inc., is obligated to pay its creditors $10,900 very soon. a. What is the market value of the shareholders’ equity if assets have a market value of $12,400? b. What if assets equal $9,600?

See Answer

Q: Eberhart Manufacturing has projected sales of $145 million next year.

Eberhart Manufacturing has projected sales of $145 million next year. Costs are expected to be $81 million and net investment is expected to be $15 million. Each of these values is expected to grow at...

See Answer

Q: The DuPont identity presented in the chapter is commonly referred to as

The DuPont identity presented in the chapter is commonly referred to as the three-factor DuPont identity. Another common way that the DuPont identity is expressed is the five-factor model, which is:...

See Answer

Q: First National Bank charges 11.2 percent compounded monthly on its

First National Bank charges 11.2 percent compounded monthly on its business loans. First United Bank charges 11.4 percent compounded semiannually. As a potential borrower, to which bank would you go f...

See Answer

Q: The treasurer of Amaro Canned Fruits, Inc., has projected the

The treasurer of Amaro Canned Fruits, Inc., has projected the cash flows of projects A, B, and C as follows: Suppose the relevant discount rate is 12 percent a year. a. Compute the profitability ind...

See Answer

Q: Etonic Inc. is considering an investment of $365,000

Etonic Inc. is considering an investment of $365,000 in an asset with an economic life of five years. The firm estimates that the nominal annual cash revenues and expenses at the end of the first year...

See Answer

Q: Hacker Software has 6.2 percent coupon bonds on the market

Hacker Software has 6.2 percent coupon bonds on the market with 9 years to maturity. The bonds make semiannual payments and currently sell for 105 percent of par. What is the current yield on the bond...

See Answer

Q: All else being the same, which has more interest rate risk

All else being the same, which has more interest rate risk, a long term bond or a short-term bond? What about a low coupon bond compared to a high coupon bond? What about a long-term, high coupon bond...

See Answer

Q: The Best Manufacturing Company is considering a new investment. Financial projections

The Best Manufacturing Company is considering a new investment. Financial projections for the investment are tabulated here. The corporate tax rate is 34 percent. Assume all sales revenue is received...

See Answer

Q: Cow Chips, Inc., a large fertilizer distributor based in California

Cow Chips, Inc., a large fertilizer distributor based in California, is planning to use a lockbox system to speed up collections from its customers located on the East Coast. A Philadelphia-area bank...

See Answer

Q: Microhard has issued a bond with the following characteristics: Par

Microhard has issued a bond with the following characteristics: Par: $1,000 Time to maturity: 15 years Coupon rate: 7 percent Semiannual payments Calculate the price of this bond if the YTM is: a. 7 p...

See Answer

Q: Redan Manufacturing uses 1,700 switch assemblies per week and then

Redan Manufacturing uses 1,700 switch assemblies per week and then reorders another 1,700. If the relevant carrying cost per switch assembly is $7, and the fixed order cost is $725, is Redan’s invento...

See Answer

Q: Hacker Software has 6.2 percent coupon bonds on the market

Hacker Software has 6.2 percent coupon bonds on the market with 9 years to maturity. The bonds make semiannual payments and currently sell for 105 percent of par. What is the current yield on the bond...

See Answer

Q: Antiques R Us is a mature manufacturing firm. The company just

Antiques R Us is a mature manufacturing firm. The company just paid a dividend of $9, but management expects to reduce the payout by 4 percent per year, indefinitely. If you require an 11 percent retu...

See Answer

Q: Barrett Pharmaceuticals is considering a drug project that costs $2.

Barrett Pharmaceuticals is considering a drug project that costs $2.5 million today and is expected to generate end-of-year annual cash flows of $227,000, forever. At what discount rate would Barrett...

See Answer

Q: In Problem 14, what is the break-even quantity for

In Problem 14, what is the break-even quantity for the new credit policy? Problem 14

See Answer

Q: Plant, Inc., is considering making an offer to purchase Palmer

Plant, Inc., is considering making an offer to purchase Palmer Corp. Plant’s vice president of finance has collected the following information: Plant also knows that securities anal...

See Answer

Q: In the previous problem, assume the equity increases by 1,

In the previous problem, assume the equity increases by 1,750 solaris due to retained earnings. If the exchange rate at the end of the year is 1.24 solaris per dollar, what does the balance sheet look...

See Answer

Q: (Refer to Table 2.3.) Corporation Growth has $

Corporation Growth has $86,000 in taxable income, and Corporation Income has $8,600,000 in taxable income. a. What is the tax bill for each firm? (Refer to Table 2.3.) b. Suppose both firms h...

See Answer

Q: Consider the following cash flows of two mutually exclusive projects for Tokyo

Consider the following cash flows of two mutually exclusive projects for Tokyo Rubber Company. Assume the discount rate for Tokyo Rubber Company is 10 percent. a. Based on the payback period, which...

See Answer

Q: Your financial planner offers you two different investment plans. Plan X

Your financial planner offers you two different investment plans. Plan X is a $15,000 annual perpetuity. Plan Y is a 10-year, $26,000 annual annuity. Both plans will make their first payment one year...

See Answer

Q: You are evaluating a proposed expansion of an existing subsidiary located in

You are evaluating a proposed expansion of an existing subsidiary located in Switzerland. The cost of the expansion would be SF 25 million. The cash flows from the project would be SF 6.9 million per...

See Answer

Q: Phillips Industries runs a small manufacturing operation. For this fiscal year

Phillips Industries runs a small manufacturing operation. For this fiscal year, it expects real net cash flows of $190,000. Phillips is an ongoing operation, but it expects competitive pressures to er...

See Answer

Q: Pembroke Co. wants to issue new 20-year bonds for

Pembroke Co. wants to issue new 20-year bonds for some much-needed expansion projects. The company currently has 7 percent coupon bonds on the market that sell for $1,063, make semiannual payments, an...

See Answer

Q: Pembroke Co. wants to issue new 20-year bonds for

Pembroke Co. wants to issue new 20-year bonds for some much-needed expansion projects. The company currently has 7 percent coupon bonds on the market that sell for $1,063, make semiannual payments, an...

See Answer

Q: Hagar Industrial Systems Company (HISC) is trying to decide between

Hagar Industrial Systems Company (HISC) is trying to decide between two different conveyor belt systems. System A costs $290,000, has a four-year life, and requires $85,000 in pretax annual operating...

See Answer

Q: Suppose we are thinking about replacing an old computer with a new

Suppose we are thinking about replacing an old computer with a new one. The old one cost us $450,000; the new one will cost $580,000. The new machine will be depreciated straight-line to zero over its...

See Answer

Q: Define each of the following investment rules and discuss any potential shortcomings

Define each of the following investment rules and discuss any potential shortcomings of each. In your definition, state the criterion for accepting or rejecting independent projects under each rule. a...

See Answer

Q: Consider four different stocks, all of which have a required return

Consider four different stocks, all of which have a required return of 17 percent and a most recent dividend of $3.50 per share. Stocks W, X, and Y are expected to maintain constant growth rates in di...

See Answer

Q: Southern California Publishing Company is trying to decide whether to revise its

Southern California Publishing Company is trying to decide whether to revise its popular textbook, Financial Psychoanalysis Made Simple. The company has estimated that the revision will cost $75,000....

See Answer

Q: Benson Enterprises is evaluating alternative uses for a three-story manufacturing

Benson Enterprises is evaluating alternative uses for a three-story manufacturing and warehousing building that it has purchased for $1,450,000. The company can continue to rent the building to the pr...

See Answer

Q: The Cornchopper Company is considering the purchase of a new harvester.

The Cornchopper Company is considering the purchase of a new harvester. Cornchopper has hired you to determine the break-even purchase price in terms of present value of the harvester. This break-even...

See Answer

Q: Most corporations pay quarterly dividends on their common stock rather than annual

Most corporations pay quarterly dividends on their common stock rather than annual dividends. Barring any unusual circumstances during the year, the board raises, lowers, or maintains the current divi...

See Answer

Q: Given the following information for O’Hara Marine Co., calculate the depreciation

Given the following information for O’Hara Marine Co., calculate the depreciation expense: sales = $41,000; costs = $26,400; addition to retained earnings = $4,900; dividends paid = $1,570; interest e...

See Answer

Q: The following Treasury bond quote appeared in The Wall Street Journal on

The following Treasury bond quote appeared in The Wall Street Journal on May 11, 2004: Why would anyone buy this Treasury bond with a negative yield to maturity? How is this possible?

See Answer

Q: Your job pays you only once a year for all the work

Your job pays you only once a year for all the work you did over the previous 12 months. Today, December 31, you just received your salary of $65,000, and you plan to spend all of it. However, you wan...

See Answer

Q: The Biological Insect Control Corporation (BICC) has hired you as

The Biological Insect Control Corporation (BICC) has hired you as a consultant to evaluate the NPV of its proposed toad ranch. BICC plans to breed toads and sell them as ecologically desirable insect...

See Answer

Q: Lewin Skis, Inc., today expects to earn $8.

Lewin Skis, Inc., today expects to earn $8.50 per share for each of the future operating periods (beginning at Time 1), today if the firm makes no new investments and returns the earnings as dividends...

See Answer

Q: What is the relationship between the value of an annuity and the

What is the relationship between the value of an annuity and the level of interest rates? Suppose you just bought a 15-year annuity of $6,800 per year at the current interest rate of 10 percent per ye...

See Answer

Q: Consider a four-year project with the following information: Initial

Consider a four-year project with the following information: Initial fixed asset investment 5 $480,000; straight-line depreciation to zero over the four-year life; zero salvage value; price 5 $37; var...

See Answer

Q: After extensive medical and marketing research, Pill, Inc., believes

After extensive medical and marketing research, Pill, Inc., believes it can penetrate the pain reliever market. It is considering two alternative products. The first is a medication for headache pain....

See Answer

Q: J. Smythe, Inc., manufactures fine furniture. The company

J. Smythe, Inc., manufactures fine furniture. The company is deciding whether to introduce a new mahogany dining room table set. The set will sell for $6,100, including a set of eight chairs. The comp...

See Answer

Q: Burklin, Inc., has earnings of $18 million and is

Burklin, Inc., has earnings of $18 million and is projected to grow at a constant rate of 5 percent forever because of the benefits gained from the learning curve. Currently, all earnings are paid out...

See Answer

Q: You need a 30-year, fixed-rate mortgage to

You need a 30-year, fixed-rate mortgage to buy a new home for $250,000. Your mortgage bank will lend you the money at a 5.3 percent APR for this 360-month loan. However, you can only afford monthly pa...

See Answer

Q: The present value of the following cash flow stream is $7

The present value of the following cash flow stream is $7,300 when discounted at 8 percent annually. What is the value of the missing cash flow? Year ……………….Cash Flow 1 …………………………..……. $1,500 2 ……………...

See Answer

Q: As discussed in the text, an annuity due is identical to

As discussed in the text, an annuity due is identical to an ordinary annuity except that the periodic payments occur at the beginning of each period and not at the end of the period. Show that the rel...

See Answer

Q: Compute the future value of $1,000 compounded annually for

Compute the future value of $1,000 compounded annually for a. 10 years at 5 percent. b. 10 years at 10 percent. c. 20 years at 5 percent. d. Why is the interest earned in part (c) not twice the amount...

See Answer

Q: You just won the TVM Lottery. You will receive $1

You just won the TVM Lottery. You will receive $1 million today plus another 10 annual payments that increase by $275,000 per year. Thus, in one year you receive $1.275 million. In two years, you get...

See Answer

Q: Consider a firm with a contract to sell an asset for $

Consider a firm with a contract to sell an asset for $115,000 three years from now. The asset costs $76,000 to produce today. Given a relevant discount rate on this asset of 13 percent per year, will...

See Answer

Q: Titan Inc.’s net income for the most recent year was

Titan Inc.’s net income for the most recent year was $8,320. The tax rate was 34 percent. The firm paid $1,940 in total interest expense and deducted $2,730 in depreciation expense. What was Titan’s c...

See Answer

Q: Sony International has an investment opportunity to produce a new HDTV.

Sony International has an investment opportunity to produce a new HDTV. The required investment on January 1 of this year is $165 million. The firm will depreciate the investment to zero using the str...

See Answer

Q: What is the present value of $5,000 per year

What is the present value of $5,000 per year, at a discount rate of 6 percent, if the first payment is received 6 years from now and the last payment is received 25 years from now?

See Answer

Q: First City Bank pays 8 percent simple interest on its savings account

First City Bank pays 8 percent simple interest on its savings account balances, whereas Second City Bank pays 8 percent interest compounded annually. If you made a $5,000 deposit in each bank, how muc...

See Answer

Q: In the chapter, we discussed one calculation of the sustainable growth

In the chapter, we discussed one calculation of the sustainable growth rate as: In practice, probably the most commonly used calculation of the sustainable growth rate is ROE 3 b. This equation is i...

See Answer

Q: Your company currently produces and sells steel shaft golf clubs. The

Your company currently produces and sells steel shaft golf clubs. The board of directors wants you to consider the introduction of a new line of titanium bubble woods with graphite shafts. Which of th...

See Answer

Q: You have your choice of two investment accounts. Investment A is

You have your choice of two investment accounts. Investment A is a 15-year annuity that features end-of-month $1,500 payments and has an interest rate of 8.7 percent compounded monthly. Investment B i...

See Answer

Q: Given an interest rate of 6.1 percent per year,

Given an interest rate of 6.1 percent per year, what is the value at Date t = 7 of a perpetual stream of $2,500 annual payments that begins at Date t = 15?

See Answer

Q: Storico Co. just paid a dividend of $3.85

Storico Co. just paid a dividend of $3.85 per share. The company will increase its dividend by 20 percent next year and will then reduce its dividend growth rate by 5 percentage points per year until...

See Answer

Q: What is the value of an investment that pays $30,

What is the value of an investment that pays $30,000 every other year forever, if the first payment occurs one year from today and the discount rate is 13 percent compounded daily? What is the value t...

See Answer

Q: An investment project has annual cash inflows of $5,000

An investment project has annual cash inflows of $5,000, $5,500, $6,000, and $7,000, and a discount rate of 14 percent. What is the discounted payback period for these cash flows if the initial cost i...

See Answer

Q: You’re prepared to make monthly payments of $350, beginning at

You’re prepared to make monthly payments of $350, beginning at the end of this month, into an account that pays 10 percent interest compounded monthly. How many payments will you have made when your a...

See Answer

Q: Universal Laser, Inc., just paid a dividend of $3

Universal Laser, Inc., just paid a dividend of $3.10 on its stock. The growth rate in dividends is expected to be a constant 6 percent per year, indefinitely. Investors require a 15 percent return on...

See Answer

Q: This one’s a little harder. Suppose the current share price for

This one’s a little harder. Suppose the current share price for the firm in the previous problem is $78.43 and all the dividend information remains the same. What required return must investors be dem...

See Answer

Q: California Real Estate, Inc., expects to earn $71 million

California Real Estate, Inc., expects to earn $71 million per year in perpetuity if it does not undertake any new projects. The firm has an opportunity to invest $16 million today and $5 million in on...

See Answer

Q: You own a lot in Key West, Florida, that is

You own a lot in Key West, Florida, that is currently unused. Similar lots have recently sold for $1.1 million. Over the past five years, the price of land in the area has increased 12 percent per yea...

See Answer

Q: Why don’t all firms simply increase their payables periods to shorten their

Why don’t all firms simply increase their payables periods to shorten their cash cycles? Last month, BlueSky Airline announced that it would stretch out its bill payments to 45 days from 30 days. The...

See Answer

Q: Travis, Inc., has sales of $387,000,

Travis, Inc., has sales of $387,000, costs of $175,000, depreciation expense of $40,000, interest expense of $21,000, and a tax rate of 35 percent. What is the net income for the firm? Suppose the com...

See Answer

Q: One of the less flattering interpretations of the acronym MIRR is “

One of the less flattering interpretations of the acronym MIRR is “meaningless internal rate of return.” Why do you think this term is applied to MIRR?

See Answer

Q: Your company has been approached to bid on a contract to sell

Your company has been approached to bid on a contract to sell 15,000 voice recognition (VR) computer keyboards a year for four years. Due to technological improvements, beyond that time they will be o...

See Answer

Q: Krell Industries has a share price of $22.00 today

Krell Industries has a share price of $22.00 today. If Krell is expected to pay a dividend of $0.88 this year and its stock price is expected to grow to $23.54 at the end of the year, what is Krell’s...

See Answer

Q: Halliford Corporation expects to have earnings this coming year of $3

Halliford Corporation expects to have earnings this coming year of $3 per share. Halliford plans to retain all of its earnings for the next two years. Then, for the subsequent two years, the firm will...

See Answer

Q: You have just received a windfall from an investment you made in

You have just received a windfall from an investment you made in a friend’s business. She will be paying you $10,000 at the end of this year, $20,000 at the end of the following year, and $30,000 at t...

See Answer

Q: Peripatetic Enterprises, a U.S. import-export trading

Peripatetic Enterprises, a U.S. import-export trading firm, is considering its international tax situation. Tax law in the Unites States requires U.S. corporations to pay taxes on their foreign earnin...

See Answer

Q: Suppose the interest on Russian government bonds is 7.5%,

Suppose the interest on Russian government bonds is 7.5%, and the current exchange rate is 28 rubles per dollar. If the forward exchange rate is 28.5 rubles per dollar, and the current U.S. risk-free...

See Answer

Q: What are some of the differences between the NYSE and NASDAQ?

What are some of the differences between the NYSE and NASDAQ?

See Answer

Q: You are a shareholder in a C corporation. The corporation earns

You are a shareholder in a C corporation. The corporation earns $2.00 per share before taxes. Once it has paid taxes it will distribute the rest of its earnings to you as a dividend. Assume the corpor...

See Answer

Q: What is the financial cycle?

What is the financial cycle?

See Answer

Q: Dorpac Corporation has a dividend yield of 1.5%. Its

Dorpac Corporation has a dividend yield of 1.5%. Its equity cost of capital is 8%, and its dividends are expected to grow at a constant rate. a. What is the expected growth rate of Dorpac’s dividends?...

See Answer

Q: How do financial institutions help with risk-bearing?

How do financial institutions help with risk-bearing?

See Answer

Q: Think back to the last time you ate at an expensive restaurant

Think back to the last time you ate at an expensive restaurant where you paid the bill. Now think about the last time you ate at a similar restaurant, but your parents paid the bill. Did you order mor...

See Answer

Q: What are some of the similarities and differences among mutual funds,

What are some of the similarities and differences among mutual funds, pension funds, and hedge funds?

See Answer

Q: Quisco Systems has 6.5 billion shares outstanding and a share

Quisco Systems has 6.5 billion shares outstanding and a share price of $18.00. Quisco is considering developing a new networking product in-house at a cost of $500 million. Alternatively, Quisco can a...

See Answer

Q: Suppose you receive $100 at the end of each year for

Suppose you receive $100 at the end of each year for the next three years. a. If the interest rate is 8%, what is the present value of these cash flows? b. What is the future value in three years of t...

See Answer

Q: In January 2009, American Airlines (AMR) had a market

In January 2009, American Airlines (AMR) had a market capitalization of $1.7 billion, debt of $11.1 billion, and cash of $4.6 billion. American Airlines had revenues of $23.8 billion. British Airways...

See Answer

Q: Find online the annual 10-K report for Peet’s Coffee and

Find online the annual 10-K report for Peet’s Coffee and Tea (PEET) for 2008 (filed in early 2009). a. Compute Peet’s net profit margin, total asset turnover, and equity multiplier. b. Verify the DuPo...

See Answer

Q: Repeat the analysis from parts a and b of the previous problem

Repeat the analysis from parts a and b of the previous problem using Starbucks Corporation (SBUX) instead. Based on the DuPont Identity, what explains the difference between the two firms’ ROEs?

See Answer

Q: Consider a retail firm with a net profit margin of 3.

Consider a retail firm with a net profit margin of 3.5%, a total asset turnover of 1.8, total assets of $44 million, and a book value of equity of $18 million. a. What is the firm’s current ROE? b. If...

See Answer

Q: Find online the 2008 annual 10-K report for Peet’s Coffee

Find online the 2008 annual 10-K report for Peet’s Coffee and Tea (PEET), filed in early 2009. Answer the following questions from its cash flow statement: a. How much cash did Peet’s generate from op...

See Answer

Q: See the cash flow statement below for H. J. Heinz

See the cash flow statement below for H. J. Heinz (HNZ) (all values in thousands of dollars) a. What were Heinz’s cumulative earnings over these four quarters? What were its cumulati...

See Answer

Q: Laurel Enterprises expects earnings next year of $4 per share and

Laurel Enterprises expects earnings next year of $4 per share and has a 40% retention rate, which it plans to keep constant. Its equity cost of capital is 10%, which is also its expected return on new...

See Answer

Q: Suppose your firm receives a $5 million order on the last

Suppose your firm receives a $5 million order on the last day of the year. You fill the order with $2 million worth of inventory. The customer picks up the entire order the same day and pays $1 millio...

See Answer

Q: Nokela Industries purchases a $40 million cyclo-converter. The

Nokela Industries purchases a $40 million cyclo-converter. The cyclo-converter will be depreciated by $10 million per year over four years, starting this year. Suppose Nokela’s tax rate is 40%. a. Wha...

See Answer

Q: The balance sheet information for Clorox Co. (CLX) in

The balance sheet information for Clorox Co. (CLX) in 2004–2005 is shown here (all values in thousands of dollars) a. What change in the book value of Clorox’s e...

See Answer

Q: Zoom Enterprises expects that one year from now it will pay a

Zoom Enterprises expects that one year from now it will pay a total dividend of $5 million and repurchase $5 million worth of shares. It plans to spend $10 million on dividends and repurchases every y...

See Answer

Q: If Local Co., the company in Problem 12, had an

If Local Co., the company in Problem 12, had an increase in selling expenses of $300,000, how would that affect each of its margins? Problem 12 data: Local Co. has sales of $10 million and cost of sal...

See Answer

Q: If Local Co., the company in Problem 12, had interest

If Local Co., the company in Problem 12, had interest expense of $800,000, how would that affect each of its margins? Problem 12 data Local Co. has sales of $10 million and cost of sales of $6 million...

See Answer

Q: Ladders, Inc. has a net profit margin of 5%

Ladders, Inc. has a net profit margin of 5% on sales of $50 million. It has book value of equity of $40 million and total liabilities with a book value of $30 million. What is Ladders’ ROE? ROA?

See Answer

Q: JPJ Corp has sales of $1 million, accounts receivable of

JPJ Corp has sales of $1 million, accounts receivable of $50,000, total assets of $5 million (of which $3 million are fixed assets), inventory of $150,000, and cost of goods sold of $600,000. What is...

See Answer

Q: If JPJ Corp (the company from the previous question) is

If JPJ Corp (the company from the previous question) is able to increase sales by 10% but keep its total and fixed asset growth to only 5%, what will its new asset turnover ratios be?

See Answer

Q: Suppose that in 2010, Global launched an aggressive marketing campaign that

Suppose that in 2010, Global launched an aggressive marketing campaign that boosted sales by 15%. However, their operating margin fell from 5.57% to 4.50%. Suppose that they had no other income, inter...

See Answer

Q: Suppose a firm’s tax rate is 35%. a. What

Suppose a firm’s tax rate is 35%. a. What effect would a $10 million operating expense have on this year’s earnings? What effect would it have on next year’s earnings? b. What effect would a $10 milli...

See Answer

Q: DFB, Inc., expects earnings this year of $5 per

DFB, Inc., expects earnings this year of $5 per share, and it plans to pay a $3 dividend to shareholders. DFB will retain $2 per share of its earnings to reinvest in new projects that have an expected...

See Answer

Q: You are analyzing the leverage of two firms and you note the

You are analyzing the leverage of two firms and you note the following (all values in millions of dollars): a. What is the market debt-to-equity ratio of each firm? b. What is the book debt-to-equity...

See Answer

Q: For 2010, Wal-Mart and Target had the following information

For 2010, Wal-Mart and Target had the following information (all values are in millions of dollars): a. What is each company’s accounts receivable days? b. What is each companyâ...

See Answer

Q: You have a loan outstanding. It requires making three annual payments

You have a loan outstanding. It requires making three annual payments of $1000 each at the end of the next three years. Your bank has offered to allow you to skip making the next two payments in lieu...

See Answer

Q: Consider the following potential events that might have occurred to Global on

Consider the following potential events that might have occurred to Global on December 30, 2010. For each one, indicate which line items in Global’s balance sheet would be affected and by how much. Al...

See Answer

Q: In June 2007, General Electric (GE) had a book

In June 2007, General Electric (GE) had a book value of equity of $117 billion, 10.3 billion shares outstanding, and a market price of $38.00 per share. GE also had cash of $16 billion, and total debt...

See Answer

Q: In July 2007, Apple had cash of $7.12

In July 2007, Apple had cash of $7.12 billion, current assets of $18.75 billion, and current liabilities of $6.99 billion. It also had inventories of $0.25 billion. a. What was Apple’s current ratio?...

See Answer

Q: In April 2010, the following information was true about Abercrombie and

In April 2010, the following information was true about Abercrombie and Fitch (ANF) and The Gap (GPS), both clothing retailers. Values (except price per share) are in millions of dollars. a. What is t...

See Answer

Q: Find online the annual 10-K report for Peet’s Coffee and

Find online the annual 10-K report for Peet’s Coffee and Tea (PEET) for 2008. Answer the following questions from the income statement: a. What were Peet’s revenues for 2008? By what percentage did re...

See Answer

Q: Local Co. has sales of $10 million and cost of

Local Co. has sales of $10 million and cost of sales of $6 million. Its selling, general and administrative expenses are $500,000 and its research and development is $1 million. It has annual deprecia...

See Answer

Q: You have $100 and a bank is offering 5% interest

You have $100 and a bank is offering 5% interest on deposits. If you deposit the money in the bank, how much will you have in one year?

See Answer

Q: A friend asks to borrow $55 from you and in return

A friend asks to borrow $55 from you and in return will pay you $58 in one year. If your bank is offering a 6% interest rate on deposits and loans: a. How much would you have in one year if you deposi...

See Answer

Q: Cooperton Mining just announced it will cut its dividend from $4

Cooperton Mining just announced it will cut its dividend from $4 to $2.50 per share and use the extra funds to expand. Prior to the announcement, Cooperton’s dividends were expected to grow at a 3% ra...

See Answer

Q: Suppose the interest rate is 4%. a. Having $

Suppose the interest rate is 4%. a. Having $200 today is equivalent to having what amount in one year? b. Having $200 in one year is equivalent to having what amount today? c. Which would you prefer,...

See Answer

Q: You want to endow a scholarship that will pay $10,

You want to endow a scholarship that will pay $10,000 per year forever, starting one year from now. If the school’s endowment discount rate is 7%, what amount must you donate to endow the scholarship?...

See Answer

Q: Consider the following alternatives: i. $100 received in

Consider the following alternatives: i. $100 received in one year ii. $200 received in 5 years iii. $300 received in 10 years a. Rank the alternatives from most valuable to least valuable if the inter...

See Answer

Q: Suppose you invest $1000 in an account paying 8% interest

Suppose you invest $1000 in an account paying 8% interest per year. a. What is the balance in the account after 3 years? How much of this balance corresponds to “interest on interest”? b. What is the...

See Answer

Q: Calculate the future value of $2000 in a. 5

Calculate the future value of $2000 in a. 5 years at an interest rate of 5% per year. b. 10 years at an interest rate of 5% per year. c. 5 years at an interest rate of 10% per year. d. Why is the amou...

See Answer

Q: What is the present value of $10,000 received

What is the present value of $10,000 received a. 12 years from today when the interest rate is 4% per year? b. 20 years from today when the interest rate is 8% per year? c. 6 years from today when th...

See Answer

Q: Your brother has offered to give you either $5000 today or

Your brother has offered to give you either $5000 today or $10,000 in 10 years. If the interest rate is 7% per year, which option is preferable?

See Answer

Q: Your cousin is currently 12 years old. She will be going

Your cousin is currently 12 years old. She will be going to college in 6 years. Your aunt and uncle would like to have $100,000 in a savings account to fund her education at that time. If the account...

See Answer

Q: Your mom is thinking of retiring. Her retirement plan will pay

Your mom is thinking of retiring. Her retirement plan will pay her either $250,000 immediately on retirement or $350,000 five years after the date of her retirement. Which alternative should she choos...

See Answer

Q: Your grandfather put some money in an account for you on the

Your grandfather put some money in an account for you on the day you were born. You are now 18 years old and are allowed to withdraw the money for the first time. The account currently has $3996 in it...

See Answer

Q: Honda Motor Company is considering offering a $2000 rebate on its

Honda Motor Company is considering offering a $2000 rebate on its minivan, lowering the vehicle’s price from $30,000 to $28,000. The marketing group estimates that this rebate will increase sales over...

See Answer

Q: Gillette Corporation will pay an annual dividend of $0.65

Gillette Corporation will pay an annual dividend of $0.65 one year from now. Analysts expect this dividend to grow at 12% per year thereafter until the fifth year. After then, growth will level off at...

See Answer

Q: How would your answer to Problem 7 change if you endow it

How would your answer to Problem 7 change if you endow it now, but it makes the first award to a student 10 years from today? Information from Problem 7: You want to endow a scholarship that will pay...

See Answer

Q: Suppose Big Bank offers an interest rate of 5.5%

Suppose Big Bank offers an interest rate of 5.5% on both savings and loans, and Bank Enn offers an interest rate of 6% on both savings and loans. a. What profit opportunity is available? b. Which bank...

See Answer

Q: Some companies cross-list their shares, meaning that their stock

Some companies cross-list their shares, meaning that their stock trades on more than one stock exchange. For example, Research In Motion, the maker of BlackBerry mobile devices, trades on both the Tor...

See Answer

Q: Bubba is a shrimp farmer. In an ironic twist, Bubba

Bubba is a shrimp farmer. In an ironic twist, Bubba is allergic to shellfish, so he cannot eat any shrimp. Each day he has a one-ton supply of shrimp. The market price of shrimp is $10,000 per ton. a....

See Answer

Q: Brett has almond orchards, but he is sick of almonds and

Brett has almond orchards, but he is sick of almonds and prefers to eat walnuts instead. The owner of the walnut orchard next door has offered to swap this year’s crop with him. Assume he produces 100...

See Answer

Q: You figure that the total cost of college will be $100

You figure that the total cost of college will be $100,000 per year 18 years from today. If your discount rate is 8% compounded annually, what is the present value today of 4 years of college costs st...

See Answer

Q: Assume that Social Security promises you $40,000 per year

Assume that Social Security promises you $40,000 per year starting when you retire 45 years from today (the first $40,000 will come 45 years from now). If your discount rate is 7%, compounded annually...

See Answer

Q: When Alex Rodriguez moved to the Texas Rangers in 2001, he

When Alex Rodriguez moved to the Texas Rangers in 2001, he received a lot of attention for his “$252 million” contract (the total of the payments promised was $252...

See Answer

Q: You are trying to decide how much to save for retirement.

You are trying to decide how much to save for retirement. Assume you plan to save $5000 per year with the first investment made 1 year from now. You think you can earn 10% per year on your investments...

See Answer

Q: A rich relative has bequeathed you a growing perpetuity. The first

A rich relative has bequeathed you a growing perpetuity. The first payment will occur in a year and will be $1000. Each year after that, you will receive a payment on the anniversary of the last payme...

See Answer

Q: You are thinking of building a new machine that will save you

You are thinking of building a new machine that will save you $1000 in the first year. The machine will then begin to wear out so that the savings decline at a rate of 2% per year forever. What is the...

See Answer

Q: The British government has a consol bond outstanding paying £100 per

The British government has a consol bond outstanding paying £100 per year forever. Assume the current interest rate is 4% per year. a. What is the value of the bond immediately after a payment is made...

See Answer

Q: Colgate-Palmolive Company has just paid an annual dividend of $

Colgate-Palmolive Company has just paid an annual dividend of $0.96. Analysts are predicting an 11% per year growth rate in earnings over the next five years. After then, Colgate’s earnings are expect...

See Answer

Q: When Alfred Nobel died, he left the majority of his estate

When Alfred Nobel died, he left the majority of his estate to fund five prizes, each to be awarded annually in perpetuity starting one year after he died (the sixth one, in economics, was added later)...

See Answer

Q: You work for a pharmaceutical company that has developed a new drug

You work for a pharmaceutical company that has developed a new drug. The patent on the drug will last 17 years. You expect that the drug’s profits will be $2 million in its first year and that this am...

See Answer

Q: A rich aunt has promised you $5000 one year from today

A rich aunt has promised you $5000 one year from today. In addition, each year after that, she has promised you a payment (on the anniversary of the last payment) that is 3% larger than the last payme...

See Answer

Q: You are thinking about buying a savings bond. The bond costs

You are thinking about buying a savings bond. The bond costs $50 today and will mature in 10 years with a value of $100. What annual interest rate will the bond earn?

See Answer

Q: You have an investment account that started with $1000 ten years

You have an investment account that started with $1000 ten years ago and which now has grown to $5000. a. What annual rate of return have you earned (you have made no additional contributions to the a...

See Answer

Q: You have an investment opportunity that requires an initial investment of $

You have an investment opportunity that requires an initial investment of $5000 today and will pay $6000 in one year. What is the rate of return of this opportunity?

See Answer

Q: You have decided to buy a perpetual bond. The bond makes

You have decided to buy a perpetual bond. The bond makes one payment at the end of every year forever and has an interest rate of 5%. If the bond initially costs $1000, what is the payment every year?...

See Answer

Q: You are thinking of purchasing a house. The house costs $

You are thinking of purchasing a house. The house costs $350,000. You have $50,000 in cash that you can use as a down payment on the house, but you need to borrow the rest of the purchase price. The b...

See Answer

Q: You are thinking about buying a piece of art that costs $

You are thinking about buying a piece of art that costs $50,000. The art dealer is proposing the following deal: He will lend you the money, and you will repay the loan by making the same payment ever...

See Answer

Q: What is the present value of $1000 paid at the end

What is the present value of $1000 paid at the end of each of the next 100 years if the interest rate is 7% per year?

See Answer

Q: You would like to buy the house and take the mortgage described

You would like to buy the house and take the mortgage described in Problem 27. You can afford to pay only $23,500 per year. The bank agrees to allow you to pay this amount each year, yet still borrow...

See Answer

Q: Your grandmother has been putting $1000 into a savings account on

Your grandmother has been putting $1000 into a savings account on every birthday since your first (that is, when you turned one). The account pays an interest rate of 3%. How much money will be in the...

See Answer

Q: Assume that your parents wanted to have $160,000 saved

Assume that your parents wanted to have $160,000 saved for college by your eighteenth birthday and they started saving on your first birthday. They saved the same amount each year on your birthday and...

See Answer

Q: You are saving for retirement. To live comfortably, you decide

You are saving for retirement. To live comfortably, you decide you will need to save $2 million by the time you are 65. Today is your twenty-second birthday, and you decide, starting today and continu...

See Answer

Q: When you purchased your car, you took out a five-

When you purchased your car, you took out a five-year annual-payment loan with an interest rate of 6% per year. The annual payment on the car is $5000. You have just made a payment and have now decide...

See Answer

Q: You have decided to refinance your mortgage. You plan to borrow

You have decided to refinance your mortgage. You plan to borrow whatever is outstanding on your current mortgage. The current monthly payment is $2356 and you have made every payment on time. The orig...

See Answer

Q: You have just sold your house for $1,000,

You have just sold your house for $1,000,000 in cash. Your mortgage was originally a 30-year mortgage with monthly payments and an initial balance of $800,000. The mortgage is currently exactly 18 1/2...

See Answer

Q: AFW Industries has 200 million shares outstanding and expects earnings at the

AFW Industries has 200 million shares outstanding and expects earnings at the end of this year of $700 million. AFW plans to pay out 60% of its earnings in total, paying 40% as a dividend and using 20...

See Answer

Q: You have just purchased a car and taken out a $50

You have just purchased a car and taken out a $50,000 loan. The loan has a five-year term with monthly payments and an APR of 6%. a. How much will you pay in interest, and how much will you pay in pri...

See Answer

Q: You are thinking about leasing a car. The purchase price of

You are thinking about leasing a car. The purchase price of the car is $30,000. The residual value (the amount you could pay to keep the car at the end of the lease) is $15,000 at the end of 36 months...

See Answer

Q: You have some extra cash this month and you are considering putting

You have some extra cash this month and you are considering putting it toward your car loan. Your interest rate is 7%, your loan payments are $600 per month, and you have 36 months left on your loan....

See Answer

Q: You have an outstanding student loan with required payments of $500

You have an outstanding student loan with required payments of $500 per month for the next four years. The interest rate on the loan is 9% APR (monthly). You are considering making an extra payment of...

See Answer

Q: Consider again the setting of Problem 19. Now that you realize

Consider again the setting of Problem 19. Now that you realize your best investment is to prepay your student loan, you decide to prepay as much as you can each month. Looking at your budget, you can...

See Answer

Q: If you decide to take the mortgage in Problem 11, Oppenheimer

If you decide to take the mortgage in Problem 11, Oppenheimer Bank will offer you the following deal: Instead of making the monthly payment you computed in that problem every month, you can make half...

See Answer

Q: You graduate and get a $10,000 check from your

You graduate and get a $10,000 check from your grandparents. You decide to save it toward a down payment on a house. You invest it earning 10% per year and you think you will need to have $20,000 save...

See Answer

Q: Your friend tells you he has a very simple trick for taking

Your friend tells you he has a very simple trick for taking one-third off the time it takes to repay your mortgage: Use your Christmas bonus to make an extra payment on January 1 of each year (that is...

See Answer

Q: The mortgage on your house is five years old. It required

The mortgage on your house is five years old. It required monthly payments of $1402, had an original term of 30 years, and had an interest rate of 10% (APR). In the intervening five years, interest ra...

See Answer

Q: You have credit card debt of $25,000 that has

You have credit card debt of $25,000 that has an APR (monthly compounding) of 15%. Each month you pay a minimum monthly payment only. You are required to pay only the outstanding interest. You have re...

See Answer

Q: Your firm has taken out a $500,000 loan with

Your firm has taken out a $500,000 loan with 9% APR (compounded monthly) for some commercial property. As is common in commercial real estate, the loan is a 5-year loan based on a 15-year amortization...

See Answer

Q: Suppose Cisco Systems pays no dividends but spent $5 billion on

Suppose Cisco Systems pays no dividends but spent $5 billion on share repurchases last year. If Cisco’s equity cost of capital is 12%, and if the amount spent on repurchases is expected to grow by 8%...

See Answer

Q: In 1975, interest rates were 7.85% and the

In 1975, interest rates were 7.85% and the rate of inflation was 12.3% in the United States. What was the real interest rate in 1975? How would the purchasing power of your savings have changed over t...

See Answer

Q: If the rate of inflation is 5%, what nominal interest rate

If the rate of inflation is 5%, what nominal interest rate is necessary for you to earn a 3% real interest rate on your investment?

See Answer

Q: You have been offered a job with an unusual bonus structure.

You have been offered a job with an unusual bonus structure. As long as you stay with the firm, you will get an extra $70,000 every seven years, starting seven years from now. What is the present valu...

See Answer

Q: Suppose Oppenheimer Bank is offering a 30-year mortgage with an

Suppose Oppenheimer Bank is offering a 30-year mortgage with an EAR of 6.80%. If you plan to borrow $150,000, what will your monthly payment be?

See Answer

Q: Assume the inflation rate is 3% APR, compounded annually.

Assume the inflation rate is 3% APR, compounded annually. Would you rather earn a nominal return of 5% APR, compounded semiannually, or a real return of 2% APR, compounded quarterly?

See Answer

Q: You have just taken out a $20,000 car loan

You have just taken out a $20,000 car loan with a 6% APR, compounded monthly. The loan is for five years. When you make your first payment in one month, how much of the payment will go toward the prin...

See Answer

Q: You are buying a house and the mortgage company offers to let

You are buying a house and the mortgage company offers to let you pay a “point” (1% of the total amount of the loan) to reduce your APR from 6.5% to 6.25% on your $400,000, 30-year mortgage with month...

See Answer

Q: Assume the current Treasury yield curve shows that the spot rates for

Assume the current Treasury yield curve shows that the spot rates for 6 months, 1 year, and 1 1/2 years are 1%, 1.1%, and 1.3%, all quoted as semiannually compounded APRs. What is the price of a $1000...

See Answer

Q: You own 20% of the stock of a company that has

You own 20% of the stock of a company that has ten directors on its board. How much representation can you get on the board if the company has cumulative voting? How much representation can you ensure...

See Answer

Q: Suppose a ten-year, $1000 bond with an 8

Suppose a ten-year, $1000 bond with an 8% coupon rate and semiannual coupons is trading for a price of $1034.74. a. What is the bond’s yield to maturity (expressed as an APR with semiannual compoundin...

See Answer

Q: Suppose a five-year, $1000 bond with annual coupons

Suppose a five-year, $1000 bond with annual coupons has a price of $900 and a yield to maturity of 6%. What is the bond’s coupon rate?

See Answer

Q: You have purchased a 10% coupon bond for $1040.

You have purchased a 10% coupon bond for $1040. What will happen to the bond’s price if market interest rates rise?

See Answer

Q: You have just turned 22 years old, have just received your

You have just turned 22 years old, have just received your bachelor’s degree, and have accepted your first job. Now you must decide how much money to put into your retirement plan. The plan works as f...

See Answer

Q: Maynard Steel plans to pay a dividend of $3 this year

Maynard Steel plans to pay a dividend of $3 this year. The company has an expected earnings growth rate of 4% per year and an equity cost of capital of 10%. a. Assuming that Maynard’s dividend payout...

See Answer

Q: Suppose a seven-year, $1000 bond with an 8

Suppose a seven-year, $1000 bond with an 8% coupon rate and semiannual coupons is trading with a yield to maturity of 6.75%. a. Is this bond currently trading at a discount, at par, or at a premium? E...

See Answer

Q: What was the price of this bond when it was issued?

What was the price of this bond when it was issued?

See Answer

Q: You are pleased to see that you have been given a 5

You are pleased to see that you have been given a 5% raise this year. However, you read on the Wall Street Journal Web site that inflation over the past year has been 2%. How much better off are you i...

See Answer

Q: Assuming the yield to maturity remains constant, what is the price

Assuming the yield to maturity remains constant, what is the price of the bond immediately before it makes its first coupon payment?

See Answer

Q: Assuming the yield to maturity remains constant, what is the price

Assuming the yield to maturity remains constant, what is the price of the bond immediately after it makes its first coupon payment?

See Answer

Q: Your company currently has $1000 par, 6% coupon bonds

Your company currently has $1000 par, 6% coupon bonds with ten years to maturity and a price of $1078. If you want to issue new ten-year coupon bonds at par, what coupon rate do you need to set? Assum...

See Answer

Q: Suppose you purchase a ten-year bond with 6% annual

Suppose you purchase a ten-year bond with 6% annual coupons. You hold the bond for four years, and sell it immediately after receiving the fourth coupon. If the bond’s yield to matur...

See Answer

Q: What is the percentage change in the price of each bond if

What is the percentage change in the price of each bond if its yield to maturity falls from 6% to 5%?

See Answer

Q: Anzio, Inc., has two classes of shares. Class B

Anzio, Inc., has two classes of shares. Class B has ten times the voting rights as Class A. If you own 10% of the class A shares and 20% of the Class B shares, what percentage of the total voting righ...

See Answer

Q: Suppose you purchase a 30-year, zero-coupon bond

Suppose you purchase a 30-year, zero-coupon bond with a yield to maturity of 6%. You hold the bond for five years before selling it. a. If the bond’s yield to maturity is 6% when you sell it, what is...

See Answer

Q: You have found three investment choices for a one-year deposit

You have found three investment choices for a one-year deposit: 10% APR compounded monthly, 10% APR compounded annually, and 9% APR compounded daily. Compute the EAR for each investment choice. (Assum...

See Answer

Q: The following table summarizes the yields to maturity on several one-

The following table summarizes the yields to maturity on several one-year, zero-coupon securities: Security……………………Yield (%) Treasury…………………… …………3.1 AAA corporate…………………………3.2 BBB corporate…………………………...

See Answer

Q: You have just taken out a five-year loan from a

You have just taken out a five-year loan from a bank to buy an engagement ring. The ring costs $5000. You plan to put down $1000 and borrow $4000. You will need to make annual payments of $1000 at the...

See Answer

Q: Andrew Industries is contemplating issuing a 30-year bond with a

Andrew Industries is contemplating issuing a 30-year bond with a coupon rate of 7% (annual coupon payments) and a face value of $1000. Andrew believes it can get a rating of A from Standard & Poor’s....

See Answer

Q: What is the shape of the yield curve given in the following

What is the shape of the yield curve given in the following term structure? What expectations are investors likely to have about future interest rates?

See Answer

Q: HMK Enterprises would like to raise $10 million to invest in

HMK Enterprises would like to raise $10 million to invest in capital expenditures. The company plans to issue five-year bonds with a face value of $1000 and a coupon rate of 6.5% (annual payments). Th...

See Answer

Q: A BBB-rated corporate bond has a yield to maturity of

A BBB-rated corporate bond has a yield to maturity of 8.2%. A U.S. Treasury security has a yield to maturity of 6.5%. These yields are quoted as APRs with semiannual compounding. Both bonds pay semian...

See Answer

Q: Consider a ten-year bond with a face value of $

Consider a ten-year bond with a face value of $1000 that has a coupon rate of 5.5%, with semiannual payments. a. What is the coupon payment for this bond? b. Draw the cash flows for the bond on a time...

See Answer

Q: Assume that a bond will make payments every six months as shown

Assume that a bond will make payments every six months as shown on the following timeline (using six-month periods): a. What is the maturity of the bond (in years)? b. What is the coupon rate (in perc...

See Answer

Q: Your company wants to raise $10 million by issuing 20-

Your company wants to raise $10 million by issuing 20-year zero-coupon bonds. If the yield to maturity on the bonds will be 6% (annually compounded APR), what total principal amount of bonds must you...

See Answer

Q: What is the price per $100 face value of a two

What is the price per $100 face value of a two-year, zero-coupon, risk-free bond?

See Answer

Q: Your bank account pays interest with an EAR of 5%. What

Your bank account pays interest with an EAR of 5%. What is the APR quote for this account based on semiannual compounding? What is the APR with monthly compounding?

See Answer

Q: What is the price per $100 face value of a four

What is the price per $100 face value of a four-year, zero-coupon, risk-free bond?

See Answer

Q: Assume Evco, Inc., has a current stock price of $

Assume Evco, Inc., has a current stock price of $50 and will pay a $2 dividend in one year; its equity cost of capital is 15%. What price must you expect Evco stock to sell for immediately after the f...

See Answer

Q: For each of the following pairs of Treasury securities (each with

For each of the following pairs of Treasury securities (each with $1000 par value), identify which will have the higher price: a. A three-year zero-coupon bond or a five-year zero coupon bond? b. A th...

See Answer

Q: You currently have a one-year-old loan outstanding on

You currently have a one-year-old loan outstanding on your car. You make monthly payments of $300. You have just made a payment. The loan has four years to go (i.e., it had an original term of five ye...

See Answer

Q: A local bank is running the following advertisement in the newspaper:

A local bank is running the following advertisement in the newspaper: “For just $1000 we will pay you $100 forever!” The fine print in the ad says that for a $1000 deposit, the bank will pay $100 ever...

See Answer

Q: The yield to maturity of a $1000 bond with a 7

The yield to maturity of a $1000 bond with a 7% coupon rate, semiannual coupons, and two years to maturity is 7.6% APR, compounded semiannually. What must its price be?

See Answer

Q: Anle Corporation has a current stock price of $20 and is

Anle Corporation has a current stock price of $20 and is expected to pay a dividend of $1 in one year. Its expected stock price right after paying that dividend is $22. a. What is Anle’s equity cost o...

See Answer

Q: Achi Corp. has preferred stock with an annual dividend of $

Achi Corp. has preferred stock with an annual dividend of $3. If the required return on Achi’s preferred stock is 8%, what is its price?

See Answer

Q: NoGrowth Corporation currently pays a dividend of $0.50 per

NoGrowth Corporation currently pays a dividend of $0.50 per quarter, and it will continue to pay this dividend forever. What is the price per share of NoGrowth stock if the firm’s equity cost of capit...

See Answer

Q: Suppose Acap Corporation will pay a dividend of $2.80

Suppose Acap Corporation will pay a dividend of $2.80 per share at the end of this year and a dividend of $3 per share next year. You expect Acap’s stock price to be $52 in two years. Assume that Acap...

See Answer

Q: Suppose the interest rate is 8% APR with monthly compounding.

Suppose the interest rate is 8% APR with monthly compounding. What is the present value of an annuity that pays $100 every six months for five years?

See Answer

Q: What four financial statements can be found in a firm’s 10-

What four financial statements can be found in a firm’s 10-K filing? What checks are there on the accuracy of these statements?

See Answer

Q: You are an international shrimp trader. A food producer in the

You are an international shrimp trader. A food producer in the Czech Republic offers to pay you 2 million Czech koruna today in exchange for a year’s supply of frozen shrimp. Your Thai supplier will p...

See Answer

Q: Suppose your employer offers you a choice between a $5000 bonus

Suppose your employer offers you a choice between a $5000 bonus and 100 shares of the company’s stock. Whichever one you choose will be awarded today. The stock is currently trading for $63 per share....

See Answer

Q: You are a broker for frozen seafood products for Choyce Products.

You are a broker for frozen seafood products for Choyce Products. You just signed a deal with a Belgian distributor. Under the terms of the contract, in one year you will deliver 4000 kilograms of fro...

See Answer

Q: You are a U.S. investor who is trying to

You are a U.S. investor who is trying to calculate the present value of a €5 million cash inflow that will occur one year in the future. The spot exchange rate is S = $1.25/€ and the forward rate is F...

See Answer

Q: You are thinking about investing $5000 in your friend’s landscaping business

You are thinking about investing $5000 in your friend’s landscaping business. Even though you know the investment is risky and you can’t be sure, you expect your investment to be worth $5750 next year...

See Answer

Q: You plan to deposit $500 in a bank account now and

You plan to deposit $500 in a bank account now and $300 at the end of one year. If the account earns 3% interest per year, what will the balance be in the account right after you make the second depos...

See Answer

Q: Mia Caruso Enterprises, a U.S. manufacturer of children’s

Mia Caruso Enterprises, a U.S. manufacturer of children’s toys, has made a sale in India and is expecting a 400 million rupee cash inflow in one year. (The currency of India is the rupee). The current...

See Answer

Q: Etemadi Amalgamated, a U.S. manufacturing firm, is

Etemadi Amalgamated, a U.S. manufacturing firm, is considering a new project in Portugal. You are in Etemadi’s corporate finance department and are responsible for deciding whether t...

See Answer

Q: Etemadi Amalgamated, the U.S. manufacturing company in Problem

Etemadi Amalgamated, the U.S. manufacturing company in Problem 7, is still considering a new project in Portugal. All information presented in Problem 7 is still accurate, except the spot rate is now...

See Answer

Q: You have been accepted into college. The college guarantees that your

You have been accepted into college. The college guarantees that your tuition will not increase for the four years you attend college. The first $10,000 tuition payment is due in six months. After tha...

See Answer

Q: You work for a U.S. firm, and your

You work for a U.S. firm, and your boss has asked you to estimate the cost of capital for countries using the euro. You know that S = $1.20/€ and F1 = $1.157/€. Suppose the dollar WACC for your compan...

See Answer

Q: Summit Systems will pay a dividend of $1.50 this

Summit Systems will pay a dividend of $1.50 this year. If you expect Summit’s dividend to grow by 6% per year, what is its price per share if the firm’s equity cost of capital is 11%?

See Answer

Q: Maryland Light, a U.S. manufacturer of light fixtures

Maryland Light, a U.S. manufacturer of light fixtures, is considering an investment in Japan. The dollar cost of equity for Maryland Light is 11%. You are in the corporate treasury department, and you...

See Answer

Q: The dollar cost of debt for Healy Consulting, a U.

The dollar cost of debt for Healy Consulting, a U.S. research firm, is 7.5%. The firm faces a tax rate of 30% on all income, no matter where it is earned. Managers in the firm need to know its yen cos...

See Answer

Q: Manzetti Foods, a U.S. food processing and distribution

Manzetti Foods, a U.S. food processing and distribution company, is considering an investment in Germany. You are in Manzetti’s corporate finance department and are responsible for d...

See Answer

Q: Tailor Johnson, a U.S. maker of fine menswear

Tailor Johnson, a U.S. maker of fine menswear, has a subsidiary in Ethiopia. This year, the subsidiary reported and repatriated earnings before interest and taxes (EBIT) of 100 million Ethiopian birrs...

See Answer

Q: Your bank is offering you an account that will pay 20%

Your bank is offering you an account that will pay 20% interest in total for a two-year deposit. Determine the equivalent discount rate for a period length of a. six months. b. one year. c. one month....

See Answer

Q: Suppose you are considering renting an apartment. You, the renter

Suppose you are considering renting an apartment. You, the renter, can be viewed as an agent while the company that owns the apartment can be viewed as the principal. What agency conflicts do you anti...

See Answer

Q: You are considering two ways of financing a spring break vacation.

You are considering two ways of financing a spring break vacation. You could put it on your credit card, at 15% APR, compounded monthly, or borrow the money from your parents, who want an 8% interest...

See Answer

Q: Which do you prefer: a bank account that pays 5%

Which do you prefer: a bank account that pays 5% per year (EAR) for three years or a. an account that pays 2.5% every six months for three years? b. an account that pays 7.5% every 18 months for thre...

See Answer

Q: You are thinking of making an investment in a new plant.

You are thinking of making an investment in a new plant. The plant will generate revenues of $1 million per year for as long as you maintain it. You expect that the maintenance costs will start at $50...

See Answer

Q: You make monthly payments on your car loan. It has a

You make monthly payments on your car loan. It has a quoted APR of 5% (monthly compounding). What percentage of the outstanding principal do you pay in interest each month?

See Answer

Q: Suppose Capital One is advertising a 60-month, 5.

Suppose Capital One is advertising a 60-month, 5.99% APR motorcycle loan. If you need to borrow $8000 to purchase your dream Harley-Davidson, what will your monthly payment be?

See Answer

Q: You have an opportunity to invest $50,000 now in

You have an opportunity to invest $50,000 now in return for $60,000 in one year. If your cost of capital is 8%, what is the NPV of this investment?

See Answer

Q: FastTrack Bikes, Inc., is thinking of developing a new composite

FastTrack Bikes, Inc., is thinking of developing a new composite road bike. Development will take six years and the cost is $200,000 per year. Once in production, the bike is expected to make $300,000...

See Answer

Q: RiverRocks’ purchase of Raft Adventures (from Problem 18) will cost

RiverRocks’ purchase of Raft Adventures (from Problem 18) will cost $100 million, but will generate cash flows that start at $15 million in one year and then grow at 4% per year forever. What is the N...

See Answer

Q: CoffeeStop primarily sells coffee. It recently introduced a premium coffee-

CoffeeStop primarily sells coffee. It recently introduced a premium coffee-flavored liquor. Suppose the firm faces a tax rate of 35% and collects the following information. If it plans to finance 11%...

See Answer

Q: Your company has two divisions: One division sells software and the

Your company has two divisions: One division sells software and the other division sells computers through a direct sales channel, primarily taking orders over the Internet. You have decided that Dell...

See Answer

Q: RiverRocks realizes that it will have to raise the financing for the

RiverRocks realizes that it will have to raise the financing for the acquisition of Raft Adventures by issuing new debt and equity. The firm estimates that the direct issuing costs will come to $7 mil...

See Answer

Q: You are planning to issue debt to finance a new project.

You are planning to issue debt to finance a new project. The project will require $20 million in financing and you estimate its NPV to be $15 million. The issue costs for the debt will be 3% of face v...

See Answer

Q: MV Corporation has debt with market value of $100 million,

MV Corporation has debt with market value of $100 million, common equity with a book value of $100 million, and preferred stock worth $20 million outstanding. Its common equity trades at $50 per share...

See Answer

Q: Andyco, Inc., has the following balance sheet and an equity

Andyco, Inc., has the following balance sheet and an equity market-to-book ratio of 1.5. Assuming the market value of debt equals its book value, what weights should it use for its WACC calculation?...

See Answer

Q: Book Co. has 1 million shares of common equity with a

Book Co. has 1 million shares of common equity with a par (book) value of $1, retained earnings of $30 million, and its shares have a market value of $50 per share. It also has debt with a par value o...

See Answer

Q: Consider a simple firm that has the following market-value balance

Consider a simple firm that has the following market-value balance sheet: Next year, there are two possible values for its assets, each equally likely: $1200 and $960. Its debt will be due with 5% int...

See Answer

Q: Avicorp has a $10 million debt issue outstanding, with a

Avicorp has a $10 million debt issue outstanding, with a 6% coupon rate. The debt has semi-annual coupons, the next coupon is due in six months, and the debt matures in five years. It is currently pri...

See Answer

Q: OpenSeas, Inc., is evaluating the purchase of a new cruise

OpenSeas, Inc., is evaluating the purchase of a new cruise ship. The ship would cost $500 million, but would operate for 20 years. OpenSeas expects annual cash flows from operating the ship to be $70...

See Answer

Q: Laurel, Inc., has debt outstanding with a coupon rate of

Laurel, Inc., has debt outstanding with a coupon rate of 6% and a yield to maturity of 7%. Its tax rate is 35%. What is Laurel’s effective (after-tax) cost of debt?

See Answer

Q: Dewyco has preferred stock trading at $50 per share. The

Dewyco has preferred stock trading at $50 per share. The next preferred dividend of $4 is due in one year. What is Dewyco’s cost of capital for preferred stock?

See Answer

Q: Steady Company’s stock has a beta of 0.20. If

Steady Company’s stock has a beta of 0.20. If the risk-free rate is 6% and the market risk premium is 7%, what is an estimate of Steady Company’s cost of equity?

See Answer

Q: HighGrowth Company has a stock price of $20. The firm

HighGrowth Company has a stock price of $20. The firm will pay a dividend next year of $1, and its dividend is expected to grow at a rate of 4% per year thereafter. What is your estimate of HighGrowth...

See Answer

Q: Slow ’n Steady, Inc., has a stock price of $

Slow ’n Steady, Inc., has a stock price of $30, will pay a dividend next year of $3, and has expected dividend growth of 1% per year. What is your estimate of Slow ’n Steady’s cost of equity capital?...

See Answer

Q: If Roundtree from Problem 7 decides to issue an extra 500,

If Roundtree from Problem 7 decides to issue an extra 500,000 shares (for a total of 2.3 million shares), how much total money will it raise? In Problem 7: Roundtree Software is going public using a...

See Answer

Q: Three years ago, you founded Outdoor Recreation, Inc., a

Three years ago, you founded Outdoor Recreation, Inc., a retailer specializing in the sale of equipment and clothing for recreational activities such as camping, skiing, and hiking. So far, your compa...

See Answer

Q: Your investment bankers price your IPO at $15 per share for

Your investment bankers price your IPO at $15 per share for 10 million shares. If the price at the end of the first day of trading is $17 per share, a. What was the percentage underpricing? b. How muc...

See Answer

Q: Margoles Publishing recently completed its IPO. The stock was offered at

Margoles Publishing recently completed its IPO. The stock was offered at a price of $14 per share. On the first day of trading, the stock closed at $19 per share. a. What was the initial return on Mar...

See Answer

Q: If Margoles Publishing from Problem 11 paid an underwriting spread of 7

If Margoles Publishing from Problem 11 paid an underwriting spread of 7% for its IPO and sold 10 million shares, what was the total cost (exclusive of underpricing) to it of going public?

See Answer

Q: In the HomeNet example from the chapter, its receivables are 15

In the HomeNet example from the chapter, its receivables are 15% of sales and its payables are 15% of COGS. Forecast the required investment in net working capital for HomeNet assuming that sales and...

See Answer

Q: Chen Brothers, Inc., sold 4 million shares in its IPO

Chen Brothers, Inc., sold 4 million shares in its IPO, at a price of $18.50 per share. Management negotiated a fee (the underwriting spread) of 7% on this transaction. What was the dollar cost of this...

See Answer

Q: Your firm is selling 3 million shares in an IPO. You

Your firm is selling 3 million shares in an IPO. You are targeting an offer price of $17.25 per share. Your underwriters have proposed a spread of 7%, but you would like to lower it to 5%. However, yo...

See Answer

Q: The firm you founded currently has 12 million shares, of which

The firm you founded currently has 12 million shares, of which you own 7 million. You are considering an IPO where you would sell 2 million shares for $20 each. If all of the shares sold are primary s...

See Answer

Q: The firm you founded currently has 12 million shares, of which

The firm you founded currently has 12 million shares, of which you own 7 million. You are considering an IPO where you would sell 2 million shares for $20 each. If all of the shares sold are from your...

See Answer

Q: The firm you founded currently has 12 million shares, of which

The firm you founded currently has 12 million shares, of which you own 7 million. You are considering an IPO where you would sell 2 million shares for $20 each. What is the maximum number of secondary...

See Answer

Q: On January 20, Metropolitan, Inc., sold 8 million shares

On January 20, Metropolitan, Inc., sold 8 million shares of stock in an SEO. The market price of Metropolitan at the time was $42.50 per share. Of the 8 million shares sold, 5 million shares were prim...

See Answer

Q: Foster Enterprises’ stock is trading for $50 per share and there

Foster Enterprises’ stock is trading for $50 per share and there are currently 10 million shares outstanding. It would like to raise $100 million. If its underwriter charges 5% of gross proceeds, a. H...

See Answer

Q: MacKenzie Corporation currently has 10 million shares of stock outstanding at a

MacKenzie Corporation currently has 10 million shares of stock outstanding at a price of $40 per share. The company would like to raise money and has announced rights issue. Every existing shareholder...

See Answer

Q: You have started a company and are in luck—a venture

You have started a company and are in luck—a venture capitalist has offered to invest. You own 100% of the company with 5 million shares. The VC offers $1 million for 800,000 new shares. a. What is th...

See Answer

Q: Starware Software was founded last year to develop software for gaming applications

Starware Software was founded last year to develop software for gaming applications. The founder initially invested $800,000 and received 8 million shares of stock. Starware now needs to raise a secon...

See Answer

Q: Elmdale Enterprises is deciding whether to expand its production facilities. Although

Elmdale Enterprises is deciding whether to expand its production facilities. Although long-term cash flows are difficult to estimate, management has projected the following cash flows for the first tw...

See Answer

Q: Your start-up company needs capital. Right now, you

Your start-up company needs capital. Right now, you own 100% of the firm with 10 million shares. You have received two offers from venture capitalists. The first offers to invest $3 million for 1 mill...

See Answer

Q: Three years ago, you founded your own company. You invested

Three years ago, you founded your own company. You invested $100,000 of your own money and received 5 million shares of Series A preferred stock. Your company has since been through three additional r...

See Answer

Q: Based on the information in Problem 4 (and that each share

Based on the information in Problem 4 (and that each share of all series of preferred stock is convertible into one share of common stock), what fractions of the firm do the Series B, C, and D investo...

See Answer

Q: Assuming that you own only the Series A preferred stock in Problem

Assuming that you own only the Series A preferred stock in Problem 4 (and that each share of all series of preferred stock is convertible into one share of common stock), what percentage of the firm d...

See Answer

Q: Roundtree Software is going public using an auction IPO. The firm

Roundtree Software is going public using an auction IPO. The firm has received the following bids: Assuming Roundtree would like to sell 1.8 million shares in its IPO, what will be the winning auction...

See Answer

Q: You own a bond with a face value of $10,

You own a bond with a face value of $10,000 and a conversion ratio of 450. What is the conversion price?

See Answer

Q: A $1000 face value bond has a conversion ratio of 40

A $1000 face value bond has a conversion ratio of 40. You estimate the transaction costs of conversion to be 3% of the face value of the bond. What price must the stock reach in order for you to conve...

See Answer

Q: You are the CFO of RealNetworks on July 1, 2008.

You are the CFO of RealNetworks on July 1, 2008. The company’s stock price is $6.74 and its convertible debt (as shown in Table 15.7) is now callable. a. What is the value of the sha...

See Answer

Q: You are finalizing a bank loan for $200,000 for

You are finalizing a bank loan for $200,000 for your small business and the closing fees payable to the bank are 2% of the loan. After paying the fees, what will be the net amount of funds from the lo...

See Answer

Q: Your firm is issuing $100 million in straight bonds at par

Your firm is issuing $100 million in straight bonds at par with a coupon rate of 6% and paying total fees of 3%. What is the net amount of funds that the debt issue will provide for your firm?

See Answer

Q: How many IRRs are there in part (a) of Problem

How many IRRs are there in part (a) of Problem 9? Does the IRR rule give the right answer in this case? Information from Problem 9: Bill Clinton reportedly was paid $10 million to write his book My L...

See Answer

Q: Your firm successfully issued new debt last year, but the debt

Your firm successfully issued new debt last year, but the debt carries covenants. Specifically, you can only pay dividends out of earnings made after the debt issue and you must maintain a minimum qui...

See Answer

Q: General Electric has just issued a callable (at par) ten

General Electric has just issued a callable (at par) ten-year, 6% coupon bond with annual coupon payments. The bond can be called at par in one year or anytime thereafter on a coupon payment date. It...

See Answer

Q: Boeing Corporation has just issued a callable (at par) three

Boeing Corporation has just issued a callable (at par) three-year, 5% coupon bond with semiannual coupon payments. The bond can be called at par in two years or anytime thereafter on a coupon payment...

See Answer

Q: How many IRRs are there in part (b) of Problem

How many IRRs are there in part (b) of Problem 9? Does the IRR rule work in this case? Information from Problem 9: Bill Clinton reportedly was paid $10 million to write his book My Life. The book too...

See Answer

Q: Professor Wendy Smith has been offered the following deal: A law

Professor Wendy Smith has been offered the following deal: A law firm would like to retain her for an upfront payment of $50,000. In return, for the next year the firm would have access to eight hours...

See Answer

Q: Innovation Company is thinking about marketing a new software product. Upfront

Innovation Company is thinking about marketing a new software product. Upfront costs to market and develop the product are $5 million. The product is expected to generate profits of $1 million per yea...

See Answer

Q: You own a coal mining company and are considering opening a new

You own a coal mining company and are considering opening a new mine. The mine itself will cost $120 million to open. If this money is spent immediately, the mine will generate $20 million for the nex...

See Answer

Q: Your firm is considering a project that will cost $4.

Your firm is considering a project that will cost $4.55 million upfront, generate cash flows of $3.5 million per year for three years, and then have a cleanup and shutdown cost of $6 million in the fo...

See Answer

Q: You have an opportunity to invest $100,000 now in

You have an opportunity to invest $100,000 now in return for $80,000 in one year and $30,000 in two years. If your cost of capital is 9%, what is the NPV of this investment?

See Answer

Q: You have just been offered a contract worth $1 million per

You have just been offered a contract worth $1 million per year for five years. However, to take the contract, you will need to purchase some new equipment. Your discount rate for this project is 12%....

See Answer

Q: You are getting ready to start a new project that will incur

You are getting ready to start a new project that will incur some cleanup and shutdown costs when it is completed. The project costs $5.4 million upfront and is expected to generate $1.1 million per y...

See Answer

Q: You are considering investing in a new gold mine in South Africa

You are considering investing in a new gold mine in South Africa. Gold in South Africa is buried very deep, so the mine will require an initial investment of $250 million. Once this investment is made...

See Answer

Q: You are considering making a movie. The movie is expected to

You are considering making a movie. The movie is expected to cost $10 million upfront and take a year to make. After that, it is expected to make $5 million in the year it is released and $2 million f...

See Answer

Q: You are choosing between two projects, but can only take one

You are choosing between two projects, but can only take one. The cash flows for the projects are given in the following table: a. What are the IRRs of the two projects? b. If your discount rate is 5%...

See Answer

Q: You are deciding between two mutually exclusive investment opportunities. Both require

You are deciding between two mutually exclusive investment opportunities. Both require the same initial investment of $10 million. Investment A will generate $2 million per year (starting at the end o...

See Answer

Q: You are considering the following two projects and can only take one

You are considering the following two projects and can only take one. Your cost of capital is 11%. a. What is the NPV of each project at your cost of capital? b. What is the IRR of each project? c. At...

See Answer

Q: You need a particular piece of equipment for your production process.

You need a particular piece of equipment for your production process. An equipment-leasing company has offered to lease you the equipment for $10,000 per year if you sign a guaranteed five-year lease....

See Answer

Q: Gateway Tours is choosing between two bus models. One is more

Gateway Tours is choosing between two bus models. One is more expensive to purchase and maintain, but lasts much longer than the other. Its discount rate is 11%. It plans to continue with one of the t...

See Answer

Q: Hassle-Free Web is bidding to provide Web-page hosting

Hassle-Free Web is bidding to provide Web-page hosting services for Hotel Lisbon. Hotel Lisbon pays its current provider $10,000 per year for hosting its Web page and handling transactions on it, etc....

See Answer

Q: Your storage firm has been offered $100,000 in one

Your storage firm has been offered $100,000 in one year to store some goods for one year. Assume your costs are $95,000, payable immediately, and the cost of capital is 8%. Should you take the contrac...

See Answer

Q: Fabulous Fabricators needs to decide how to allocate space in its production

Fabulous Fabricators needs to decide how to allocate space in its production facility this year. It is considering the following contracts: a. What are the profitability indexes of the projects? b. Wh...

See Answer

Q: Kartman Corporation is evaluating four real estate investments. Management plans to

Kartman Corporation is evaluating four real estate investments. Management plans to buy the properties today and sell them three years from today. The annual discount rate for these investments is 15%...

See Answer

Q: Orchid Biotech Company is evaluating several development projects for experimental drugs.

Orchid Biotech Company is evaluating several development projects for experimental drugs. Although the cash flows are difficult to forecast, the company has come up with the following estimates of the...

See Answer

Q: Cellular Access, Inc., is a cellular telephone service provider that

Cellular Access, Inc., is a cellular telephone service provider that reported net income of $250 million for the most recent fiscal year. The firm had depreciation expenses of $100 million, capital ex...

See Answer

Q: Recall the HomeNet example from the chapter. Suppose HomeNet’s lab will

Recall the HomeNet example from the chapter. Suppose HomeNet’s lab will be housed in warehouse space that the company could have otherwise rented out for $200,000 per year during years 1–4. How does t...

See Answer

Q: One year ago, your company purchased a machine used in manufacturing

One year ago, your company purchased a machine used in manufacturing for $110,000. You have learned that a new machine is available that offers many advantages; you can purchase it for $150,000 today....

See Answer

Q: Beryl’s Iced Tea currently rents a bottling machine for $50,

Beryl’s Iced Tea currently rents a bottling machine for $50,000 per year, including all maintenance expenses. It is considering purchasing a machine instead and is comparing two options: a. Purchase t...

See Answer

Q: You have just completed a $20,000 feasibility study for

You have just completed a $20,000 feasibility study for a new coffee shop in some retail space you own. You bought the space two years ago for $100,000, but if you sold it today, you would net $115,00...

See Answer

Q: You purchased a machine for $1 million three years ago and

You purchased a machine for $1 million three years ago and have been applying straight-line depreciation to zero for a seven-year life. Your tax rate is 35%. If you sell the machine right now (after t...

See Answer

Q: The Jones Company has just completed the third year of a five

The Jones Company has just completed the third year of a five-year MACRS recovery period for a piece of equipment it originally purchased for $300,000. a. What is the book value of the equipment? b. I...

See Answer

Q: You run a construction firm. You have just won a contract

You run a construction firm. You have just won a contract to build a government office building. Building it will require an investment of $10 million today and $5 million in one year. The government...

See Answer

Q: Just before it is about to sell the equipment from Problem 20

Just before it is about to sell the equipment from Problem 20, Jones receives a new order. It can take the new order if it keeps the old equipment. Is there a cost to taking the order and if so, what...

See Answer

Q: Home Builder Supply, a retailer in the home improvement industry,

Home Builder Supply, a retailer in the home improvement industry, currently operates seven retail outlets in Georgia and South Carolina. Management is contemplating building an eighth retail store acr...

See Answer

Q: If Daily Enterprises uses MACRS instead of straight-line depreciation,

If Daily Enterprises uses MACRS instead of straight-line depreciation, which incremental free cash flows from Problem 10 would increase and which would decrease?

See Answer

Q: Markov Manufacturing recently spent $15 million to purchase some equipment used

Markov Manufacturing recently spent $15 million to purchase some equipment used in the manufacture of disk drives. The firm expects that this equipment will have a useful life of five years, and its m...

See Answer

Q: You are a manager at Percolated Fiber, which is considering expanding

You are a manager at Percolated Fiber, which is considering expanding its operations in synthetic fiber manufacturing. Your boss comes into your office, drops a consultant’s report o...

See Answer

Q: Bauer Industries is an automobile manufacturer. Management is currently evaluating a

Bauer Industries is an automobile manufacturer. Management is currently evaluating a proposal to build a plant that will manufacture lightweight trucks. Bauer plans to use a cost of capital of 12% to...

See Answer

Q: Billingham Packaging is considering expanding its production capacity by purchasing a new

Billingham Packaging is considering expanding its production capacity by purchasing a new machine, the XC-750. The cost of the XC-750 is $2.75 million. Unfortunately, installing this machine will take...

See Answer

Q: Why is it that real options must have positive value?

Why is it that real options must have positive value?

See Answer

Q: What kind of real option does the XC-900 machine provide

What kind of real option does the XC-900 machine provide to Billingham in Problem 27?

See Answer

Q: If Billingham knows that it can sell the XC-750 to

If Billingham knows that it can sell the XC-750 to another firm for $2 million in two years, what kind of real option would that provide?

See Answer

Q: You have been offered a unique investment opportunity. If you invest

You have been offered a unique investment opportunity. If you invest $10,000 today, you will receive $500 one year from now, $1500 two years from now, and $10,000 ten years from now. a. What is the NP...

See Answer

Q: Daily Enterprises is purchasing a $10 million machine. It will

Daily Enterprises is purchasing a $10 million machine. It will cost $50,000 to transport and install the machine. The machine has a depreciable life of five years and will have no salvage value. If Da...

See Answer

Q: The machine in Problem 1 will generate incremental revenues of $4

The machine in Problem 1 will generate incremental revenues of $4 million per year along with incremental costs of $1.2 million per year. If Daily’s marginal tax rate is 35%, what are the incremental...

See Answer

Q: You are upgrading to better production equipment for your firm’s only product

You are upgrading to better production equipment for your firm’s only product. The new equipment will allow you to make more of your product in the same amount of time. Thus, you forecast that total s...

See Answer

Q: Pisa Pizza, a seller of frozen pizza, is considering introducing

Pisa Pizza, a seller of frozen pizza, is considering introducing a healthier version of its pizza that will be low in cholesterol and contain no trans fats. The firm expects that sales of the new pizz...

See Answer

Q: Kokomochi is considering the launch of an advertising campaign for its latest

Kokomochi is considering the launch of an advertising campaign for its latest dessert product, the Mini Mochi Munch. Kokomochi plans to spend $5 million on TV, radio, and print advertising this year f...

See Answer

Q: Hyperion, Inc., currently sells its latest high-speed color

Hyperion, Inc., currently sells its latest high-speed color printer, the Hyper 500, for $350. It plans to lower the price to $300 next year. Its cost of goods sold for the Hyper 500 is $200 per unit,...

See Answer

Q: You have a depreciation expense of $500,000 and a

You have a depreciation expense of $500,000 and a tax rate of 35%. What is your depreciation tax shield?

See Answer

Q: You have forecast pro-forma earnings of $1,000

You have forecast pro-forma earnings of $1,000,000. This includes the effect of $200,000 in depreciation. You also forecast a decrease in working capital of $100,000 that year. What is your forecast o...

See Answer

Q: Your pro-forma income statement shows sales of $1,

Your pro-forma income statement shows sales of $1,000,000, cost of goods sold as $500,000, depreciation expense of $100,000, and taxes of $160,000 due to a tax rate of 40%. What are your pro-forma ear...

See Answer

Q: You are forecasting incremental free cash flows for Daily Enterprises. Based

You are forecasting incremental free cash flows for Daily Enterprises. Based on the information in Problems 1 and 2, what are the incremental free cash flows associated with the new machine?

See Answer

Q: Marian Plunket owns her own business and is considering an investment.

Marian Plunket owns her own business and is considering an investment. If she undertakes the investment, it will pay $4000 at the end of each of the next three years. The opportunity requires an initi...

See Answer

Q: Castle View Games would like to invest in a division to develop

Castle View Games would like to invest in a division to develop software for video games. To evaluate this decision, the firm first attempts to project the working capital needs for this operation. It...

See Answer

Q: What was your dividend yield from investing in the stock in Problem

What was your dividend yield from investing in the stock in Problem 8? What was your capital gain? In Problem 8:

See Answer

Q: Your factory has been offered a contract to produce a part for

Your factory has been offered a contract to produce a part for a new printer. The contract would last for three years and your cash flows from the contract would be $5 million per year. Your upfront s...

See Answer

Q: Given $100,000 to invest, construct a value-

Given $100,000 to invest, construct a value-weighted portfolio of the four stocks listed below.

See Answer

Q: Consider the following five monthly returns: / *b

Consider the following five monthly returns: *b. Calculate the geometric average monthly return over this period. c. Calculate the monthly variance over this period. d. Calculate the monthly standard...

See Answer

Q: You hear on the news that the S&P 500 was

You hear on the news that the S&P 500 was down 2% today relative to the risk-free rate (the market’s excess return was -2% ). You are thinking about your portfolio and your investments in Apple and Pr...

See Answer

Q: You are considering opening a new plant. The plant will cost

You are considering opening a new plant. The plant will cost $100 million upfront and will take one year to build. After that, it is expected to produce profits of $30 million at the end of every year...

See Answer

Q: Suppose the risk-free return is 4% and the market

Suppose the risk-free return is 4% and the market portfolio has an expected return of 10% and a standard deviation of 16%. Johnson & Johnson Corporation stock has a beta of 0.32. What is its expected...

See Answer

Q: What is the sign of the risk premium of a negative-

What is the sign of the risk premium of a negative-beta stock? Explain. (Assume the risk premium of the market portfolio is positive.)

See Answer

Q: EJH has a beta of 1.2, CSH has a

EJH has a beta of 1.2, CSH has a beta of 0.6, and KMS has a beta of 1.0. If you put 25% of your money in EJH, 25% in CSH, and 50% in KMS, what is the beta of your portfolio?

See Answer

Q: Suppose Intel stock has a beta of 1.6, whereas

Suppose Intel stock has a beta of 1.6, whereas Boeing stock has a beta of 1. If the risk-free interest rate is 4% and the expected return of the market portfolio is 10%, according to the CAPM, a. What...

See Answer

Q: You are thinking of buying a stock priced at $100 per

You are thinking of buying a stock priced at $100 per share. Assume that the risk free rate is about 4.5% and the market risk premium is 6%. If you think the stock will rise to $117 per share by the e...

See Answer

Q: You are analyzing a stock that has a beta of 1.

You are analyzing a stock that has a beta of 1.2. The risk-free rate is 5% and you estimate the market risk premium to be 6%. If you expect the stock to have a return of 11% over the next year, should...

See Answer

Q: The following table contains prices and dividends for a stock. All

The following table contains prices and dividends for a stock. All prices are after the dividend has been paid. If you bought the stock on January 1 and sold it on December 31, what is your realized r...

See Answer

Q: You buy 100 shares of Tidepool Co. for $40 each

You buy 100 shares of Tidepool Co. for $40 each and 200 shares of Madfish, Inc., for $15 each. What are the weights in your portfolio?

See Answer

Q: Fremont Enterprises has an expected return of 15% and Laurelhurst News

Fremont Enterprises has an expected return of 15% and Laurelhurst News has an expected return of 20%. If you put 70% of your portfolio in Laurelhurst and 30% in Fremont, what is the expected return of...

See Answer

Q: Explain the difference between the arithmetic average return you calculated in Problem

Explain the difference between the arithmetic average return you calculated in Problem 10a and the geometric average return you calculated in Problem 10b. Are both numbers useful? If so, explain why....

See Answer

Q: You are considering how to invest part of your retirement savings.

You are considering how to invest part of your retirement savings. You have decided to put $200,000 into three stocks: 50% of the money in GoldFinger (currently $25/share), 25% of the money in Moosehe...

See Answer

Q: You have $70,000. You put 20% of

You have $70,000. You put 20% of your money in a stock with an expected return of 12%, $30,000 in a stock with an expected return of 15%, and the rest in a stock with an expected return of 20%. What i...

See Answer

Q: Bill Clinton reportedly was paid $10 million to write his book

Bill Clinton reportedly was paid $10 million to write his book My Life. The book took three years to write. In the time he spent writing, Clinton could have been paid to make speeches. Given his popul...

See Answer

Q: Stocks A and B have the following returns (see My Finance

Stocks A and B have the following returns (see My Finance Lab for the data in Excel format): a. What are the expected returns of the two stocks? b. What are the standard deviations of the returns of t...

See Answer

Q: Using the data in the following table, estimate the average return

Using the data in the following table, estimate the average return and volatility for each stock.

See Answer

Q: Using your estimates from Problem 8 and the fact that the correlation

Using your estimates from Problem 8 and the fact that the correlation of A and B is 0.48, calculate the volatility (standard deviation) of a portfolio that is 70% invested in stock A and 30% invested...

See Answer

Q: The following spreadsheet contains monthly returns for Cola Co. and Gas

The following spreadsheet contains monthly returns for Cola Co. and Gas Co. for 2010. Using these data, estimate the average monthly return and volatility for each stock.

See Answer

Q: Ten annual returns are listed in the following table. /

Ten annual returns are listed in the following table. *b. What is the geometric average return over the ten-year period? c. If you invested $100 at the beginning, how much would you have at the end?...

See Answer

Q: Mackenzie Company has a price of $36 and will issue a

Mackenzie Company has a price of $36 and will issue a dividend of $2 next year. It has a beta of 1.2, the risk-free rate is 5.5%, and it estimates the market risk premium to be 5%. a. Estimate the equ...

See Answer

Q: Pfd Company has debt with a yield to maturity of 7%,

Pfd Company has debt with a yield to maturity of 7%, a cost of equity of 13%, and a cost of preferred stock of 9%. The market values of its debt, preferred stock, and equity are $10 million, $3 millio...

See Answer

Q: Growth Company’s current share price is $20 and it is expected

Growth Company’s current share price is $20 and it is expected to pay a $1 dividend per share next year. After that, the firm’s dividends are expected to grow at a rate of 4% per year. a. What is an e...

See Answer

Q: The last four years of returns for a stock are as follows

The last four years of returns for a stock are as follows: a. What is the average annual return? b. What is the variance of the stock’s returns? c. What is the standard deviation of...

See Answer

Q: A retail coffee company is planning to open 100 new coffee outlets

A retail coffee company is planning to open 100 new coffee outlets that are expected to generate, in total, $15 million in free cash flows per year, with a growth rate of 3% in perpetuity. If the coff...

See Answer

Q: RiverRocks, Inc., is considering a project with the following projected

RiverRocks, Inc., is considering a project with the following projected free cash flows: The firm believes that, given the risk of this project, the WACC method is the appropriate approach to valuing...

See Answer

Q: RiverRocks (whose WACC is 12%) is considering an acquisition of

RiverRocks (whose WACC is 12%) is considering an acquisition of Raft Adventures (whose WACC is 15%). What is the appropriate discount rate for RiverRocks to use to evaluate the acquisition? Why?

See Answer

Q: If returns of S&P 500 stocks are normally distributed,

If returns of S&P 500 stocks are normally distributed, what range of returns would you expect to see 95% of the time? Base your answer on Figures 11.3 and 11.4.

See Answer

Q: You observe a portfolio for five years and determine that its average

You observe a portfolio for five years and determine that its average return is 12% and the standard deviation of its returns is 20%. Can you be 95% confident that this portfolio will not lose more th...

See Answer

Q: Using the data in Critical Thinking Question 6, calculate a

Using the data in Critical Thinking Question 6, calculate a. The expected overall payoff of each bank. b. The standard deviation of the overall payoff of each bank.

See Answer

Q: Using the data in the table below, calculate the return for

Using the data in the table below, calculate the return for investing in this stock from January 1 to December 31. Prices are after the dividend has been paid.

See Answer

Q: How much of the return in Problem 1 came from dividend yield

How much of the return in Problem 1 came from dividend yield and how much came from capital gain? In Problem 1 You bought a stock one year ago for $50 per share and sold it today for $55 per share. I...

See Answer

Q: You have just purchased a share of stock for $20.

You have just purchased a share of stock for $20. The company is expected to pay a dividend of $0.50 per share in exactly one year. If you want to earn a 10% return on your investment, what price do y...

See Answer

Q: The fact that Cola Co. and Gas Co. have a

The fact that Cola Co. and Gas Co. have a correlation of 0.6083, calculate the volatility (standard deviation) of a portfolio that is 55% invested in Cola Co. stock and 45% invested in Gas Co. stock....

See Answer

Q: Suppose Johnson & Johnson and the Walgreen Company have the expected returns

Suppose Johnson & Johnson and the Walgreen Company have the expected returns and volatilities shown below, with a correlation of 22%. For a portfolio that is equally invested in Johnson & John...

See Answer

Q: You have a portfolio with a standard deviation of 30% and

You have a portfolio with a standard deviation of 30% and an expected return of 18%. You are considering adding one of the two stocks in the table below. If after adding the stock you will have 20% of...

See Answer

Q: Your client has $100,000 invested in stock A.

Your client has $100,000 invested in stock A. She would like to build a two-stock portfolio by investing another $100,000 in either stock B or C. She wants a portfolio with an expected return of at le...

See Answer

Q: Suppose all possible investment opportunities in the world are limited to the

Suppose all possible investment opportunities in the world are limited to the five stocks listed in the table below. What are the market portfolio weights?

See Answer

Q: Suppose the corporate tax rate is 40%. Consider a firm that

Suppose the corporate tax rate is 40%. Consider a firm that earns $1000 before interest and taxes each year with no risk. The firm’s capital expenditures equal its depreciation expenses each year, and...

See Answer

Q: Kohwe Corporation plans to issue equity to raise $50 million to

Kohwe Corporation plans to issue equity to raise $50 million to finance a new investment. After making the investment, Kohwe expects to earn free cash flows of $10 million each year. Kohwe currently h...

See Answer

Q: You are long two calls on the same share of stock with

You are long two calls on the same share of stock with the same exercise date. The exercise price of the first call is $40 and the exercise price of the second call is $60. In addition, you are short...

See Answer

Q: Dynamic Energy Systems stock is currently trading for $33 per share

Dynamic Energy Systems stock is currently trading for $33 per share. The stock pays no dividends. A one-year European put option on Dynamic with a strike price of $35 is currently trading for $2.10. I...

See Answer

Q: You happen to be checking the newspaper and notice an arbitrage opportunity

You happen to be checking the newspaper and notice an arbitrage opportunity. The current stock price of Intrawest is $20 per share and the one-year risk-free interest rate is 8%. A one-year put on Int...

See Answer

Q: Express the position of an equity holder in terms of put options

Express the position of an equity holder in terms of put options.

See Answer

Q: Below is an option quote on IBM from the CBOE Web site

Below is an option quote on IBM from the CBOE Web site. a. Which option contract had the most trades today? b. Which option contract is being held the most overall? c. Suppose you purchase one option...

See Answer

Q: You own a call option on Intuit stock with a strike price

You own a call option on Intuit stock with a strike price of $40. The option will expire in exactly three months’ time. a. If the stock is trading at $55 in three months, what will be the payoff of th...

See Answer

Q: Assume that you have shorted the call option in Problem 2.

Assume that you have shorted the call option in Problem 2. a. If the stock is trading at $55 in three months, what will you owe? b. If the stock is trading at $35 in three months, what will you owe? c...

See Answer

Q: You own a put option on Ford stock with a strike price

You own a put option on Ford stock with a strike price of $10. The option will expire in exactly six months’ time. a. If the stock is trading at $8 in six months, what will be the payoff of the put? b...

See Answer

Q: Loki, Inc., and Thor, Inc., have entered into

Loki, Inc., and Thor, Inc., have entered into a stock-swap merger agreement whereby Loki will pay a 40% premium over Thor’s premerger price. If Thor’s premerger price per share was $40 and Loki’s was...

See Answer

Q: The NFF Corporation has announced plans to acquire LE Corporation. NFF

The NFF Corporation has announced plans to acquire LE Corporation. NFF is trading for $35 per share and LE is trading for $25 per share, implying a premerger value of LE of $4 billion. If the projecte...

See Answer

Q: Suppose Kohwe borrows the $50 million instead. The firm will

Suppose Kohwe borrows the $50 million instead. The firm will pay interest only on this loan each year, and it will maintain an outstanding balance of $50 million on the loan. Suppose that Kohwe’s corp...

See Answer

Q: Let’s reconsider part (b) of Problem 1. The actual

Let’s reconsider part (b) of Problem 1. The actual premium that your company will pay for TargetCo when it completes the transaction will not be 20%, because on the announcement the target price will...

See Answer

Q: ABC has 1 million shares outstanding, each of which has a

ABC has 1 million shares outstanding, each of which has a price of $20. It has made a takeover offer of XYZ Corporation, which has 1 million shares outstanding and a price per share of $2.50. Assume t...

See Answer

Q: BAD Company’s stock price is $20, and the firm has

BAD Company’s stock price is $20, and the firm has 2 million shares outstanding. You believe you can increase the company’s value if you buy it and replace the management. Assume that BAD has a poison...

See Answer

Q: You work for a leveraged buyout firm and are evaluating a potential

You work for a leveraged buyout firm and are evaluating a potential buyout of UnderWater Company. UnderWater’s stock price is $20, and it has 2 million shares outstanding. You believe that if you buy...

See Answer

Q: Your company has earnings per share of $4. It has

Your company has earnings per share of $4. It has 1 million shares outstanding, each of which has a price of $40. You are thinking of buying TargetCo, which has earnings per share of $2, 1 million sha...

See Answer

Q: If companies in the same industry as TargetCo (from Problem 1

If companies in the same industry as TargetCo (from Problem 1) are trading at multiples of 14 times earnings, what would be one estimate of an appropriate premium for TargetCo? Information from Proble...

See Answer

Q: You are invested in GreenFrame, Inc. The CEO owns 3

You are invested in GreenFrame, Inc. The CEO owns 3% of GreenFrame and is considering an acquisition. If the acquisition destroys $50 million of GreenFrame’s value, but the present value of the CEO’s...

See Answer

Q: Sora Industries has 60 million outstanding shares, $120 million in

Sora Industries has 60 million outstanding shares, $120 million in debt, $40 million in cash, and the following projected free cash flow for the next four years (see MyFinanceLab for the data in Excel...

See Answer

Q: Consider the valuation of Nike given in Example 10.1.

Consider the valuation of Nike given in Example 10.1. a. Suppose you believe Nike’s initial revenue growth rate will be between 7% and 11% (with growth always slowing linearly to 5% by year 2015). Wha...

See Answer

Q: You notice that Dell Computers has a stock price of $27

You notice that Dell Computers has a stock price of $27.85 and EPS of $1.26. Its competitor Hewlett-Packard has EPS of $2.47. What is one estimate of the value of a share of Hewlett-Packard stock?

See Answer

Q: Now suppose that with leverage, Kohwe’s expected free cash flows will

Now suppose that with leverage, Kohwe’s expected free cash flows will decline to $9 million per year due to reduced sales and other financial distress costs. Assume that the appropriate discount rate...

See Answer

Q: CSH has EBITDA of $5 million. You feel that an

CSH has EBITDA of $5 million. You feel that an appropriate EV/EBITDA ratio for CSH is 9. CSH has $10 million in debt, $2 million in cash, and 800,000 shares outstanding. What is your estimate of CSH’s...

See Answer

Q: After researching the competitors of EJH Enterprises, you determine that most

After researching the competitors of EJH Enterprises, you determine that most comparable firms have the following valuation ratios (see MyFinanceLab for the data in Excel format): EJH Enterprises has...

See Answer

Q: Suppose that in May 2010, Nike had EPS of $3

Suppose that in May 2010, Nike had EPS of $3.51 and a book value of equity of $18.92 per share. a. Using the average P/E multiple in Table 10.1, estimate Nike’s share price. b. What...

See Answer

Q: Suppose that in May 2010, Nike had sales of $19

Suppose that in May 2010, Nike had sales of $19,176 million, EBITDA of $2,809 million, excess cash of $3,500 million, $437 million of debt, and 485.7 million shares outstanding. a. Using the average e...

See Answer

Q: Suppose Rocky Shoes and Boots has earnings per share of $2

Suppose Rocky Shoes and Boots has earnings per share of $2.30 and EBITDA of $30.7 million. The firm also has 5.4 million shares outstanding and debt of $125 million (net of cash). You believe Deckers...

See Answer

Q: Summit Systems has an equity cost of capital of 11%, will

Summit Systems has an equity cost of capital of 11%, will pay a dividend of $1.50 in one year and its dividends had been expected to grow by 6% per year. You read in the paper that Summit has revised...

See Answer

Q: Assume that Cola Company has a share price of $43.

Assume that Cola Company has a share price of $43. The firm will pay a dividend of $1.24 in one year, and you expect Cola Co. to raise this dividend by approximately 7% per year in perpetuity. a. If C...

See Answer

Q: Roybus, Inc., a manufacturer of flash memory, just reported

Roybus, Inc., a manufacturer of flash memory, just reported that its main production facility in Taiwan was destroyed in a fire. Although the plant was fully insured, the loss of production will decre...

See Answer

Q: Apnex, Inc., is a biotechnology firm that is about to

Apnex, Inc., is a biotechnology firm that is about to announce the results of its clinical trials of a potential new cancer drug. If the trials were successful, Apnex stock will be worth $70 per share...

See Answer

Q: You have a $100,000 portfolio made up of 15

You have a $100,000 portfolio made up of 15 stocks. You trade each stock five times this year and each time you trade, you pay about $30 in commissions and spread. You have no special knowledge, so yo...

See Answer

Q: Hawar International is a shipping firm with a current share price of

Hawar International is a shipping firm with a current share price of $5.50 and 10 million shares outstanding. Suppose that Hawar announces plans to lower its corporate taxes by borrowing $20 million a...

See Answer

Q: Assume the annual return for the lowest turnover portfolio is 18%

Assume the annual return for the lowest turnover portfolio is 18% and the annual return for the highest turnover portfolio is 12%. If you invest $100,000 and have the highest turnover, how much lower...

See Answer

Q: This year, FCF, Inc., has earnings before interest and

This year, FCF, Inc., has earnings before interest and taxes of $10 million, depreciation expenses of $1 million, capital expenditures of $1.5 million, and has increased its net working capital by $50...

See Answer

Q: Victoria Enterprises expects earnings before interest and taxes (EBIT) next

Victoria Enterprises expects earnings before interest and taxes (EBIT) next year of $1 million. Its depreciation and capital expenditures will both be $300,000, and it expects its capital expenditures...

See Answer

Q: The present value of JECK Co.’s expected free cash flows

The present value of JECK Co.’s expected free cash flows is $100 million. If JECK has $30 million in debt, $6 million in cash, and 2 million shares outstanding, what is its share price?

See Answer

Q: Portage Bay Enterprises has $1 million in excess cash, no

Portage Bay Enterprises has $1 million in excess cash, no debt and is expected to have free cash flow of $10 million next year. Its FCF is then expected to grow at a rate of 3% per year forever. If Po...

See Answer

Q: Heavy Metal Corporation is expected to generate the following free cash flows

Heavy Metal Corporation is expected to generate the following free cash flows over the next five years: After then, the free cash flows are expected to grow at the industry average of 4% per year. Usi...

See Answer

Q: Covan, Inc., is expected to have the following free cash

Covan, Inc., is expected to have the following free cash flows: a. Covan has 8 million shares outstanding, $3 million in excess cash, and it has no debt. If its cost of capital is 12%, what should its...

See Answer

Q: Consider a project with free cash flows in one year of $

Consider a project with free cash flows in one year of $130,000 or $180,000, with each outcome being equally likely. The initial investment required for the project is $100,000, and the project’s cost...

See Answer

Q: You are an entrepreneur starting a biotechnology firm. If your research

You are an entrepreneur starting a biotechnology firm. If your research is successful, the technology can be sold for $30 million. If your research is unsuccessful, it will be worth nothing. To fund y...

See Answer

Q: Acort Industries owns assets that will have an 80% probability of

Acort Industries owns assets that will have an 80% probability of having a market value of $50 million in one year. There is a 20% chance that the assets will be worth only $20 million. The current ri...

See Answer

Q: Marpor Industries has no debt and expects to generate free cash flows

Marpor Industries has no debt and expects to generate free cash flows of $16 million each year. Marpor believes that if it permanently increases its level of debt to $40 million, the risk of financial...

See Answer

Q: Suppose there are no taxes. Firm ABC has no debt,

Suppose there are no taxes. Firm ABC has no debt, and firm XYZ has debt of $5000 on which it pays interest of 10% each year. Both companies have identical projects that generate free cash flows of $80...

See Answer

Q: Hardmon Enterprises is currently an all-equity firm with an expected

Hardmon Enterprises is currently an all-equity firm with an expected return of 12%. It is considering borrowing money to buy back some of its existing shares, thus increasing its leverage. a. Suppose...

See Answer

Q: Suppose Microsoft has no debt and a WACC of 9.2

Suppose Microsoft has no debt and a WACC of 9.2%. The average debt-to-value ratio for the software industry is 5%. What would its cost of equity be if it took on the average amount of debt for its ind...

See Answer

Q: Pelamed Pharmaceuticals had EBIT of $325 million in 2010. In

Pelamed Pharmaceuticals had EBIT of $325 million in 2010. In addition, Pelamed had interest expenses of $125 million and a corporate tax rate of 40%. a. What was Pelamed’s 2010 net income? b. What was...

See Answer

Q: Grommit Engineering expects to have net income next year of $20

Grommit Engineering expects to have net income next year of $20.75 million and free cash flow of $22.15 million. Grommit’s marginal corporate tax rate is 35%. a. If Grommit increases leverage so that...

See Answer

Q: Assume that Microsoft has a total market value of $300 billion

Assume that Microsoft has a total market value of $300 billion and a marginal tax rate of 35%. If it permanently changes its leverage from no debt by taking on new debt in the amount of 13% of its cur...

See Answer

Q: The HNH Corporation will pay a constant dividend of $2 per

The HNH Corporation will pay a constant dividend of $2 per share, per year, in perpetuity. Assume all investors pay a 20% tax on dividends and that there is no capital gains tax. The cost of capital f...

See Answer

Q: Consider a firm whose only asset is a plot of vacant land

Consider a firm whose only asset is a plot of vacant land, and whose only liability is debt of $15 million due in one year. If left vacant, the land will be worth $10 million in one year. Alternativel...

See Answer

Q: Zymase is a biotechnology start-up firm. Researchers at Zymase

Zymase is a biotechnology start-up firm. Researchers at Zymase must choose one of three different research strategies. The payoffs (after taxes) and their likelihood for each strategy are shown below....

See Answer

Q: You own a firm, and you want to raise $30

You own a firm, and you want to raise $30 million to fund an expansion. Currently, you own 100% of the firm’s equity, and the firm has no debt. To raise the $30 million solely through equity, you will...

See Answer

Q: Empire Industries forecasts net income this coming year as shown below (

Empire Industries forecasts net income this coming year as shown below (in thousands of dollars): Approximately $200,000 of Empire’s earnings will be needed to make new, positive-NP...

See Answer

Q: Your firm currently has $100 million in debt outstanding with a

Your firm currently has $100 million in debt outstanding with a 10% interest rate. The terms of the loan require the firm to repay $25 million of the balance each year. Suppose that the marginal corpo...

See Answer

Q: Info Systems Technology (IST) manufactures microprocessor chips for use in

Info Systems Technology (IST) manufactures microprocessor chips for use in appliances and other applications. IST has no debt and 100 million shares outstanding. The correct price for these shares is...

See Answer

Q: You purchased CSH stock for $40 and it is now selling

You purchased CSH stock for $40 and it is now selling for $50. The company has announced that it plans a $10 special dividend. a. Assuming 2010 tax rates, if you sell the stock or wait and receive the...

See Answer

Q: Assume perfect capital markets. Kay Industries currently has $100 million

Assume perfect capital markets. Kay Industries currently has $100 million invested in short-term Treasury securities paying 7%, and it pays out the interest payments on these securities as a dividend....

See Answer

Q: Redo Problem 11, but assume that Kay must pay a corporate

Redo Problem 11, but assume that Kay must pay a corporate tax rate of 35%, and that investors pay no taxes. Data from Problem 11: Assume perfect capital markets. Kay Industries currently has $100 mil...

See Answer

Q: Redo Problem 11, but assume that investors pay a 15%

Redo Problem 11, but assume that investors pay a 15% tax on dividends but no capital gains taxes, and that Kay does not pay corporate taxes. Data from Problem 11: Assume perfect capital markets. Kay...

See Answer

Q: AMC Corporation currently has an enterprise value of $400 million and

AMC Corporation currently has an enterprise value of $400 million and $100 million in excess cash. The firm has 10 million shares outstanding and no debt. Suppose AMC uses its excess cash to repurchas...

See Answer

Q: AMC Corporation currently has an enterprise value of $400 million and

AMC Corporation currently has an enterprise value of $400 million and $100 million in excess cash. The firm has 10 million shares outstanding and no debt. Suppose AMC uses its excess cash to repurchas...

See Answer

Q: AMC Corporation currently has an enterprise value of $400 million and

AMC Corporation currently has an enterprise value of $400 million and $100 million in excess cash. The firm has 10 million shares outstanding and no debt. Suppose AMC uses its excess cash to repurchas...

See Answer

Q: AMC Corporation currently has an enterprise value of $400 million and

AMC Corporation currently has an enterprise value of $400 million and $100 million in excess cash. The firm has 10 million shares outstanding and no debt. Suppose AMC uses its excess cash to repurchas...

See Answer

Q: AMC Corporation currently has an enterprise value of $400 million and

AMC Corporation currently has an enterprise value of $400 million and $100 million in excess cash. The firm has 10 million shares outstanding and no debt. Suppose AMC uses its excess cash to repurchas...

See Answer

Q: Arnell Industries has $10 million in permanent debt outstanding. The

Arnell Industries has $10 million in permanent debt outstanding. The firm will pay interest only on this debt. Arnell’s marginal tax rate is expected to be 35% for the foreseeable future. a. Suppose A...

See Answer

Q: FCF Co. has 20,000 shares outstanding and a total

FCF Co. has 20,000 shares outstanding and a total market value of $1 million, $300 thousand of which is debt and the other $700 thousand is equity. It is planning a 10% stock dividend. a. What is the...

See Answer

Q: Suppose the stock of Host Hotels & Resorts is currently trading for

Suppose the stock of Host Hotels & Resorts is currently trading for $20 per share. a. If Host issues a 20% stock dividend, what would its new share price be? b. If Host does a 3:2 stock split, what wo...

See Answer

Q: If Berkshire Hathaway’s A shares are trading at $120,000

If Berkshire Hathaway’s A shares are trading at $120,000, what split ratio would it need to bring its stock price down to $50?

See Answer

Q: After the market close on May 11, 2001, Adaptec,

After the market close on May 11, 2001, Adaptec, Inc., distributed a dividend of shares of the stock of its software division, Roxio, Inc. Each Adaptec shareholder received 0.1646 share of Roxio stock...

See Answer

Q: ABC Corporation announced that it would pay a dividend to all shareholders

ABC Corporation announced that it would pay a dividend to all shareholders of record as of Monday, April 5, 2010. It takes three business days after a purchase for the new owners of a share of stock t...

See Answer

Q: RFC Corp. has announced a $1 dividend. If RFC’s

RFC Corp. has announced a $1 dividend. If RFC’s last price while trading cum dividend is $50, what should its first ex-dividend price be (assuming perfect capital markets)?

See Answer

Q: ECB Co. has 1 million shares outstanding selling at $20

ECB Co. has 1 million shares outstanding selling at $20 per share. It plans to repurchase 100,000 shares at the market price. What will its market capitalization be after the repurchase? What will its...

See Answer

Q: KMS corporation has assets of $500 million, $50 million

KMS corporation has assets of $500 million, $50 million of which are cash. It has debt of $200 million. If KMS repurchases $20 million of its stock: a. What changes will occur on its balance sheet? b...

See Answer

Q: Suppose that KMS in Problem 4 decides to initiate a dividend instead

Suppose that KMS in Problem 4 decides to initiate a dividend instead, but it wants the present value of the payout to be the same $20 million. If its cost of equity capital is 10%, to what amount per...

See Answer

Q: EJH Company has a market capitalization of $1 billion and 20

EJH Company has a market capitalization of $1 billion and 20 million shares outstanding. It plans to distribute $100 million through an open market repurchase. Assuming perfect capital markets: a. Wha...

See Answer

Q: Rogot Instruments makes fine violins, violas, and cellos. It

Rogot Instruments makes fine violins, violas, and cellos. It has $1 million in debt outstanding, equity valued at $2 million, and pays corporate income tax at a rate of 35%. Its cost of equity is 12%...

See Answer

Q: Natsam Corporation has $250 million of excess cash. The firm

Natsam Corporation has $250 million of excess cash. The firm has no debt and 500 million shares outstanding with a current market price of $15 per share. Natsam’s board has decided to pay out this cas...

See Answer

Q: Suppose the board of Natsam Corporation decided to do the share repurchase

Suppose the board of Natsam Corporation decided to do the share repurchase in Problem 7(b), but you as an investor would have preferred to receive a dividend payment. How can you leave yourself in the...

See Answer

Q: Use the following income statement and balance sheet for Global Corp.:

Use the following income statement and balance sheet for Global Corp.: Assume that Global pays out 50% of its net income. Use the percent of sales method to forecast stockholders’ eq...

See Answer

Q: Use the following income statement and balance sheet for Global Corp.:

Use the following income statement and balance sheet for Global Corp.: What is the amount of net new financing needed for Global?

See Answer

Q: Use the following income statement and balance sheet for Global Corp.:

Use the following income statement and balance sheet for Global Corp.: If Global decides that it will limit its net new financing to no more than $9 million, how will this affect its payout policy?

See Answer

Q: Assume that KMS’s market share will increase by 0.25%

Assume that KMS’s market share will increase by 0.25% per year rather than the 1% used in the chapter (see Table 18.5) and that its prices remain as in the chapter. What production capacity will KMS r...

See Answer

Q: Under the assumption that KMS’s market share will increase by 0.

Under the assumption that KMS’s market share will increase by 0.25% per year, you determine that the plant will require an expansion in 2012. The expansion will cost $20 million. Assuming that the fin...

See Answer

Q: Under the assumption that KMS’s market share will increase by 0.

Under the assumption that KMS’s market share will increase by 0.25% per year, you project the following depreciation: Using this information, project net income through 2015 (that is...

See Answer

Q: Assuming that KMS’s market share will increase by 0.25%

Assuming that KMS’s market share will increase by 0.25% per year (implying that the investment, financing, and depreciation will be adjusted as described in Problems 13 and 14), and...

See Answer

Q: Using the information in the table below, calculate this company’s:

Using the information in the table below, calculate this company’s: a. Internal growth rate. b. Sustainable growth rate. c. Sustainable growth rate if it pays out 40% of its net inco...

See Answer

Q: Rumolt Motors has 30 million shares outstanding with a price of $

Rumolt Motors has 30 million shares outstanding with a price of $15 per share. In addition, Rumolt has issued bonds with a total current market value of $150 million. Suppose Rumolt’s equity cost of c...

See Answer

Q: Did KMS’s expansion plan call for it to grow slower or faster

Did KMS’s expansion plan call for it to grow slower or faster than its sustainable growth rate?

See Answer

Q: Your firm has an ROE of 12%, a payout ratio of

Your firm has an ROE of 12%, a payout ratio of 25%, $600,000 of stockholders’ equity, and $400,000 of debt. If you grow at your sustainable growth rate this year, how much additional debt will you nee...

See Answer

Q: IZAX, Co. had the following items on its balance sheet

IZAX, Co. had the following items on its balance sheet at the beginning of the year: Its net income this year is $20,000 and it pays dividends of $5,000. If its assets grew at its internal growth rate...

See Answer

Q: Forecast KMS’s free cash flows (reproduce Table 18.13),

Forecast KMS’s free cash flows (reproduce Table 18.13), assuming KMS’s market share will increase by 0.25% per year; investment, financing, and depreciation will be adjusted accordingly; and working c...

See Answer

Q: Calculate the continuation value of KMS using your reproduction of Table 18

Calculate the continuation value of KMS using your reproduction of Table 18.8 from Problem 14, and assuming an EBITDA multiple of 8.5. In Problem 14

See Answer

Q: Assuming a cost of capital of 10%, compute the value of

Assuming a cost of capital of 10%, compute the value of KMS under the 0.25% growth scenario.

See Answer

Q: Your company has sales of $100,000 this year and

Your company has sales of $100,000 this year and cost of goods sold of $72,000. You forecast sales to increase to $110,000 next year. Using the percent of sales method, forecast next year’s cost of go...

See Answer

Q: For the next fiscal year, you forecast net income of $

For the next fiscal year, you forecast net income of $50,000 and ending assets of $500,000. Your firm’s payout ratio is 10%. Your beginning stockholders’ equity is $300,000 and your beginning total li...

See Answer

Q: Assume your beginning debt in Problem 2 is $100,000

Assume your beginning debt in Problem 2 is $100,000. What amount of equity and what amount of debt would you need to issue to cover the net new financing in order to keep your debt-equity ratio consta...

See Answer

Q: Use the following income statement and balance sheet for Jim’s Espresso:

Use the following income statement and balance sheet for Jim’s Espresso: Jim’s expects sales to grow by 10% next year. Using the percent of sales method, forecast:...

See Answer

Q: Your supplier offers terms of 1/10, net 45.

Your supplier offers terms of 1/10, net 45. What is the effective annual cost of trade credit if you choose to forgo the discount and pay on day 45?

See Answer

Q: The Fast Reader Company supplies bulletin board services to numerous hotel chains

The Fast Reader Company supplies bulletin board services to numerous hotel chains nationwide. The owner of the firm is investigating the desirability of employing a billing firm to do her billing and...

See Answer

Q: The Saban Corporation is trying to decide whether to switch to a

The Saban Corporation is trying to decide whether to switch to a bank that will accommodate electronic funds transfers from Saban’s customers. Saban’s financial manager believes the new system would d...

See Answer

Q: Milton Industries expects free cash flows of $5 million each year

Milton Industries expects free cash flows of $5 million each year. Milton’s corporate tax rate is 35%, and its unlevered cost of capital is 15%. The firm also has outstanding debt of $19.05 million, a...

See Answer

Q: The Manana Corporation had sales of $60 million this year.

The Manana Corporation had sales of $60 million this year. Its accounts receivable balance averaged $2 million. How long, on average, does it take the firm to collect on its sales?

See Answer

Q: The Mighty Power Tool Company has the following accounts on its books

The Mighty Power Tool Company has the following accounts on its books: The firm extends credit on terms of 1/15, net 30. Develop an aging schedule using 15-day increments through 60 days, and then ind...

See Answer

Q: Simple Simon’s Bakery purchases supplies on terms of 1/10,

Simple Simon’s Bakery purchases supplies on terms of 1/10, net 25. If Simple Simon’s chooses to take the discount offered, it must obtain a bank loan to meet its short-term financing needs. A local ba...

See Answer

Q: Your firm purchases goods from its supplier on terms of 3/

Your firm purchases goods from its supplier on terms of 3/15, net 40. a. What is the effective annual cost to your firm if it chooses not to take the discount and makes its payment on day 40? b. What...

See Answer

Q: Use the financial statements supplied below and on the next page for

Use the financial statements supplied below and on the next page for International Motor Corporation (IMC) to answer the following questions: a. Calculate the cash conversion cycle for IMC for both 20...

See Answer

Q: Your company had $10 million in sales last year. Its

Your company had $10 million in sales last year. Its cost of goods sold was $7 million and its average inventory balance was $1,200,000. What was its average days of inventory?

See Answer

Q: Summit Builders has a market debt-equity ratio of 0.

Summit Builders has a market debt-equity ratio of 0.65 and a corporate tax rate of 40%, and it pays 7% interest on its debt. By what amount does the interest tax shield from its debt lower Summit’s WA...

See Answer

Q: Happy Valley Homecare Suppliers, Inc. (HVHS), had $

Happy Valley Homecare Suppliers, Inc. (HVHS), had $20 million in sales in 2010. Its cost of goods sold was $8 million, and its average inventory balance was $2,000,000. a. Calculate the average numbe...

See Answer

Q: Use the following income statement and balance sheet for Jim’s Espresso:

Use the following income statement and balance sheet for Jim’s Espresso: Assume that Jim’s pays out 90% of its net income. Use the percent of sales method to foreca...

See Answer

Q: Sailboats Etc. is a retail company specializing in sailboats and other

Sailboats Etc. is a retail company specializing in sailboats and other sailing-related equipment. The following table contains financial forecasts as well as current (month 0) working capital levels....

See Answer

Q: FastChips Semiconductors has inventory days of 75, accounts receivable days of

FastChips Semiconductors has inventory days of 75, accounts receivable days of 30, and accounts payable days of 90. What is its cash conversion cycle?

See Answer

Q: Emerald City Umbrellas sells umbrellas and rain gear in Seattle, so

Emerald City Umbrellas sells umbrellas and rain gear in Seattle, so its sales are fairly level across the year. However, it is branching out to other markets where it expects demand to be much more va...

See Answer

Q: Use the following income statement and balance sheet for Jim’s Espresso:

Use the following income statement and balance sheet for Jim’s Espresso: What is the amount of net new financing needed for Jim’s?

See Answer

Q: What are the permanent working capital needs of your company? What

What are the permanent working capital needs of your company? What are the temporary needs?

See Answer

Q: If you chose to use only long-term financing, what

If you chose to use only long-term financing, what total amount of borrowing would you need to have on a permanent basis? Forecast your excess cash levels under this scenario.

See Answer

Q: If you hold only $100 in cash at any time,

If you hold only $100 in cash at any time, what is your maximum short-term borrowing and when?

See Answer

Q: If you choose to enter the year with $400 total in

If you choose to enter the year with $400 total in cash, what is your maximum short-term borrowing?

See Answer

Q: Assume that you have shorted the put option in Problem 4.

Assume that you have shorted the put option in Problem 4. a. If the stock is trading at $8 in three months, what will you owe? b. If the stock is trading at $23 in three months, what will you owe? c....

See Answer

Q: You are long both a call and a put on the same

You are long both a call and a put on the same share of stock with the same exercise date. The exercise price of the call is $40 and the exercise price of the put is $45. Plot the value of this combin...

See Answer

Q: The Treadwater Bank wants to raise $1 million using three-

The Treadwater Bank wants to raise $1 million using three-month commercial paper. The net proceeds to the bank will be $985,000. What is the effective annual rate of this financing for Treadwater?

See Answer

Q: Use the following income statement and balance sheet for Jim’s Espresso:

Use the following income statement and balance sheet for Jim’s Espresso: If Jim’s adjusts its payout policy to 70% of net income, how will the net new financing cha...

See Answer

Q: Use the following income statement and balance sheet for Global Corp.:

Use the following income statement and balance sheet for Global Corp.: Global expects sales to grow by 8% next year. Using the percent of sales method, forecast: a. Costs except depreciation b. Deprec...

See Answer

Q: Westerly Industries has the following financial information. What is its cash

Westerly Industries has the following financial information. What is its cash conversion cycle?

See Answer

Q: Aberdeen Outboard Motors is contemplating building a new plant. The company

Aberdeen Outboard Motors is contemplating building a new plant. The company anticipates that the plant will require an initial investment of $2 million in net working capital today. The plant will las...

See Answer

Q: Your firm currently has net working capital of $100,000

Your firm currently has net working capital of $100,000 that it expects to grow at a rate of 4% per year forever. You are considering some suggestions that could slow that growth to 3% per year. If yo...

See Answer

Q: Assume the credit terms offered to your firm by your suppliers are

Assume the credit terms offered to your firm by your suppliers are 3/5, net 30. Calculate the cost of the trade credit if your firm does not take the discount and pays on day 30.

See Answer

Q: NatNah, a builder of acoustic accessories, has no debt and

NatNah, a builder of acoustic accessories, has no debt and an equity cost of capital of 15%. NatNah decides to increase its leverage to maintain a market debt-to-value ratio of 0.5. Suppose its debt c...

See Answer

Q: If you want to limit your maximum short-term borrowing to

If you want to limit your maximum short-term borrowing to $500, how much excess cash must you carry?

See Answer

Q: The Hand-to-Mouth Company needs a $10,

The Hand-to-Mouth Company needs a $10,000 loan for the next 30 days. It is trying to decide which of three alternatives to use: Alternative A: Forgo the discount on its trade credit agreement that off...

See Answer

Q: Consider two loans with one-year maturities and identical face values

Consider two loans with one-year maturities and identical face values: an 8% loan with a 1% loan origination fee and an 8% loan with a 5% (no-interest) compensating balance requirement. Which loan wou...

See Answer

Q: Magna Corporation has an issue of commercial paper with a face value

Magna Corporation has an issue of commercial paper with a face value of $1,000,000 and a maturity of six months. Magna received net proceeds of $973,710 when it sold the paper. What is the effective a...

See Answer

Q: Which of the following one-year, $1000 bank loans

Which of the following one-year, $1000 bank loans offers the lowest effective annual rate? a. A loan with an APR of 6%, compounded monthly b. A loan with an APR of 6%, compounded annually, with a com...

See Answer

Q: The Needy Corporation borrowed $10,000 from Bank Ease.

The Needy Corporation borrowed $10,000 from Bank Ease. According to the terms of the loan, Needy must pay the bank $400 in interest every three months for the three-year life of the loan, with the pri...

See Answer

Q: Assume that the prime rate is 8% APR, compounded quarterly

Assume that the prime rate is 8% APR, compounded quarterly. How much dollar savings in interest did Treadwater (Problem 12) and Magna (Problem 11) achieve by accessing the commercial paper market? Da...

See Answer

Q: The Signet Corporation has issued four-month commercial paper with a

The Signet Corporation has issued four-month commercial paper with a $6 million face value. The firm netted $5,870,850 on the sale. What effective annual rate is Signet paying for these funds?

See Answer

Q: The Ohio Valley Steel Corporation has borrowed $5 million for one

The Ohio Valley Steel Corporation has borrowed $5 million for one month at a stated annual rate of 9%, using inventory stored in a field warehouse as collateral. The warehouser charges a $5000 fee, pa...

See Answer

Q: Kurz Manufacturing is currently an all-equity firm with 20 million

Kurz Manufacturing is currently an all-equity firm with 20 million shares outstanding and a stock price of $7.50 per share. Although investors currently expect Kurz to remain an all-equity firm, Kurz...

See Answer

Q: The Rasputin Brewery is considering using a public warehouse loan as part

The Rasputin Brewery is considering using a public warehouse loan as part of its short-term financing. The firm will require a loan of $500,000. Interest on the loan will be 10% (APR, annual compoundi...

See Answer

Q: Construct a short-term financial plan for Springfield Snowboards based on

Construct a short-term financial plan for Springfield Snowboards based on its expansion opportunity described in the “Positive Cash Flow Shocks” part of Section 20....

See Answer

Q: The term globalization has become very widely used in recent years.

The term globalization has become very widely used in recent years. How would you define it?

See Answer

Q: How do the motivations of individuals, both inside and outside the

How do the motivations of individuals, both inside and outside the organization or business, define the limits of financial globalization?

See Answer

Q: Name the four main types of transactions from which transaction exposure arises

Name the four main types of transactions from which transaction exposure arises.

See Answer

Q: Why would anyone write an option, knowing that the gain from

Why would anyone write an option, knowing that the gain from receiving the option premium is fixed, but the loss, if the underlying price goes in the wrong direction, can be extremely large?

See Answer

Q: Infrastructure weakness was one of the causes of the emerging market crisis

Infrastructure weakness was one of the causes of the emerging market crisis in Thailand in 1997. Define infrastructure weakness, and explain how it could affect a country’s exchange rate.

See Answer

Q: Define the Fisher effect. To what extent do empirical tests confirm

Define the Fisher effect. To what extent do empirical tests confirm that the Fisher effect exists in practice?

See Answer

Q: Define the international Fisher effect. To what extent do empirical tests

Define the international Fisher effect. To what extent do empirical tests confirm that the international Fisher effect exists in practice?

See Answer

Q: Define interest rate parity. What is the relationship between interest rate

Define interest rate parity. What is the relationship between interest rate parity and forward rates?

See Answer

Q: Describe four arguments in favor of a firm pursuing an active currency

Describe four arguments in favor of a firm pursuing an active currency risk management program.

See Answer

Q: Define the terms covered interest arbitrage and uncovered interest arbitrage. What

Define the terms covered interest arbitrage and uncovered interest arbitrage. What is the difference between these two transactions?

See Answer

Q: Some forecasters believe that foreign exchange markets for the major floating currencies

Some forecasters believe that foreign exchange markets for the major floating currencies are “efficient” and forward exchange rates are unbiased predictors of future spot exchange rates. What is meant...

See Answer

Q: Define the following terms: a. The law of one

Define the following terms: a. The law of one price. b. Absolute purchasing power parity. c. Relative purchasing power parity.

See Answer

Q: Explain how a nominal effective exchange rate index is constructed.

Explain how a nominal effective exchange rate index is constructed.

See Answer

Q: What are the main advantages that Trident gains by developing a multinational

What are the main advantages that Trident gains by developing a multinational presence?

See Answer

Q: A newspaper shows the following prices for the previous day’s trading in

A newspaper shows the following prices for the previous day’s trading in the U.S. dollar-euro currency futures: What do the above terms indicate?

See Answer

Q: Give an example of a transaction exposure that arises from borrowing in

Give an example of a transaction exposure that arises from borrowing in a foreign currency.

See Answer

Q: What is the basic difference between a put on British pounds sterling

What is the basic difference between a put on British pounds sterling and a call on sterling?

See Answer

Q: You read that exchange-traded American call options on pounds sterling

You read that exchange-traded American call options on pounds sterling having a strike price of 1.460 and a maturity of next March are now quoted at 3.67. What does this mean if you are a potential bu...

See Answer

Q: What happens to the premium you paid for the above option in

What happens to the premium you paid for the above option in the event you decide to let the option expire unexercised? What happens to this amount in the event you decide to exercise the option?

See Answer

Q: You have the same information as in question 4 above, except

You have the same information as in question 4 above, except that the pricing is for a European option. What is different? Information from Question 4: A newspaper shows the following prices for the...

See Answer

Q: Explain why infrastructure strengths have helped to offset the large BOP deficits

Explain why infrastructure strengths have helped to offset the large BOP deficits on current account in the United States.

See Answer

Q: The emerging market crises of 199–2002 were worsened because of

The emerging market crises of 199–2002 were worsened because of rampant speculation. Do speculators cause such a crisis, or do they simply respond to market signals of weakness? How can a government m...

See Answer

Q: Swings in foreign direct investment flows into and out of emerging markets

Swings in foreign direct investment flows into and out of emerging markets contribute to exchange rate volatility. Describe one concrete historical example of this phenomenon during the last 10 years....

See Answer

Q: What were the main causes of Thailand’s crisis of 1997? What

What were the main causes of Thailand’s crisis of 1997? What lessons were learned and what steps were eventually taken to normalize Thailand’s economy?

See Answer

Q: What were the main causes of Russia’s crisis of 1998? What

What were the main causes of Russia’s crisis of 1998? What lessons were learned and what steps were taken to normalize Russia’s economy?

See Answer

Q: What were the main causes of Argentina’s crisis of 2001–2002

What were the main causes of Argentina’s crisis of 2001–2002? What lessons were learned and what steps were taken to normalize Argentina’s economy?

See Answer

Q: How would a high degree of leverage (debt/assets)

How would a high degree of leverage (debt/assets) be viewed by the shareholder wealth maximization model compared to the stakeholder wealth maximization model?

See Answer

Q: Explain why foreign currency cash balances do not cause transaction exposure.

Explain why foreign currency cash balances do not cause transaction exposure.

See Answer

Q: The value of an option is stated to be the sum of

The value of an option is stated to be the sum of its intrinsic value and its time value. Explain what is meant by these terms.

See Answer

Q: Ultimately a treasurer must choose among alternative strategies to manage transaction exposure

Ultimately a treasurer must choose among alternative strategies to manage transaction exposure. Explain the two main decision criteria that must be used.

See Answer

Q: Many MNEs have established transaction exposure risk management policies that mandate proportional

Many MNEs have established transaction exposure risk management policies that mandate proportional hedging. Explain and give an example of how proportional hedging can be implemented.

See Answer

Q: Define the following terms: a. Foreign exchange exposure.

Define the following terms: a. Foreign exchange exposure. b. The three types of foreign exchange exposure.

See Answer

Q: Define the following terms: a. Hedging. b

Define the following terms: a. Hedging. b. Currency risk.

See Answer

Q: Describe six arguments against a firm pursuing an active currency risk management

Describe six arguments against a firm pursuing an active currency risk management program.

See Answer

Q: Define and give an example of each of the following quotes:

Define and give an example of each of the following quotes: a. Bid quote. b. Ask quote.

See Answer

Q: What are the major differences between short-term and long-

What are the major differences between short-term and long-term forecasts for a fixed exchange rate versus a floating exchange rate?

See Answer

Q: Conglomerates are firms that have diversified into unrelated fields. How would

Conglomerates are firms that have diversified into unrelated fields. How would a policy of conglomeration be viewed by the shareholder wealth maximization model compared to the stakeholder wealth maxi...

See Answer

Q: How is risk defined in the shareholder wealth maximization model compared to

How is risk defined in the shareholder wealth maximization model compared to the stakeholder wealth maximization model?

See Answer

Q: How would stock options granted to a firm’s management and employees be

How would stock options granted to a firm’s management and employees be viewed by the shareholder wealth maximization model compared to the stakeholder wealth maximization model?

See Answer

Q: From the point of view of a borrowing corporation, what are

From the point of view of a borrowing corporation, what are credit and repricing risks? Explain steps a company might take to minimize both.

See Answer

Q: What alternative actions can shareholders take if they are dissatisfied with their

What alternative actions can shareholders take if they are dissatisfied with their company?

See Answer

Q: In many countries it is common for a firm to have two

In many countries it is common for a firm to have two or more classes of common stock with differential voting rights. In the United States the norm is for a firm to have one class of common stock wit...

See Answer

Q: Define the following terms: a. Corporate governance b

Define the following terms: a. Corporate governance b. The market for corporate control c. Agency theory d. Stakeholder capitalism

See Answer

Q: In recent years emerging market MNEs have improved their corporate governance policies

In recent years emerging market MNEs have improved their corporate governance policies and become more shareholder-friendly. What do you think is driving this phenomenon?

See Answer

Q: What have been the main causes of recent corporate governance failures in

What have been the main causes of recent corporate governance failures in the United States and Europe?

See Answer

Q: Which assets play the most critical role in linking the major institutions

Which assets play the most critical role in linking the major institutions which make up the global financial marketplace?

See Answer

Q: What are the key differences in the goals and motivations of family

What are the key differences in the goals and motivations of family ownership of a business as opposed to those of a widely held publicly traded business?

See Answer

Q: What is meant by the term “overshooting”? What causes it

What is meant by the term “overshooting”? What causes it and how is it corrected?

See Answer

Q: Do markets appear to be willing to pay for good governance?

Do markets appear to be willing to pay for good governance?

See Answer

Q: What are the primary principles behind corporate governance reform today? Are

What are the primary principles behind corporate governance reform today? Are these culturally specific in your opinion?

See Answer

Q: How can a business firm that has borrowed on a floating-

How can a business firm that has borrowed on a floating-rate basis use a forward rate agreement to reduce interest rate risk?

See Answer

Q: What should be the primary operational goal of an MNE?

What should be the primary operational goal of an MNE?

See Answer

Q: “Knowledge assets” are a firm’s intangible assets, the sources

“Knowledge assets” are a firm’s intangible assets, the sources and uses of its intellectual talent—its competitive advantage. What are some of the most important “knowledge assets” that create shareho...

See Answer

Q: In Germany and Scandinavia, among others, labor unions have representation

In Germany and Scandinavia, among others, labor unions have representation on boards of directors or supervisory boards. How might such union representation be viewed under the shareholder wealth maxi...

See Answer

Q: In an interlocking directorate members of the board of directors of one

In an interlocking directorate members of the board of directors of one firm also sit on the board of directors of other firms. How would interlocking directorates be viewed by the shareholder wealth...

See Answer

Q: A leveraged buyout is a financial strategy in which a group of

A leveraged buyout is a financial strategy in which a group of investors gain voting control of a firm and then liquidate its assets in order to repay the loans used to purchase the firm’s shares. How...

See Answer

Q: How does ownership alter the goals and governance of a business?

How does ownership alter the goals and governance of a business?

See Answer

Q: Why is this separation so critical to the understanding of how businesses

Why is this separation so critical to the understanding of how businesses are structured and led?

See Answer

Q: Explain the assumptions and objectives of the shareholder wealth maximization model.

Explain the assumptions and objectives of the shareholder wealth maximization model.

See Answer

Q: What is meant by the term “fundamental equilibrium path” for

What is meant by the term “fundamental equilibrium path” for a currency value? What is “noise”?

See Answer

Q: Explain the assumptions and objectives of the stakeholder wealth maximization model.

Explain the assumptions and objectives of the stakeholder wealth maximization model.

See Answer

Q: Why would one company with interest payments due in pounds sterling want

Why would one company with interest payments due in pounds sterling want to swap those payments for interest payments due in U.S. dollars?

See Answer

Q: Fixed exchange rate regimes are sometimes implemented through a currency board (

Fixed exchange rate regimes are sometimes implemented through a currency board (Hong Kong) or through dollarization (Ecuador). What is the difference between the two approaches?

See Answer

Q: High capital mobility is forcing emerging market nations to choose between free

High capital mobility is forcing emerging market nations to choose between free-floating regimes and currency board or dollarization regimes. What are the main outcomes of each of these regimes from t...

See Answer

Q: How did the Argentine currency board function from 1991 to January 2002

How did the Argentine currency board function from 1991 to January 2002, and why did it collapse?

See Answer

Q: On January 4, 1999, 11 member states of the European

On January 4, 1999, 11 member states of the European Union initiated the European Monetary Union (EMU) and established a single currency, the euro, which replaced the individual currencies of particip...

See Answer

Q: The United Kingdom, Denmark, and Sweden have chosen not to

The United Kingdom, Denmark, and Sweden have chosen not to adopt the euro, but rather to maintain their individual currencies. What are the motivations of each of these three countries, which are also...

See Answer

Q: The IMF was established by the Bretton Woods Agreement (1944).

The IMF was established by the Bretton Woods Agreement (1944). What were its original objectives?

See Answer

Q: What are Special Drawing Rights?

What are Special Drawing Rights?

See Answer

Q: What are the attributes of the ideal currency?

What are the attributes of the ideal currency?

See Answer

Q: Why did the fixed exchange rate regime of 1945–1973 eventually

Why did the fixed exchange rate regime of 1945–1973 eventually fail?

See Answer

Q: Explain how the asset market approach can be used to forecast future

Explain how the asset market approach can be used to forecast future spot exchange rates. How does the asset market approach differ from the BOP approach to forecasting?

See Answer

Q: What are the main phases that Trident passed through as it evolved

What are the main phases that Trident passed through as it evolved into a truly global firm? What are the advantages and disadvantages of each?

See Answer

Q: Define and explain the theory of comparative advantage.

Define and explain the theory of comparative advantage.

See Answer

Q: Under the gold standard all national governments promised to follow the “

Under the gold standard all national governments promised to follow the “rules of the game.” This meant defending a fixed exchange rate. What did this promise imply about a country’s money supply?

See Answer

Q: What are the advantages and disadvantages of fixed exchange rates?

What are the advantages and disadvantages of fixed exchange rates?

See Answer

Q: Explain what is meant by the term impossible trinity, and why

Explain what is meant by the term impossible trinity, and why it is true.

See Answer

Q: What are the main component accounts of the current account? Give

What are the main component accounts of the current account? Give one debit and one credit example for each component account for the United States.

See Answer

Q: What is the difference between a direct foreign investment and a portfolio

What is the difference between a direct foreign investment and a portfolio foreign investment? Give an example of each. Which type of investment is a multinational industrial company more likely to ma...

See Answer

Q: What are the main components of the financial accounts? Give one

What are the main components of the financial accounts? Give one debit and one credit example for each component account for the United States.

See Answer

Q: Classify the following as a transaction reported in a sub-component

Classify the following as a transaction reported in a sub-component of the current account, or the capital and financial accounts of the two countries involved: a. A U.S. food chain imports wine from...

See Answer

Q: What are the main summary statements of the balance of payments accounts

What are the main summary statements of the balance of payments accounts, and what do they measure?

See Answer

Q: Where in the balance of payments accounts do the flows of “

Where in the balance of payments accounts do the flows of “laundered” money by drug dealers and international terrorist organizations appear?

See Answer

Q: How does organized exchange trading in swaps remove any risk that the

How does organized exchange trading in swaps remove any risk that the counterparty in a swap agreement will not complete the agreement?

See Answer

Q: Explain how technical analysis can be used to forecast future spot exchange

Explain how technical analysis can be used to forecast future spot exchange rates. How does technical analysis differ from the BOP and asset market approaches to forecasting?

See Answer

Q: Identify the correct BOP account for each of the following transactions.

Identify the correct BOP account for each of the following transactions. a. A German-based pension fund buys U.S. government 30-year bonds for its investment portfolio. b. Scandinavian Airlines Syste...

See Answer

Q: The key to understanding most theories is what they say and what

The key to understanding most theories is what they say and what they don’t. What are four or five key limitations to the theory of comparative advantage?

See Answer

Q: Business managers and investors need BOP data to anticipate changes in host

Business managers and investors need BOP data to anticipate changes in host country economic policies that might be driven by BOP events. From the perspective of business managers and investors, list...

See Answer

Q: What are the two main types of economic activity measured by a

What are the two main types of economic activity measured by a country’s BOP?

See Answer

Q: Why does the BOP always “balance”?

Why does the BOP always “balance”?

See Answer

Q: Why were LIBOR rates so much higher than Treasury yields in 2007

Why were LIBOR rates so much higher than Treasury yields in 2007 and 2008? What is needed to return LIBOR rates to the lower, more stable levels of the past?

See Answer

Q: What were the three key elements of the package used by the

What were the three key elements of the package used by the U.S. government to resolve the 2008–2009 credit crisis?

See Answer

Q: Why are the sovereign debtors of the eurozone considered to have a

Why are the sovereign debtors of the eurozone considered to have a problem that is different from these of any other heavily indebted country, like the United States?

See Answer

Q: What is the European Financial Stability Facility (EFSF), and what

What is the European Financial Stability Facility (EFSF), and what role might it play in the resolution of the eurozone debt crisis?

See Answer

Q: Explain the difference between foreign currency options and futures and when either

Explain the difference between foreign currency options and futures and when either might be most appropriately used.

See Answer

Q: Why has the case of Portugal been termed a “case of

Why has the case of Portugal been termed a “case of contagion” rather than a sovereign debt crisis?

See Answer

Q: Numerous exchange rate forecasting services exist. Trident’s CFO Maria Gonzalez is

Numerous exchange rate forecasting services exist. Trident’s CFO Maria Gonzalez is considering whether to subscribe to one of these services at a cost of $20,000 per year. The price includes online ac...

See Answer

Q: What are the three primary methods that might be used individually or

What are the three primary methods that might be used individually or in combination to resolve the debt crisis?

See Answer

Q: What were the three major forces behind the credit crisis of 2007

What were the three major forces behind the credit crisis of 2007–2008?

See Answer

Q: After reading this chapter’s description of Trident’s globalization process, how would

After reading this chapter’s description of Trident’s globalization process, how would you explain the distinctions between international, multinational, and global companies?

See Answer

Q: What is a structured investment vehicle (SIV)?

What is a structured investment vehicle (SIV)?

See Answer

Q: What is a credit default swap (CDS)?

What is a credit default swap (CDS)?

See Answer

Q: Why do you believe it is important for many of the world’s

Why do you believe it is important for many of the world’s largest commercial and investment banks to be considered on-the-run in the interbank market?

See Answer

Q: What are the three major functions of the foreign exchange market?

What are the three major functions of the foreign exchange market?

See Answer

Q: At what point in the globalization process did Trident become a multinational

At what point in the globalization process did Trident become a multinational enterprise (MNE)?

See Answer

Q: What formula is used to convert a nominal effective exchange rate index

What formula is used to convert a nominal effective exchange rate index into a real effective exchange rate index?

See Answer

Q: Exhibit 7.3 compares the real effective exchange rates for Japan

Exhibit 7.3 compares the real effective exchange rates for Japan, the United States, and the Euro area. If the comparative real effective exchange rate was the main determinant, does Japan or the Unit...

See Answer

Q: Incomplete exchange rate pass-through is one reason that a country’s

Incomplete exchange rate pass-through is one reason that a country’s real effective exchange rate can deviate for lengthy periods from its purchasing power equilibrium level of 100. What is meant by t...

See Answer

Q: For each of the foreign exchange market participants, identify their motive

For each of the foreign exchange market participants, identify their motive for buying or selling foreign exchange.

See Answer

Q: Explain the meaning and probable significance for international business of the following

Explain the meaning and probable significance for international business of the following contract specifications: • Specific-sized contract • Standard method of stating exchange rates • Standard matu...

See Answer

Q: Define each of the following types of foreign exchange transactions:

Define each of the following types of foreign exchange transactions: a. Spot. b. Outright forward. c. Forward-forward swap.

See Answer

Q: With reference to foreign exchange turnover in 2010: a.

With reference to foreign exchange turnover in 2010: a. Rank the relative size of spot, forwards, and swaps as of 2010. b. Rank the five most important geographic locations for foreign exchange turno...

See Answer

Q: Explain the meaning of “cross-rate consistency” as used

Explain the meaning of “cross-rate consistency” as used by MNEs. How do MNEs use a check of cross-rate consistency in practice?

See Answer

Q: With reference to interbank quotations, what is the difference between American

With reference to interbank quotations, what is the difference between American terms and European terms?

See Answer

Q: Define and give an example of the following: a.

Define and give an example of the following: a. Direct quote between the U.S. dollar and the Mexican peso, where the United States is designated as the home country. b. Indirect quote between the Japa...

See Answer

Q: Define the following terms: • Foreign exchange market •

Define the following terms: • Foreign exchange market • Foreign exchange transaction •Foreign exchange

See Answer

Q: What does the word translation mean? Why is translation exposure sometimes

What does the word translation mean? Why is translation exposure sometimes called accounting exposure?

See Answer

Q: Explain how back-to-back loans can hedge foreign exchange

Explain how back-to-back loans can hedge foreign exchange operating exposure.

See Answer

Q: What is real option analysis? How is it a better method

What is real option analysis? How is it a better method of making investment decisions than using traditional capital budgeting analysis?

See Answer

Q: What is a foreign tax credit? Why do countries give credit

What is a foreign tax credit? Why do countries give credit for taxes paid on foreign-source income?

See Answer

Q: a. What is a value-added tax? b

a. What is a value-added tax? b. Although the value-added tax has been proposed numerous times, the Unites States has never adopted it. Why do you think the United States is so negative on it, when t...

See Answer

Q: What is a transfer price, and can a government regulate it

What is a transfer price, and can a government regulate it? What difficulties and motives does a parent multinational firm face in setting transfer prices?

See Answer

Q: Subsidiary Alpha in Country Able faces a 40% income tax rate

Subsidiary Alpha in Country Able faces a 40% income tax rate. Subsidiary Beta in Country Baker faces only a 20% income tax rate. Presently each subsidiary imports from the other an amount of goods and...

See Answer

Q: Section 482 of the U.S. Internal Revenue Code specifies

Section 482 of the U.S. Internal Revenue Code specifies use of a “correct” transfer price, and the burden of proof that the transfer price is “correct” lies with the company. What guidelines exist for...

See Answer

Q: What are the major differences in translating assets between the current rate

What are the major differences in translating assets between the current rate method and the temporal method?

See Answer

Q: Answer the following questions: a. What is meant by

Answer the following questions: a. What is meant by the term “tax haven”? b. What are the desired characteristics for a country if it expects to be used as a tax haven? c. What are the advantages lea...

See Answer

Q: How do tax treaties affect the operations and structure of MNEs?

How do tax treaties affect the operations and structure of MNEs?

See Answer

Q: Why do the U.S. tax authorities tax passive income

Why do the U.S. tax authorities tax passive income generated offshore differently from active income?

See Answer

Q: What is meant by the term “tax morality”?

What is meant by the term “tax morality”?

See Answer

Q: Capital budgeting for a foreign project uses the same theoretical framework as

Capital budgeting for a foreign project uses the same theoretical framework as does domestic capital budgeting. What are the basic steps in domestic capital budgeting?

See Answer

Q: a. Define the term “tax neutrality.” b.

a. Define the term “tax neutrality.” b. What is the difference between domestic neutrality and foreign neutrality? c. What are a country’s objectives when determining tax policy on foreign-source inc...

See Answer

Q: Nations typically structure their tax systems along one of two basic approaches

Nations typically structure their tax systems along one of two basic approaches: the worldwide approach or the territorial approach. Explain these two approaches and how they differ from each other.

See Answer

Q: Define in words (without graphics) how the optimal domestic portfolio

Define in words (without graphics) how the optimal domestic portfolio is constructed.

See Answer

Q: If the primary benefit of portfolio diversification is risk reduction, is

If the primary benefit of portfolio diversification is risk reduction, is the investor always better off choosing the portfolio with the lowest expected risk?

See Answer

Q: When asked why they do not internationally diversify their portfolios, answer

When asked why they do not internationally diversify their portfolios, answer that “the risks are not worth the expected returns.” Using the theory of international diversification, how would you eval...

See Answer

Q: The benefits of portfolio construction, domestically or internationally, arise from

The benefits of portfolio construction, domestically or internationally, arise from the lack of correlation among assets and markets. The increasing globalization of business is expected to change the...

See Answer

Q: The key to managing operating exposure at the strategic level is for

The key to managing operating exposure at the strategic level is for management to recognize a disequilibrium in parity conditions when it occurs, and to be prepositioned to react most appropriately....

See Answer

Q: Conceptually, how do the Sharpe and Treynor performance measures define risk

Conceptually, how do the Sharpe and Treynor performance measures define risk differently? Which do you believe is a more useful measure in an internationally diversified portfolio?

See Answer

Q: As the newest member of the asset allocation team in your firm

As the newest member of the asset allocation team in your firm, you constantly find yourself being quizzed by your fellow group members. The topic is international diversification. One analyst asks yo...

See Answer

Q: Firms with operations and assets across the globe, true MNEs,

Firms with operations and assets across the globe, true MNEs, are in many ways as international in composition as the most internationally diversified portfolio of unrelated securities. Why do investo...

See Answer

Q: Explain how currency swaps can hedge foreign exchange operating exposure. What

Explain how currency swaps can hedge foreign exchange operating exposure. What are the accounting advantages of currency swaps?

See Answer

Q: When you are constructing your portfolio, you know you want to

When you are constructing your portfolio, you know you want to include Cementos de Mexico (Mexico), but you cannot decide whether you wish to hold it in the form of ADRs traded on the NYSE, or directl...

See Answer

Q: How does the diversification of a portfolio change its expected returns and

How does the diversification of a portfolio change its expected returns and expected risks? Is this in principle any different for internationally diversified portfolios?

See Answer

Q: What types of risk are present in a diversified portfolio? Which

What types of risk are present in a diversified portfolio? Which type of risk remains after the portfolio has been diversified?

See Answer

Q: How, according to portfolio theory, is the risk of the

How, according to portfolio theory, is the risk of the portfolio measured exactly?

See Answer

Q: The currency risk associated with international diversification is a serious concern for

The currency risk associated with international diversification is a serious concern for portfolio managers. Is it possible for currency risk ever to benefit the portfolio’s return?

See Answer

Q: The decision about where to invest abroad is influenced by behavioral factors

The decision about where to invest abroad is influenced by behavioral factors. a. Explain the behavioral approach to FDI. b. Explain the international network theory explanation of FDI.

See Answer

Q: What are the advantages and disadvantages of limiting a firm’s activities to

What are the advantages and disadvantages of limiting a firm’s activities to exporting compared to producing abroad?

See Answer

Q: What is hyperinflation, and what are the consequences for translating foreign

What is hyperinflation, and what are the consequences for translating foreign financial statements?

See Answer

Q: What are the advantages and disadvantages of licensing and management contracts compared

What are the advantages and disadvantages of licensing and management contracts compared to producing abroad?

See Answer

Q: What are the advantages and disadvantages of forming a joint venture to

What are the advantages and disadvantages of forming a joint venture to serve a foreign market compared to serving that market with a wholly owned production subsidiary?

See Answer

Q: Capital budgeting for a foreign project is considerably more complex than the

Capital budgeting for a foreign project is considerably more complex than the domestic case. What are the factors that add complexity?

See Answer

Q: What are the advantages and disadvantages of serving a foreign market through

What are the advantages and disadvantages of serving a foreign market through a greenfield foreign direct investment compared to an acquisition of a local firm in the target market?

See Answer

Q: The term “cross-border strategic alliance” conveys different meanings

The term “cross-border strategic alliance” conveys different meanings to different observers. What are the meanings?

See Answer

Q: Answer the following questions: a. What is meant

Answer the following questions: a. What is meant by the term “governance risk”? b. What is the most important type of governance risk?

See Answer

Q: An investment agreement spells out specific rights and responsibilities of both the

An investment agreement spells out specific rights and responsibilities of both the foreign firm and the host government. What are the main financial policies that should be included in an investment...

See Answer

Q: Answer the following questions: a. What is OPIC?

Answer the following questions: a. What is OPIC? b. What types of political risks can OPIC insure against?

See Answer

Q: The following operating strategies, among others, are expected to reduce

The following operating strategies, among others, are expected to reduce damage from political risk. Explain each, and how it reduces damage. a. Local sourcing. b. Facility location. c. Control of tec...

See Answer

Q: Define the following terms: a. Transfer risk.

Define the following terms: a. Transfer risk. b. Blocked funds.

See Answer

Q: Identify and explain the main types of cultural and institutional risks,

Identify and explain the main types of cultural and institutional risks, except protectionism.

See Answer

Q: What is the primary difference between losses from transaction exposure, operating

What is the primary difference between losses from transaction exposure, operating exposure, and translation exposure?

See Answer

Q: Respond to the following: a. Define protectionism and identify

Respond to the following: a. Define protectionism and identify the industries that are typically protected. b. Explain the “infant industry” argument for protectionism.

See Answer

Q: a. Why should a foreign project be evaluated from both a

a. Why should a foreign project be evaluated from both a project and a parent viewpoint? b. Which viewpoint, project or parent, gives results closer to the traditional meaning of net present value in...

See Answer

Q: a. What are the traditional methods for countries to implement protectionism

a. What are the traditional methods for countries to implement protectionism? b. What are some typical non-tariff barriers to trade? c. How can MNEs overcome host country protectionism?

See Answer

Q: The United States has a law prohibiting U.S. firms

The United States has a law prohibiting U.S. firms from bribing foreign officials and business persons, even in countries where bribery is a normal practice. Some U.S. firms claim this places the Unit...

See Answer

Q: As a firm evolves from purely domestic into a true multinational enterprise

As a firm evolves from purely domestic into a true multinational enterprise, it must consider (1) its competitive advantages, (2) its production location, (3) the type of control it wants to have over...

See Answer

Q: What is the essence of the theory of comparative advantage?

What is the essence of the theory of comparative advantage?

See Answer

Q: MNEs strive to take advantage of market imperfections in national markets for

MNEs strive to take advantage of market imperfections in national markets for products, factors of production, and financial assets. Large international firms are better able to exploit such imperfect...

See Answer

Q: In deciding whether to invest abroad, management must first determine whether

In deciding whether to invest abroad, management must first determine whether the firm has a sustainable competitive advantage that enables it to compete effectively in the home market. What are the n...

See Answer

Q: Explain briefly how economies of scale and scope can be developed in

Explain briefly how economies of scale and scope can be developed in production, marketing, finance, research and development, transportation, and purchasing.

See Answer

Q: A strongly competitive home market can sharpen a firm’s competitive advantage relative

A strongly competitive home market can sharpen a firm’s competitive advantage relative to firms located in less competitive markets. Explain what is meant by the “competitive advantage of nations.”

See Answer

Q: The OLI Paradigm is an attempt to create an overall framework to

The OLI Paradigm is an attempt to create an overall framework to explain why MNEs choose FDI, rather than serve foreign markets through alternative modes. Explain what is meant by the O, the L, and th...

See Answer

Q: In the context of unbundling cash flows from subsidiary to parent,

In the context of unbundling cash flows from subsidiary to parent, why might a host government be more lenient in its treatment of fees than its treatment of dividends? What difference does it make to...

See Answer

Q: The operating cycle of a firm, domestic or multinational, consists

The operating cycle of a firm, domestic or multinational, consists of the following four time periods. For each of these periods, explain whether a cash outflow or a cash inflow is associated with the...

See Answer

Q: Exhibit 19.1 shows the accounts payable period to be longer

Exhibit 19.1 shows the accounts payable period to be longer than the inventory period. Could this be otherwise, and what would be the cash implications?

See Answer

Q: As a financial manager, would you prefer that the accounts payable

As a financial manager, would you prefer that the accounts payable period end before, at the same time, or after the beginning of the accounts receivable period? Explain.

See Answer

Q: Assuming the flow illustrated in Exhibit 19.1, where does

Assuming the flow illustrated in Exhibit 19.1, where does transaction exposure begin and end if inputs are purchased with one currency at t1, and proceeds from the sale are received at t5? Is there mo...

See Answer

Q: Financial strategies are directly related to the OLI Paradigm. a

Financial strategies are directly related to the OLI Paradigm. a. Explain how proactive financial strategies are related to OLI. b. Explain how reactive financial strategies are related to OLI.

See Answer

Q: Is any operating exposure created during the course of a firm’s operating

Is any operating exposure created during the course of a firm’s operating cycle?

See Answer

Q: Is any accounting exposure created during the course of a firm’s operating

Is any accounting exposure created during the course of a firm’s operating cycle?

See Answer

Q: Assume a firm purchases inventory with one foreign currency and sells it

Assume a firm purchases inventory with one foreign currency and sells it for another foreign currency, neither currency being the home currency of the parent or subsidiary where the manufacturing proc...

See Answer

Q: Merck is an MNE that has undertaken contractual hedging of its operating

Merck is an MNE that has undertaken contractual hedging of its operating exposure. a. How do they accomplish this task? b. What assumptions do they make in order to justify contractual hedging of the...

See Answer

Q: Roberts and Sons, Inc., of Great Britain has just purchased

Roberts and Sons, Inc., of Great Britain has just purchased inventory items costing kroner 1,000,000 from a Swedish supplier. The supplier has quoted terms 3/15, net 45. Under what conditions might Ro...

See Answer

Q: Japanese industry is often praised for its “just-in-

Japanese industry is often praised for its “just-in-time” inventory practice between industrial buyers and industrial sellers. In the context of the “Day’s Receivables” turnover in Exhibit 19.5, what...

See Answer

Q: Why might the time lag for intramultinational firm accounts receivable and payable

Why might the time lag for intramultinational firm accounts receivable and payable (that is, all received or paid to a parent or sister subsidiary) differ substantially from the time lags reported for...

See Answer

Q: Merlin Corporation of the United States imports raw material from Indonesia on

Merlin Corporation of the United States imports raw material from Indonesia on terms of 2/10, net 30. Merlin expects a 36% devaluation of the Indonesian rupiah at any moment. Should Merlin take the di...

See Answer

Q: What are the advantages of a free trade zone? Are there

What are the advantages of a free trade zone? Are there any disadvantages?

See Answer

Q: Explain the difference between the “transaction motive” and the “

Explain the difference between the “transaction motive” and the “precautionary motive” for holding cash.

See Answer

Q: Capital projects provide both operating cash flows and financial cash flows.

Capital projects provide both operating cash flows and financial cash flows. Why are operating cash flows preferred for domestic capital budgeting, but financial cash flows given major consideration i...

See Answer

Q: The operating cash cycle of a multinational firm goes from cash collection

The operating cash cycle of a multinational firm goes from cash collection from customers, cash holding for anticipated transaction needs (the transaction motive for holding cash), possible cash repos...

See Answer

Q: Electro-Beam Company generates and disburses cash in the currencies of

Electro-Beam Company generates and disburses cash in the currencies of four countries, Singapore, Malaysia, Thailand, and Vietnam. What would be the characteristics you might consider if charged with...

See Answer

Q: During the era of the French franc, France imposed a rule

During the era of the French franc, France imposed a rule on its banks and subsidiaries of international companies operating in France that precluded those subsidiaries from netting cash flow obligati...

See Answer

Q: What is the difference between a foreign branch and a foreign subsidiary

What is the difference between a foreign branch and a foreign subsidiary of a home-country bank?

See Answer

Q: Answer the following: a. How can an MNE

Answer the following: a. How can an MNE diversify operations? b. How can an MNE diversify financing?

See Answer

Q: Each of the following factors is sometimes a constraint on the free

Each of the following factors is sometimes a constraint on the free movement of funds internationally. Why would a government impose such a constraint? How might the management of a multinational argu...

See Answer

Q: What does this term mean? Why would unbundling be needed for

What does this term mean? Why would unbundling be needed for international cash flows from foreign subsidiaries, but not for domestic cash flows between related domestic subsidiaries and their parent?...

See Answer

Q: In the context of unbundling cash flows from subsidiary to parent,

In the context of unbundling cash flows from subsidiary to parent, explain how each of the following creates a conduit. What are the tax consequences of each? a. Imports of components from the parent....

See Answer

Q: Subsidiary Alpha in Country Able faces a 40% income tax rate

Subsidiary Alpha in Country Able faces a 40% income tax rate. Subsidiary Beta in Country Baker faces only a 20% income tax rate. At present each subsidiary imports from the other an amount of goods an...

See Answer

Q: What are the differences between a “license fee” and a

What are the differences between a “license fee” and a “royalty fee”? Do you think license and royalty fees should be covered by the tax rules that regulate transfer pricing? Why?

See Answer

Q: Should the anticipated internal rate of return (IRR) for a

Should the anticipated internal rate of return (IRR) for a proposed foreign project be compared to (1) alternative home country proposals, (2) returns earned by local companies in the same industry...

See Answer

Q: What is the difference between a “management fee”, a “

What is the difference between a “management fee”, a “technical assistance fee”, and a “license fee for patent usage”? Should they be treated differently for income tax purposes?

See Answer

Q: What methods might the U.S. Internal Revenue Service use

What methods might the U.S. Internal Revenue Service use to determine whether allocations of distributed overhead are being fairly allocated to foreign subsidiaries?

See Answer

Q: Explain the difference between a letter of credit (L/C

Explain the difference between a letter of credit (L/C) and a draft. How are they linked?

See Answer

Q: What is the major difference between “currency risk” and “

What is the major difference between “currency risk” and “risk of noncompletion”? How are these risks handled in a typical international trade transaction?

See Answer

Q: Identify each party to a letter of credit (L/C

Identify each party to a letter of credit (L/C), and indicate its responsibility.

See Answer

Q: Define the following terms: a. Operating exposure

Define the following terms: a. Operating exposure b. Economic exposure c. Competitive exposure

See Answer

Q: Why would an exporter insist on a confirmed letter of credit?

Why would an exporter insist on a confirmed letter of credit?

See Answer

Q: List the steps involved in the export of computer hard disk drives

List the steps involved in the export of computer hard disk drives from Penang, Malaysia, to San Jose, California, using an unconfirmed letter of credit authorizing payment on sight.

See Answer

Q: List the steps involved in the export of lumber from Portland,

List the steps involved in the export of lumber from Portland, Oregon, to Yokohama, Japan, using a confirmed letter of credit, payment to be made in 120 days.

See Answer

Q: Inca Breweries of Lima, Peru, has received an order for

Inca Breweries of Lima, Peru, has received an order for 10,000 cartons of beer from Alicante Importers of Alicante, Spain. The beer will be exported to Spain under the terms of a letter of credit issu...

See Answer

Q: In the context of evaluating foreign investment proposals, how should a

In the context of evaluating foreign investment proposals, how should a multinational firm evaluate cash flows in the host foreign country that are blocked from being repatriated to the firm’s home co...

See Answer

Q: Swishing Shoe Company of Durham, North Carolina, has received an

Swishing Shoe Company of Durham, North Carolina, has received an order for 50,000 cartons of athletic shoes from Southampton Footware, Ltd., of Great Britain, payment to be in British pounds sterling....

See Answer

Q: Assume that Great Britain charges a duty of 10% on shoes

Assume that Great Britain charges a duty of 10% on shoes imported into the United Kingdom. Swishing Shoe Company, in question 11, discovers that it can manufacture shoes in Ireland and import them int...

See Answer

Q: Various governments have established agencies to insure against nonpayment for exports and

Various governments have established agencies to insure against nonpayment for exports and/or to provide export credit. This shifts credit risk away from private banks and to the citizen taxpayers of...

See Answer

Q: Why might different documentation be used for an export to a nonaffiliated

Why might different documentation be used for an export to a nonaffiliated foreign buyer who is a new customer, as compared with an export to a nonaffiliated foreign buyer to whom the exporter has bee...

See Answer

Q: For what reason might an exporter use standard international trade documentation (

For what reason might an exporter use standard international trade documentation (letter of credit, draft, order bill of lading) on an intrafirm export to its parent or sister subsidiary?

See Answer

Q: What reasons can you give for the observation that intrafirm trade is

What reasons can you give for the observation that intrafirm trade is now greater than trade between nonaffiliated exporters and importers?

See Answer

Q: Answer the following: a. Why do unexpected exchange

Answer the following: a. Why do unexpected exchange rate changes contribute to operating exposure, but expected exchange rate changes do not? b. Explain the time horizons used to analyze unexpected ch...

See Answer

Q: Novo believed that the Danish capital market was segmented from world capital

Novo believed that the Danish capital market was segmented from world capital markets. Explain the six characteristics of the Danish equity market that were responsible for its segmentation.

See Answer

Q: a. What was Novo’s strategy to internationalize its cost of capital

a. What was Novo’s strategy to internationalize its cost of capital? b. What is the evidence that Novo’s strategy succeeded?

See Answer

Q: It has been suggested that firms located in illiquid and segmented emerging

It has been suggested that firms located in illiquid and segmented emerging markets could follow Novo’s proactive strategy to internationalize their own cost of capital. What are the preconditions tha...

See Answer

Q: Theoretically MNEs should be in a better position than their domestic counterparts

Theoretically MNEs should be in a better position than their domestic counterparts to support higher debt ratios, because their cash flows are diversified internationally. However, recent empirical st...

See Answer

Q: The “Riddle” The riddle is an attempt to explain under

The “Riddle” The riddle is an attempt to explain under what conditions an MNE would have a higher or lower debt ratio and beta than its domestic counterpart. Explain and diagram these conditions.

See Answer

Q: Global integration has given many firms access to new and cheaper sources

Global integration has given many firms access to new and cheaper sources of funds beyond those available in their home markets. What are the dimensions of a strategy to capture this lower cost and gr...

See Answer

Q: What are the differences in the cash flows used in a project

What are the differences in the cash flows used in a project point of view analysis and a parent point of view analysis?

See Answer

Q: Define the following terms: a. Systematic risk.

Define the following terms: a. Systematic risk. b. Beta (in the Capital Asset Pricing Model).

See Answer

Q: Answer the following; a. What is an equity

Answer the following; a. What is an equity risk premium? b. What is the difference between calculating an equity risk premium using arithmetic returns compared to using geometric returns? c. In Exhi...

See Answer

Q: What is the central problem involved in consolidating the financial statements of

What is the central problem involved in consolidating the financial statements of a foreign subsidiary?

See Answer

Q: Apart from improving liquidity and escaping from a segmented home market,

Apart from improving liquidity and escaping from a segmented home market, why might emerging market MNEs further lower their cost of capital by listing and selling equity abroad?

See Answer

Q: Define what is meant by a “Euroequity public share issue.”

Define what is meant by a “Euroequity public share issue.”

See Answer

Q: Answer the following questions: a. What is SEC Rule

Answer the following questions: a. What is SEC Rule 144A? b. Why might a foreign firm choose to sell its equity in the United States under SEC Rule 144A?

See Answer

Q: a. What is a private equity fund? b.

a. What is a private equity fund? b. How do they differ from traditional venture capital firms? c. How do private equity funds raise their own capital, and how does this action give them a competitive...

See Answer

Q: What, in simple wording, is the objective sought by finding

What, in simple wording, is the objective sought by finding an optimal capital structure?

See Answer

Q: Answer the following questions: a. What is the “

Answer the following questions: a. What is the “cost of debt” and how is it determined? b. What is the “cost of equity” and how is it determined?

See Answer

Q: As debt in a firm’s capital structure is increased from no debt

As debt in a firm’s capital structure is increased from no debt to a significant proportion of debt (say, 60%), what tends to happen to the cost of debt, to the cost of equity, and to the overall weig...

See Answer

Q: How does the availability of capital influence the theory of optimal capital

How does the availability of capital influence the theory of optimal capital structure for a multinational enterprise?

See Answer

Q: How is foreign exchange risk sensitivity factored into the capital budgeting analysis

How is foreign exchange risk sensitivity factored into the capital budgeting analysis of a foreign project?

See Answer

Q: Define “marginal weighted average cost of capital”.

Define “marginal weighted average cost of capital”.

See Answer

Q: If a multinational firm is able to diversify its sources of cash

If a multinational firm is able to diversify its sources of cash inflow so as to receive those flows from several countries and in several currencies, do you think that tends to increase or decrease i...

See Answer

Q: Both domestic and international portfolio managers are asset allocators. a

Both domestic and international portfolio managers are asset allocators. a. What is their portfolio management objective? b. What is the main advantage that international portfolio managers have comp...

See Answer

Q: Many firms in many countries every year borrow at nominal costs which

Many firms in many countries every year borrow at nominal costs which prove to be very different after the fact. For example, not too long ago Deutsche Bank borrowed funds at a nominal cost of 9.59% p...

See Answer

Q: What is the difference between a self-sustaining foreign subsidiary and

What is the difference between a self-sustaining foreign subsidiary and an integrated foreign subsidiary?

See Answer

Q: Should foreign subsidiaries of multinational firms conform to the capital structure norms

Should foreign subsidiaries of multinational firms conform to the capital structure norms of the host country, or to the norms of their parent’s country? Discuss.

See Answer

Q: What is the difference between “internal” financing and “external

What is the difference between “internal” financing and “external” financing for a subsidiary? List three types of internal financing and three types of external financing available to a foreign subsi...

See Answer

Q: Bank borrowing has long been the manner by which corporations and governments

Bank borrowing has long been the manner by which corporations and governments borrowed funds for short periods. What then, is the advantage over bank borrowing for each of the following? a. Syndicated...

See Answer

Q: What is the difference between a eurobond and a foreign bond,

What is the difference between a eurobond and a foreign bond, and why do two types of international bonds exist?

See Answer

Q: Exhibit 14.1 illustrates alternative paths to globalizing the cost and

Exhibit 14.1 illustrates alternative paths to globalizing the cost and availability of capital. Identify the specific steps in Exhibit 14.1 that were taken by Novo Industry (Chapter 13) in chronologic...

See Answer

Q: Define the following terms: a. ADRs. b

Define the following terms: a. ADRs. b. GRSs. c. Sponsored depositary receipts. d. Unsponsored depositary receipts.

See Answer

Q: How is expropriation risk factored into the capital budgeting analysis of a

How is expropriation risk factored into the capital budgeting analysis of a foreign project?

See Answer

Q: Distinguish between the three levels of commitment for ADRs traded in the

Distinguish between the three levels of commitment for ADRs traded in the United States.

See Answer

Q: Portfolio asset allocation can be accomplished along many dimensions, depending on

Portfolio asset allocation can be accomplished along many dimensions, depending on the investment objective of the portfolio manager. Identify the various dimensions.

See Answer

Q: Give five reasons why a firm might cross-list and sell

Give five reasons why a firm might cross-list and sell its shares on a very liquid stock exchange.

See Answer

Q: What are the main reasons causing U.S. firms to

What are the main reasons causing U.S. firms to cross-list abroad?

See Answer

Q: What are the main barriers to cross-listing abroad?

What are the main barriers to cross-listing abroad?

See Answer

Q: What is a functional currency? What is a non-functional

What is a functional currency? What is a non-functional currency?

See Answer

Q: What are five alternative instruments that can be used to source equity

What are five alternative instruments that can be used to source equity in global markets?

See Answer

Q: Answer the following questions: a. Define what is meant

Answer the following questions: a. Define what is meant by a “directed public share issue”. b. Why did Novo choose to make a $61 million directed public share issue in the United States in 1981?

See Answer

Q: a. What is meant by the term “tax deferral”?

a. What is meant by the term “tax deferral”? b. Why do countries allow tax deferral on foreign-source income?

See Answer

Q: a. What is a bilateral tax treaty? b.

a. What is a bilateral tax treaty? b. What is the purpose of a bilateral tax treaty? c. What policies do most tax treaties cover?

See Answer

Q: Taxes are classified on the basis of whether they are applied directly

Taxes are classified on the basis of whether they are applied directly to income, called direct taxes, or to some other measurable performance characteristic of the firm, called indirect taxes. Identi...

See Answer

Q: A foreign subsidiary does not have an independent cost of capital.

A foreign subsidiary does not have an independent cost of capital. However, in order to estimate the discount rate for a comparable host country firm, the analyst should try to calculate a hypothetica...

See Answer

Q: Answer the following questions: a. Define what is meant

Answer the following questions: a. Define what is meant by the term market liquidity. b. What are the main disadvantages for a firm to be located in an illiquid market? c. If a firm is limited to ra...

See Answer

Q: In the context of preparing consolidated financial statements, are the words

In the context of preparing consolidated financial statements, are the words translate and convert synonyms?

See Answer

Q: a. Define market segmentation. b. What are the

a. Define market segmentation. b. What are the six main causes of market segmentation? c. What are the main disadvantages for a firm to be located in a segmented market?

See Answer

Q: Why did Novo believe that its cost of capital was too high

Why did Novo believe that its cost of capital was too high compared to its competitors? Why did Novo’s relatively high cost of capital create a competitive disadvantage?

See Answer

Q: A stock currently sells for $32.00. A 6

A stock currently sells for $32.00. A 6-month call option with a strike of $30.00 has a premium of $4.29, and a 6-month put with the same strike has a premium of $2.64. Assume a 4% continuously compou...

See Answer

Q: Assuming a $10m investment in one stock, compute the 95

Assuming a $10m investment in one stock, compute the 95% and 99% VaR for stocks A and B over 1-day, 10-day, and 20-day horizons.

See Answer

Q: What is a collateralized debt obligation (CDO)?

What is a collateralized debt obligation (CDO)?

See Answer

Q: The newspaper reports that a given June Eurodollar future settled at 93

The newspaper reports that a given June Eurodollar future settled at 93.55. What was the annual yield? How many dollars does this represent?

See Answer

Q: A stock currently sells for $32.00. A 6

A stock currently sells for $32.00. A 6-month call option with a strike of $35.00 has a premium of $2.27. Assuming a 4% continuously compounded risk-free rate and a 6% continuous dividend yield, what...

See Answer

Q: Suppose you observe the following effective annual zero-coupon bond yields

Suppose you observe the following effective annual zero-coupon bond yields: 0.030 (1-year), 0.035 (2-year), 0.040 (3-year), 0.045 (4-year), 0.050 (5-year). For each maturity year compute the zero-coup...

See Answer

Q: Suppose the stock price is $35 and the continuously compounded interest

Suppose the stock price is $35 and the continuously compounded interest rate is 5%. a. What is the 6-month forward price, assuming dividends are zero? b. If the 6-month forward price is $35.50, what i...

See Answer

Q: Using the information in Table 8.9, what is the

Using the information in Table 8.9, what is the swap price of a 4-quarter oil swap with the first settlement occurring in the third quarter?

See Answer

Q: Using the zero-coupon bond prices and oil forward prices in

Using the zero-coupon bond prices and oil forward prices in Table 8.9, what is the price of an 8-period swap for which two barrels of oil are delivered in even-numbered quarters and one barrel of oil...

See Answer

Q: Using the assumptions in Tables 8.5 and 8.6

Using the assumptions in Tables 8.5 and 8.6, verify that equation (8.13) equals 6%. Equation (8.13)

See Answer

Q: Suppose that 1- and 2-year oil forward prices are

Suppose that 1- and 2-year oil forward prices are $22/barrel and $23/barrel. The 1-and 2-year interest rates are 6% and 6.5%. Show that the new 2-year swap price is $22.483.

See Answer

Q: Verify that the 1-year yield volatility of the 4-

Verify that the 1-year yield volatility of the 4-year zero-coupon bond price generated by the tree in Figure 25.5 is 0.14.

See Answer

Q: This problem builds on the previous problem using the same parameters,

This problem builds on the previous problem using the same parameters, only valuing a call option instead of a bond. Using Monte Carlo, simulate the Vasicek process for 3 years. For each simulation tr...

See Answer

Q: What is the price of a 3-year interest rate cap

What is the price of a 3-year interest rate cap with an 11.5% (effective annual) cap rate? For the first three problems, use the following information:

See Answer

Q: A stock currently sells for $32.00. A 6

A stock currently sells for $32.00. A 6-month call option with a strike of $30.00 has a premium of $4.29, and a 6-month put with the same strike has a premium of $2.64. Assume a 4% continuously compou...

See Answer

Q: Suppose the exchange rate is 0.95 $/=C, the

Suppose the exchange rate is 0.95 $/=C, the euro-denominated continuously compounded interest rate is 4%, the dollar-denominated continuously compounded interest rate is 6%, and the price of a 1-year...

See Answer

Q: Let S = $100, K = $105, r

Let S = $100, K = $105, r = 8%, T = 0.5, and δ = 0. Let u = 1.3, d = 0.8, and n =1. a. What are the premium, ∆, and B for a European call? b. What are the premium, ∆, and B for a European put?

See Answer

Q: Assuming a $10m investment in one stock, compute the 95

Assuming a $10m investment in one stock, compute the 95% and 99% VaR for stocks A and B over 1-day, 10-day, and 20-day horizons.

See Answer

Q: Using the same assumptions as in Example 26.3, compute

Using the same assumptions as in Example 26.3, compute VaR with and without the mean, assuming correlations of −1, −0.5, 0, 0.5, and 1. Is risk eliminated with a correlation of −1? If not, why not?

See Answer

Q: Consider the expression in equation (26.6). What is

Consider the expression in equation (26.6). What is the exact probability that, over a 1-day horizon, stock A will have a loss?

See Answer

Q: Suppose you observe the following 1-year implied forward rates:

Suppose you observe the following 1-year implied forward rates: 0.050000 (1- year), 0.034061 (2-year), 0.036012 (3-year), 0.024092 (4-year), 0.001470 (5-year). For each maturity year compute the zero-...

See Answer

Q: Suppose you observe the following continuously compounded zero-coupon bond yields

Suppose you observe the following continuously compounded zero-coupon bond yields: 0.06766 (1-year), 0.05827 (2-year), 0.04879 (3-year), 0.04402 (4-year), 0.03922 (5-year). For each maturity year comp...

See Answer

Q: If all national markets have market risk, is all market risk

If all national markets have market risk, is all market risk the same?

See Answer

Q: Explain how the concept of macroeconomic uncertainty expands the scope of analyzing

Explain how the concept of macroeconomic uncertainty expands the scope of analyzing operating exposure.

See Answer

Q: The objective of both operating and transaction exposure management is to anticipate

The objective of both operating and transaction exposure management is to anticipate and influence the effect of unexpected changes in exchange rates on a firm’s future cash flows. What strategic alte...

See Answer

Q: Operating and transaction exposures can be partially managed by adopting operating or

Operating and transaction exposures can be partially managed by adopting operating or financing policies that offset anticipated foreign exchange exposures. What are four of the most commonly employed...

See Answer

Q: Answer the following: a. Explain how matching currency

Answer the following: a. Explain how matching currency cash flows can offset operating exposure. b. Give an example of matching currency cash flows.

See Answer

Q: An alternative arrangement for managing operating exposure between firms with a continuing

An alternative arrangement for managing operating exposure between firms with a continuing buyer-supplier relationship is risk sharing. Explain how risk sharing works.

See Answer

Q: What are the major differences in translating liabilities between the current rate

What are the major differences in translating liabilities between the current rate method and the temporal method?

See Answer

Q: Name the four main contractual instruments used to hedge transaction exposure.

Name the four main contractual instruments used to hedge transaction exposure.

See Answer

Q: What is a mortgage-backed security (MBS)?

What is a mortgage-backed security (MBS)?

See Answer

Q: Check The Wall Street Journal to find the United States locations for

Check The Wall Street Journal to find the United States locations for trading foreign exchange futures contracts.

See Answer

Q: The newspaper reports that a given June Eurodollar future settled at 93

The newspaper reports that a given June Eurodollar future settled at 93.55. What was the annual yield? How many dollars does this represent?

See Answer

Q: How should an MNE factor host country inflation into its evaluation of

How should an MNE factor host country inflation into its evaluation of an investment proposal?

See Answer

Q: Smith Company and Jones Company enter into an interest rate swap,

Smith Company and Jones Company enter into an interest rate swap, with Smith paying fixed interest to Jones, and Jones paying floating interest to Smith. Smith now goes bankrupt, and so defaults on it...

See Answer

Q: Convert the following indirect quotes to direct quotes, and direct quotes

Convert the following indirect quotes to direct quotes, and direct quotes to indirect quotes: a. Euro: €1.22/$ (indirect quote) b. Russia: Rubl 30/$ (indirect quote c. Canada: $0.72/C$ (direct quote)...

See Answer

Q: The IMF classifies all exchange rate regimes into eight specific categories that

The IMF classifies all exchange rate regimes into eight specific categories that are summarized in this chapter. Under which exchange rate regime would you classify each of the following countries?

See Answer

Q: The U.S. dollar has maintained or increased its value

The U.S. dollar has maintained or increased its value over the past 20 years, despite running a gradually increasing current account deficit. Why has this phenomenon occurred?

See Answer

Q: Brazil has experienced periodic depreciation of its currency over the past 20

Brazil has experienced periodic depreciation of its currency over the past 20 years, despite occasionally running a current account surplus. Why has this phenomenon occurred?

See Answer

Q: Technically, what is a sovereign default?

Technically, what is a sovereign default?

See Answer

Q: It has been claimed that failures in corporate governance have hampered the

It has been claimed that failures in corporate governance have hampered the growth and profitability of some prominent firms located in emerging markets. What are some typical causes of these failures...

See Answer

Q: The measurement of all international economic transactions between the residents of a

The measurement of all international economic transactions between the residents of a country and foreign residents is called the balance of payments (BOP). What institution provides the primary sourc...

See Answer

Q: If a country follows a fixed exchange rate regime, what macroeconomic

If a country follows a fixed exchange rate regime, what macroeconomic variables could cause the fixed exchange rate to be devalued?

See Answer

Q: Why have eurocurrencies and LIBOR remained the centerpiece of the global financial

Why have eurocurrencies and LIBOR remained the centerpiece of the global financial marketplace for so long?

See Answer

Q: What is the effect of market liquidity and segmentation on a firm

What is the effect of market liquidity and segmentation on a firm capital.

See Answer

Q: What is a collateralized debt obligation (CDO)?

What is a collateralized debt obligation (CDO)?

See Answer

Q: If the BOP were viewed as an accounting statement, would it

If the BOP were viewed as an accounting statement, would it be a balance sheet of the country’s wealth, an income statement of the country’s earnings, or a funds flow statement of money into and out o...

See Answer

Q: Why does LIBOR receive so much attention in the global financial markets

Why does LIBOR receive so much attention in the global financial markets?

See Answer

Q: What is the difference between a “real” asset and a

What is the difference between a “real” asset and a “financial” asset?

See Answer

Q: What are the benefits of achieving a lower cost and greater availability

What are the benefits of achieving a lower cost and greater availability of capital?

See Answer

Q: In January 2002 the government of Argentina broke away from its currency

In January 2002 the government of Argentina broke away from its currency board system that had tied the peso to the U.S. dollar, and devalued the peso from APs1.0000/$ to APs1.4000/$. This caused some...

See Answer

Q: Explain the strategies used by an MNE to counter blocked funds.

Explain the strategies used by an MNE to counter blocked funds. What can a multinational firm do to transfer funds out of countries having exchange or remittance restrictions? At least six popular str...

See Answer

Q: Why would anyone, individual or corporation, want to deposit U

Why would anyone, individual or corporation, want to deposit U.S. dollars in a bank outside of the United States, when the natural location for such deposits would be a bank within the United States?...

See Answer

Q: What, if anything, is a “euro-euro?”

What, if anything, is a “euro-euro?”

See Answer

Q: What are the main types of political risks that are global in

What are the main types of political risks that are global in origin?

See Answer

Q: Diagram the life span of an exposure arising from selling a product

Diagram the life span of an exposure arising from selling a product on open account. On the diagram define and show quotation, backlog, and billing exposures.

See Answer

Q: Given that eGain Communications was up by almost 412 percent for 2011

Given that eGain Communications was up by almost 412 percent for 2011, why didn’t all investors hold eGain Communications?

See Answer

Q: Define financial distress using the stock-based and flow-based

Define financial distress using the stock-based and flow-based approaches.

See Answer

Q: Companies pay rating agencies such as Moody’s and S&P to

Companies pay rating agencies such as Moody’s and S&P to rate their bonds, and the costs can be substantial. However, companies are not required to have their bonds rated in the first place; doing so...

See Answer

Q: True or false: The unsystematic risk of a share of stock

True or false: The unsystematic risk of a share of stock is irrelevant for valuing the stock because it can be diversified away; therefore, it is also irrelevant for valuing a call option on the stock...

See Answer

Q: Why is it not necessarily bad for the operating cash flow to

Why is it not necessarily bad for the operating cash flow to be negative for a particular period?

See Answer

Q: How can the return on a portfolio be expressed in terms of

How can the return on a portfolio be expressed in terms of a factor model?

See Answer

Q: Critically evaluate the following statements: Playing the stock market is like

Critically evaluate the following statements: Playing the stock market is like gambling. Such speculative investing has no social value other than the pleasure people get from this form of gambling.

See Answer

Q: Why is the use of debt financing referred to as financial “

Why is the use of debt financing referred to as financial “leverage”?

See Answer

Q: A put option and a call option with an exercise price of

A put option and a call option with an exercise price of $85 and three months to expiration sell for $2.40 and $5.09, respectively. If the risk-free rate is 4.8 percent per year, compounded continuous...

See Answer

Q: Your company currently uses traditional capital budgeting techniques, including net present

Your company currently uses traditional capital budgeting techniques, including net present value. After hearing about the use of real option analysis, your boss decides that your company should use r...

See Answer

Q: Why will convertible bonds not be voluntarily converted to stock before expiration

Why will convertible bonds not be voluntarily converted to stock before expiration?

See Answer

Q: What impact did this change in payables policy have on BlueSky’s operating

What impact did this change in payables policy have on BlueSky’s operating cycle? Its cash cycle?

See Answer

Q: Why does the value of a share of stock depend on dividends

Why does the value of a share of stock depend on dividends?

See Answer

Q: If a company moves to a JIT inventory management system, what

If a company moves to a JIT inventory management system, what will happen to inventory turnover? What will happen to total asset turnover? What will happen to return on equity (ROE)?

See Answer

Q: Suppose the current exchange rate for the Polish zloty is Z 3

Suppose the current exchange rate for the Polish zloty is Z 3.14. The expected exchange rate in three years is Z 3.23. What is the difference in the annual inflation rates for the United States and Po...

See Answer

Q: What are the three factors that determine a company’s price−earnings

What are the three factors that determine a company’s price−earnings ratio?

See Answer

Q: Suppose a certain stock currently sells for $30 per share.

Suppose a certain stock currently sells for $30 per share. If a put option and a call option are available with $30 exercise prices, which do you think will sell for more? Explain.

See Answer

Q: You own a callable, convertible bond with a conversion ratio of

You own a callable, convertible bond with a conversion ratio of 24.25. The stock is currently selling for $48 per share. The issuer of the bond has announced a call at a call price of 110. What are yo...

See Answer

Q: Both ROA and ROE measure profitability. Which one is more useful

Both ROA and ROE measure profitability. Which one is more useful for comparing two companies? Why?

See Answer

Q: What is homemade leverage?

What is homemade leverage?

See Answer

Q: Firms sometimes use the threat of a bankruptcy filing to force creditors

Firms sometimes use the threat of a bankruptcy filing to force creditors to renegotiate terms. Critics argue that in such cases the firm is using bankruptcy laws “as a sword rather than a shield.” Is...

See Answer

Q: Explain why the after tax borrowing rate is the appropriate discount rate

Explain why the after tax borrowing rate is the appropriate discount rate to use in lease evaluation. Refer to the following example for Questions 10–12. In May 2011, Air Lease Corporation (ALC) annou...

See Answer

Q: A put option and a call option with an exercise price of

A put option and a call option with an exercise price of $55 expire in two months and sell for $2.65 and $5.32, respectively. If the stock is currently priced at $57.30, what is the annual continuousl...

See Answer

Q: As you increase the length of time involved, what happens to

As you increase the length of time involved, what happens to future values? What happens to present values?

See Answer

Q: What impact did the announcement have on BlueSky’s suppliers?

What impact did the announcement have on BlueSky’s suppliers?

See Answer

Q: It is sometimes argued that excess cash held by a firm can

It is sometimes argued that excess cash held by a firm can aggravate agency problems and, more generally, reduce incentives for shareholder wealth maximization. How would you describe the issue here?...

See Answer

Q: No More Pencils, Inc., disburses checks every two weeks that

No More Pencils, Inc., disburses checks every two weeks that average $58,000 and take seven days to clear. How much interest can the company earn annually if it delays transfer of funds from an intere...

See Answer

Q: If a company’s inventory carrying costs are $5 million per year

If a company’s inventory carrying costs are $5 million per year and its fixed order costs are $8 million per year, do you think the firm keeps too much inventory on hand or too little? Why?

See Answer

Q: Suppose a company in which you own stock has attracted two take

Suppose a company in which you own stock has attracted two take over offers. Would it ever make sense for your company’s management to favor the lower offer? Does the form of payment affect your answe...

See Answer

Q: We discussed five international capital market relationships: Relative PPP, IRP

We discussed five international capital market relationships: Relative PPP, IRP, UFR, UIP, and the international Fisher effect. Which of these would you expect to hold most closely? Which do you think...

See Answer

Q: Is it unfair or unethical for corporations to create classes of stock

Is it unfair or unethical for corporations to create classes of stock with unequal voting rights?

See Answer

Q: Suppose that when TMCC offered the security for $24,099

Suppose that when TMCC offered the security for $24,099 the U.S. Treasury had offered an essentially identical security. Do you think it would have had a higher or lower price? Why?

See Answer

Q: What is the difference between the term structure of interest rates and

What is the difference between the term structure of interest rates and the yield curve?

See Answer

Q: Suppose the interest rate on T-bills suddenly and unexpectedly rises

Suppose the interest rate on T-bills suddenly and unexpectedly rises. All other things being the same, what is the impact on call option values? On put option values?

See Answer

Q: Could a company’s cash flow to stockholders be negative in a given

Could a company’s cash flow to stockholders be negative in a given year? Explain how this might come about. What about cash flow to creditors?

See Answer

Q: What is wrong with measuring the performance of a U.S

What is wrong with measuring the performance of a U.S. growth stock manager against a benchmark composed of British stocks?

See Answer

Q: What is the difference between internal financing and external financing?

What is the difference between internal financing and external financing?

See Answer

Q: What is the basic goal of financial management with regard to capital

What is the basic goal of financial management with regard to capital structure?

See Answer

Q: Is it ethical for large firms to unilaterally lengthen their payables periods

Is it ethical for large firms to unilaterally lengthen their payables periods, particularly when dealing with smaller suppliers?

See Answer

Q: One option a firm usually has with any excess cash is to

One option a firm usually has with any excess cash is to pay its suppliers more quickly. What are the advantages and disadvantages of this use of excess cash?

See Answer

Q: If you are an exporter who must make payments in foreign currency

If you are an exporter who must make payments in foreign currency three months after receiving each shipment and you predict that the domestic currency will appreciate in value over this period, is th...

See Answer

Q: Evaluate the following statement: Managers should not focus on the current

Evaluate the following statement: Managers should not focus on the current stock value because doing so will lead to an overemphasis on short-term profits at the expense of long-term profits.

See Answer

Q: In evaluating the Cayenne, would you consider the possible damage to

In evaluating the Cayenne, would you consider the possible damage to Porsche’s reputation as erosion?

See Answer

Q: In evaluating the Cayenne, would you consider the possible damage to

In evaluating the Cayenne, would you consider the possible damage to Porsche’s reputation as erosion?

See Answer

Q: In the context of capital budgeting, what is an opportunity cost

In the context of capital budgeting, what is an opportunity cost?

See Answer

Q: Looking back at the crossover bonds we discussed in the chapter,

Looking back at the crossover bonds we discussed in the chapter, why do you think split ratings such as these occur?

See Answer

Q: You own a stock portfolio invested 10 percent in Stock Q ,

You own a stock portfolio invested 10 percent in Stock Q , 35 percent in Stock R , 20 percent in Stock S , and 35 percent in Stock T . The betas for these four stocks are .75, 1.90, 1.38, and 1.16, re...

See Answer

Q: Nina Corp. uses no debt. The weighted average cost of

Nina Corp. uses no debt. The weighted average cost of capital is 9 percent. If the current market value of the equity is $37 million and there are no taxes, what is EBIT?

See Answer

Q: Suppose a firm enters a fixed for floating interest rate swap with

Suppose a firm enters a fixed for floating interest rate swap with a swap dealer. Describe the cash flows that will occur as a result of the swap.

See Answer

Q: Referring back to the Bank of America example at the beginning of

Referring back to the Bank of America example at the beginning of the chapter, note that we suggested that Bank of America’s stockholders probably didn’t suffer as a result of the reported loss. What...

See Answer

Q: Another option usually available is to reduce the firm’s outstanding debt.

Another option usually available is to reduce the firm’s outstanding debt. What are the advantages and disadvantages of this use of excess cash?

See Answer

Q: What factors influence a firm’s choice of external versus internal equity financing

What factors influence a firm’s choice of external versus internal equity financing?

See Answer

Q: Porsche was one of the last manufacturers to enter the sports utility

Porsche was one of the last manufacturers to enter the sports utility vehicle market. Why would one company decide to proceed with a product when other companies, at least initially, decide not to ent...

See Answer

Q: Porsche was one of the last manufacturers to enter the sports utility

Porsche was one of the last manufacturers to enter the sports utility vehicle market. Why would one company decide to proceed with a product when other companies, at least initially, decide not to ent...

See Answer

Q: Do you think the company would have suffered the same fate if

Do you think the company would have suffered the same fate if its product had been less popular? Why or why not?

See Answer

Q: In the context of capital budgeting, what is an opportunity cost

In the context of capital budgeting, what is an opportunity cost?

See Answer

Q: BlueSky lengthened its payables period to “control costs and optimize

BlueSky lengthened its payables period to “control costs and optimize cash flow.” Exactly what is the cash benefit to BlueSky from this change? Last month, BlueSky Airline announced that it would stre...

See Answer

Q: An unfortunately common practice goes like this (Warning: Don’t try

An unfortunately common practice goes like this (Warning: Don’t try this at home): Suppose you are out of money in your checking account; however, your local grocery store will, as a convenience to yo...

See Answer

Q: What do you suppose happens to the plane at the end of

What do you suppose happens to the plane at the end of the lease period?

See Answer

Q: In evaluating the Cayenne, what do you think Porsche needs to

In evaluating the Cayenne, what do you think Porsche needs to assume regarding the substantial profit margins that exist in this market? Is it likely that they will be maintained as the market becomes...

See Answer

Q: What are the implications for bond investors of the lack of transparency

What are the implications for bond investors of the lack of transparency in the bond market?

See Answer

Q: A stock has a beta of 1.15, the expected

A stock has a beta of 1.15, the expected return on the market is 11 percent, and the risk-free rate is 5 percent. What must the expected return on this stock be?

See Answer

Q: What is the impact of an increase in the volatility of the

What is the impact of an increase in the volatility of the underlying stock’s return on an option’s value? Explain.

See Answer

Q: In evaluating the Cayenne, what do you think Porsche needs to

In evaluating the Cayenne, what do you think Porsche needs to assume regarding the substantial profit margins that exist in this market? Is it likely that they will be maintained as the market becomes...

See Answer

Q: Show that the NPV of a merger can be expressed as the

Show that the NPV of a merger can be expressed as the value of the synergistic benefits, D V , less the merger premium.

See Answer

Q: If the firm was so successful at selling, why wouldn’t a

If the firm was so successful at selling, why wouldn’t a bank or some other lender step in and provide it with the cash it needed to continue?

See Answer

Q: Is it true that a U.S. Treasury security is

Is it true that a U.S. Treasury security is risk-free?

See Answer

Q: If financial markets are perfectly competitive and the Eurodollar rate is above

If financial markets are perfectly competitive and the Eurodollar rate is above that offered in the U.S. loan market, you would immediately want to borrow money in the United States and invest it in E...

See Answer

Q: A controversy erupted regarding bond-rating agencies when some agencies began

A controversy erupted regarding bond-rating agencies when some agencies began to provide unsolicited bond ratings. Why do you think this is controversial?

See Answer

Q: A stock has an expected return of 13.4 percent,

A stock has an expected return of 13.4 percent, its beta is 1.60, and the risk-free rate is 5.5 percent. What must the expected return on the market be?

See Answer

Q: Sysco Corporation, the distributor of food and food-related products

Sysco Corporation, the distributor of food and food-related products (not to be confused with Cisco Systems), announced it had signed an interest rate swap. The interest rate swap effectively converte...

See Answer

Q: Which was the biggest culprit here: Too many orders, too

Which was the biggest culprit here: Too many orders, too little cash, or too little production capacity?

See Answer

Q: A stock has an expected return of 13.1 percent,

A stock has an expected return of 13.1 percent, a beta of 1.28, and the expected return on the market is 11 percent. What must the risk-free rate be?

See Answer

Q: What are some actions a small company like The Grandmother Calendar Company

What are some actions a small company like The Grandmother Calendar Company can take (besides expansion of capacity) if it finds itself in a situation in which growth in sales outstrips production?

See Answer

Q: A call option matures in six months. The underlying stock price

A call option matures in six months. The underlying stock price is $75, and the stock’s return has a standard deviation of 30 percent per year. The risk-free rate is 4 percent per year, compounded con...

See Answer

Q: One thing put–call parity tells us is that given any

One thing put–call parity tells us is that given any three of a stock, a call, a put, and a T-bill, the fourth can be synthesized or replicated using the other three. For example, how can we replicate...

See Answer

Q: A call option has an exercise price of $80 and matures

A call option has an exercise price of $80 and matures in six months. The current stock price is $84, and the risk-free rate is 5 percent per year, compounded continuously. What is the price of the ca...

See Answer

Q: A researcher has determined that a two-factor model is appropriate

A researcher has determined that a two-factor model is appropriate to determine the return on a stock. The factors are the percentage change in GNP and an interest rate. GNP is expected to grow by 3.6...

See Answer

Q: A stock is currently priced at $35. A call option

A stock is currently priced at $35. A call option with an expiration of one year has an exercise price of $50. The risk-free rate is 7 percent per year, compounded continuously, and the standard devia...

See Answer

Q: Using information from the previous chapter about capital market history, determine

Using information from the previous chapter about capital market history, determine the return on a portfolio that is equally invested in large company stocks and long-term government bonds. What is t...

See Answer

Q: Your broker commented that well-managed firms are better investments than

Your broker commented that well-managed firms are better investments than poorly managed firms. As evidence your broker cited a recent study examining 100 small manufacturing firms that eight years ea...

See Answer

Q: A famous economist just announced in The Wall Street Journal his findings

A famous economist just announced in The Wall Street Journal his findings that the recession is over and the economy is again entering an expansion. Assume market efficiency. Can you profit from inves...

See Answer

Q: Draw up an income statement and balance sheet for this company for

Draw up an income statement and balance sheet for this company for 2011 and 2012.

See Answer

Q: For 2012, calculate the cash flow from assets, cash flow

For 2012, calculate the cash flow from assets, cash flow to creditors, and cash flow to stockholders.

See Answer

Q: The efficient market hypothesis implies that all mutual funds should obtain the

The efficient market hypothesis implies that all mutual funds should obtain the same expected risk-adjusted returns. Therefore, we can simply pick mutual funds at random. Is this statement true or fal...

See Answer

Q: Assume that markets are efficient. During a trading day American Golf

Assume that markets are efficient. During a trading day American Golf Inc. announces that it has lost a contract for a large golfing project that, prior to the news, it was widely believed to have sec...

See Answer

Q: In the previous problem, what is the probability that the return

In the previous problem, what is the probability that the return is less than 2100 percent? (Think.) What are the implications for the distribution of returns?

See Answer

Q: Some people argue that the efficient market hypothesis cannot explain the 1987

Some people argue that the efficient market hypothesis cannot explain the 1987 market crash or the high price-to-earnings ratios of Internet stocks during the late 1990s. What alternative hypothesis i...

See Answer

Q: A convertible bond has a conversion ratio of 24.6.

A convertible bond has a conversion ratio of 24.6. What is the conversion price?

See Answer

Q: True or false: All assets are liquid at some price.

True or false: All assets are liquid at some price. Explain.

See Answer

Q: If you can borrow all the money you need for a project

If you can borrow all the money you need for a project at 6 percent, doesn’t it follow that 6 percent is your cost of capital for the project?

See Answer

Q: Consider the following statement: For the APT to be useful,

Consider the following statement: For the APT to be useful, the number of systematic risk factors must be small. Do you agree or disagree with this statement? Why?

See Answer

Q: Why do we use an after tax figure for cost of debt

Why do we use an after tax figure for cost of debt but not for cost of equity?

See Answer

Q: Why do we use an after tax figure for cost of debt

Why do we use an after tax figure for cost of debt but not for cost of equity?

See Answer

Q: What is the main difference between the WACC and APV methods?

What is the main difference between the WACC and APV methods?

See Answer

Q: Taxes are an important consideration in the leasing decision. Which is

Taxes are an important consideration in the leasing decision. Which is more likely to lease: A profitable corporation in a high tax bracket or a less profitable one in a low tax bracket? Why?

See Answer

Q: What are the cash flows from the lease from the lessor’s viewpoint

What are the cash flows from the lease from the lessor’s viewpoint? Assume a 35 percent tax bracket.

See Answer

Q: What are the two options that many businesses have?

What are the two options that many businesses have?

See Answer

Q: What are some of the characteristics of a firm with a long

What are some of the characteristics of a firm with a long cash cycle?

See Answer

Q: What options are available to a firm if it believes it has

What options are available to a firm if it believes it has too much cash? How about too little?

See Answer

Q: In what form is trade credit most commonly offered? What is

In what form is trade credit most commonly offered? What is the credit instrument in this case?

See Answer

Q: The Dybvig Corporation’s common stock has a beta of 1.21

The Dybvig Corporation’s common stock has a beta of 1.21. If the risk-free rate is 3.5 percent and the expected return on the market is 11 percent, what is Dybvig’s cost of equity capital?

See Answer

Q: The Paden Corporation has annual sales of $34 million. The

The Paden Corporation has annual sales of $34 million. The average collection period is 33 days. What is the average investment in accounts receivable as shown on the balance sheet?

See Answer

Q: What are some benefits of financial distress?

What are some benefits of financial distress?

See Answer

Q: Suppose the rate of inflation in Mexico will run about 3 percent

Suppose the rate of inflation in Mexico will run about 3 percent higher than the U.S. inflation rate over the next several years. All other things being the same, what will happen to the Mexican peso...

See Answer

Q: Classify the following events as mostly systematic or mostly unsystematic. Is

Classify the following events as mostly systematic or mostly unsystematic. Is the distinction clear in every case? a. Short-term interest rates increase unexpectedly. b. The interest rate a company pa...

See Answer

Q: What happens to the future value of an annuity if you increase

What happens to the future value of an annuity if you increase the rate r? What happens to the present value?

See Answer

Q: Which has greater interest rate risk, a 30-year Treasury

Which has greater interest rate risk, a 30-year Treasury bond or a 30-year BB corporate bond?

See Answer

Q: A convertible bond has a conversion price of $61.50

A convertible bond has a conversion price of $61.50. What is the conversion ratio of the bond?

See Answer

Q: So-called “same-store sales” are a very

So-called “same-store sales” are a very important measure for companies as diverse as McDonald’s and Sears. As the name suggests, examining same-store sales means comparing revenues from the same stor...

See Answer

Q: We have seen that over long periods stock investments have tended to

We have seen that over long periods stock investments have tended to substantially outperform bond investments. However, it is not at all uncommon to observe investors with long horizons holding their...

See Answer

Q: How is the APV of a project calculated?

How is the APV of a project calculated?

See Answer

Q: If a portfolio has a positive investment in every asset, can

If a portfolio has a positive investment in every asset, can the expected return on the portfolio be greater than that on every asset in the portfolio? Can it be less than that on every asset in the p...

See Answer

Q: If you use the stock beta and the security market line to

If you use the stock beta and the security market line to compute the discount rate for a project, what assumptions are you implicitly making?

See Answer

Q: If you use the stock beta and the security market line to

If you use the stock beta and the security market line to compute the discount rate for a project, what assumptions are you implicitly making?

See Answer

Q: What is the main difference between the FTE approach and the other

What is the main difference between the FTE approach and the other two approaches?

See Answer

Q: Why does a strict NPV calculation typically understate the value of a

Why does a strict NPV calculation typically understate the value of a company or project?

See Answer

Q: Suppose you are evaluating a callable, convertible bond. If the

Suppose you are evaluating a callable, convertible bond. If the stock price volatility increases, how will this affect the price of the bond?

See Answer

Q: Are stockholders and creditors likely to agree on how much cash a

Are stockholders and creditors likely to agree on how much cash a firm should keep on hand?

See Answer

Q: What costs are associated with carrying receivables? What costs are associated

What costs are associated with carrying receivables? What costs are associated with not granting credit? What do we call the sum of the costs for different levels of receivables?

See Answer

Q: What is prepackaged bankruptcy? What is the main benefit of prepackaged

What is prepackaged bankruptcy? What is the main benefit of prepackaged bankruptcy?

See Answer

Q: Evaluate the following statement: Managers should not focus on the

Evaluate the following statement: Managers should not focus on the current stock value because doing so will lead to an over emphasis on short-term profits at the expense of long-term profit.

See Answer

Q: Why do companies issue options to executives if they cost the company

Why do companies issue options to executives if they cost the company more than they are worth to the executive? Why not just give cash and split the difference? Wouldn’t that make both the company an...

See Answer

Q: Suppose two athletes sign 10-year contracts for $80 million

Suppose two athletes sign 10-year contracts for $80 million. In one case, we’re told that the $80 million will be paid in 10 equal installments. In the other case, we’re told that the $80 million will...

See Answer

Q: With regard to bid and ask prices on a Treasury bond,

With regard to bid and ask prices on a Treasury bond, is it possible for the bid price to be higher? Why or why not?

See Answer

Q: You own a portfolio that is 25 percent invested in Stock X

You own a portfolio that is 25 percent invested in Stock X , 40 percent in Stock Y , and 35 percent in Stock Z . The expected returns on these three stocks are 11 percent, 17 percent, and 14 percent,...

See Answer

Q: Looking at the accounting statement of cash flows, what does the

Looking at the accounting statement of cash flows, what does the bottom line number mean? How useful is this number for analyzing a company?

See Answer

Q: True or false: The most important characteristic in determining the expected

True or false: The most important characteristic in determining the expected return of a well-diversified portfolio is the variances of the individual assets in the portfolio. Explain.

See Answer

Q: What is the quirk in the tax code that makes a levered

What is the quirk in the tax code that makes a levered firm more valuable than an otherwise identical unlevered firm?

See Answer

Q: You are determining whether your company should undertake a new project and

You are determining whether your company should undertake a new project and have calculated the NPV of the project using the WACC method when the CFO, a former accountant, notices that you did not use...

See Answer

Q: Assets Grohl Manufacturing, Inc., has recently installed a justin-

Assets Grohl Manufacturing, Inc., has recently installed a justin- time (JIT) inventory system. Describe the effect this is likely to have on the company’s carrying costs, shortage costs, and operatin...

See Answer

Q: What is the difference between cash management and liquidity management?

What is the difference between cash management and liquidity management?

See Answer

Q: What is the primary difference between a warrant and a traded call

What is the primary difference between a warrant and a traded call option?

See Answer

Q: Tidwell, Inc., has weekly credit sales of $27,

Tidwell, Inc., has weekly credit sales of $27,500, and the average collection period is 27 days. The cost of production is 75 percent of the selling price. What is the average accounts receivable figu...

See Answer

Q: Why doesn’t financial distress always cause firms to die?

Why doesn’t financial distress always cause firms to die?

See Answer

Q: Are exchange rate changes necessarily good or bad for a particular company

Are exchange rate changes necessarily good or bad for a particular company?

See Answer

Q: Critically evaluate the following statement: Playing the stock market is like

Critically evaluate the following statement: Playing the stock market is like gambling. Such speculative investing has no social value, other than the pleasure people get from this form of gambling.

See Answer

Q: Treasury bid and ask quotes are sometimes given in terms of yields

Treasury bid and ask quotes are sometimes given in terms of yields, so there would be a bid yield and an ask yield. Which do you think would be larger? Explain.

See Answer

Q: What is the intrinsic value of a call option? Of a

What is the intrinsic value of a call option? Of a put option? How do we interpret these values?

See Answer

Q: Should lending laws be changed to require lenders to report EARs instead

Should lending laws be changed to require lenders to report EARs instead of APRs? Why or why not?

See Answer

Q: Suppose a company has a preferred stock issue and a common stock

Suppose a company has a preferred stock issue and a common stock issue. Both have just paid a $2 dividend. Which do you think will have a higher price, a share of the preferred or a share of the commo...

See Answer

Q: Look at Table 10.1 and Figure 10.7 in

Look at Table 10.1 and Figure 10.7 in the text. When were T-bill rates at their highest over the period from 1926 through 2011? Why do you think they were so high during this period? What relationship...

See Answer

Q: What are some of the characteristics of a firm with a long

What are some of the characteristics of a firm with a long operating cycle?

See Answer

Q: If a portfolio has a positive investment in every asset, can

If a portfolio has a positive investment in every asset, can the standard deviation on the portfolio be less than that on every asset in the portfolio? What about the portfolio beta?

See Answer

Q: What are the differences between a k -factor model and the

What are the differences between a k -factor model and the market model?

See Answer

Q: Mullineaux Corporation has a target capital structure of 70 percent common stock

Mullineaux Corporation has a target capital structure of 70 percent common stock and 30 percent debt. Its cost of equity is 13 percent, and the cost of debt is 6 percent. The relevant tax rate is 35 p...

See Answer

Q: A stock market analyst is able to identify mispriced stocks by comparing

A stock market analyst is able to identify mispriced stocks by comparing the average price for the last 10 days to the average price for the last 60 days. If this is true, what do you know about the m...

See Answer

Q: Your company owns a vacant lot in a suburban area. What

Your company owns a vacant lot in a suburban area. What is the advantage of waiting to develop the lot?

See Answer

Q: Why is a preferred stock with a dividend tied to short-

Why is a preferred stock with a dividend tied to short-term interest rates an attractive short-term investment for corporations with excess cash?

See Answer

Q: Are poison pills good or bad for stockholders? How do you

Are poison pills good or bad for stockholders? How do you think acquiring firms are able to get around poison pills?

See Answer

Q: What is the difference between liquidation and reorganization?

What is the difference between liquidation and reorganization?

See Answer

Q: At one point, Duracell International confirmed that it was planning to

At one point, Duracell International confirmed that it was planning to open battery manufacturing plants in China and India. Manufacturing in these countries allows Duracell to avoid import duties of...

See Answer

Q: Would the goal of maximizing the value of the stock differ for

Would the goal of maximizing the value of the stock differ for financial management in a foreign country? Why or why not?

See Answer

Q: Is it possible for a firm to have too much cash?

Is it possible for a firm to have too much cash? Why would shareholders care if a firm accumulates large amounts of cash?

See Answer

Q: You notice that shares of stock in the Patel Corporation are going

You notice that shares of stock in the Patel Corporation are going for $50 per share. Call options with an exercise price of $35 per share are selling for $10. What’s wrong here? Describe how you can...

See Answer

Q: On subsidized Stafford loans, a common source of financial aid for

On subsidized Stafford loans, a common source of financial aid for college students, interest does not begin to accrue until repayment begins. Who receives a bigger subsidy, a freshman or a senior? Ex...

See Answer

Q: If a market is semi strong form efficient, is it also

If a market is semi strong form efficient, is it also weak form efficient? Explain.

See Answer

Q: A stock is currently selling for $38 per share. A

A stock is currently selling for $38 per share. A call option with an exercise price of $40 sells for $3.80 and expires in three months. If the risk-free rate of interest is 2.6 percent per year, comp...

See Answer

Q: What is wrong with the simple view that it is cheaper to

What is wrong with the simple view that it is cheaper to issue a bond with a warrant or a convertible feature because the required coupon is lower?

See Answer

Q: Which would a firm prefer: A net collection float or a

Which would a firm prefer: A net collection float or a net disbursement float? Why?

See Answer

Q: Bell Tolls, Inc., has an average collection period of 36

Bell Tolls, Inc., has an average collection period of 36 days. Its average daily investment in receivables is $58,300. What are annual credit sales? What is the receivables turnover?

See Answer

Q: Miller Manufacturing has a target debt–equity ratio of .55

Miller Manufacturing has a target debt–equity ratio of .55. Its cost of equity is 14 percent, and its cost of debt is 7 percent. If the tax rate is 35 percent, what is Miller’s WACC?

See Answer

Q: Why would TMCC be willing to accept such a small amount today

Why would TMCC be willing to accept such a small amount today ($24,099) in exchange for a promise to repay about four times that amount ($100,000) in the future?

See Answer

Q: Evan, Inc., has offered $340 million cash for all

Evan, Inc., has offered $340 million cash for all of the common stock in Tanner Corporation. Based on recent market information, Tanner is worth $317 million as an independent operation. If the merger...

See Answer

Q: If the risk of a stock increases, what is likely to

If the risk of a stock increases, what is likely to happen to the price of call options on the stock? To the price of put options? Why?

See Answer

Q: Why is it not necessarily bad for the cash flow from assets

Why is it not necessarily bad for the cash flow from assets to be negative for a particular period?

See Answer

Q: What is the relationship between the one-factor model and the

What is the relationship between the one-factor model and the CAPM?

See Answer

Q: What are the implications of the efficient market hypothesis for investors who

What are the implications of the efficient market hypothesis for investors who buy and sell stocks in an attempt to “beat the market”?

See Answer

Q: What is a proxy?

What is a proxy?

See Answer

Q: A put option that expires in six months with an exercise price

A put option that expires in six months with an exercise price of $65 sells for $4.89. The stock is currently priced at $61, and the risk-free rate is 3.6 percent per year, compounded continuously. Wh...

See Answer

Q: Suppose a firm has a book balance of $2 million.

Suppose a firm has a book balance of $2 million. At the automatic teller machine (ATM), the cash manager finds out that the bank balance is $2.5 million. What is the situation here? If this is an ongo...

See Answer

Q: What does it mean to say that a proposed merger will take

What does it mean to say that a proposed merger will take advantage of available economies of scale? Suppose Eastern Power Co. and Western Power Co. are located in different time zones. Both operate a...

See Answer

Q: In the context of the dividend growth model, is it true

In the context of the dividend growth model, is it true that the growth rate in dividends and the growth rate in the price of the stock are identical?

See Answer

Q: One of the implicit assumptions we made in calculating the external funds

One of the implicit assumptions we made in calculating the external funds needed was that the company was operating at full capacity. If the company is operating at less than full capacity, how will t...

See Answer

Q: Why might a firm announce a reverse stock split?

Why might a firm announce a reverse stock split?

See Answer

Q: Using the information in the table, compute the required return for

Using the information in the table, compute the required return for each company using both CAPM and the constant growth model. Compare and discuss the results. Assume that the market portfolio will e...

See Answer

Q: Suppose that LilyMac Photography has annual sales of $230,000

Suppose that LilyMac Photography has annual sales of $230,000, cost of goods sold of $165,000, average inventories of $4,500, average accounts receivable of $25,000, and an average accounts payable ba...

See Answer

Q: Suppose that the Ken-Z Art Gallery has annual sales of

Suppose that the Ken-Z Art Gallery has annual sales of $870,000, cost of goods sold of $560,000, average inventories of $244,500, average accounts receivable of $265,000, and an average accounts payab...

See Answer

Q: Suppose your firm is seeking an eight-year, amortizing $

Suppose your firm is seeking an eight-year, amortizing $800,000 loan with annual payments and your bank is offering you the choice between an $850,000 loan with a $50,000 compensating balance and an $...

See Answer

Q: Suppose your firm is seeking a four-year, amortizing $

Suppose your firm is seeking a four-year, amortizing $200,000 loan with annual payments and your bank is offering you the choice between a $205,000 loan with a $5,000 compensating balance and a $200,0...

See Answer

Q: Veggie Burgers, Inc., would like to maintain their cash account

Veggie Burgers, Inc., would like to maintain their cash account at a minimum level of $245,000, but expect the standard deviation in net daily cash flows to be $12,000, the effective annual rate on ma...

See Answer

Q: Suppose that Dunn Industries has annual sales of $2,300

Suppose that Dunn Industries has annual sales of $2,300,000, cost of goods sold of $1,650,000, average inventories of $1,116,000, and average accounts receivable of $750,000. Assuming that all of Dunn...

See Answer

Q: Suppose that LilyMac Photography has annual sales of $230,000

Suppose that LilyMac Photography has annual sales of $230,000, cost of goods sold of $165,000, average inventories of $4,500, and average accounts receivable of $25,000. Assuming that all of LilyMac’s...

See Answer

Q: Rose Axels faces a smooth annual demand for cash of $5

Rose Axels faces a smooth annual demand for cash of $5,000,000, incurs transaction costs of $275 every time they sell marketable securities, and can earn 4.3 percent on their marketable securities. Wh...

See Answer

Q: Watkins Resources faces a smooth annual demand for cash of $1

Watkins Resources faces a smooth annual demand for cash of $1,500,000, incurs transaction costs of $75 every time they sell marketable securities, and can earn 3.7 percent on their marketable securiti...

See Answer

Q: HotFoot Shoes would like to maintain their cash account at a minimum

HotFoot Shoes would like to maintain their cash account at a minimum level of $25,000, but expect the standard deviation in net daily cash flows to be $4,000, the effective annual rate on marketable s...

See Answer

Q: Using the information in the table, compute the required return for

Using the information in the table, compute the required return for each company using both CAPM and the constant growth model. Compare and discuss the results. Assume that the market portfolio will e...

See Answer

Q: Veggie Burgers, Inc., would like to maintain their cash account

Veggie Burgers, Inc., would like to maintain their cash account at a minimum level of $245,000, but expect the standard deviation in net daily cash flows to be $12,000, the effective annual rate on ma...

See Answer

Q: Your bank offers you a $140,000 line of credit

Your bank offers you a $140,000 line of credit with an interest rate of 2.30 percent per quarter. The loan agreement also requires that 7 percent of the unused portion of the credit line be deposited...

See Answer

Q: Go to the SEC’s Edgar site at http://www.sec

Go to the SEC’s Edgar site at http://www.sec.gov/edgar.shtml and download the latest annual (“10-K”) report for the firm of your choice. Use the financial statements in the report to calculate the fir...

See Answer

Q: Is it possible for a firm to have negative net working capital

Is it possible for a firm to have negative net working capital? How?

See Answer

Q: Would it be possible for a decision to deny credit to your

Would it be possible for a decision to deny credit to your customers be value maximizing? How?

See Answer

Q: Which of the following will result in an increase in net working

Which of the following will result in an increase in net working capital?

See Answer

Q: Would it be possible for a firm to have a negative cash

Would it be possible for a firm to have a negative cash cycle? How?

See Answer

Q: If a firm’s inventory turnover ratio increases, what will happen to

If a firm’s inventory turnover ratio increases, what will happen to the firm’s operating cycle?

See Answer

Q: If a firm’s inventory turnover ratio increases, what will happen to

If a firm’s inventory turnover ratio increases, what will happen to the firm’s cash cycle?

See Answer

Q: Everything else held constant, will an increase in the amount of

Everything else held constant, will an increase in the amount of inventory on hand increase or decrease the firm’s profitability?

See Answer

Q: Using beta as a risk measure has been fully integrated into corporate

Using beta as a risk measure has been fully integrated into corporate finance and the investment industry. You can obtain a beta for most companies at many financial Web sites. Sites that list a beta...

See Answer

Q: Would a firm ever use short-term debt to finance permanent

Would a firm ever use short-term debt to finance permanent current assets? Why or why not?

See Answer

Q: Suppose that short-term borrowing actually becomes more expensive than long

Suppose that short-term borrowing actually becomes more expensive than long-term borrowing. How would this affect the firm’s choice between a flexible financing policy and a restrictive policy?

See Answer

Q: If asset-backed loans are cheaper than unsecured loans, what

If asset-backed loans are cheaper than unsecured loans, what is the disadvantage to the firm in using an asset-backed loan?

See Answer

Q: Is an increase in the cash account a source of funds or

Is an increase in the cash account a source of funds or a use of funds?

See Answer

Q: What will be the carrying cost associated with a compensating balance requirement

What will be the carrying cost associated with a compensating balance requirement?

See Answer

Q: What will be the shortage cost associated with a compensating balance requirement

What will be the shortage cost associated with a compensating balance requirement?

See Answer

Q: What would be the shortage costs associated with a restaurant not having

What would be the shortage costs associated with a restaurant not having enough cash on hand to make change?

See Answer

Q: If a firm needs to keep a minimum cash balance on hand

If a firm needs to keep a minimum cash balance on hand and faces both cash inflows and outflows, which of the cash management models discussed in this chapter would be more appropriate for them to use...

See Answer

Q: What effect will increasing the trading costs associated with selling marketable securities

What effect will increasing the trading costs associated with selling marketable securities have on the optimal replenishment level in the Baumol model? Why?

See Answer

Q: What effect will an increase in the standard deviation of daily cash

What effect will an increase in the standard deviation of daily cash flows have on the return point in the Miller-Orr model? Why?

See Answer

Q: Investors can choose from many thousands of stocks. The large number

Investors can choose from many thousands of stocks. The large number to choose from can be quite daunting to new investors. Fortunately, some good stock screeners are available for free on the Interne...

See Answer

Q: Could a firm ever have negative collection float? Why or why

Could a firm ever have negative collection float? Why or why not?

See Answer

Q: Could a firm ever have negative disbursement float? Why or why

Could a firm ever have negative disbursement float? Why or why not?

See Answer

Q: Would a draft have availability float? Why or why not?

Would a draft have availability float? Why or why not?

See Answer

Q: From our discussion of capital markets elsewhere in this book, why

From our discussion of capital markets elsewhere in this book, why would you expect a firm to have a time delay between raising funds to finance a project and the expenditure of those funds on that pr...

See Answer

Q: What purpose does a discount on credit terms serve? What is

What purpose does a discount on credit terms serve? What is the cost of such a discount to the offering firm?

See Answer

Q: Derive an accept/reject rule for IRR similar to equation 13

Derive an accept/reject rule for IRR similar to equation 13-8 that would make the correct decision on cash flows that are non-normal, but that always have one large positive cash flow at time zero fol...

See Answer

Q: Is the set of cash flows depicted in the following table normal

Is the set of cash flows depicted in the following table normal or non-normal? Explain.

See Answer

Q: Is it possible for a company to initiate two products that target

Is it possible for a company to initiate two products that target the same market that are not mutually exclusive?

See Answer

Q: Suppose that your company used “APV,” or “All-

Suppose that your company used “APV,” or “All-the-Present Value-Except-CF0”, to analyze capital budgeting projects. What would this rule’s benchmark value be?

See Answer

Q: Under what circumstances could payback and discounted payback be equal?

Under what circumstances could payback and discounted payback be equal?

See Answer

Q: Following stocks in a portfolio is easier than ever. Many financial

Following stocks in a portfolio is easier than ever. Many financial Web sites have the capability to follow the stocks in your portfolio over time. Just enter your stocks, the number of shares, your...

See Answer

Q: Could a project’s MIRR ever exceed its IRR?

Could a project’s MIRR ever exceed its IRR?

See Answer

Q: If you had two mutually exclusive, normal-cash-flow

If you had two mutually exclusive, normal-cash-flow projects whose NPV profiles crossed at all points, for which range of interest rates would IRR give the right accept/reject answer?

See Answer

Q: Suppose a company wanted to double their firm’s value with the next

Suppose a company wanted to double their firm’s value with the next round of capital budgeting project decisions. To what would they set the PI benchmark to make this goal?

See Answer

Q: Suppose a company faced different borrowing and lending rates. How would

Suppose a company faced different borrowing and lending rates. How would this range change the way that you would compute the MIRR statistic?

See Answer

Q: Graph the NPV profiles for both projects on a common chart,

Graph the NPV profiles for both projects on a common chart, making sure that you identify all of the “crucial” points.

See Answer

Q: For what range of possible interest rates would you want to use

For what range of possible interest rates would you want to use IRR to choose between these two projects? For what range of rates would you NOT want to use IRR?

See Answer

Q: Construct an NPV profile and determine EXACTLY how many nonnegative IRRs you

Construct an NPV profile and determine EXACTLY how many nonnegative IRRs you can find for the following set of cash flows:

See Answer

Q: Construct an NPV profile and determine EXACTLY how many nonnegative IRRs you

Construct an NPV profile and determine EXACTLY how many nonnegative IRRs you can find for the following set of cash flows:

See Answer

Q: Compute the MIRR statistic for Project J and advise whether to accept

Compute the MIRR statistic for Project J and advise whether to accept or reject the project with the cash flows shown as follows if the appropriate cost of capital is 10 percent. Project J

See Answer

Q: Compute the discounted payback statistic for Project C and recommend whether the

Compute the discounted payback statistic for Project C and recommend whether the firm should accept or reject the project with the cash flows shown as follows if the appropriate cost of capital is 8 p...

See Answer

Q: How will passive and active capital structure changes differ?

How will passive and active capital structure changes differ?

See Answer

Q: Compute the MIRR statistic for Project I and tell whether to accept

Compute the MIRR statistic for Project I and tell whether to accept or reject the project with the cash flows shown as follows if the appropriate cost of capital is 12 percent. Project I

See Answer

Q: Compute the NPV statistic for Project Y and note whether the firm

Compute the NPV statistic for Project Y and note whether the firm should accept or reject the project with the cash flows shown as follows if the appropriate cost of capital is 12 percent. Project Y...

See Answer

Q: Compute the NPV statistic for Project U and recommend whether the firm

Compute the NPV statistic for Project U and recommend whether the firm should accept or reject the project with the cash flows shown as follows if the appropriate cost of capital is ten percent. Proje...

See Answer

Q: Compute the NPV statistic for Project K and recommend whether the firm

Compute the NPV statistic for Project K and recommend whether the firm should accept or reject the project with the cash flows shown as follows if the appropriate cost of capital is six percent. Proje...

See Answer

Q: Compute the payback statistic for Project B and decide whether the firm

Compute the payback statistic for Project B and decide whether the firm should accept or reject the project with the cash flows shown as follows if the appropriate cost of capital is 12 percent and th...

See Answer

Q: Compute the payback statistic for Project A and recommend whether the firm

Compute the payback statistic for Project A and recommend whether the firm should accept or reject the project with the cash flows shown as follows if the appropriate cost of capital is 8 percent and...

See Answer

Q: Compute the discounted payback statistic for Project D and recommend whether the

Compute the discounted payback statistic for Project D and recommend whether the firm should accept or reject the project with the cash flows shown as follows if the appropriate cost of capital is 12...

See Answer

Q: Compute the IRR statistic for Project E and note whether the firm

Compute the IRR statistic for Project E and note whether the firm should accept or reject the project with the cash flows shown as follows if the appropriate cost of capital is 8 percent. Project E

See Answer

Q: Compute the IRR statistic for project F and note whether the firm

Compute the IRR statistic for project F and note whether the firm should accept or reject the project with the cash flows shown as follows if the appropriate cost of capital is 12 percent. Project F...

See Answer

Q: Compute the PI statistic for Project Z for and advise the firm

Compute the PI statistic for Project Z for and advise the firm whether to accept or reject the project with the cash flows shown as follows if the appropriate cost of capital is 8 percent. Project Z...

See Answer

Q: Compare and contrast the use of pro forma financial statements in corporate

Compare and contrast the use of pro forma financial statements in corporate financial planning with their use in accounting.

See Answer

Q: Compute the PI statistic for Project Q and tell whether you would

Compute the PI statistic for Project Q and tell whether you would accept or reject the project with the cash flows shown as follows if the appropriate cost of capital is 12 percent. Project Q

See Answer

Q: How many possible IRRs could you find for the following set of

How many possible IRRs could you find for the following set of cash flows?

See Answer

Q: How many possible IRRs could you find for the following set of

How many possible IRRs could you find for the following set of cash flows?

See Answer

Q: Compute the NPV for Project M and accept or reject the project

Compute the NPV for Project M and accept or reject the project with the cash flows shown as follows if the appropriate cost of capital is 8 percent. Project M

See Answer

Q: Use the PI decision rule to evaluate this project; should it

Use the PI decision rule to evaluate this project; should it be accepted or rejected?

See Answer

Q: Use the payback decision rule to evaluate this project; should it

Use the payback decision rule to evaluate this project; should it be accepted or rejected?

See Answer

Q: Use the discounted payback decision rule to evaluate this project; should

Use the discounted payback decision rule to evaluate this project; should it be accepted or rejected?

See Answer

Q: Use the MIRR decision rule to evaluate this project; should it

Use the MIRR decision rule to evaluate this project; should it be accepted or rejected?

See Answer

Q: Use the PI decision rule to evaluate this project; should it

Use the PI decision rule to evaluate this project; should it be accepted or rejected?

See Answer

Q: Use the payback decision rule to evaluate this project; should it

Use the payback decision rule to evaluate this project; should it be accepted or rejected?

See Answer

Q: Why might current liabilities be considered a spontaneous source of funding for

Why might current liabilities be considered a spontaneous source of funding for a firm?

See Answer

Q: Use the discounted payback decision rule to evaluate this project; should

Use the discounted payback decision rule to evaluate this project; should it be accepted or rejected?

See Answer

Q: Use the MIRR decision rule to evaluate this project; should it

Use the MIRR decision rule to evaluate this project; should it be accepted or rejected?

See Answer

Q: Use the NPV decision rule to evaluate this project; should it

Use the NPV decision rule to evaluate this project; should it be accepted or rejected?

See Answer

Q: Use the IRR decision rule to evaluate this project; should it

Use the IRR decision rule to evaluate this project; should it be accepted or rejected?

See Answer

Q: Use the NPV decision rule to evaluate this project; should it

Use the NPV decision rule to evaluate this project; should it be accepted or rejected?

See Answer

Q: Use the IRR decision rule to evaluate this project; should it

Use the IRR decision rule to evaluate this project; should it be accepted or rejected?

See Answer

Q: Suppose your firm is considering investing in a project with the cash

Suppose your firm is considering investing in a project with the cash flows shown as follows, that the required rate of return on projects of this risk class is 11 percent, and that the maximum allowa...

See Answer

Q: Your company is contemplating replacing their current fleet of delivery vehicles with

Your company is contemplating replacing their current fleet of delivery vehicles with Nissan NV vans. You will be replacing 5 fully-depreciated vans, which you think you can sell for $3,000 apiece and...

See Answer

Q: You are considering adding a new software title to those published by

You are considering adding a new software title to those published by your highly successful software company. If you add the new product, it will use capacity on your disk duplicating machines that y...

See Answer

Q: You are evaluating a project for The Ultimate recreational tennis racket,

You are evaluating a project for The Ultimate recreational tennis racket, guaranteed to correct that wimpy backhand. You estimate the sales price of The Ultimate to be $400 per unit and sales volume t...

See Answer

Q: Why is debt often referred to as leverage in finance?

Why is debt often referred to as leverage in finance?

See Answer

Q: Mom's Cookies, Inc. is considering the purchase of a new

Mom's Cookies, Inc. is considering the purchase of a new cookie oven. The original cost of the old oven was $30,000; it is now five years old, and it has a current market value of $13,333.33. The old...

See Answer

Q: You are evaluating a project for The Tiff-any golf club

You are evaluating a project for The Tiff-any golf club, guaranteed to correct that nasty slice. You estimate the sales price of The Tiff-any to be $400 per unit and sales volume to be 1,000 units in...

See Answer

Q: Continuing the previous problem, what is the operating cash flow for

Continuing the previous problem, what is the operating cash flow for the project in year 2?

See Answer

Q: You are considering the purchase of one of two machines used in

You are considering the purchase of one of two machines used in your manufacturing plant. Machine A has a life of two years, costs $80 initially, and then $125 per year in maintenance costs. Machine B...

See Answer

Q: Your company is considering a new project that will require $1

Your company is considering a new project that will require $1 million of new equipment at the start of the project. The equipment will have a depreciable life of 10 years and will be depreciated to a...

See Answer

Q: You are trying to pick the least-expensive car for your

You are trying to pick the least-expensive car for your new delivery service. You have two choices: the Scion xA, which will cost $14,000 to purchase and which will have OCF of -$1,200 annually throug...

See Answer

Q: You are evaluating two different cookie-baking ovens. The Pillsbury

You are evaluating two different cookie-baking ovens. The Pillsbury 707 costs $57,000, has a five-year life, and has an annual OCF (after tax) of -$10,000 per year. The Keebler CookieMunster costs $90...

See Answer

Q: Suppose you sell a fixed asset for $109,000 when

Suppose you sell a fixed asset for $109,000 when its book value is $129,000. If your company’s marginal tax rate is 39 percent, what will be the effect on cash flows of this sale (i.e., what will be t...

See Answer

Q: Your firm needs a computerized machine tool lathe which costs $50

Your firm needs a computerized machine tool lathe which costs $50,000, and requires $12,000 in maintenance for each year of its three-year life.

See Answer

Q: If the lathe in the previous problem can be sold for $

If the lathe in the previous problem can be sold for $5,000 at the end of year 3, what is the after-tax salvage value?

See Answer

Q: In M&M’s perfect world, will the debt holders ever

In M&M’s perfect world, will the debt holders ever bear any of the risk of the firm?

See Answer

Q: You have been asked by the president of your company to evaluate

You have been asked by the president of your company to evaluate the proposed acquisition of a new special-purpose truck for $60,000. The truck falls into the MACRS three-year class, and it will be so...

See Answer

Q: KADS, Inc. has spent $400,000 on research

KADS, Inc. has spent $400,000 on research to develop a new computer game. The firm is planning to spend $200,000 on a machine to produce the new game. Shipping and installation costs of the machine wi...

See Answer

Q: Your company, Dawgs “R” Us, is evaluating a

Your company, Dawgs “R” Us, is evaluating a new project involving the purchase of a new oven to bake your hotdog buns. If purchased, the new oven will replace your existing oven, which was purchased...

See Answer

Q: How is the pro forma statement we used in this chapter for

How is the pro forma statement we used in this chapter for computing OCF different from an accountant’s income statement?

See Answer

Q: Suppose you paid your old college finance professor to evaluate a project

Suppose you paid your old college finance professor to evaluate a project for you. If you would pay him regardless of your decision concerning whether to proceed with the project, should his fee for e...

See Answer

Q: Why does a decrease in NWC result in a cash inflow to

Why does a decrease in NWC result in a cash inflow to the firm?

See Answer

Q: Everything else held constant, would you rather depreciate a project with

Everything else held constant, would you rather depreciate a project with straight-line depreciation or with DDB?

See Answer

Q: Everything else held constant, would you rather depreciate a project with

Everything else held constant, would you rather depreciate a project with DDB depreciation or deduct it under a Section 179 deduction?

See Answer

Q: In a replacement problem, would we ever see changes in NWC

In a replacement problem, would we ever see changes in NWC?

See Answer

Q: In a replacement problem, will incremental net depreciation always be less

In a replacement problem, will incremental net depreciation always be less than the gross depreciation on the new piece of equipment?

See Answer

Q: Would it be possible for a firm to announce a “reverse

Would it be possible for a firm to announce a “reverse stock dividend”?

See Answer

Q: Why does allowing for the existence of corporate taxation cause firms to

Why does allowing for the existence of corporate taxation cause firms to prefer the maximum amount of debt possible?

See Answer

Q: In a cost-cutting proposal, what might cause you to

In a cost-cutting proposal, what might cause you to sometimes have negative EBIT?

See Answer

Q: How many TVM formulas do you use every time you calculate EAC

How many TVM formulas do you use every time you calculate EAC for a project?

See Answer

Q: Will an increase in flotation costs increase or decrease the initial cash

Will an increase in flotation costs increase or decrease the initial cash flow for a project?

See Answer

Q: Explain why the divisional cost of capital approach may cause problems if

Explain why the divisional cost of capital approach may cause problems if new projects are assigned to the wrong division.

See Answer

Q: How would you handle calculating the cost of capital if a firm

How would you handle calculating the cost of capital if a firm were planning to issue two different classes of common stock?

See Answer

Q: Why don’t we multiply the cost of preferred stock by one minus

Why don’t we multiply the cost of preferred stock by one minus the tax rate, as we do for debt?

See Answer

Q: Expressing WACC in terms of iE, iP, and iD,

Expressing WACC in terms of iE, iP, and iD, what is the theoretical minimum for the WACC?

See Answer

Q: Under what situations would you want to use the CAPM approach for

Under what situations would you want to use the CAPM approach for estimating the component cost of equity? The constant-growth model?

See Answer

Q: Could you calculate the component cost of equity for a stock with

Could you calculate the component cost of equity for a stock with nonconstant expected growth rates in dividends if you didn’t have the information necessary to compute the component cost using the CA...

See Answer

Q: Why do we use market-based weights instead of book-

Why do we use market-based weights instead of book-value-based weights when computing the WACC?

See Answer

Q: If a firm increased the amount of debt in its capital structure

If a firm increased the amount of debt in its capital structure, but a shareholder wanted to switch back to the mixture of expected return and risk she had before the switch, how would she go about do...

See Answer

Q: Suppose your firm wanted to expand into a new line of business

Suppose your firm wanted to expand into a new line of business quickly, and that management anticipated that the new line of business would constitute over 80 percent of your firm’s operations within...

See Answer

Q: When will the subjective approach to forming divisional WACCs be better than

When will the subjective approach to forming divisional WACCs be better than using the firmwide WACC to evaluate all projects?

See Answer

Q: Suppose a new project was going to be financed partially with retained

Suppose a new project was going to be financed partially with retained earnings. What flotation costs should you use for retained earnings?

See Answer

Q: Suppose your firm has decided to use a divisional WACC approach to

Suppose your firm has decided to use a divisional WACC approach to analyze projects. The firm currently has four divisions, A through D, with average betas for each division of 0.9, 1.1, 1.3, and 1.5,...

See Answer

Q: An all-equity firm is considering the projects shown as follows

An all-equity firm is considering the projects shown as follows. The T-bill rate is 4 percent and the market risk premium is 7 percent. If the firm uses its current WACC of 12 percent to evaluate thes...

See Answer

Q: An all-equity firm is considering the projects shown as follows

An all-equity firm is considering the projects shown as follows. The T-bill rate is 4 percent and the market risk premium is 7 percent. If the firm uses its current WACC of 12 percent to evaluate thes...

See Answer

Q: Suppose your firm has decided to use a divisional WACC approach to

Suppose your firm has decided to use a divisional WACC approach to analyze projects. The firm currently has four divisions, A through D, with average betas for each division of 0.6, 1.0, 1.3, and 1.6,...

See Answer

Q: A firm is considering a project that will generate perpetual after-

A firm is considering a project that will generate perpetual after-tax cash flows of $15,000 per year beginning next year. The project has the same risk as the firm's overall operations and must be fi...

See Answer

Q: PDQ, Inc. expects EBIT to be approximately $11 million

PDQ, Inc. expects EBIT to be approximately $11 million per year for the foreseeable future, and that they have 25,000 20-years, 8 percent annual coupon bonds outstanding. What would the appropriate ta...

See Answer

Q: KatyDid Clothes has a $150 million (face value) 30

KatyDid Clothes has a $150 million (face value) 30-year bond issue selling for 104 percent of par that carries a coupon rate of 11 percent, paid semiannually. What would be Katydid’s before-tax compon...

See Answer

Q: If an investor wanted to reduce the risk of a levered stock

If an investor wanted to reduce the risk of a levered stock in their portfolio, how could they go about doing so while still retaining shares in the company?

See Answer

Q: Suppose that LilyMac Photography expects EBIT to be approximately $200,

Suppose that LilyMac Photography expects EBIT to be approximately $200,000 per year for the foreseeable future, and that they have 1,000 10-years, 9 percent annual coupon bonds outstanding. What would...

See Answer

Q: JaiLai Cos. stock has a beta of 0.9,

JaiLai Cos. stock has a beta of 0.9, the current risk-free rate is 6.2 percent, and the expected return on the market is 12 percent. What is JaiLai’s cost of equity?

See Answer

Q: Oberon, Inc. has a $20 million (face value

Oberon, Inc. has a $20 million (face value) 10-year bond issue selling for 97 percent of par that pays an annual coupon of 8.25 percent. What would be Oberon’s before-tax component cost of debt?

See Answer

Q: ILK has preferred stock selling for 97 percent of par that pays

ILK has preferred stock selling for 97 percent of par that pays an 8 percent annual coupon. What would be ILK’s component cost of preferred stock?

See Answer

Q: Marme, Inc. has preferred stock selling for 96 percent of

Marme, Inc. has preferred stock selling for 96 percent of par that pays an 11 percent annual coupon. What would be Marme’s component cost of preferred stock?

See Answer

Q: FarCry Industries, a maker of telecommunications equipment, has two million

FarCry Industries, a maker of telecommunications equipment, has two million shares of common stock outstanding, one million shares of preferred stock outstanding, and 10,000 bonds. If the common share...

See Answer

Q: OMG Inc. has four million shares of common stock outstanding,

OMG Inc. has four million shares of common stock outstanding, three million shares of preferred stock outstanding, and 5,000 bonds. If the common shares are selling for $17 per share, the preferred sh...

See Answer

Q: FarCry Industries, a maker of telecommunications equipment, has two million

FarCry Industries, a maker of telecommunications equipment, has two million shares of common stock outstanding, one million shares of preferred stock outstanding, and 10,000 bonds. If the common share...

See Answer

Q: OMG Inc. has four million shares of common stock outstanding,

OMG Inc. has four million shares of common stock outstanding, three million shares of preferred stock outstanding, and 5,000 bonds. If the common shares are selling for $27 per share, the preferred sh...

See Answer

Q: FarCry Industries, a maker of telecommunications equipment, has two million

FarCry Industries, a maker of telecommunications equipment, has two million shares of common stock outstanding, one million shares of preferred stock outstanding, and 10,000 bonds. If the common share...

See Answer

Q: Suppose you were the financial manager for a firm and were considering

Suppose you were the financial manager for a firm and were considering a proposed increase in the amount of debt in the firm’s capital structure. If you thought the firm was going to consistently earn...

See Answer

Q: OMG Inc. has four million shares of common stock outstanding,

OMG Inc. has four million shares of common stock outstanding, three million shares of preferred stock outstanding, and 5,000 bonds. If the common shares sell for $17 per share, the preferred shares se...

See Answer

Q: Diddy Corp. stock has a beta of 1.2,

Diddy Corp. stock has a beta of 1.2, the current risk-free rate is 5 percent, and the expected return on the market is 13.5 percent. What is Diddy’s cost of equity?

See Answer

Q: WhackAmOle has two million shares of common stock outstanding, 1.

WhackAmOle has two million shares of common stock outstanding, 1.5 million shares of preferred stock outstanding, and 50,000 bonds. If the common shares are selling for $63 per share, the preferred sh...

See Answer

Q: TAFKAP Industries has three million shares of stock outstanding selling at $

TAFKAP Industries has three million shares of stock outstanding selling at $17 per share and an issue of $20 million in 7.5 percent, annual coupon bonds with a maturity of 15 years, selling at 106 per...

See Answer

Q: Johnny Cake Ltd. has ten million shares of stock outstanding selling

Johnny Cake Ltd. has ten million shares of stock outstanding selling at $23 per share and an issue of $50 million in 9 percent, annual coupon bonds with a maturity of 17 years, selling at 93.5 percent...

See Answer

Q: BetterPie Industries has three million shares of common stock outstanding, two

BetterPie Industries has three million shares of common stock outstanding, two million shares of preferred stock outstanding, and 10,000 bonds. If the common shares are selling for $47 per share, the...

See Answer

Q: Suppose that MNINK Industries’ capital structure features 63 percent equity, 7

Suppose that MNINK Industries’ capital structure features 63 percent equity, 7 percent preferred stock, and 30 percent debt. If the before-tax component costs of equity, preferred stock and debt are 1...

See Answer

Q: Suppose that Brown-Murphies’ common shares sell for $19.

Suppose that Brown-Murphies’ common shares sell for $19.50 per share, that the firm is expected to set their next annual dividend at $0.57 per share, and that all future dividends are expected to grow...

See Answer

Q: Suppose that TapDance, Inc.’s capital structure features 65 percent

Suppose that TapDance, Inc.’s capital structure features 65 percent equity, 35 percent debt, and that its before-tax cost of debt is 8 percent, while its cost of equity is 13 percent. If the appropria...

See Answer

Q: Suppose that JB Cos. has a capital structure of 78 percent

Suppose that JB Cos. has a capital structure of 78 percent equity, 22 percent debt, and that its before-tax cost of debt is 11 percent while its cost of equity is 15 percent. If the appropriate weight...

See Answer

Q: Explain why, in a world with both corporate taxes and the

Explain why, in a world with both corporate taxes and the chance of bankruptcy, a small firm with volatile EBIT is unlikely to have much debt.

See Answer

Q: Suppose that B2B, Inc. has a capital structure of 37

Suppose that B2B, Inc. has a capital structure of 37 percent equity, 17 percent preferred stock, and 46 percent debt. If the before-tax component costs of equity, preferred stock and debt are 14.5 per...

See Answer

Q: LilyMac Studios, a national chain of photography studios, is considering

LilyMac Studios, a national chain of photography studios, is considering opening up a chain of coffee shop/art galleries. While the existing operations of the firm have a beta of 1.17, the new chain i...

See Answer

Q: LilyMac Studios, a national chain of photography studios, is considering

LilyMac Studios, a national chain of photography studios, is considering opening up a chain of coffee shop/art galleries. While the existing operations of the firm have a beta of 1.17, the new chain i...

See Answer

Q: Why do we use market-based weights instead of book-

Why do we use market-based weights instead of book-value-based weights when computing the WACC?

See Answer

Q: Suppose your firm wanted to expand into a new line of business

Suppose your firm wanted to expand into a new line of business quickly, and that management anticipated that the new line of business would constitute over 80 percent of your firm’s operations within...

See Answer

Q: When will the subjective approach to forming divisional WACCs be better than

When will the subjective approach to forming divisional WACCs be better than using the firmwide WACC to evaluate all projects?

See Answer

Q: Suppose a new project was going to be financed partially with retained

Suppose a new project was going to be financed partially with retained earnings. What flotation costs should you use for retained earnings?

See Answer

Q: How would you handle calculating the cost of capital if a firm

How would you handle calculating the cost of capital if a firm were planning to issue two different classes of common stock?

See Answer

Q: Why don’t we multiply the cost of preferred stock by one minus

Why don’t we multiply the cost of preferred stock by one minus the tax rate, as we do for debt?

See Answer

Q: Expressing WACC in terms of iE, iP, and iD,

Expressing WACC in terms of iE, iP, and iD, what is the theoretical minimum for the WACC?

See Answer

Q: If the U.S. government completely eliminated taxation at the

If the U.S. government completely eliminated taxation at the corporate level, how would this influence the capital structures of firms in a world with bankruptcy?

See Answer

Q: Under what situations would you want to use the CAPM approach for

Under what situations would you want to use the CAPM approach for estimating the component cost of equity? The constant-growth model?

See Answer

Q: Could you calculate the component cost of equity for a stock with

Could you calculate the component cost of equity for a stock with nonconstant expected growth rates in dividends if you didn’t have the information necessary to compute the component cost using the CA...

See Answer

Q: The average annual return on the S&P 500 Index from

The average annual return on the S&P 500 Index from 1996 to 2005 was 10.8 percent. The average annual T-bill yield during the same period was 3.6 percent. What was the market risk premium during these...

See Answer

Q: Hastings Entertainment has a beta of 0.65. If the

Hastings Entertainment has a beta of 0.65. If the market return is expected to be 11 percent and the risk-free rate is 4 percent, what is Hastings’ required return?

See Answer

Q: Nanometrics, Inc. has a beta of 3.15.

Nanometrics, Inc. has a beta of 3.15. If the market return is expected to be 10 percent and the risk-free rate is 3.5 percent, what is Nanometrics’ required return?

See Answer

Q: Netflix, Inc. has a beta of 3.61.

Netflix, Inc. has a beta of 3.61. If the market return is expected to be 13 percent and the risk-free rate is 3 percent, what is Netflix’ risk premium?

See Answer

Q: Paycheck, Inc. has a beta of 0.94.

Paycheck, Inc. has a beta of 0.94. If the market return is expected to be 11 percent and the risk-free rate is 3 percent, what is Paycheck’s risk premium?

See Answer

Q: You have a portfolio with a beta of 1.35.

You have a portfolio with a beta of 1.35. What will be the new portfolio beta if you keep 85 percent of your money in the old portfolio and 5 percent in a stock with a beta of 0.78?

See Answer

Q: You have a portfolio with a beta of 1.1.

You have a portfolio with a beta of 1.1. What will be the new portfolio beta if you keep 85 percent of your money in the old portfolio and 15 percent in a stock with a beta of 0.5?

See Answer

Q: The Nasdaq stock market bubble peaked at 4,816 in 2000

The Nasdaq stock market bubble peaked at 4,816 in 2000. Two and a half years later it had fallen to 1,000. What was the percentage decline?

See Answer

Q: Would you expect a utility company to have high or low debt

Would you expect a utility company to have high or low debt levels? Why?

See Answer

Q: The Japanese stock market bubble peaked at 38,916 in 1989

The Japanese stock market bubble peaked at 38,916 in 1989. Two and a half years later it had fallen to 15,900. What was the percentage decline?

See Answer

Q: Paccar’s current stock price is $48.20 and it is

Paccar’s current stock price is $48.20 and it is likely to pay a $0.80 dividend next year. Since analysts estimate Paccar will have an 8.8% growth rate, what is its required return?

See Answer

Q: Universal Forest’s current stock price is $57.50 and it

Universal Forest’s current stock price is $57.50 and it is likely to pay a $0.26 dividend next year. Since analysts estimate Universal Forest will have a 9.5 percent growth rate, what is its required...

See Answer

Q: Compute the expected return given these three economic states, their likelihoods

Compute the expected return given these three economic states, their likelihoods, and the potential returns:

See Answer

Q: You own $7,000 of Human Genome stock that has

You own $7,000 of Human Genome stock that has a beta of 3.5. You also own $8,000 of Frozen Food Express (beta = 1.6) and $10,000 of Molecular Devices (beta = 0.4). What is the beta of your portfolio?...

See Answer

Q: You own $10,000 of Olympic Steel stock that has

You own $10,000 of Olympic Steel stock that has a beta of 2.2. You also own $7,000 of Rent-a-Center (beta = 1.5) and $8,000 of Lincoln Educational (beta = 0.5). What is the beta of your portfolio?

See Answer

Q: For the same economic state probability distribution in Problem 10-2

For the same economic state probability distribution in Problem 10-2, determine the standard deviation of the expected return.

See Answer

Q: A manager believes his firm will earn a 14 percent return next

A manager believes his firm will earn a 14 percent return next year. His firm has a beta of 1.5, the expected return on the market is 12 percent, and the risk-free rate is 4 percent. Compute the re...

See Answer

Q: A manager believes his firm will earn a 14 percent return next

A manager believes his firm will earn a 14 percent return next year. His firm has a beta of 1.2, the expected return on the market is 11 percent, and the risk-free rate is 5 percent. Compute the ret...

See Answer

Q: For the same economic state probability distribution in Problem 10-1

For the same economic state probability distribution in Problem 10-1, determine the standard deviation of the expected return.

See Answer

Q: No Nuns Cos. has a 25 percent tax rate and has

No Nuns Cos. has a 25 percent tax rate and has $350 million in assets, currently financed entirely with equity. Equity is worth $37 per share, and book value of equity is equal to market value of equi...

See Answer

Q: When you go on the Web to find a firm’s beta,

When you go on the Web to find a firm’s beta, you do not know how recently it was computed, what index was used as a proxy for the market portfolio, or which time series of returns t...

See Answer

Q: Build a spreadsheet that automatically computes the expected market return and risk

Build a spreadsheet that automatically computes the expected market return and risk for different assumptions about the state of the economy. a. First, create the following spreadsheet and compute th...

See Answer

Q: As discussed in the text, beta estimates for one firm will

As discussed in the text, beta estimates for one firm will vary depending on various factors like such as the time over which the estimation is conducted, the market portfolio proxy, and the return in...

See Answer

Q: Compare and contrast the assumptions that need to be made to compute

Compare and contrast the assumptions that need to be made to compute a required return using CAPM and the constant growth rate model.

See Answer

Q: Consider an asset that provides the same return no matter what economic

Consider an asset that provides the same return no matter what economic state occurs. What would be the standard deviation (or risk) of this asset? Explain.

See Answer

Q: Why expected return is considered “forward-looking”? What are

Why expected return is considered “forward-looking”? What are the challenges for practitioners to utilize expected return?

See Answer

Q: In 2000, the S&P 500 Index earned −9

In 2000, the S&P 500 Index earned −9.1 percent while the T-bill yield was 5.9 percent. Does this mean the market risk premium was negative? Explain.

See Answer

Q: How might the magnitude of the market risk premium impact people’s desire

How might the magnitude of the market risk premium impact people’s desire to buy stocks?

See Answer

Q: Describe how adding a risk-free security to modern portfolio theory

Describe how adding a risk-free security to modern portfolio theory allows investors to do better than the efficient frontier.

See Answer

Q: Show on a graph like Figure 10-2 where a stock

Show on a graph like Figure 10-2 where a stock with a beta of 1.3 would be located on the security market line. Then show where that stock would be located if it is undervalued.

See Answer

Q: GTB, Inc., has a 34 percent tax rate and has

GTB, Inc., has a 34 percent tax rate and has $100 million in assets, currently financed entirely with equity. Equity is worth $7 per share, and book value of equity is equal to market value of equity....

See Answer

Q: Consider that you have three stocks in your portfolio and wish to

Consider that you have three stocks in your portfolio and wish to add a fourth. You want to know if the fourth stock will make the portfolio riskier or less risky. Compare and contrast how this would...

See Answer

Q: Describe how different allocations between the risk-free security and the

Describe how different allocations between the risk-free security and the market portfolio can achieve any level of market risk desired. Give examples of a portfolio from a person who is very risk ave...

See Answer

Q: Note from Table 10-2 that some technology-oriented firms

Note from Table 10-2 that some technology-oriented firms (Apple) in the Dow Jones Industrial Average have high market risk while others (Intel and Verizon) have low market risk. How do you explain thi...

See Answer

Q: Explain how the concept of a positive risk-return relationship breaks

Explain how the concept of a positive risk-return relationship breaks down if you can systematically find stocks that are overvalued and undervalued.

See Answer

Q: Why do most investment scams conducted over the Internet and e-

Why do most investment scams conducted over the Internet and e-mail involve penny stocks instead of S&P 500 Index stocks?

See Answer

Q: Describe a stock market bubble. Can a bubble occur in a

Describe a stock market bubble. Can a bubble occur in a single stock?

See Answer

Q: If stock prices are not strong-form efficient, then what

If stock prices are not strong-form efficient, then what might be the price reaction to a firm announcing a stock buyback? Explain.

See Answer

Q: Find a beta estimate from three different sources for General Electric (

Find a beta estimate from three different sources for General Electric (GE). Compare these three values. Why might they be different?

See Answer

Q: If you were to compute beta yourself, what choices would you

If you were to compute beta yourself, what choices would you make regarding the market portfolio, the holding period for the returns (daily, weekly, etc.), and the number of returns? Justify your choi...

See Answer

Q: Determine what level of market efficiency each event is consistent with:

Determine what level of market efficiency each event is consistent with: a. Immediately after an earnings announcement the stock price jumps and then stays at the new level. b. The CEO buys 50,000 sha...

See Answer

Q: NoNuns Cos. has a 25 percent tax rate and has $

NoNuns Cos. has a 25 percent tax rate and has $350 million in assets, currently financed entirely with equity. Equity is worth $37 per share, and book value of equity is equal to market value of equit...

See Answer

Q: How should you handle a case where required return computations from CAPM

How should you handle a case where required return computations from CAPM and the constant growth rate model are very different?

See Answer

Q: Cisco Systems has a beta of 1.25. Does this

Cisco Systems has a beta of 1.25. Does this mean that you should expect Cisco to earn a return 25 percent higher than the S&P 500 Index return? Explain.

See Answer

Q: Create the spreadsheet below. The spreadsheet should use the returns for

Create the spreadsheet below. The spreadsheet should use the returns for assets A and B to form a portfolio return using the weights for each asset shown in cells E1 and E2. The average portfolio retu...

See Answer

Q: You have a portfolio with an asset allocation of 50 percent stocks

You have a portfolio with an asset allocation of 50 percent stocks, 40 percent long-term Treasury bonds, and 10 percent T-bills. Use these weights and the returns in Table 9.2 to compute the return of...

See Answer

Q: You have a portfolio with an asset allocation of 35 percent stocks

You have a portfolio with an asset allocation of 35 percent stocks, 55 percent long-term Treasury bonds, and 10 percent T-bills. Use these weights and the returns in Table 9.2 to compute the return of...

See Answer

Q: You have $15,000 to invest. You want to

You have $15,000 to invest. You want to purchase shares of Alaska Air at $42.88, Best Buy at $51.32, and Ford Motor at $8.51. How many shares of each company should you purchase so that your portfoli...

See Answer

Q: The table below shows your stock positions at the beginning of the

The table below shows your stock positions at the beginning of the year, the dividends that each stock paid during the year, and the stock prices at the end of the year. What is your portfolio dollar...

See Answer

Q: The table below shows your stock positions at the beginning of the

The table below shows your stock positions at the beginning of the year, the dividends that each stock paid during the year, and the stock prices at the end of the year. What is your portfolio dollar...

See Answer

Q: Consider the following annual returns of Estee Lauder and Lowe’s Companies:

Consider the following annual returns of Estee Lauder and Lowe’s Companies: Compute each stock’s average return, standard deviation, and coefficient of variation. W...

See Answer

Q: Consider the following annual returns of Molson Coors and International Paper:

Consider the following annual returns of Molson Coors and International Paper: Compute each stock’s average return, standard deviation, and coefficient of variation. Which stock appe...

See Answer

Q: Why might firms prefer to conduct stock repurchases through open-market

Why might firms prefer to conduct stock repurchases through open-market operations rather than through fixed-price tender offers?

See Answer

Q: GTB, Inc., has a 34 percent tax rate and has

GTB, Inc., has a 34 percent tax rate and has $100 million in assets, currently financed entirely with equity. Equity is worth $7 per share, and book value of equity is equal to market value of equity....

See Answer

Q: Below are the monthly returns for March 2011 to February 2016 of

Below are the monthly returns for March 2011 to February 2016 of three international stock indices; All Ordinaries of Australia, Nikkei 225 of Japan, and FTSE 100 of England. A. Compute and compare e...

See Answer

Q: You have $20,000 to invest. You want to

You have $20,000 to invest. You want to purchase shares of Xerox at $17.34, Qwest at $8.15, and Liz Claiborne at $44.73. How many shares of each company should you purchase so that your portfolio cons...

See Answer

Q: FedEx Corp stock ended the previous year at $103.39

FedEx Corp stock ended the previous year at $103.39 per share. It paid a $0.35 per share dividend last year. It ended last year at $106.69. If you owned 200 shares of FedEx, what was your dollar retur...

See Answer

Q: Sprint Nextel Corp stock ended the previous year at $23.

Sprint Nextel Corp stock ended the previous year at $23.36 per share. It paid a $2.37 per share dividend last year. It ended last year at $18.89. If you owned 500 shares of Sprint, what was your doll...

See Answer

Q: A corporate bond that you own at the beginning of the year

A corporate bond that you own at the beginning of the year is worth $975. During the year, it pays $35 in interest payments and ends the year valued at $965. What was your dollar return and percent re...

See Answer

Q: A Treasury bond that you own at the beginning of the year

A Treasury bond that you own at the beginning of the year is worth $1,055. During the year, it pays $35 in interest payments and ends the year valued at $1,065. What was your dollar return and percent...

See Answer

Q: Rank the following three stocks by their level of total risk,

Rank the following three stocks by their level of total risk, highest to lowest. Rail Haul has an average return of 12 percent and standard deviation of 25 percent. The average return and standard dev...

See Answer

Q: Rank the following three stocks by their total risk level, highest

Rank the following three stocks by their total risk level, highest to lowest. Night Ryder has an average return of 12 percent and standard deviation of 32 percent. The average return and standard devi...

See Answer

Q: Rank the following three stocks by their risk-return relationship,

Rank the following three stocks by their risk-return relationship, best to worst. Rail Haul has an average return of 12 percent and standard deviation of 25 percent. The average return and standard de...

See Answer

Q: Rank the following three stocks by their risk-return relationship,

Rank the following three stocks by their risk-return relationship, best to worst. Night Ryder has an average return of 12 percent and standard deviation of 32 percent. The average return and standard...

See Answer

Q: NoNuns Cos. has a 25 percent tax rate and has $

NoNuns Cos. has a 25 percent tax rate and has $350 million in assets, currently financed entirely with equity. Equity is worth $37 per share, and book value of equity is equal to market value of equit...

See Answer

Q: Determine which one of these three portfolios dominates another. Name the

Determine which one of these three portfolios dominates another. Name the dominated portfolio and the portfolio that dominates it. Portfolio Blue has an expected return of 12 percent and risk of 18 pe...

See Answer

Q: Determine which one of the three portfolios dominates another. Name the

Determine which one of the three portfolios dominates another. Name the dominated portfolio and the portfolio that dominates it. Portfolio Green has an expected return of 15 percent and risk of 21 p...

See Answer

Q: An investor owns $6,000 of Adobe Systems stock,

An investor owns $6,000 of Adobe Systems stock, $5,000 of Dow Chemical, and $5,000 of Office Depot. What are the portfolio weights of each stock?

See Answer

Q: An investor owns $3,000 of Adobe Systems stock,

An investor owns $3,000 of Adobe Systems stock, $6,000 of Dow Chemical, and $7,000 of Office Depot. What are the portfolio weights of each stock?

See Answer

Q: Year-to-date, Oracle had earned a −1

Year-to-date, Oracle had earned a −1.34 percent return. During the same time period, Valero Energy earned 7.96 percent and McDonalds earned 0.88 percent. If you have a portfolio made up of 30 percent...

See Answer

Q: Year to date, Yum Brands had earned a 3.80

Year to date, Yum Brands had earned a 3.80 percent return. During the same time period, Raytheon earned 4.26 percent and Coca-Cola earned −0.46 percent. If you have a portfolio made up of 30 percent Y...

See Answer

Q: At the beginning of the month, you owned $6,

At the beginning of the month, you owned $6,000 of News Corp, $5,000 of First Data, and $8,500 of Whirlpool. The monthly returns for News Corp, First Data, and Whirlpool were 8.24 percent, −2.59 perc...

See Answer

Q: Assess the risk-return relationship of the bond market (see

Assess the risk-return relationship of the bond market (see Tables 9.2 and 9.4) during each decade since 1950.

See Answer

Q: Assess the risk-return relationship in T-bills (see

Assess the risk-return relationship in T-bills (see Tables 9.2 and 9.4) during each decade since 1950.

See Answer

Q: Consider the characteristics of the following three stocks: /

Consider the characteristics of the following three stocks: The correlation between Thumb Devices and Air Comfort is −0.12. The correlation between Thumb Devices and Sport Garb is &a...

See Answer

Q: GTB, Inc., has a 34 percent tax rate and has

GTB, Inc., has a 34 percent tax rate and has $100 million in assets, currently financed entirely with equity. Equity is worth $7 per share, and book value of equity is equal to market value of equity....

See Answer

Q: Consider the characteristics of the following three stocks: /

Consider the characteristics of the following three stocks: The correlation between Pic Image and Tax Help is 0.88. The correlation between Pic Image and Warm Wear is −0.21. The cor...

See Answer

Q: If you own 200 shares of Alaska Air at $42.

If you own 200 shares of Alaska Air at $42.88, 350 shares of Best Buy at $51.32, and 250 shares of Ford Motor at $8.51, what are the portfolio weights of each stock?

See Answer

Q: At the beginning of the month, you owned $5,

At the beginning of the month, you owned $5,500 of General Dynamics, $7,500 of Starbucks, and $8,000 of Nike. The monthly returns for General Dynamics, Starbucks, and Nike were 7.44 percent, −1.36 per...

See Answer

Q: The past five monthly returns for Kohl’s are 4.11 percent

The past five monthly returns for Kohl’s are 4.11 percent, 3.62 percent, −1.68 percent, 9.25 percent, and −2.56 percent. What is the average monthly return?

See Answer

Q: The past five monthly returns for PG&E are −3

The past five monthly returns for PG&E are −3.17 percent, 3.88 percent, 3.77 percent, 6.47 percent, and 3.58 percent. What is the average monthly return?

See Answer

Q: Compute the standard deviation of Kohls’ monthly returns shown in Problem 9

Compute the standard deviation of Kohls’ monthly returns shown in Problem 9-15.

See Answer

Q: Compute the standard deviation of PG&E’s monthly returns shown in

Compute the standard deviation of PG&E’s monthly returns shown in Problem 9-16.

See Answer

Q: If you own 400 shares of Xerox at $17.34

If you own 400 shares of Xerox at $17.34, 500 shares of Qwest at $8.15, and 350 shares of Liz Claiborne at $44.73, what are the portfolio weights of each stock?

See Answer

Q: Many more types of investments are available besides stocks, bonds,

Many more types of investments are available besides stocks, bonds, and cash securities. Many people invest in real estate and in precious metals, primarily gold. What are the risk and return characte...

See Answer

Q: Say you own 200 shares of Mattel and 100 shares of Staples

Say you own 200 shares of Mattel and 100 shares of Staples. Would your portfolio return be different if you instead owned 100 shares of Mattel and 200 shares of Staples? Why?

See Answer

Q: Suppose that Papa Bell, Inc.’s, equity is currently

Suppose that Papa Bell, Inc.’s, equity is currently selling for $45 per share, with 4 million shares outstanding. The firm also has seven thousand bonds outstanding, which are selling at 94 percent of...

See Answer

Q: Why is the percentage return a more useful measure than the dollar

Why is the percentage return a more useful measure than the dollar return?

See Answer

Q: Characterize the historical return, risk, and risk-return relationship

Characterize the historical return, risk, and risk-return relationship of the stock, bond and cash markets.

See Answer

Q: How do we define risk in this chapter and how do we

How do we define risk in this chapter and how do we measure it?

See Answer

Q: What are the two components of total risk? Which component is

What are the two components of total risk? Which component is part of the risk-return relationship? Why?

See Answer

Q: Which company is likely to have lower total risk, General Electric

Which company is likely to have lower total risk, General Electric or Coca-Cola? Why?

See Answer

Q: Can a company change its total risk level over time? How

Can a company change its total risk level over time? How?

See Answer

Q: What does the coefficient of variation measure? Why is a lower

What does the coefficient of variation measure? Why is a lower value better for the investor?

See Answer

Q: You receive an investment newsletter advertisement in the mail. The letter

You receive an investment newsletter advertisement in the mail. The letter claims that you should invest in a stock that has doubled the return of the S&P 500 Index over the last three months. It also...

See Answer

Q: What does diversification do to the risk and return characteristics of a

What does diversification do to the risk and return characteristics of a portfolio?

See Answer

Q: Describe the diversification potential of two assets with a −0.

Describe the diversification potential of two assets with a −0.8 correlation. What’s the potential if the correlation is +0.8?

See Answer

Q: Suppose that Lil John Industries’ equity is currently selling for $27

Suppose that Lil John Industries’ equity is currently selling for $27 per share and that there are 2 million shares outstanding. The firm also has 50 thousand bonds outstanding, which are selling at 1...

See Answer

Q: You own only two stocks in your portfolio but want to add

You own only two stocks in your portfolio but want to add more. When you add a third stock, the total risk of your portfolio declines. When you add a tenth stock to the portfolio, the total risk decl...

See Answer

Q: Many employees believe that their employer’s stock is less likely to lose

Many employees believe that their employer’s stock is less likely to lose half of its value than a well diversified portfolio of stocks. Explain why this belief is erroneous.

See Answer

Q: If an investor’s desired risk level changes over time, should the

If an investor’s desired risk level changes over time, should the investor change the composition of his or her portfolio? How?

See Answer

Q: You are a risk adverse investor with a low-risk portfolio

You are a risk adverse investor with a low-risk portfolio of bonds. How is it possible that adding some stocks (which are riskier than bonds) to the portfolio can lower the total risk of the portfolio...

See Answer

Q: Consider a firm that had been priced using an 11.5

Consider a firm that had been priced using an 11.5 percent growth rate and a 13.5 percent required return. The firm recently paid a $1.50 dividend. The firm has just announced that because of a new jo...

See Answer

Q: A 3.85 percent coupon municipal bond has 18 years left

A 3.85 percent coupon municipal bond has 18 years left to maturity and has a price quote of 103.20. The bond can be called in eight years. The call premium is one year of coupon payments. Compute and...

See Answer

Q: A corporate bond with a 6.5 percent coupon has 15

A corporate bond with a 6.5 percent coupon has 15 years left to maturity. It has had a credit rating of BBB and a yield to maturity of 7.2 percent. The firm has recently gotten into some trouble and t...

See Answer

Q: Consider a 3.5 percent TIPS with an issue CPI reference

Consider a 3.5 percent TIPS with an issue CPI reference of 185.6. At the beginning of this year, the CPI was 193.5 and was at 199.6 at the end of the year. What was the capital gain of the TIPS in dol...

See Answer

Q: Consider a 2.25 percent TIPS with an issue CPI reference

Consider a 2.25 percent TIPS with an issue CPI reference of 187.2. At the beginning of this year, the CPI was 197.1 and was at 203.8 at the end of the year. What was the capital gain of the TIPS in...

See Answer

Q: Compute the price of a 3.8 percent coupon bond with

Compute the price of a 3.8 percent coupon bond with 15 years left to maturity and a market interest rate of 6.8 percent. (Assume interest payments are semiannual.) Is this a discount or premium bond?...

See Answer

Q: Compute the price of a 5.6 percent coupon bond with

Compute the price of a 5.6 percent coupon bond with ten years left to maturity and a market interest rate of 7.0 percent. (Assume interest payments are semiannual.) Is this a discount or premium bo...

See Answer

Q: Calculate the price of a 5.2 percent coupon bond with

Calculate the price of a 5.2 percent coupon bond with 18 years left to maturity and a market interest rate of 4.6 percent. (Assume interest payments are semiannual.) Is this a discount or premium bond...

See Answer

Q: Calculate the price of a 5.7 percent coupon bond with

Calculate the price of a 5.7 percent coupon bond with 22 years left to maturity and a market interest rate of 6.5 percent. (Assume interest payments are semiannual.) Is this a discount or premium bond...

See Answer

Q: If the risk-free rate is 3 percent and the risk

If the risk-free rate is 3 percent and the risk premium is 5 percent, what is the required return?

See Answer

Q: A 5.65 percent coupon bond with 18 years left to

A 5.65 percent coupon bond with 18 years left to maturity is offered for sale at $1,035.25. What yield to maturity is the bond offering? (Assume interest payments are semiannual.)

See Answer

Q: Suppose that a firm’s recent earnings per share and dividend per share

Suppose that a firm’s recent earnings per share and dividend per share are $2.50 and $1.30, respectively. Both are expected to grow at 8 percent. However, the firm’s current P/E ratio of 22 seems high...

See Answer

Q: A 4.30 percent coupon bond with 14 years left to

A 4.30 percent coupon bond with 14 years left to maturity is offered for sale at $943.22. What yield to maturity is the bond offering? (Assume interest payments are semiannual.)

See Answer

Q: A 6.75 percent coupon bond with 26 years left to

A 6.75 percent coupon bond with 26 years left to maturity can be called in six years. The call premium is one year of coupon payments. It is offered for sale at $1,135.25. What is the yield to call o...

See Answer

Q: A 5.25 percent coupon bond with 14 years left to

A 5.25 percent coupon bond with 14 years left to maturity can be called in four years. The call premium is one year of coupon payments. It is offered for sale at $1,075.50. What is the yield to call...

See Answer

Q: A client in the 39 percent marginal tax bracket is comparing a

A client in the 39 percent marginal tax bracket is comparing a municipal bond that offers a 4.5 percent yield to maturity and a similar-risk corporate bond that offers a 6.45 percent yield. Which bond...

See Answer

Q: A client in the 28 percent marginal tax bracket is comparing a

A client in the 28 percent marginal tax bracket is comparing a municipal bond that offers a 4.5 percent yield to maturity and a similar-risk corporate bond that offers a 6.45 percent yield. Which bond...

See Answer

Q: Land’o’Toys is a profitable, medium-sized, retail company.

Land’o’Toys is a profitable, medium-sized, retail company. Several years ago, it issued a 6½ percent coupon bond, which pays interest semiannually. Th...

See Answer

Q: Say that in June of this year, a company issued bonds

Say that in June of this year, a company issued bonds that are scheduled to mature in three years in June. The coupon rate is 5.75 percent and is paid semiannually. The bond issue was rated AAA. a. Bu...

See Answer

Q: You have a portfolio of three bonds. The Long Bond will

You have a portfolio of three bonds. The Long Bond will mature in 19 years and has a 5.5% coupon rate. The Midterm Bond matures in 9 years and has a 6.6% coupon rate. The Short Bond matures in only 2...

See Answer

Q: Describe the difference between a bond issued as a high-yield

Describe the difference between a bond issued as a high-yield bond and one that has become a “fallen angel.”

See Answer

Q: If the risk-free rate is 4 percent and the risk

If the risk-free rate is 4 percent and the risk premium is 6 percent, what is the required return?

See Answer

Q: Waller Co. paid a $0.286 dividend per share

Waller Co. paid a $0.286 dividend per share in 2006, which grew to $0.55 in 2012. This growth is expected to continue. What is the value of this stock at the beginning of 2013 when the required return...

See Answer

Q: What does a call provision allow issuers to do, and why

What does a call provision allow issuers to do, and why would they do it?

See Answer

Q: List the differences between the new TIPS and traditional Treasury bonds.

List the differences between the new TIPS and traditional Treasury bonds.

See Answer

Q: Explain how mortgage-backed securities work.

Explain how mortgage-backed securities work.

See Answer

Q: Provide the definitions of a discount bond and a premium bond.

Provide the definitions of a discount bond and a premium bond. Give examples.

See Answer

Q: Describe the differences in interest payments and bond price between a 5

Describe the differences in interest payments and bond price between a 5 percent coupon bond and a zero coupon bond.

See Answer

Q: All else equal, which bond’s price is more affected by a

All else equal, which bond’s price is more affected by a change in interest rates, a short-term bond or a longer-term bond? Why?

See Answer

Q: All else equal, which bond’s price is more affected by a

All else equal, which bond’s price is more affected by a change in interest rates, a bond with a large coupon or a small coupon? Why?

See Answer

Q: Compare and contrast the advantages and disadvantages of the current yield computation

Compare and contrast the advantages and disadvantages of the current yield computation versus yield to maturity calculations.

See Answer

Q: What is the yield to call and why is it important to

What is the yield to call and why is it important to a bond investor?

See Answer

Q: What is the purpose of computing the equivalent taxable yield of a

What is the purpose of computing the equivalent taxable yield of a municipal bond?

See Answer

Q: Campbell Supper Co. paid a $0.632 dividend per

Campbell Supper Co. paid a $0.632 dividend per share in 2013, which grew to $0.76 in 2016. This growth is expected to continue. What is the value of this stock at the beginning of 2017 when the requir...

See Answer

Q: The average annual return on the S&P 500 Index from

The average annual return on the S&P 500 Index from 1986 to 1995 was 15.8 percent. The average annual T-bill yield during the same period was 5.6 percent. What was the market risk premium during these...

See Answer

Q: Why does a Treasury bond offer a lower yield than a corporate

Why does a Treasury bond offer a lower yield than a corporate bond with the same time to maturity? Could a corporate bond with a different time to maturity offer a lower yield? Explain.

See Answer

Q: Explain how a bond’s interest rate can change over time even if

Explain how a bond’s interest rate can change over time even if interest rates in the economy do not change.

See Answer

Q: Explain why high income and wealthy people are more likely to buy

Explain why high income and wealthy people are more likely to buy a municipal bond than a corporate bond.

See Answer

Q: What is the difference in the trading volume between Treasury bonds and

What is the difference in the trading volume between Treasury bonds and corporate bonds? Give examples and/or evidence.

See Answer

Q: From discussions with your broker, you have determined that the expected

From discussions with your broker, you have determined that the expected inflation premium is 1.35 percent next year, 1.50 percent in year 2, 1.75 percent in year 3, and 2.00 percent in year 4 and bey...

See Answer

Q: A recent edition of The Wall Street Journal reported interest rates of

A recent edition of The Wall Street Journal reported interest rates of 1.25 percent, 1.60 percent, 1.98 percent, and 2.25 percent for 3-year, 4-year, 5-year, and 6-year Treasury security yields, respe...

See Answer

Q: Suppose that the current 1-year rate (1-year

Suppose that the current 1-year rate (1-year spot rate) and expected 1-year T-bill rates over the following three years (i.e., years 2, 3, and 4, respectively) are as follows:

See Answer

Q: Suppose that the current 1-year rate (1-year

Suppose that the current 1-year rate (1-year spot rate) and expected 1-year T-bill rates over the following three years (i.e., years 2, 3, and 4, respectively) are as follows:

See Answer

Q: Based on economists’ forecasts and analysis, 1-year Treasury bill

Based on economists’ forecasts and analysis, 1-year Treasury bill rates and liquidity premiums for the next four years are expected to be as follows: R1 = 0.65% E(2r1) = 1.75% L2 = 0.05% E(3r1) = 1.8...

See Answer

Q: A fast growing firm recently paid a dividend of $0.

A fast growing firm recently paid a dividend of $0.35 per share. The dividend is expected to increase at a 20 percent rate for the next three years. Afterwards, a more stable 12 percent growth rate ca...

See Answer

Q: Based on economists’ forecasts and analysis, 1-year Treasury bill

Based on economists’ forecasts and analysis, 1-year Treasury bill rates and liquidity premiums for the next four years are expected to be as follows:

See Answer

Q: If an investor wanted to reduce the risk of a levered stock

If an investor wanted to reduce the risk of a levered stock in their portfolio, how could they go about doing so while still retaining shares in the company?

See Answer

Q: On March 11, 20XX, the existing or current (spot

On March 11, 20XX, the existing or current (spot) 1-, 2-, 3-, and 4-year zero-coupon Treasury security rates were as follows:

See Answer

Q: You are considering an investment in 30-year bonds issued by

You are considering an investment in 30-year bonds issued by Moore Corporation. The bonds have no special covenants. The Wall Street Journal reports that 1-year T-bills are currently earning 1.25 perc...

See Answer

Q: Dakota Corporation 15-year bonds have an equilibrium rate of return

Dakota Corporation 15-year bonds have an equilibrium rate of return of 8 percent. For all securities, the inflation risk premium is 1.75 percent and the real risk-free rate is 3.50 percent. The securi...

See Answer

Q: A 2-year Treasury security currently earns 1.94 percent

A 2-year Treasury security currently earns 1.94 percent. Over the next two years, the real risk-free rate is expected to be 1.00 percent per year and the inflation premium is expected to be 0.50 perce...

See Answer

Q: One-year Treasury bills currently earn 1.45 percent.

One-year Treasury bills currently earn 1.45 percent. You expect that one year from now, 1-year Treasury bill rates will increase to 1.65 percent. If the unbiased expectations theory is correct, what s...

See Answer

Q: One-year Treasury bills currently earn 2.15 percent.

One-year Treasury bills currently earn 2.15 percent. You expect that one year from now, 1-year Treasury bill rates will increase to 2.65 percent and that two years from now, 1-year Treasury bill rates...

See Answer

Q: One-year Treasury bills currently earn 3.45 percent.

One-year Treasury bills currently earn 3.45 percent. You expect that one year from now, 1-year Treasury bill rates will increase to 3.65 percent. The liquidity premium on 2-year securities is 0.05 per...

See Answer

Q: One-year Treasury bills currently earn 2.25 percent.

One-year Treasury bills currently earn 2.25 percent. You expect that one year from now, 1-year Treasury bill rates will increase to 2.45 percent and that two years from now, 1-year Treasury bill rates...

See Answer

Q: A fast growing firm recently paid a dividend of $0.

A fast growing firm recently paid a dividend of $0.40 per share. The dividend is expected to increase at a 25 percent rate for the next four years. Afterwards, a more stable 11 percent growth rate can...

See Answer

Q: Tom and Sue’s Flowers, Inc.’s, 15-year

Tom and Sue’s Flowers, Inc.’s, 15-year bonds are currently yielding a return of 8.25 percent. The expected inflation premium is 2.25 percent annually and the real risk-free rate is expected to be 3.50...

See Answer

Q: Nikki G’s Corporation’s 10- year bonds are currently yielding a return

Nikki G’s Corporation’s 10- year bonds are currently yielding a return of 6.05 percent. The expected inflation premium is 1.00 percent annually and the real risk-free rate is expected to be 2.10 perce...

See Answer

Q: Suppose you were the financial manager for a firm and were considering

Suppose you were the financial manager for a firm and were considering a proposed increase in the amount of debt in the firm’s capital structure. If you thought the firm was going to consistently earn...

See Answer

Q: Suppose we observe the following rates: 1R1 = 8%, 1R2

Suppose we observe the following rates: 1R1 = 8%, 1R2 = 10%. If the unbiased expectations theory of the term structure of interest rates holds, what is the 1-year interest rate expected one year from...

See Answer

Q: The Wall Street Journal reports that the rate on 4-year

The Wall Street Journal reports that the rate on 4-year Treasury securities is 1.60 percent and the rate on 5-year Treasury securities is 2.15 percent. According to the unbiased expectations theories,...

See Answer

Q: The Wall Street Journal reports that the rate on 3-year

The Wall Street Journal reports that the rate on 3-year Treasury securities is 5.25 percent and the rate on 4-year Treasury securities is 5.50 percent. The 1-year interest rate expected in three years...

See Answer

Q: Suppose we observe the following rates: 1R1 = 0.75

Suppose we observe the following rates: 1R1 = 0.75%, 1R2 = 1.20%, and E(2r1) = 0.907%. If the liquidity premium theory of the term structure of interest rates holds, what is the liquidity premium for...

See Answer

Q: You note the following yield curve in The Wall Street Journal.

You note the following yield curve in The Wall Street Journal. According to the unbiased expectations theory, what is the 1-year forward rate for the period beginning one year from today, 2f1? Maturit...

See Answer

Q: The Wall Street Journal reports that the current rate on 10-

The Wall Street Journal reports that the current rate on 10-year Treasury bonds is 7.25 percent, on 20-year Treasury bonds is 7.85 percent, and on a 20-year corporate bond issued by MHM Corp. is 8.75...

See Answer

Q: The Wall Street Journal reports that the current rate on 8-

The Wall Street Journal reports that the current rate on 8-year Treasury bonds is 5.85 percent, on 15-year Treasury bonds is 6.25 percent, and on a 15-year corporate bond issued by MHM Corp. is 7.35 p...

See Answer

Q: On March 5, 2013, the Dow Jones Industrial Average set

On March 5, 2013, the Dow Jones Industrial Average set a new high. The index closed at 14,253.77, which was up 125.95 that day. What was the return (in percent) of the stock market that day?

See Answer

Q: The Wall Street Journal reports that the current rate on 5-

The Wall Street Journal reports that the current rate on 5-year Treasury bonds is 1.85 percent and on 10-year Treasury bonds is 3.35 percent. Assume that the maturity risk premium is zero. Calculate t...

See Answer

Q: The Wall Street Journal reports that the current rate on 10-

The Wall Street Journal reports that the current rate on 10-year Treasury bonds is 2.25 percent and on 20-year Treasury bonds is 4.50 percent. Assume that the maturity risk premium is zero. Calculate...

See Answer

Q: Suppose we observe the 3-year Treasury security rate (1R3

Suppose we observe the 3-year Treasury security rate (1R3) to be 8 percent, the expected 1-year rate next year−E(2r1)−to be 4 percent, and the expected 1-year rate the following year−E(3r1)−to be 6 pe...

See Answer

Q: Explain why, in a world with both corporate taxes and the

Explain why, in a world with both corporate taxes and the chance of bankruptcy, a small firm with volatile EBIT is unlikely to have much debt.

See Answer

Q: The Wall Street Journal reports that the rate on 3-year

The Wall Street Journal reports that the rate on 3-year Treasury securities is 1.20 percent and the rate on 5-year Treasury securities is 2.15 percent. According to the unbiased expectations theories,...

See Answer

Q: Assume the current interest rate on a 1-year Treasury bond

Assume the current interest rate on a 1-year Treasury bond (1R1) is 4.50 percent, the current rate on a 2-year Treasury bond (1R2) is 5.25 percent, and the current rate on a 3-year Treasury bond (1R3)...

See Answer

Q: A particular security’s default risk premium is 2 percent. For all

A particular security’s default risk premium is 2 percent. For all securities, the inflation risk premium is 1.75 percent and the real risk-free rate is 3.5 percent. The security’s liquidity risk prem...

See Answer

Q: Are the unbiased expectations and liquidity premium theories explanations for the shape

Are the unbiased expectations and liquidity premium theories explanations for the shape of the yield curve completely independent theories? Explain why or why not.

See Answer

Q: Classify the following transactions as taking place in the primary or secondary

Classify the following transactions as taking place in the primary or secondary markets:

See Answer

Q: Classify the following financial instruments as money market securities or capital market

Classify the following financial instruments as money market securities or capital market securities:

See Answer

Q: On March 9, 2009, the Dow Jones Industrial Average reached

On March 9, 2009, the Dow Jones Industrial Average reached a new low. The index closed at 6,547.05, which was down 79.89 that day. What was the return (in percent) of the stock market that day?

See Answer

Q: What are the different types of financial institutions? Include a description

What are the different types of financial institutions? Include a description of the main services offered by each.

See Answer

Q: How would economic transactions between suppliers of funds (e.g

How would economic transactions between suppliers of funds (e.g., households) and users of funds (e.g., corporations) occur in a world without FIs?

See Answer

Q: Why would a world limited to the direct transfer of funds from

Why would a world limited to the direct transfer of funds from suppliers of funds to users of funds likely result in quite low levels of fund flows?

See Answer

Q: How do FIs reduce monitoring costs associated with the flow of funds

How do FIs reduce monitoring costs associated with the flow of funds from fund suppliers to fund users?

See Answer

Q: The capital budgeting decision techniques that we’ve discussed all have strengths and

The capital budgeting decision techniques that we’ve discussed all have strengths and weaknesses, but they do comprise the most popular rules for valuing projects. Valuing entire bus...

See Answer

Q: If the U.S. government completely eliminated taxation at the

If the U.S. government completely eliminated taxation at the corporate level, how would this influence the capital structures of firms in a world with bankruptcy?

See Answer

Q: How do FIs alleviate the problem of liquidity risk faced by investors

How do FIs alleviate the problem of liquidity risk faced by investors wishing to invest in securities of corporations?

See Answer

Q: Who are the suppliers of loanable funds?

Who are the suppliers of loanable funds?

See Answer

Q: Who are the demanders of loanable funds?

Who are the demanders of loanable funds?

See Answer

Q: What factors cause the supply of funds curve to shift?

What factors cause the supply of funds curve to shift?

See Answer

Q: At your discount brokerage firm, it costs $7.95

At your discount brokerage firm, it costs $7.95 per stock trade. How much money do you need to buy 200 shares of Pfizer, Inc. (PFE), which trades at $31.40?

See Answer

Q: What factors cause the demand for funds curve to shift?

What factors cause the demand for funds curve to shift?

See Answer

Q: Discuss and compare the three explanations for the shape of the yield

Discuss and compare the three explanations for the shape of the yield curve.

See Answer

Q: What are six factors that determine the nominal interest rate on a

What are six factors that determine the nominal interest rate on a security?

See Answer

Q: What should happen to a security’s equilibrium interest rate as the security’s

What should happen to a security’s equilibrium interest rate as the security’s liquidity risk increases?

See Answer

Q: What is a forward interest rate?

What is a forward interest rate?

See Answer

Q: If we observe a 1-year Treasury security rate that is

If we observe a 1-year Treasury security rate that is higher than the 2-year Treasury security rate, what can we infer about the 1-year rate expected one year from now?

See Answer

Q: Would you expect a utility company to have high or low debt

Would you expect a utility company to have high or low debt levels? Why?

See Answer

Q: Given a 9 percent interest rate, compute the year 6 future

Given a 9 percent interest rate, compute the year 6 future value if deposits of $1,500 and $2,500 are made in years 2 and 3, respectively, and a withdrawal of $600 is made in year 5.

See Answer

Q: To borrow $2,000, you are offered an add

To borrow $2,000, you are offered an add-on interest loan at 10 percent with 12 monthly payments. First, compute the 12 equal payments and then compute the EAR of the loan:

See Answer

Q: To borrow $700, you are offered an add-on

To borrow $700, you are offered an add-on interest loan at 9 percent with 12 monthly payments. First, compute the 12 equal payments and then compute the EAR of the loan:

See Answer

Q: Suppose that Lil John Industries’ equity is currently selling for $37

Suppose that Lil John Industries’ equity is currently selling for $37 per share and that there are 2 million shares outstanding. If the firm also has 30 thousand bonds outstanding, which are selling a...

See Answer

Q: A car company is offering a choice of deals. You can

A car company is offering a choice of deals. You can receive $500 cash back on the purchase, or a 3 percent APR, 4-year loan. The price of the car is $15,000 and you could obtain a 4-year loan from yo...

See Answer

Q: A car company is offering a choice of deals. You can

A car company is offering a choice of deals. You can receive $1,000 cash back on the purchase, or a 2 percent APR, 5-year loan. The price of the car is $20,000 and you could obtain a 5-year loan from...

See Answer

Q: Create the amortization schedule for a loan of $15,000

Create the amortization schedule for a loan of $15,000, paid monthly over three years using a 9 percent APR.

See Answer

Q: Create the amortization schedule for a loan of $5,000

Create the amortization schedule for a loan of $5,000, paid monthly over two years using an 8 percent APR.

See Answer

Q: Monica has decided that she wants to build enough retirement wealth that

Monica has decided that she wants to build enough retirement wealth that, if invested at 8 percent per year, will provide her with $3,500 of monthly income for 20 years. To date, she has saved nothing...

See Answer

Q: Ross has decided that he wants to build enough retirement wealth that

Ross has decided that he wants to build enough retirement wealth that, if invested at 7 percent per year, will provide him with $3,000 of monthly income for 30 years. To date, he has saved nothing, bu...

See Answer

Q: Rachel purchased a $15,000 car three years ago using

Rachel purchased a $15,000 car three years ago using an 8 percent, 4-year loan. She has decided that she would sell the car now, if she could get a price that would pay off the balance of her loan. Wh...

See Answer

Q: Why is debt often referred to as leverage in finance?

Why is debt often referred to as leverage in finance?

See Answer

Q: Hank purchased a $20,000 car two years ago using

Hank purchased a $20,000 car two years ago using a 9 percent, 5-year loan. He has decided that he would sell the car now, if he could get a price that would pay off the balance of his loan. What’s the...

See Answer

Q: A mortgage broker is offering a $183,900, 30

A mortgage broker is offering a $183,900, 30-year mortgage with a teaser rate. In the first two years of the mortgage, the borrower makes monthly payments on only a 4 percent APR interest rate. After...

See Answer

Q: Explain what we mean when we say that one portfolio dominates another

Explain what we mean when we say that one portfolio dominates another portfolio?

See Answer

Q: At your discount brokerage firm, it costs $9.50

At your discount brokerage firm, it costs $9.50 per stock trade. How much money do you need to buy 300 shares of Time Warner, Inc. (TWX), which trades at $22.62?

See Answer

Q: A mortgage broker is offering a $279,000, 30

A mortgage broker is offering a $279,000, 30-year mortgage with a teaser rate. In the first two years of the mortgage, the borrower makes monthly payments on only a 4.5 percent APR interest rate. Aft...

See Answer

Q: Given an 8 percent interest rate, compute the year 7 future

Given an 8 percent interest rate, compute the year 7 future value if deposits of $1,000 and $2,000 are made in years 1 and 3, respectively, and a withdrawal of $700 is made in year 4.

See Answer

Q: Compute the future value in year 9 of a $2,

Compute the future value in year 9 of a $2,000 deposit in year 1 and another $1,500 deposit at the end of year 3 using a 10 percent interest rate.

See Answer

Q: Compute the future value in year 7 of a $2,

Compute the future value in year 7 of a $2,000 deposit in year 1 and another $2,500 deposit at the end of year 4 using an 8% interest rate.

See Answer

Q: What is the future value of a $900 annuity payment over

What is the future value of a $900 annuity payment over five years if interest rates are 8 percent?

See Answer

Q: What is the future value of a $700 annuity payment over

What is the future value of a $700 annuity payment over six years if interest rates are 10 percent?

See Answer

Q: Compute the present value of a $2,000 deposit in

Compute the present value of a $2,000 deposit in year 1 and another $1,500 deposit at the end of year 3 if interest rates are 10 percent.

See Answer

Q: Compute the present value of a $2,000 deposit in

Compute the present value of a $2,000 deposit in year 1 and another $2,500 deposit at the end of year 4 using an 8 percent interest rate.

See Answer

Q: Suppose that the 2016 actual and 2017 projected financial statements for AFS

Suppose that the 2016 actual and 2017 projected financial statements for AFS are initially shown as follows. In these tables, sales are projected to rise by 14 percent in the coming year, and the comp...

See Answer

Q: What’s the present value of a $900 annuity payment over five

What’s the present value of a $900 annuity payment over five years if interest rates are 8 percent?

See Answer

Q: At your full-service brokerage firm, it costs $140

At your full-service brokerage firm, it costs $140 per stock trade. How much money do you receive after selling 200 shares of Nokia Corporation (NOK), which trades at $20.13?

See Answer

Q: What’s the present value of a $700 annuity payment over six

What’s the present value of a $700 annuity payment over six years if interest rates are 10 percent?

See Answer

Q: What’s the present value, when interest rates are 7.5

What’s the present value, when interest rates are 7.5 percent, of a $50 payment made every year forever?

See Answer

Q: What’s the present value, when interest rates are 8.5

What’s the present value, when interest rates are 8.5 percent, of a $75 payment made every year forever?

See Answer

Q: If the present value of an ordinary, 7-year annuity

If the present value of an ordinary, 7-year annuity is $6,500 and interest rates are 7.5 percent, what’s the present value of the same annuity due?

See Answer

Q: If the present value of an ordinary, 6-year annuity

If the present value of an ordinary, 6-year annuity is $8,500 and interest rates are 9.5 percent, what’s the present value of the same annuity due?

See Answer

Q: If the future value of an ordinary, 7-year annuity

If the future value of an ordinary, 7-year annuity is $6,500 and interest rates are 8.5 percent, what is the future value of the same annuity due?

See Answer

Q: If the future value of an ordinary, 6-year annuity

If the future value of an ordinary, 6-year annuity is $8,500 and interest rates are 9.5 percent, what’s the future value of the same annuity due?

See Answer

Q: A loan is offered with monthly payments and a 10 percent APR

A loan is offered with monthly payments and a 10 percent APR. What’s the loan’s effective annual rate (EAR)?

See Answer

Q: A loan is offered with monthly payments and a 13 percent APR

A loan is offered with monthly payments and a 13 percent APR. What’s the loan’s effective annual rate (EAR)?

See Answer

Q: John’s Bait and Fish shop has had the monthly sales amounts listed

John’s Bait and Fish shop has had the monthly sales amounts listed as follows for the last four years. Assuming that there is both seasonality and a trend, estimate monthly sales for...

See Answer

Q: At your full-service brokerage firm, it costs $135

At your full-service brokerage firm, it costs $135 per stock trade. How much money do you receive after selling 250 shares of International Business Machines (IBM), which trades at $96.17?

See Answer

Q: Consider that you are 45 years old and have just changed to

Consider that you are 45 years old and have just changed to a new job. You have $150,000 in the retirement plan from your former employer. You can roll that money into the retirement plan of the new...

See Answer

Q: Your client has been given a trust fund valued at $1

Your client has been given a trust fund valued at $1 million. He cannot access the money until he turns 65 years old, which is in 25 years. At that time, he can withdrawal $25,000 per month. If the tr...

See Answer

Q: Your client has been given a trust fund valued at $1

Your client has been given a trust fund valued at $1.5 million. She cannot access the money until she turns 65 years old, which is in 15 years. At that time, she can withdraw $20,000 per month. If th...

See Answer

Q: A local furniture store is advertising a deal in which you buy

A local furniture store is advertising a deal in which you buy a $3,000 dining room set and do not need to pay for two years (no interest cost is incurred). How much money would you have to deposit no...

See Answer

Q: A local furniture store is advertising a deal in which you buy

A local furniture store is advertising a deal in which you buy a $5,000 living room set with three years before you need to make any payments (no interest cost is incurred). How much money would you h...

See Answer

Q: Say that you purchase a house for $200,000 by

Say that you purchase a house for $200,000 by getting a mortgage for $180,000 and paying a $20,000 down payment. If you get a 30-year mortgage with a 7 percent interest rate, what are the monthly paym...

See Answer

Q: Say that you purchase a house for $150,000 by

Say that you purchase a house for $150,000 by getting a mortgage for $135,000 and paying a $15,000 down payment. If you get a 15-year mortgage with a 7 percent interest rate, what are the monthl...

See Answer

Q: You have secured a loan from your bank for two years to

You have secured a loan from your bank for two years to build your home. The terms of the loan are that you will borrow $200,000 now and an additional $100,000 in one year. Interest of 10 percent APR...

See Answer

Q: You have secured a loan from your bank for two years to

You have secured a loan from your bank for two years to build your home. The terms of the loan are that you will borrow $100,000 now and an additional $50,000 in one year. Interest of 9 percent APR wi...

See Answer

Q: Consider that you are 35 years old and have just changed to

Consider that you are 35 years old and have just changed to a new job. You have $80,000 in the retirement plan from your former employer. You can roll that money into the retirement plan of the new em...

See Answer

Q: You would like to buy shares of Sirius Satellite Radio (SIRI

You would like to buy shares of Sirius Satellite Radio (SIRI). The current ask and bid quotes are $3.96 and $3.93, respectively. You place a market buy-order for 500 shares that executes at these quot...

See Answer

Q: Suppose that the 2016 actual and 2017 projected financial statements for Comfy

Suppose that the 2016 actual and 2017 projected financial statements for Comfy Corners Catbeds are initially shown as follows. In these tables, sales are projected to rise by 22 percent in the coming...

See Answer

Q: You wish to buy a $10,000 dining room set

You wish to buy a $10,000 dining room set. The furniture store offers you a 3-year loan with an 11 percent APR. What are the monthly payments? How would the payment differ if you paid interest only?...

See Answer

Q: Given a 4 percent interest rate, compute the year 6 future

Given a 4 percent interest rate, compute the year 6 future value of deposits made in years 1, 2, 3, and 4 of $1,100, $1,200, $1,200, and $1,500.

See Answer

Q: Given a 5 percent interest rate, compute the year 6 future

Given a 5 percent interest rate, compute the year 6 future value of deposits made in years 1, 2, 3, and 4 of $1,000, $1,300, $1,300, and $1,400.

See Answer

Q: Assume that you contribute $150 per month to a retirement plan

Assume that you contribute $150 per month to a retirement plan for 15 years. Then you are able to increase the contribution to $350 per month for the next 25 years. Given an 8 percent interest rate,...

See Answer

Q: Given a 6 percent interest rate, compute the present value of

Given a 6 percent interest rate, compute the present value of payments made in years 1, 2, 3, and 4 of $1,000, $1,200, $1,200, and $1,500.

See Answer

Q: Given a 7 percent interest rate, compute the present value of

Given a 7 percent interest rate, compute the present value of payments made in years 1, 2, 3, and 4 of $1,000, $1,300, $1,300, and $1,400.

See Answer

Q: A small business owner visits her bank to ask for a loan

A small business owner visits her bank to ask for a loan. The owner states that she can repay a loan at $1,000 per month for the next three years and then $2,000 per month for two years after that. If...

See Answer

Q: A small business owner visits his bank to ask for a loan

A small business owner visits his bank to ask for a loan. The owner states that he can repay a loan at $1,500 per month for the next three years and then $500 per month for two years after that. If th...

See Answer

Q: If you start making $50 monthly contributions today and continue them

If you start making $50 monthly contributions today and continue them for five years, what’s their future value if the compounding rate is 10 percent APR? What is the present value of this annuity?

See Answer

Q: You would like to buy shares of Coldwater Creek, Inc.

You would like to buy shares of Coldwater Creek, Inc. (CWTR). The current ask and bid quotes are $20.70 and $20.66, respectively. You place a market buy-order for 200 shares that executes at these quo...

See Answer

Q: If you start making $75 monthly contributions today and continue them

If you start making $75 monthly contributions today and continue them for four years, what is their future value if the compounding rate is 12 percent APR? What is the present value of this annuity?

See Answer

Q: Sara’s Ice Cream Shop is closed for six months out of the

Sara’s Ice Cream Shop is closed for six months out of the year, but has had the monthly sales amounts listed as follows for the last four years. Assuming that there is both seasonali...

See Answer

Q: You wish to buy a $25,000 car. The

You wish to buy a $25,000 car. The dealer offers you a 4-year loan with a 9 percent APR. What are the monthly payments? How would the payment differ if you paid interest only? What would the conseque...

See Answer

Q: Assume that you contribute $200 per month to a retirement plan

Assume that you contribute $200 per month to a retirement plan for 20 years. Then you are able to increase the contribution to $300 per month for another 30 years. Given a 7 percent interest rate, wha...

See Answer

Q: You are looking to buy a car. You can afford $

You are looking to buy a car. You can afford $450 in monthly payments for four years. In addition to the loan, you can make a $1,000 down payment. If interest rates are 5% APR, what price of car can y...

See Answer

Q: You are looking to buy a car. You can afford $

You are looking to buy a car. You can afford $650 in monthly payments for five years. In addition to the loan, you can make a $750 down payment. If interest rates are 8% APR, what price of car can you...

See Answer

Q: A perpetuity pays $100 per year and interest rates are 7

A perpetuity pays $100 per year and interest rates are 7.5 percent. How much would its value change if interest rates increased to 9 percent? Did the value increase or decrease?

See Answer

Q: A perpetuity pays $50 per year and interest rates are 9

A perpetuity pays $50 per year and interest rates are 9 percent. How much would its value change if interest rates decreased to 7.5 percent? Did the value increase or decrease?

See Answer

Q: Payday loans are very short-term loans that charge very high

Payday loans are very short-term loans that charge very high interest rates. You can borrow $225 today and repay $300 in two weeks. What is the compounded annual rate implied by this 33.33 percent rat...

See Answer

Q: Payday loans are very short-term loans that charge very high

Payday loans are very short-term loans that charge very high interest rates. You can borrow $500 today and repay $590 in two weeks. What is the compounded annual rate implied by this 18 percent rat...

See Answer

Q: You would like to sell 200 shares of Xenith Bankshares (XBKS

You would like to sell 200 shares of Xenith Bankshares (XBKS). The current ask and bid quotes are $4.66 and $4.62, respectively. You place a limit sell-order at $4.65. If the trade executes, how much...

See Answer

Q: What’s the interest rate of a 6-year, annual $

What’s the interest rate of a 6-year, annual $5,000 annuity with present value of $20,000?

See Answer

Q: What’s the interest rate of a 7-year, annual $

What’s the interest rate of a 7-year, annual $4,000 annuity with present value of $20,000?

See Answer

Q: Suppose that Wind Em Corp. currently has the following balance sheet

Suppose that Wind Em Corp. currently has the following balance sheet, and that sales for the year just ended were $7 million. The firm also has a profit margin of 27 percent, a retention ratio of 20 p...

See Answer

Q: What annual interest rate would you need to earn if you wanted

What annual interest rate would you need to earn if you wanted a $1,000 per month contribution to grow to $75,000 in six years?

See Answer

Q: What annual interest rate would you need to earn if you wanted

What annual interest rate would you need to earn if you wanted a $600 per month contribution to grow to $45,000 in six years?

See Answer

Q: To borrow $500, you are offered an add-on

To borrow $500, you are offered an add-on interest loan at 8 percent. Two loan payments are to be made, one at six months and the other at the end of the year. Compute the two equal payments.

See Answer

Q: To borrow $800, you are offered an add-on

To borrow $800, you are offered an add-on interest loan at 7 percent. Three loan payments are to be made, one at four months, another at eight months, and the last one at the end of the year. Compute...

See Answer

Q: Joey realizes that he has charged too much on his credit card

Joey realizes that he has charged too much on his credit card and has racked up $5,000 in debt. If he can pay $150 each month and the card charges 17 percent APR (compounded monthly), how long will it...

See Answer

Q: Phoebe realizes that she has charged too much on her credit card

Phoebe realizes that she has charged too much on her credit card and has racked up $6,000 in debt. If she can pay $200 each month and the card charges 18 percent APR (compounded monthly), how long wil...

See Answer

Q: Consider Gavin, a new freshman who has just received a Stafford

Consider Gavin, a new freshman who has just received a Stafford student loan and started college. He plans to obtain the maximum loan from Stafford at the beginning of each year. Although Gavin does n...

See Answer

Q: You would like to sell 100 shares of Echo Global Logistics,

You would like to sell 100 shares of Echo Global Logistics, Inc. (ECHO). The current ask and bid quotes are $15.33 and $15.28, respectively. You place a limit sell-order at $15.31. If the trade execut...

See Answer

Q: Consider a person who begins contributing to a retirement plan at age

Consider a person who begins contributing to a retirement plan at age 25 and contributes for 40 years until retirement at age 65. For the first ten years, she contributes $3,000 per year. She increase...

See Answer

Q: When paying off a home mortgage, extra principle payments can have

When paying off a home mortgage, extra principle payments can have a dramatic impact on the time needed to pay off the mortgage. (a) Create an amortization schedule for a $200,000, 3-year mortgage wit...

See Answer

Q: Given a certain amount of savings, how much can I spend

Given a certain amount of savings, how much can I spend annually during retirement? Your annual income is estimated to be $70,000. Information entered 1. Savings Amount saved…………………………………… ………………………$...

See Answer

Q: Suppose that Gyp Sum Industries currently has the following balance sheet,

Suppose that Gyp Sum Industries currently has the following balance sheet, and that sales for the year just ended were $10 million. The firm also has a profit margin of 25 percent, a retention ratio o...

See Answer

Q: How can you use the concepts illustrated in computing the number of

How can you use the concepts illustrated in computing the number of payments in an annuity to figure how to pay off a credit card balance? How does the magnitude of the payment impact the number of mo...

See Answer

Q: How can you use the present value of an annuity concept to

How can you use the present value of an annuity concept to determine the price of a house you can afford?

See Answer

Q: Since perpetuity payments continue forever, how can a present value be

Since perpetuity payments continue forever, how can a present value be computed? Why isn’t the present value infinite?

See Answer

Q: People can become millionaires in their retirement years quite easily if they

People can become millionaires in their retirement years quite easily if they start saving early in employer 401(k) or 403(b) programs (or even if their employers don’t offer such programs). Demonstra...

See Answer

Q: When you discount multiple cash flows, how does the future period

When you discount multiple cash flows, how does the future period that a cash flow is paid affect its present value and its contribution to the value of all the cash flows?

See Answer

Q: Explain why you use the same adjustment factor, (1 +

Explain why you use the same adjustment factor, (1 + i), when you adjust annuity due payments for both future value and present value.

See Answer

Q: A preferred stock from Duquesne Light Company (DQUPRA) pays $

A preferred stock from Duquesne Light Company (DQUPRA) pays $3.55 in annual dividends. If the required return on the preferred stock is 6.7 percent, what’s the value of the stock?

See Answer

Q: Use the idea of compound interest to explain why EAR is larger

Use the idea of compound interest to explain why EAR is larger than APR.

See Answer

Q: Would you rather pay $10,000 for a 5-

Would you rather pay $10,000 for a 5-year $2,500 annuity or a 10-year $1,250 annuity? Why?

See Answer

Q: The interest on your home mortgage is tax deductible. Why are

The interest on your home mortgage is tax deductible. Why are the early years of the mortgage more helpful in reducing taxes than in the later years?

See Answer

Q: How can you add a cash flow in year 2 and a

How can you add a cash flow in year 2 and a cash flow in year 4 in year 7?

See Answer

Q: Suppose a firm has had the historic sales figures shown as follows

Suppose a firm has had the historic sales figures shown as follows. What would be the forecast for next year’s sales using the naïve approach?

See Answer

Q: People have had a fascination with gold for thousands of years.

People have had a fascination with gold for thousands of years. Archaeologists have discovered gold jewelry in Southern Iraq dating to 3000 BC and gold ornaments in Peru dating to 1200 BC. The ancient...

See Answer

Q: At age 25 you invest $1,500 that earns 8

At age 25 you invest $1,500 that earns 8 percent each year. At age 40 you invest $1,500 that earns 11 percent per year. In which case would you have more money at age 65?

See Answer

Q: You invested $2,000 in the stock market one year

You invested $2,000 in the stock market one year ago. Today, the investment is valued at $1,500. What return did you earn? What return would you need to get next year to break even overall?

See Answer

Q: You invested $3,000 in the stock market one year

You invested $3,000 in the stock market one year ago. Today, the investment is valued at $3,750. What return did you earn? What return would you suffer next year for your investment to be valued at th...

See Answer

Q: What annual rate of return is earned on a $4,

What annual rate of return is earned on a $4,000 investment made in year 2 when it grows to $6,500 by the end of year seven?

See Answer

Q: A preferred stock from Hecla Mining Co. (HLPRB) pays

A preferred stock from Hecla Mining Co. (HLPRB) pays $3.50 in annual dividends. If the required return on the preferred stock is 6.8 percent, what is the value of the stock?

See Answer

Q: What annual rate of return is implied on a $2,

What annual rate of return is implied on a $2,500 loan taken next year when $3,500 must be repaid in year 4?

See Answer

Q: Ten years ago, Hailey invested $2,000 and locked

Ten years ago, Hailey invested $2,000 and locked in a 9 percent annual interest rate for 30 years (end 20 years from now). Aidan can make a 20-year investment today and lock in a 10 percent interest r...

See Answer

Q: Ten years ago, Hailey invested $3,000 and locked

Ten years ago, Hailey invested $3,000 and locked in an 8 percent annual interest rate for 30 years (end 20 years from now). Aidan can make a 20-year investment today and lock in a 10 percent interest...

See Answer

Q: You are scheduled to receive a $500 cash flow in one

You are scheduled to receive a $500 cash flow in one year, a $1,000 cash flow in two years, and pay an $800 payment in three years. If interest rates are 10 percent per year, what is the combined pres...

See Answer

Q: At age 30 you invest $1,000 that earns 8

At age 30 you invest $1,000 that earns 8 percent each year. At age 40 you invest $1,000 that earns 12 percent per year. In which case would you have more money at age 60?

See Answer

Q: You own $10,000 of Denny’s Corp stock that has

You own $10,000 of Denny’s Corp stock that has a beta of 2.9. You also own $15,000 of Qwest Communications (beta = 1.5) and $5,000 of Southwest Airlines (beta = 0.7). Assume that the market return wil...

See Answer

Q: Suppose a firm has had the historic sales figures shown as follows

Suppose a firm has had the historic sales figures shown as follows. What would be the forecast for next year’s sales using the naïve approach?

See Answer

Q: A deposit of $350 earns the following interest rates:

A deposit of $350 earns the following interest rates: • 8 percent in the first year, • 6 percent in the second year, and • 5.5 percent in the third year. What would be the third year future value?

See Answer

Q: Show the time line for a $500 cash inflow today,

Show the time line for a $500 cash inflow today, a $605 cash outflow in year 2, and a 10 percent interest rate.

See Answer

Q: Show the time line for a $400 cash outflow today,

Show the time line for a $400 cash outflow today, a $518 cash inflow in year 3, and a 9 percent interest rate.

See Answer

Q: Ultra Petroleum (UPL) has earnings per share of $1

Ultra Petroleum (UPL) has earnings per share of $1.56 and a P/E ratio of 32.48. What’s the stock price?

See Answer

Q: What is the future value of $500 deposited for one year

What is the future value of $500 deposited for one year earning a 8 percent interest rate annually.

See Answer

Q: What is the future value of $400 deposited for one year

What is the future value of $400 deposited for one year earning an interest rate of 9 percent per year?

See Answer

Q: How much would be in your savings account in eleven years after

How much would be in your savings account in eleven years after depositing $150 today if the bank pays 8 percent per year?

See Answer

Q: Compute the value in 25 years of a $1,000

Compute the value in 25 years of a $1,000 deposit earning 10 percent per year.

See Answer

Q: A deposit of $750 earns interest rates of 9 percent in

A deposit of $750 earns interest rates of 9 percent in the first year and 12 percent in the second year. What would be the second year future value?

See Answer

Q: What is the present value of a $350 payment in one

What is the present value of a $350 payment in one year when the discount rate is 10 percent?

See Answer

Q: What is the present value of a $200 payment in one

What is the present value of a $200 payment in one year when the discount rate is 7 percent?

See Answer

Q: Suppose a firm has had the historic sales figures shown as follows

Suppose a firm has had the historic sales figures shown as follows. What would be the forecast for next year’s sales using the average approach?

See Answer

Q: What is the present value of a $1,500 payment

What is the present value of a $1,500 payment made in nine years when the discount rate is 8 percent?

See Answer

Q: Compute the present value of an $850 payment made in 10

Compute the present value of an $850 payment made in 10 years when the discount rate is 12 percent.

See Answer

Q: Explain what the efficient frontier is and why it is important to

Explain what the efficient frontier is and why it is important to investors.

See Answer

Q: HiLo, Inc., doesn’t face any taxes and has $150

HiLo, Inc., doesn’t face any taxes and has $150 million in assets, currently financed entirely with equity. Equity is worth $7 per share, and book value of equity is equal to market...

See Answer

Q: Compute the present value of $1,000 paid in three

Compute the present value of $1,000 paid in three years using the following discount rates: 6 percent in the first year, 7 percent in the second year, and 8 percent in the third year.

See Answer

Q: Compute the present value of $5,000 paid in two

Compute the present value of $5,000 paid in two years using the following discount rates: 8 percent in the first year and 7 percent in the second year.

See Answer

Q: Approximately how many years does it take to double a $100

Approximately how many years does it take to double a $100 investment when interest rates are 7 percent per year?

See Answer

Q: Approximately how many years does it take to double a $500

Approximately how many years does it take to double a $500 investment when interest rates are 10 percent per year?

See Answer

Q: Approximately what interest rate is needed to double an investment over five

Approximately what interest rate is needed to double an investment over five years?

See Answer

Q: Approximately what interest rate is earned when an investment doubles over 12

Approximately what interest rate is earned when an investment doubles over 12 years?

See Answer

Q: Determine the interest rate earned on a $1,400 deposit

Determine the interest rate earned on a $1,400 deposit when $1,800 is paid back in one year.

See Answer

Q: Determine the interest rate earned on a $2,300 deposit

Determine the interest rate earned on a $2,300 deposit when $2,900 is paid back in one year.

See Answer

Q: Suppose a firm has had the historic sales figures shown as follows

Suppose a firm has had the historic sales figures shown as follows. What would be the forecast for next year’s sales using the average approach?

See Answer

Q: How long will it take $2,000 to reach $

How long will it take $2,000 to reach $5,000 when it grows at 10 percent per year?

See Answer

Q: JP Morgan Chase Co. (JPM) has earnings per share

JP Morgan Chase Co. (JPM) has earnings per share of $3.53 and a P/E ratio of 13.81. What is the price of the stock?

See Answer

Q: Consider a $2,000 deposit earning 8 percent interest per

Consider a $2,000 deposit earning 8 percent interest per year for five years. What is the future value, and how much total interest is earned on the original deposit vs. how much is interest earned on...

See Answer

Q: Consider a $5,000 deposit earning 10 percent interest per

Consider a $5,000 deposit earning 10 percent interest per year for 10 years. What is the future value, how much total interest is earned on the original deposit, and how much is interest earned on int...

See Answer

Q: What would be more valuable, receiving $500 today or receiving

What would be more valuable, receiving $500 today or receiving $625 in three years if interest rates are 7 percent? Why?

See Answer

Q: Which cash flow would you rather pay, $425 today or

Which cash flow would you rather pay, $425 today or $500 in two years if interest rates are 10 percent? Why?

See Answer

Q: How many years (and months) will it take $2

How many years (and months) will it take $2 million to grow to $5 million with an annual interest rate of 7 percent?

See Answer

Q: What is the value in year 3 of a $700 cash

What is the value in year 3 of a $700 cash flow made in year 6 if interest rates are 10 percent?

See Answer

Q: What is the value in year 4 of a $1,

What is the value in year 4 of a $1,000 cash flow made in year 6 if interest rates are 8 percent?

See Answer

Q: What is the value in year 10 of a $1,

What is the value in year 10 of a $1,000 cash flow made in year 3 if interest rates are 9 percent?

See Answer

Q: What is the value in year 15 of a $250 cash

What is the value in year 15 of a $250 cash flow made in year 3 if interest rates are 11 percent?

See Answer

Q: Suppose a firm has had the historic sales figures shown as follows

Suppose a firm has had the historic sales figures shown as follows. What would be the forecast for next year’s sales using regression to estimate a trend? //

See Answer

Q: A firm is expected to pay a dividend of $1.

A firm is expected to pay a dividend of $1.35 next year and $1.50 the following year. Financial analysts believe the stock will be at their price target of $68 in two years. Compute the value of this...

See Answer

Q: What annual rate of return is earned on a $1,

What annual rate of return is earned on a $1,000 investment when it grows to $1,800 in six years?

See Answer

Q: What annual rate of return is earned on a $5,

What annual rate of return is earned on a $5,000 investment when it grows to $9,500 in five years?

See Answer

Q: What kind of returns might you expect in the stock market?

What kind of returns might you expect in the stock market? One way to measure how the stock market has performed is to examine the rate of return of the S&P 500 Index. To see historical prices of the...

See Answer

Q: Consider that you are the marketing manager of a firm. You

Consider that you are the marketing manager of a firm. You need to have approximately 1 additional salesperson for every $10 million in sales. You currently have $50 million in sales and have 5 employ...

See Answer

Q: Oil prices have increased a great deal in the last decade.

Oil prices have increased a great deal in the last decade. The table below shows the average oil price for each year since 1949. Many companies use oil products as a resource in their own business ope...

See Answer

Q: Show how the Rule of 72 can be used to approximate the

Show how the Rule of 72 can be used to approximate the number of years to quadruple an investment.

See Answer

Q: List and describe the purpose of each part of a time line

List and describe the purpose of each part of a time line with an initial cash inflow and a future cash outflow. Which cash flows should be negative and which positive? Why?

See Answer

Q: How are the present value and future value related?

How are the present value and future value related?

See Answer

Q: How are present values affected by changes in interest rates?

How are present values affected by changes in interest rates?

See Answer

Q: What do you think about the following statement. “I am

What do you think about the following statement. “I am going to receive $100 two years from now and $200 three years from now, so I am getting a $300 future value.” How could the two cash flows be com...

See Answer

Q: A firm is expected to pay a dividend of $2.

A firm is expected to pay a dividend of $2.05 next year and $2.35 the following year. Financial analysts believe the stock will be at their price target of $110 in two years. Compute the value of this...

See Answer

Q: Suppose that Psy Ops Industries currently has the following balance sheet,

Suppose that Psy Ops Industries currently has the following balance sheet, and that sales for the year just ended were $5 million. The firm also has a profit margin of 25 percent, a retention ratio of...

See Answer

Q: Without making any computations, indicate which of each pair has a

Without making any computations, indicate which of each pair has a higher interest rate? a. $100 doubles to $200 in 5 years or 7 years. b. $500 increases in 4 years to $750 or to $800. c. $300 increas...

See Answer

Q: A $1,000 investment has doubled to $2,

A $1,000 investment has doubled to $2,000 in 8 years because of a 9 percent rate of return. How much longer will it take for the investment to reach $4,000 if it continues to earn a 9 percent rate?...

See Answer

Q: Would you prefer to have an investment earning 5 percent for 40

Would you prefer to have an investment earning 5 percent for 40 years or an investment earning 10 percent for 20 years? Explain.

See Answer

Q: Listed below are the 2018 financial statements for Garners’ Platoon Mental Health

Listed below are the 2018 financial statements for Garners’ Platoon Mental Health Care, Inc. Spread the balance sheet and income statement. Calculate the financial ratios for the fir...

See Answer

Q: You are considering investing in Annie’s Eatery. You have been able

You are considering investing in Annie’s Eatery. You have been able to locate the following information on the firm: Total assets are $40 million, accounts receivable are $6.0 million, ACP is 30 days,...

See Answer

Q: Maggie’s Skunk Removal Corp.’s 2018 income statement listed net sales

Maggie’s Skunk Removal Corp.’s 2018 income statement listed net sales = $12.5 million, gross profit of $6.9 million, EBIT = $5.6 million, net income available to common stockholders = $3.2 million, an...

See Answer

Q: In 2018, Jake’s Jamming Music, Inc. announced an ROA

In 2018, Jake’s Jamming Music, Inc. announced an ROA of 8.56 percent, ROE of 14.5 percent, and profit margin of 20.5 percent. The firm had total assets of $9.5 million at year-end 2018. Calculate the...

See Answer

Q: Mandesa, Inc., has current liabilities of $8 million,

Mandesa, Inc., has current liabilities of $8 million, current ratio of 2 times, inventory turnover ratio of 12 times, average collection period of 30 days, and credit sales of $64 million. Calculate t...

See Answer

Q: Use the following information to complete the balance sheet below. Sales

Use the following information to complete the balance sheet below. Sales are $8.8 million, capital intensity ratio is 2.10 times, debt ratio is 55 percent, and fixed asset turnover ratio is 1.2 times....

See Answer

Q: Annual dividends of ATTA Corp grew from $0.96 in

Annual dividends of ATTA Corp grew from $0.96 in 2005 to $1.76 in 2017. What was the annual growth rate?

See Answer

Q: Tiggie’s Dog Toys, Inc., reported a debt-to-

Tiggie’s Dog Toys, Inc., reported a debt-to-equity ratio of 1.75 times at the end of 2018. If the firm’s total assets at year-end were $25 million, how much of their assets are financed with debt and...

See Answer

Q: Suppose that Wall-E Corp. currently has the following balance

Suppose that Wall-E Corp. currently has the following balance sheet, and that sales for the year just ended were $7 million. The firm also has a profit margin of 27 percent, a retention ratio of 20 pe...

See Answer

Q: Calculate the times interest earned ratio for LaTonya’s Flop Shops, Inc

Calculate the times interest earned ratio for LaTonya’s Flop Shops, Inc., using the following information. Sales are $1.5 million, cost of goods sold is $600,000, depreciation expense is $150,000, oth...

See Answer

Q: Rick’s Travel Service has asked you to help piece together financial information

Rick’s Travel Service has asked you to help piece together financial information on the firm for the most current year. Managers give you the following information: sales are $8.2 million, total debt...

See Answer

Q: Leonatti Labs’ year-end price on its common stock is $

Leonatti Labs’ year-end price on its common stock is $35. The firm has total assets of $50 million, debt ratio of 65 percent, no preferred stock, and 3 million shares of common stock outstanding. Calc...

See Answer

Q: Leonatti Labs’ year-end price on its common stock is $

Leonatti Labs’ year-end price on its common stock is $15. The firm has a profit margin of 8 percent, total assets of $42 million, a total asset turnover ratio of 0.75, no preferred stock, and 3 millio...

See Answer

Q: Last year, Stumble-on-Inn, Inc., reported

Last year, Stumble-on-Inn, Inc., reported an ROE of 18 percent. The firm’s debt ratio was 55 percent, sales were $15 million, and the capital intensity was 1.25 times. Calculate the net income for Stu...

See Answer

Q: You are considering investing in Nuran Security Services. You have been

You are considering investing in Nuran Security Services. You have been able to locate the following information on the firm: total assets are $24 million, accounts receivable are $3.3 million, ACP is...

See Answer

Q: You have located the following information on Webb’s Heating & Air Conditioning

You have located the following information on Webb’s Heating & Air Conditioning: debt ratio is 54 percent, capital intensity ratio is 1.10 times, profit margin is 12.5 percent, and dividend payout rat...

See Answer

Q: Spread the balance sheets and income statements of Lake of Egypt Marina

Spread the balance sheets and income statements of Lake of Egypt Marina, Inc. for 2018 and 2017.

See Answer

Q: Annual dividends of Generic Electrical grew from $0.66 in

Annual dividends of Generic Electrical grew from $0.66 in 2012 to $1.03 in 2017. What was the annual growth rate?

See Answer

Q: Calculate the following ratios for Lake of Egypt Marina, Inc.

Calculate the following ratios for Lake of Egypt Marina, Inc. as of year-end 2018.

See Answer

Q: Use the following information to complete the balance sheet below.

Use the following information to complete the balance sheet below. Current ratio = 2.5 times Profit margin = 10% Sales = $1,200m ROE = 20% Long-term debt to Long-term debt and equity = 55%

See Answer

Q: Suppose a firm has had the historic sales figures shown as follows

Suppose a firm has had the historic sales figures shown as follows. What would be the forecast for next year’s sales using regression to estimate a trend?

See Answer

Q: Use the following information to complete the balance sheet below.

Use the following information to complete the balance sheet below. Current ratio = 2.2 times Credit sales = $1,200m Average collection period = 60 days Inventory turnover = 1.50 times Total asset t...

See Answer

Q: Last year, K9 Webb Wear, Inc., reported an ROE

Last year, K9 Webb Wear, Inc., reported an ROE of 20 percent. The firm’s debt ratio was 55 percent, sales were $20 million, and the capital intensity was 1.25 times. Calculate the net income and profi...

See Answer

Q: You are considering investing in Dakota’s Security Services. You have been

You are considering investing in Dakota’s Security Services. You have been able to locate the following information on the firm: Total assets are $32 million, accounts receivable are $4.4 million, ACP...

See Answer

Q: Last year, Marly Brown, Inc., reported an ROE of

Last year, Marly Brown, Inc., reported an ROE of 20 percent. The firm’s debt-to-equity was 1.50 times, sales were $20 million, the capital intensity was 1.25 times, and dividends paid to common stockh...

See Answer

Q: The top part of Ramakrishnan, Inc,’s 2018 and 2017

The top part of Ramakrishnan, Inc,’s 2018 and 2017 balance sheets is listed below (in millions of dollars). Calculate Ramakrishnan, Inc.’s current ratio, quick rati...

See Answer

Q: Tater and Pepper Corp. reported sales for 2018 of $23

Tater and Pepper Corp. reported sales for 2018 of $23 million. Tater and Pepper listed $5.6 million of inventory on its balance sheet. Using a 365 day year, how many days did Tater and Pepper’s invent...

See Answer

Q: Mr. Husker’s Tuxedos Corp. ended the year 2018 with an

Mr. Husker’s Tuxedos Corp. ended the year 2018 with an average collection period of 32 days. The firm’s credit sales for 2018 were $56.1 million. What is the year- end 2018 balance in accounts receiva...

See Answer

Q: Financial analysts forecast Safeco Corp.’s (SAF) growth rate

Financial analysts forecast Safeco Corp.’s (SAF) growth rate for the future to be 8 percent. Safeco’s recent dividend was $0.88. What is the value of Safeco stock when the required return is 12 percen...

See Answer

Q: Tiggie’s Dog Toys, Inc. reported a debt-to-

Tiggie’s Dog Toys, Inc. reported a debt-to-equity ratio of 1.75 times at the end of 2018. If the firm’s total debt at year-end was $25 million, how much equity does Tiggie’s have on its balance sheet?...

See Answer

Q: You are considering a stock investment in one of two firms (

You are considering a stock investment in one of two firms (Lots of Debt, Inc. and Lots of Equity, Inc.), both of which operate in the same industry. Lots of Debt, Inc. finances its $30 million in ass...

See Answer

Q: You are considering an investment in Roxie’s Bed & Breakfast Corp.

You are considering an investment in Roxie’s Bed & Breakfast Corp. During the last year the firm’s income statement listed an addition to retained earnings of $4.8 million and common stock dividends o...

See Answer

Q: Suppose that the 2016 actual and 2017 projected financial statements for your

Suppose that the 2016 actual and 2017 projected financial statements for your firm are initially shown as follows. In these tables, sales are projected to rise by 18 percent in the coming year, and th...

See Answer

Q: Dudley Hill Golf Club’s market-to-book ratio is currently

Dudley Hill Golf Club’s market-to-book ratio is currently 2.5 times and the PE ratio is 6.75 times. If Dudley Hill Golf Club’s common stock is currently selling at $22.50 per share, what is the book v...

See Answer

Q: If Silas 4-Wheeler, Inc. has an ROE of

If Silas 4-Wheeler, Inc. has an ROE of 18 percent, equity multiplier of 2, and a profit margin of 18.75 percent, what is the total asset turnover and the capital intensity?

See Answer

Q: Last year, Hassan’s Mad hatter, Inc. had an ROA

Last year, Hassan’s Mad hatter, Inc. had an ROA of 7.5 percent, a profit margin of 12 percent, and sales of $25 million. Calculate Hassan’s Mad hatter’s total assets.

See Answer

Q: Last year, Lakesha’s Lounge Furniture Corporation had an ROA of 7

Last year, Lakesha’s Lounge Furniture Corporation had an ROA of 7.5 percent and a dividend payout ratio of 25 percent. What is the internal growth rate?

See Answer

Q: Last year, Lakesha’s Lounge Furniture Corporation had an ROE of 17

Last year, Lakesha’s Lounge Furniture Corporation had an ROE of 17.5 percent and a dividend payout ratio of 20 percent. What is the sustainable growth rate?

See Answer

Q: Brenda’s Bar and Grill has current liabilities of $15 million.

Brenda’s Bar and Grill has current liabilities of $15 million. Cash makes up 10 percent of the current assets and accounts receivable makes up another 40 percent of current assets. Brenda’s current r...

See Answer

Q: Financial analysts forecast Limited Brands’ (LTD) growth rate for the

Financial analysts forecast Limited Brands’ (LTD) growth rate for the future to be 12.5 percent. LTD’s recent dividend was $0.60. What is the value of Limited Brands’ stock when the required retur...

See Answer

Q: You have the following information on Els’ Putters, Inc.: sales

You have the following information on Els’ Putters, Inc.: sales to working capital is 4.6 times, profit margin is 20 percent, net income available to common stockholders is $5 million, and current lia...

See Answer

Q: You are thinking of investing in Nikki T’s, Inc. You

You are thinking of investing in Nikki T’s, Inc. You have only the following information on the firm at year-end 2018: net income is $250,000, total debt is $2.5 million, and debt ratio is 55 percent....

See Answer

Q: Dogs R Us reported a profit margin of 10.5 percent

Dogs R Us reported a profit margin of 10.5 percent, total asset turnover of 0.75 times, debt-to-equity of 0.80 times, net income of $500,000, and dividends paid to common stockholders of $200,000. The...

See Answer

Q: Construct the DuPont ROA and ROE breakdowns for Lake of Egypt Marina

Construct the DuPont ROA and ROE breakdowns for Lake of Egypt Marina, Inc.

See Answer

Q: What is the optimal length of time over which to take an

What is the optimal length of time over which to take an average of historic sales when using the average approach?

See Answer

Q: Calculate the internal and sustainable growth rate for Lake of Egypt Marina

Calculate the internal and sustainable growth rate for Lake of Egypt Marina, Inc.

See Answer

Q: Using the ratios from question 3-30 for Lake of Egypt

Using the ratios from question 3-30 for Lake of Egypt Marina, Inc. and the industry, what can you conclude about Lake of Egypt Marina’s financial performance for 2018.

See Answer

Q: You are evaluating the balance sheet for Patty Cake’s Corporation. From

You are evaluating the balance sheet for Patty Cake’s Corporation. From the balances heet you find the following balances: cash and marketable securities = $400,000; accounts receivable = $1,200,000;...

See Answer

Q: What does it mean when a firm window dresses its financial statements

What does it mean when a firm window dresses its financial statements?

See Answer

Q: Classify each of the following ratios according to a ratio category (

Classify each of the following ratios according to a ratio category (liquidity ratio, asset management ratio, debt management ratio, profitability ratio, or market value ratio).

See Answer

Q: Ecolap, Inc. (ECL) recently paid a $0

Ecolap, Inc. (ECL) recently paid a $0.46 dividend. The dividend is expected to grow at a 14.5 percent rate. At a current stock price of $44.12, what is the return shareholders are expecting?

See Answer

Q: For each of the following actions, determine what would happen to

For each of the following actions, determine what would happen to the current ratio. Assume nothing else on the balance sheet changes and that net working capital is positive.

See Answer

Q: Explain the meaning and significance of the following ratios

Explain the meaning and significance of the following ratios

See Answer

Q: A firm has an average collection period of 10 days. The

A firm has an average collection period of 10 days. The industry average ACP is 25 days. Is this a good or poor sign about the management of the firm’s accounts receivable?

See Answer

Q: A firm has a debt ratio of 20 percent. The industry

A firm has a debt ratio of 20 percent. The industry average debt ratio is 65 percent. Is this a good or poor sign about the management of the firm’s financial leverage?

See Answer

Q: Why is the DuPont system of analysis an important tool when evaluating

Why is the DuPont system of analysis an important tool when evaluating firm performance?

See Answer

Q: What approach should be used to forecast sales if a firm believes

What approach should be used to forecast sales if a firm believes that sales will increase over time?

See Answer

Q: A firm has an ROE of 10 percent. The industry

A firm has an ROE of 10 percent. The industry average ROE is 15 percent. How can the DuPont system of analysis help the firm’s managers identify the reasons for this difference?

See Answer

Q: What is the difference between the internal growth rate and the sustainable

What is the difference between the internal growth rate and the sustainable growth rate?

See Answer

Q: What information does time series and cross-sectional analysis provide for

What information does time series and cross-sectional analysis provide for firm managers, analysts, and investors?

See Answer

Q: Why is it important to know a firm’s accounting rules before making

Why is it important to know a firm’s accounting rules before making any conclusions about its performance from ratios analysis?

See Answer

Q: Paychex Inc. (PAYX) recently paid an $0.

Paychex Inc. (PAYX) recently paid an $0.84 dividend. The dividend is expected to grow at a 15 percent rate. At a current stock price of $40.11, what is the return shareholders are expecting?

See Answer

Q: What is the difference between time series analysis and cross-sectional

What is the difference between time series analysis and cross-sectional analysis?

See Answer

Q: A firm has an ROE of 20 percent. The industry average

A firm has an ROE of 20 percent. The industry average ROE is 12 percent. Is this a good or poor sign about the management of the firm?

See Answer

Q: Shown below are partial financial statements for Garners’ Platoon Mental Health Care

Shown below are partial financial statements for Garners’ Platoon Mental Health Care, Inc. Fill in the blanks on the four financial statements. Garners’ Platoon Me...

See Answer

Q: Vinny’s Overhead Construction had free cash flow during 2018 of $25

Vinny’s Overhead Construction had free cash flow during 2018 of $25.4 million. The change in gross fixed assets on Vinny’s balance sheet during 2018 was $7.0 million and the change in net operating wo...

See Answer

Q: You are evaluating the balance sheet for Goodman’s Bees Corporation. From

You are evaluating the balance sheet for Goodman’s Bees Corporation. From the balance sheet you find the following balances: cash and marketable securities = $400,000, accounts receivable = $1,200,000...

See Answer

Q: Casello Mowing & Landscaping’s year-end 2018 balance sheet lists current

Casello Mowing & Landscaping’s year-end 2018 balance sheet lists current assets of $435,200, fixed assets of $550,800, current liabilities of $416,600, and long-term debt of $314,500. Calculate Casell...

See Answer

Q: You own $15,000 of Opsware, Inc. stock

You own $15,000 of Opsware, Inc. stock that has a beta of 3.8. You also own $10,000 of Lowe’s Companies (beta = 1.6) and $10,000 of New York Times (beta = 0.8). Assume that the market return will be 1...

See Answer

Q: What is the theoretical minimum value for MAPE?

What is the theoretical minimum value for MAPE?

See Answer

Q: Consider a firm with an EBIT of $850,000.

Consider a firm with an EBIT of $850,000. The firm finances its assets with $2,500,000 debt (costing 7.5 percent) and 400,000 shares of stock selling at $5.00 per share. To reduce firm’s risk associat...

See Answer

Q: Consider a firm with an EBIT of $550,000.

Consider a firm with an EBIT of $550,000. The firm finances its assets with $1,000,000 debt (costing 5.5 percent) and 200,000 shares of stock selling at $12.00 per share. The firm is considering incre...

See Answer

Q: What’s the source of firm-specific risk? What’s the source

What’s the source of firm-specific risk? What’s the source of market risk?

See Answer

Q: A firm does not pay a dividend. It is expected to

A firm does not pay a dividend. It is expected to pay its first dividend of $0.20 per share in three years. This dividend will grow at 11 percent indefinitely. Using a 12 percent discount rate, compu...

See Answer

Q: Oakdale Fashions, Inc., had $245,000 in 2018

Oakdale Fashions, Inc., had $245,000 in 2018 taxable income. Using the tax schedule in Table 2.3, calculate the company’s 2018 income taxes. What is the average tax rate? What is the marginal tax rate...

See Answer

Q: Hunt Taxidermy, Inc., is concerned about the taxes paid by

Hunt Taxidermy, Inc., is concerned about the taxes paid by the company in 2018. In addition to $42.4 million of taxable income, the firm received $2,975,000 of interest on state-issued bonds and $1,00...

See Answer

Q: Ramakrishnan Inc. reported 2018 net income of $15 million and

Ramakrishnan Inc. reported 2018 net income of $15 million and depreciation of $2,650,000. The top part of Ramakrishnan, Inc.’s 2018 and 2017 balance sheets is listed below (in millions of dollars).

See Answer

Q: You are considering an investment in Fields and Struthers, Inc.,

You are considering an investment in Fields and Struthers, Inc., and want to evaluate the firm’s free cash flow. From the income statement, you see that Fields and Struthers earned an EBIT of $62 mill...

See Answer

Q: Tater and Pepper Corp. reported free cash flows for 2018 of

Tater and Pepper Corp. reported free cash flows for 2018 of $39.1 million and investment in operating capital of $22.1 million. Tater and Pepper incurred $13.6 million in depreciation expense and paid...

See Answer

Q: Use the following information to find dividends paid to common stockholders during

Use the following information to find dividends paid to common stockholders during 2018.

See Answer

Q: Brenda’s Bar and Grill has total assets of $15 million of

Brenda’s Bar and Grill has total assets of $15 million of which $5 million are current assets. Cash makes up 10 percent of the current assets and accounts receivable makes up another 40 percent of cur...

See Answer

Q: Glen’s Tobacco Shop has total assets of $91.8 million

Glen’s Tobacco Shop has total assets of $91.8 million. Fifty percent of these assets are financed with debt of which $28.9 million is current liabilities. The firm has no preferred stock but the balan...

See Answer

Q: If a firm needs to keep a minimum cash balance on hand

If a firm needs to keep a minimum cash balance on hand and faces both cash inflows and outflows, which of the cash management models discussed in this chapter would be more appropriate for it to use?...

See Answer

Q: Muffin’s Masonry, Inc’s balance sheet lists net fixed asset as $

Muffin’s Masonry, Inc’s balance sheet lists net fixed asset as $14 million. The fixed assets could currently be sold for $19 million. Muffin’s current balance sheet shows current liabilities of $5.5 m...

See Answer

Q: Daddi Mac, Inc., doesn’t face any taxes and has $

Daddi Mac, Inc., doesn’t face any taxes and has $350 million in assets, currently financed entirely with equity. Equity is worth $37 per share, and book value of equity is equal to m...

See Answer

Q: Value Ava’s Spin Ball Corp. lists fixed assets of $12

Value Ava’s Spin Ball Corp. lists fixed assets of $12 million on its balance sheet. The firm’s fixed assets have recently been appraised at $16 million. Ava’s Spin Ball Corp.’s balance sheet also list...

See Answer

Q: You have been given the following information for Corky’s Bedding Corp.:

You have been given the following information for Corky’s Bedding Corp.: a. Net sales = $11,250,000. b. Cost of goods sold = $7,500,000. c. Other operating expenses = $250,000. d. Addition to retained...

See Answer

Q: You have been given the following information for Moore’s HoneyBee Corp.:

You have been given the following information for Moore’s HoneyBee Corp.: a. Net sales = $32,000,000. b. Gross profits = $18,700,000. c. Other operating expenses = $2,500,000. d. Addition to retained...

See Answer

Q: Consider a firm with an EBIT of $1,000,

Consider a firm with an EBIT of $1,000,000. The firm finances its assets with $4,500,000 debt (costing 8 percent) and 200,000 shares of stock selling at $16.00 per share. To reduce risk associated wit...

See Answer

Q: Consider a firm with an EBIT of $10,500,

Consider a firm with an EBIT of $10,500,000. The firm finances its assets with $50,000,000 debt (costing 6.5 percent) and 10,000,000 shares of stock selling at $10.00 per share. The firm is considerin...

See Answer

Q: The Dakota Corporation had a 2018 taxable income of $33,

The Dakota Corporation had a 2018 taxable income of $33,365,000 from operations after all operating costs but before (1) interest charges of $8,500,000; (2) dividends received of $750,000; (3) dividen...

See Answer

Q: Suppose that in addition to $17.85 million of taxable

Suppose that in addition to $17.85 million of taxable income, Texas Taco, Inc., received $1,105,000 of interest on state-issued bonds and $760,000 of dividends on common stock it owns in Arizona Taco,...

See Answer

Q: Use the balance sheet and income statement below to construct a statement

Use the balance sheet and income statement below to construct a statement of cash flows for Clancy’s Dog Biscuit Corporation. / Dividends per share (DPS) $0.60 $0.60 Book value p...

See Answer

Q: Use the balance sheet and income statement below to construct a statement

Use the balance sheet and income statement below to construct a statement of cash flows for Valium’s Medical Supply Corporation.

See Answer

Q: Can the procedure described in this chapter for adjusting for seasonality apply

Can the procedure described in this chapter for adjusting for seasonality apply to periods longer than a year? How?

See Answer

Q: A firm does not pay a dividend. It is expected to

A firm does not pay a dividend. It is expected to pay its first dividend of $0.25 per share in two years. This dividend will grow at 10 percent indefinitely. Using an 11.5 percent discount rate, comp...

See Answer

Q: Chris’ Outdoor Furniture, Inc., has net cash flows from operating

Chris’ Outdoor Furniture, Inc., has net cash flows from operating activities for the last year of $340 million. The income statement shows that net income is $315 million and depreciation expense is $...

See Answer

Q: Dogs 4 U Corporation has net cash flow from financing activities for

Dogs 4 U Corporation has net cash flow from financing activities for the last year of $34 million. The company paid $178 million in dividends last year. During the year, the change in notes payable on...

See Answer

Q: The 2018 income statement for Duffy’s Pest Control shows that depreciation expense

The 2018 income statement for Duffy’s Pest Control shows that depreciation expense was $197 million, EBIT was $494 million, and the tax rate was 30 percent. At the beginning of the year, the balance o...

See Answer

Q: The 2018 income statement for Egyptian Noise Blasters shows that depreciation expense

The 2018 income statement for Egyptian Noise Blasters shows that depreciation expense is $85 million, NOPAT is $246 million. At the end of the year, the balance of gross fixed assets was $655 million....

See Answer

Q: Thelma and Louie, Inc., started the year with a balance

Thelma and Louie, Inc., started the year with a balance of retained earnings of $543 million and ended the year with retained earnings of $589 million. The company paid dividends of $35 million to the...

See Answer

Q: Jamaica Tours, Inc., started the year with a balance of

Jamaica Tours, Inc., started the year with a balance of retained earnings of $1,780 million. The company reported net income for the year of $284 million and paid dividends of $17 million to the prefe...

See Answer

Q: Listed below is the 2018 income statement for Tom and Sue Travels

Listed below is the 2018 income statement for Tom and Sue Travels, Inc.

See Answer

Q: You have been given the following information for Patty Cake’s Athletic Wear

You have been given the following information for Patty Cake’s Athletic Wear Corp. for the year 2018: a. Net sales = $38,250,000. b. Cost of goods sold = $22,070,000. c. Other operating expenses = $5,...

See Answer

Q: Rebecky’s Flowers 4U, Inc., had free cash flows during 2018

Rebecky’s Flowers 4U, Inc., had free cash flows during 2018 of $43 million, NOPAT of $85 million, and depreciation of $14 million. Using this information, fill in the blanks on Rebecky’s balance sheet...

See Answer

Q: The Fitness Studio, Inc.’s 2018 income statement lists the

The Fitness Studio, Inc.’s 2018 income statement lists the following income and expenses: EBIT = $538,000, interest expense = $63,000, and net income = $435,000. Calculate the 2018 taxes reported on t...

See Answer

Q: Kellogg Co. (K) recently earned a profit of $

Kellogg Co. (K) recently earned a profit of $2.52 per share and has a P/E ratio of 13.5. The dividend has been growing at a 5 percent rate over the past few years. If this growth rate continues, what...

See Answer

Q: Everything else held constant, which will be greater: AFN for

Everything else held constant, which will be greater: AFN for a firm with excess fixed- asset capacity, or AFN for a firm with no excess fixed-asset capacity? Why?

See Answer

Q: The Fitness Studio, Inc.’s 2018 income statement lists the

The Fitness Studio, Inc.’s 2018 income statement lists the following income and expenses: EBIT = $773,500, interest expense = $100,000, and taxes = $234,500. The firm has no preferred stock outstandin...

See Answer

Q: In 2018, Usher Sports Shop had cash flows from investing activities

In 2018, Usher Sports Shop had cash flows from investing activities of -$4,364,000 and cash flows from financing activities of -$5,880,000. The balance in the firm’s cash account was $1,615,000 at the...

See Answer

Q: Mr. Husker’s Tuxedos, Corp. began the year 2018 with

Mr. Husker’s Tuxedos, Corp. began the year 2018 with $256 million in retained earnings. The firm earned net income of $33 million in 2018 and paid dividends of $5 million to its preferred stockholders...

See Answer

Q: You are considering a stock investment in one of two firms (

You are considering a stock investment in one of two firms (NoEquity, Inc., and NoDebt, Inc.), both of which operate in the same industry and have identical operating income of $32.5 million. NoEquity...

See Answer

Q: You are considering a stock investment in one of two firms (

You are considering a stock investment in one of two firms (All Debt, Inc., and All Equity, Inc.), both of which operate in the same industry and have identical operating income of $12.5 million. AllD...

See Answer

Q: List and describe the four major financial statements.

List and describe the four major financial statements.

See Answer

Q: On which of the four major financial statements (balance sheet,

On which of the four major financial statements (balance sheet, income statement, statement of cash flows, or statement of retained earnings) would you find the following items?

See Answer

Q: How does the choice of accounting method used to record fixed asset

How does the choice of accounting method used to record fixed asset depreciation affect management of the balance sheet?

See Answer

Q: What are the costs and benefits of holding liquid securities on a

What are the costs and benefits of holding liquid securities on a firm’s balance sheet?

See Answer

Q: New York Times Co. (NYT) recently earned a profit

New York Times Co. (NYT) recently earned a profit of $1.21 per share and has a P/E ratio of 19.59. The dividend has been growing at a 7.25 percent rate over the past six years. If this growth rate c...

See Answer

Q: Why can the book value and market value of a firm differ

Why can the book value and market value of a firm differ?

See Answer

Q: What does a negative value for AFN mean?

What does a negative value for AFN mean?

See Answer

Q: From a firm manager’s or investor’s point of view, which is

From a firm manager’s or investor’s point of view, which is more important―the book value of a firm or the market value of the firm?

See Answer

Q: What do we mean by a progressive tax structure?

What do we mean by a progressive tax structure?

See Answer

Q: What is the difference between an average tax rate and a marginal

What is the difference between an average tax rate and a marginal tax rate?

See Answer

Q: How does the payment of interest on debt affect the amount of

How does the payment of interest on debt affect the amount of taxes the firm must pay?

See Answer

Q: The income statement is prepared using GAAP. How does this affect

The income statement is prepared using GAAP. How does this affect the reported revenue and expense measures listed on the balance sheet?

See Answer

Q: Why do financial managers and investors find cash flows to be more

Why do financial managers and investors find cash flows to be more important than accounting profit?

See Answer

Q: What is the difference between cash flows from operating activities, cash

What is the difference between cash flows from operating activities, cash flows from investing activities, and cash flows from financing activities?

See Answer

Q: What are free cash flows for a firm? What does it

What are free cash flows for a firm? What does it mean when a firm’s free cash flow is negative?

See Answer

Q: Carnival Corp. (CCL) provides cruises to major vacation destinations

Carnival Corp. (CCL) provides cruises to major vacation destinations. Carnival operates 99 cruise ships in North America, Europe, Australia, and Asia. The company also operates hotels, sightseeing mot...

See Answer

Q: What is earnings management?

What is earnings management?

See Answer

Q: What does the Sarbanes-Oxley Act require of firm managers?

What does the Sarbanes-Oxley Act require of firm managers?

See Answer

Q: Which specific item of a pro forma income statement should be most

Which specific item of a pro forma income statement should be most expected to vary proportionately with sales? Why?

See Answer

Q: What is the difference between current liabilities and long-term debt

What is the difference between current liabilities and long-term debt?

See Answer

Q: Which of the following activities result in an increase (decrease)

Which of the following activities result in an increase (decrease) in a firm’s cash?

See Answer

Q: Everything else held constant, if a firm announces that it will

Everything else held constant, if a firm announces that it will halve the length of time between its ex-dividend date and its payment date, what should be the effect on the stock price?

See Answer

Q: Show mathematically that, with a tax rate on both dividends and

Show mathematically that, with a tax rate on both dividends and capital gains of 5 percent, it doesn’t matter whether earnings are paid out as dividends or kept in the firm to cause g to grow for a co...

See Answer

Q: Show mathematically that, with a tax rate on both dividends and

Show mathematically that, with a tax rate on both dividends and capital gains of 15 percent, it doesn’t matter whether earnings are paid out as dividends or kept in the firm to cause g to grow for a c...

See Answer

Q: Everything else held constant, if a firm announces that it will

Everything else held constant, if a firm announces that it will double the length of time between its ex-dividend date and its payment date, what should be the effect on the stock price?

See Answer

Q: Annually Suppose that a firm always announces a yearly dividend at the

Annually Suppose that a firm always announces a yearly dividend at the end of the first quarter of the year, but then pays the dividend out as four equal quarterly payments. If the next such “annual”...

See Answer

Q: Design a spreadsheet similar to the one below to compute the value

Design a spreadsheet similar to the one below to compute the value of a variable growth rate firm over a five-year horizon. A. What is the value of the stock if the current dividend is $1.30, the firs...

See Answer

Q: Suppose that a firm always announces a yearly dividend at the end

Suppose that a firm always announces a yearly dividend at the end of the first quarter of the year, but then pays the dividend out as four equal quarterly payments. If the next such “annual” dividend...

See Answer

Q: If a firm has retained earnings of $23 million, a

If a firm has retained earnings of $23 million, a common shares account of $275 million, and additional paid-in-capital of $100 million, how would these accounts change in response to a 20 percent sto...

See Answer

Q: If a firm has retained earnings of $3 million, a

If a firm has retained earnings of $3 million, a common shares account of $5 million, and additional paid-in-capital of $10 million, how would these accounts change in response to a 10 percent stock d...

See Answer

Q: Explain why we need to use the iterative calculation approach described in

Explain why we need to use the iterative calculation approach described in the text to get a complete solution for AFN.

See Answer

Q: Suppose a firm pays total dividends of $500,000 out

Suppose a firm pays total dividends of $500,000 out of net income of $2 million. What would the firm’s payout ratio be?

See Answer

Q: Suppose a firm pays total dividends of $750,000 out

Suppose a firm pays total dividends of $750,000 out of net income of $5 million. What would the firm’s payout ratio be?

See Answer

Q: Suppose a firm has a retention ratio of 35 percent and net

Suppose a firm has a retention ratio of 35 percent and net income of $5 million. How much does it pay out in dividends?

See Answer

Q: Suppose a firm has a retention ratio of 56 percent and net

Suppose a firm has a retention ratio of 56 percent and net income of $9 million. How much does it pay out in dividends?

See Answer

Q: Suppose a firm has a retention ratio of 40 percent, net

Suppose a firm has a retention ratio of 40 percent, net income of $17 million, and 10 million shares outstanding. What would be the dividend per share paid out on the firm’s stock?

See Answer

Q: Suppose a firm has a retention ratio of 60 percent, net

Suppose a firm has a retention ratio of 60 percent, net income of $35 million, and 140 million shares outstanding. What would be the dividend per share paid out on the firm’s stock?

See Answer

Q: Spreadsheets are especially useful for computing stock value under different assumptions.

Spreadsheets are especially useful for computing stock value under different assumptions. Consider a firm that is expected to pay the following dividends: Year 1 2 3 4 5 6 $1.20 $1.2...

See Answer

Q: MMK Cos. Normally pays an annual dividend. The last such

MMK Cos. Normally pays an annual dividend. The last such dividend paid was $2.25, all future dividends are expected to grow at a rate of 7 percent per year, and the firm faces a required rate of retur...

See Answer

Q: Gen Corp. is expected to pay a dividend of $3

Gen Corp. is expected to pay a dividend of $3.50 per year indefinitely. If the appropriate rate of return on this stock is 11 percent per year, and the stock consistently goes ex-dividend 35 days befo...

See Answer

Q: Kenzie Cos. is expected to pay a dividend of $2

Kenzie Cos. is expected to pay a dividend of $2.75 per year indefinitely. If the appropriate rate of return on this stock is 16 percent per year, and the stock consistently goes ex-dividend 40 days be...

See Answer

Q: JBK, Inc., normally pays an annual dividend. The last

JBK, Inc., normally pays an annual dividend. The last such dividend paid was $2.50, all future dividends are expected to grow at 5 percent, and the firm faces a required rate of return on equity of 11...

See Answer

Q: What approach should be used to forecast sales if a firm believes

What approach should be used to forecast sales if a firm believes that sales will be stable over time?

See Answer

Q: If a firm announces a dividend decrease, would you expect the

If a firm announces a dividend decrease, would you expect the stock price to go down more or less than the present value of that decrease? Why?

See Answer

Q: Explain how an announced increase in a firm’s dividend payout might be

Explain how an announced increase in a firm’s dividend payout might be perceived as either a good or a bad information signal.

See Answer

Q: Why might the government actually want the capital gains tax rate to

Why might the government actually want the capital gains tax rate to be lower than the dividend tax rate?

See Answer

Q: What condition would have to be necessary in order for the riskiness

What condition would have to be necessary in order for the riskiness of the firm’s cash flows to investors to be affected by the firm’s dividend payout policy?

See Answer

Q: We talked about how a firm might attract a different clientele by

We talked about how a firm might attract a different clientele by switching dividend payout policies: Might a particular clientele change its preference for dividends versus capital gains through no a...

See Answer

Q: Describe the process for using the P/E ratio to estimate

Describe the process for using the P/E ratio to estimate a future stock price.

See Answer

Q: Suppose that federal banking regulators in the United States announced that they

Suppose that federal banking regulators in the United States announced that they are going to allow banks to take on significant equity investments in firms to which they have lent. What would you exp...

See Answer

Q: If a firm follows the modified residual dividend model discussed in this

If a firm follows the modified residual dividend model discussed in this chapter, are extraordinary dividends paid out of residual net income?

See Answer

Q: Suppose a firm announces a new dividend amount every year with the

Suppose a firm announces a new dividend amount every year with the first quarterly dividend declaration, but never explicitly states that the dividend will be continued for the other three quarters of...

See Answer

Q: Could the record date ever be before the ex-dividend date

Could the record date ever be before the ex-dividend date? Why or why not?

See Answer

Q: Suppose a firm managed to consistently lower the length of time between

Suppose a firm managed to consistently lower the length of time between the ex-dividend date and the payment date. On average, how would this affect the firm’s stock price?

See Answer

Q: The company from the text, Yellow Jacket, has decided to

The company from the text, Yellow Jacket, has decided to change its production strategy. Instead of a steady production throughout the year, they will produce the coats they estimate to sell in the mo...

See Answer

Q: How big of a stock dividend would a firm have to announce

How big of a stock dividend would a firm have to announce for the stock price to be affected as much as it would through a 3-for-1 stock split?

See Answer

Q: A firm has estimated the following two month cash budget. What

A firm has estimated the following two month cash budget. What is the cash surplus or deficit for these two months?

See Answer

Q: You hold the positions in the table below. What is the

You hold the positions in the table below. What is the beta of your portfolio? If you expect the market to earn 12 percent and the risk-free rate is 3.5 percent, what is the required return of the por...

See Answer

Q: A firm has estimated the following two month cash budget. What

A firm has estimated the following two month cash budget. What is the cash surplus or deficit for these two months?

See Answer

Q: As owners, what rights and advantages do shareholders obtain?

As owners, what rights and advantages do shareholders obtain?

See Answer

Q: The net cash flow for a firm in January, February,

The net cash flow for a firm in January, February, and March are $-2.5 million, $-3.0 million, and $2.4 million. What is the cumulative net cash flow for March?

See Answer

Q: The Net Cash Flow for a firm in January, February,

The Net Cash Flow for a firm in January, February, and March are $3.5 million, $-1.0 million, and $1.4 million. What is the Cumulative Net Cash Flow for March?

See Answer

Q: The Hug’a’Bear company makes its teddy bears the month before they are

The Hug’a’Bear company makes its teddy bears the month before they are sold. If sales of $2.5 million are expected in November and the firm pays 50 percent of sales in material costs, then what is the...

See Answer

Q: The Snow Adventures Company makes its snowboards the month before they are

The Snow Adventures Company makes its snowboards the month before they are sold. If sales of $7.8 million are expected in November and the firm pays 65 percent of sales in material costs, then what is...

See Answer

Q: Consider a company that has sales in May, June, and

Consider a company that has sales in May, June, and July of $10 million, $12 million, and $9 million, respectively. The firm is paid by 35 percent of its customers in the month of the sale, 40 perce...

See Answer

Q: Consider a company that has sales in May, June, and

Consider a company that has sales in May, June, and July of $11 million, $10 million, and $12 million, respectively. The firm is paid by 25 percent of its customers in the month of the sale, 50 percen...

See Answer

Q: Would it be worth it to incur a compensating balance of $

Would it be worth it to incur a compensating balance of $7,500 in order to get a 0.65-percent-lower interest rate on a two-year, pure discount loan of $150,000?

See Answer

Q: Would it be worth it to incur a compensating balance of $

Would it be worth it to incur a compensating balance of $10,000 in order to get a 1-percent-lower interest rate on a one-year, pure discount loan of $225,000?

See Answer

Q: Dandee Lions, Inc., has a cash balance of $105

Dandee Lions, Inc., has a cash balance of $105,000, accounts payable of $220,000, inventory of $203,000, accounts receivable of $319,000, notes payable of $65,000, and accrued wages and taxes of $75,0...

See Answer

Q: You hold the positions in the table below. What is the

You hold the positions in the table below. What is the beta of your portfolio? If you expect the market to earn 12 percent and the risk-free rate is 3.5 percent, what is the required return of the por...

See Answer

Q: Suppose that a firm’s recent earnings per share and dividend per share

Suppose that a firm’s recent earnings per share and dividend per share are $2.75 and $1.60, respectively. Both are expected to grow at 9 percent. However, the firm’s current P/E ratio of 23 seems high...

See Answer

Q: Obtain a current quote of McDonald’s (MCD) from the Internet

Obtain a current quote of McDonald’s (MCD) from the Internet. Describe what has changed since the quote in Figure 8.1.

See Answer

Q: Dabble, Inc., has sales of $980,000 and

Dabble, Inc., has sales of $980,000 and cost of goods sold of $640,000. The firm had a beginning inventory of $36,000 and an ending inventory of $46,000. What is the length of the days’ sales in inven...

See Answer

Q: Sow Tire, Inc., has sales of $1,450

Sow Tire, Inc., has sales of $1,450,000 and cost of goods sold of $980,000. The firm had a beginning inventory of $97,000 and an ending inventory of $82,000. What is the length of the days’ sales in i...

See Answer

Q: If a firm has a cash cycle of 67 days and an

If a firm has a cash cycle of 67 days and an operating cycle of 104 days, what is its average payment period?

See Answer

Q: If a firm has a cash cycle of 45 days and an

If a firm has a cash cycle of 45 days and an operating cycle of 77 days, what is its average payment period?

See Answer

Q: If a firm has a cash cycle of 73 days and an

If a firm has a cash cycle of 73 days and an operating cycle of 127 days, what is its payables turnover?

See Answer

Q: If a firm has a cash cycle of 54 days and an

If a firm has a cash cycle of 54 days and an operating cycle of 77 days, what is its payables turnover?

See Answer

Q: CM Enterprises estimates that it takes, on average, three days

CM Enterprises estimates that it takes, on average, three days for their customers’ payments to reach them, one day for the payments to be processed and deposited by their bookkeeping department, and...

See Answer

Q: Smelpank, Inc.,estimates that it takes, on average,

Smelpank, Inc.,estimates that it takes, on average, four days for their customers’ payments to reach them, three days for the payments to be processed and deposited by their bookkeeping department, an...

See Answer

Q: JohnBoy Industries has a cash balance of $45,000,

JohnBoy Industries has a cash balance of $45,000, accounts payable of $125,000, inventory of $175,000, accounts receivable of $210,000, notes payable of $120,000, and accrued wages and taxes of $37,00...

See Answer

Q: HotFoot Shoes would like to maintain their cash account at a minimum

HotFoot Shoes would like to maintain their cash account at a minimum level of $25,000, but expect the standard deviation in net daily cash flows to be $2,000, the effective annual rate on marketable s...

See Answer

Q: Get the trading statistics for the three main U.S.

Get the trading statistics for the three main U.S. stock exchanges. Compare the trading activity to that of Table 8.1.

See Answer

Q: HiLo, Inc., doesn’t face any taxes and has $100

HiLo, Inc., doesn’t face any taxes and has $100 million in assets, currently financed entirely with equity. Equity is worth $7 per share, and book value of equity is equal to market...

See Answer

Q: Why might the Standard & Poor’s 500 Index be a better measure

Why might the Standard & Poor’s 500 Index be a better measure of stock market performance than the Dow Jones Industrial Average? Why is the DJIA more popular than the S&P 500?

See Answer

Q: Which is higher, the ask quote or the bid quote?

Which is higher, the ask quote or the bid quote? Why?

See Answer

Q: Illustrate through examples how trading commission costs impact an investor’s return.

Illustrate through examples how trading commission costs impact an investor’s return.

See Answer

Q: Describe the difference in the timing of trade execution and the certainty

Describe the difference in the timing of trade execution and the certainty of trade price between market orders and limit orders.

See Answer

Q: What are the differences between common stock and preferred stock?

What are the differences between common stock and preferred stock?

See Answer

Q: How important is growth to a stock’s value? Illustrate with examples

How important is growth to a stock’s value? Illustrate with examples.

See Answer

Q: The expected return derived from the constant growth rate model relies on

The expected return derived from the constant growth rate model relies on dividend yield and capital gain. Where do these two parts of the return come from?

See Answer

Q: A firm recently paid a $0.45 annual dividend.

A firm recently paid a $0.45 annual dividend. The dividend is expected to increase by 10 percent in each of the next four years. In the fourth year, the stock price is expected to be $80. If the requi...

See Answer

Q: Describe, in words, how to use the variable growth rate

Describe, in words, how to use the variable growth rate technique to value a stock.

See Answer

Q: Can the variable growth rate model be used to value a firm

Can the variable growth rate model be used to value a firm that has a negative growth rate in Stage 1 and a stable and positive growth rate in Stage 2? Explain.

See Answer

Q: Explain why using the P/E relative value approach may be

Explain why using the P/E relative value approach may be useful for companies that do not pay dividends.

See Answer

Q: Why might a firm’s investors wish to delay receiving cash from the

Why might a firm’s investors wish to delay receiving cash from the firm?

See Answer

Q: Daddi Mac, Inc., doesn’t face any taxes and has $

Daddi Mac, Inc., doesn’t face any taxes and has $290 million in assets, currently financed entirely with equity. Equity is worth $37 per share, and book value of equity is equal to m...

See Answer

Q: How is a firm’s changing P/E ratio reflected in the

How is a firm’s changing P/E ratio reflected in the stock price? Give examples.

See Answer

Q: Differentiate the characteristics of growth stocks and value stocks?

Differentiate the characteristics of growth stocks and value stocks?

See Answer

Q: What’s the relationship between the P/E ratio and a firm’s

What’s the relationship between the P/E ratio and a firm’s growth rate?

See Answer

Q: Describe how being a residual claimant can be very valuable.

Describe how being a residual claimant can be very valuable.

See Answer

Q: Explain how it is possible for the DJIA to increase one day

Explain how it is possible for the DJIA to increase one day while the Nasdaq Composite decreases during the same day.

See Answer

Q: A firm recently paid a $0.60 annual dividend.

A firm recently paid a $0.60 annual dividend. The dividend is expected to increase by 12 percent in each of the next four years. In the fourth year, the stock price is expected to be $110. If the r...

See Answer

Q: Under what conditions would the constant growth rate model not be appropriate

Under what conditions would the constant growth rate model not be appropriate?

See Answer

Q: Determine the interest payment for the following three bonds: 3 ½

Determine the interest payment for the following three bonds: 3 ½ percent coupon corporate bond (paid semiannually), 4.25 percent coupon Treasury note, and a corporate zero coupon bond maturing in...

See Answer

Q: Determine the interest payment for the following three bonds: 4 ½

Determine the interest payment for the following three bonds: 4 ½ percent coupon corporate bond (paid semiannually), 5.15 percent coupon Treasury note, and a corporate zero coupon bond maturing in 15...

See Answer

Q: A bond issued by Ford on May 15, 1997 is scheduled

A bond issued by Ford on May 15, 1997 is scheduled to mature on May 15, 2097. If today is November 16, 2014, what is this bond’s time to maturity?

See Answer

Q: A bond issued by IBM on December 1, 1996 is scheduled

A bond issued by IBM on December 1, 1996 is scheduled to mature on December 1, 2096. If today is December 2, 2015, what is this bond’s time to maturity?

See Answer

Q: Compute the expected return and standard deviation given these four economic states

Compute the expected return and standard deviation given these four economic states, their likelihoods, and the potential returns:

See Answer

Q: A 6 percent corporate coupon bond is callable in five years for

A 6 percent corporate coupon bond is callable in five years for a call premium of one year of coupon payments. Assuming a par value of $1,000, what is the price paid to the bondholder if the issue...

See Answer

Q: A 5.5 percent corporate coupon bond is callable in ten

A 5.5 percent corporate coupon bond is callable in ten years for a call premium of one year of coupon payments. Assuming a par value of $1,000, what is the price paid to the bondholder if the issuer...

See Answer

Q: A 2 ¾ percent TIPS has an original reference CPI of 185

A 2 ¾ percent TIPS has an original reference CPI of 185.4. If the current CPI is 210.7, what is the current interest payment and par value of the TIPS?

See Answer

Q: A 3 1/8 percent TIPS has an original reference CPI

A 3 1/8 percent TIPS has an original reference CPI of 180.5. If the current CPI is 206.8, what is the current interest payment and par value of the TIPS?

See Answer

Q: Suppose that Papa Bell, Inc.’s, equity is currently

Suppose that Papa Bell, Inc.’s, equity is currently selling for $55 per share, with 4 million shares outstanding. If the firm also has 17 thousand bonds outstanding, which are selling at 94 percent of...

See Answer

Q: Consider the following three bond quotes; a Treasury note quoted at

Consider the following three bond quotes; a Treasury note quoted at 97.844, and a corporate bond quoted at 103.25, and a municipal bond quoted at 101.90. If the Treasury and corporate bonds have a par...

See Answer

Q: Consider the following three bond quotes; a Treasury bond quoted at

Consider the following three bond quotes; a Treasury bond quoted at 106.438, a corporate bond quoted at 96.55, and a municipal bond quoted at 100.95. If the Treasury and corporate bonds have a par...

See Answer

Q: Calculate the price of a zero coupon bond that matures in 20

Calculate the price of a zero coupon bond that matures in 20 years if the market interest rate is 3.8 percent.

See Answer

Q: Calculate the price of a zero coupon bond that matures in 15

Calculate the price of a zero coupon bond that matures in 15 years if the market interest rate is 5.75 percent.

See Answer

Q: What’s the current yield of a 3.8 percent coupon corporate

What’s the current yield of a 3.8 percent coupon corporate bond quoted at a price of 102.08?

See Answer

Q: What’s the current yield of a 5.2 percent coupon corporate

What’s the current yield of a 5.2 percent coupon corporate bond quoted at a price of 96.78?

See Answer

Q: Compute the expected return and standard deviation given these four economic states

Compute the expected return and standard deviation given these four economic states, their likelihoods, and the potential returns:

See Answer

Q: What’s the taxable equivalent yield on a municipal bond with a yield

What’s the taxable equivalent yield on a municipal bond with a yield to maturity of 3.5 percent for an investor in the 33 percent marginal tax bracket?

See Answer

Q: What’s the taxable equivalent yield on a municipal bond with a yield

What’s the taxable equivalent yield on a municipal bond with a yield to maturity of 2.9 percent for an investor in the 28 percent marginal tax bracket?

See Answer

Q: Rank from highest credit risk to lowest risk the following bonds,

Rank from highest credit risk to lowest risk the following bonds, with the same time to maturity, by their yield to maturity: Treasury bond with yield of 5.55 percent, IBM bond with yield of 7.49 per...

See Answer

Q: Consider a firm that had been priced using a 10 percent growth

Consider a firm that had been priced using a 10 percent growth rate and a 12 percent required return. The firm recently paid a $1.20 dividend. The firm has just announced that because of a new joint v...

See Answer

Q: Rank the following bonds in order from lowest credit risk to highest

Rank the following bonds in order from lowest credit risk to highest risk, all with the same time to maturity, by their yield to maturity: Treasury bond with yield of 4.65 percent, United Airline bond...

See Answer

Q: A corporate bond with a 6.75 percent coupon has ten

A corporate bond with a 6.75 percent coupon has ten years left to maturity. It has had a credit rating of BB and a yield to maturity of 8.2 percent. The firm has recently become more financially stabl...

See Answer

Q: A 5.75 percent coupon bond with ten years left to

A 5.75 percent coupon bond with ten years left to maturity is priced to offer a 6.5 percent yield to maturity. You believe that in one year, the yield to maturity will be 6.0 percent. What is the chan...

See Answer

Q: A 6.5 percent coupon bond with 14 years left to

A 6.5 percent coupon bond with 14 years left to maturity is priced to offer a 7.2 percent yield to maturity. You believe that in one year, the yield to maturity will be 6.8 percent. What is the change...

See Answer

Q: Reconsider the 3.5 percent TIPS discussed in problem 7-

Reconsider the 3.5 percent TIPS discussed in problem 7-19. It was issued with CPI reference of 185.6. The bond is purchased at the beginning of the year (after the interest payment), when the CPI was...

See Answer

Q: Reconsider the 2.25 percent TIPS discussed in problem 7-

Reconsider the 2.25 percent TIPS discussed in problem 7-20. It was issued with CPI reference of 187.2. The bond is purchased at the beginning of the year (after the interest payment), when the CPI was...

See Answer

Q: A 6.25 percent coupon bond with 22 years left to

A 6.25 percent coupon bond with 22 years left to maturity is priced to offer a 5.5 percent yield to maturity. You believe that in one year, the yield to maturity will be 6.0 percent. If this occurs, w...

See Answer

Q: Compute the expected return given these three economic states, their likelihoods

Compute the expected return given these three economic states, their likelihoods, and the potential returns:

See Answer

Q: A 7.5 percent coupon bond with 13 years left to

A 7.5 percent coupon bond with 13 years left to maturity is priced to offer a 6.25 percent yield to maturity. You believe that in one year, the yield to maturity will be 7.0 percent. If this occurs, w...

See Answer

Q: A 2.50 percent coupon municipal bond has 12 years left

A 2.50 percent coupon municipal bond has 12 years left to maturity and has a price quote of 98.45. The bond can be called in four years. The call premium is one year of coupon payments. Compute and di...

See Answer

Q: Describe the difference between a merger and an acquisition.

Describe the difference between a merger and an acquisition.

See Answer

Q: Describe the similarities and the differences of exchange rate/cross rate

Describe the similarities and the differences of exchange rate/cross rate arbitrage and spot rate/forwardrate arbitrage.

See Answer

Q: The Justice Department has been asked to review a merger request for

The Justice Department has been asked to review a merger request for a market with the following four firms. Firm Assets...

See Answer

Q: Stubborn Motors, Inc. is asking a price of $75

Stubborn Motors, Inc. is asking a price of $75 million to be purchased by Rubber Tire Motor Corp. Stubborn Motors currently has total cash flows of $2 million that are expected to grow by 1 percent an...

See Answer

Q: Disaster Airlines is a firm in severe financial distress. The firm

Disaster Airlines is a firm in severe financial distress. The firm can no longer pay its bills on time and it is far behind on payments to its banks and long-term debt holders. The firm has decided to...

See Answer

Q: What is a credit-scoring model?

What is a credit-scoring model?

See Answer

Q: How does a best efforts underwriting differ from a firm commitment underwriting

How does a best efforts underwriting differ from a firm commitment underwriting? If you operated a company issuing stock for the first time, which type of underwriting would you prefer? Why might you...

See Answer

Q: How does a competitive sale of corporate bonds differ from a negotiated

How does a competitive sale of corporate bonds differ from a negotiated sale? Which type of underwriting would you prefer? Why might you still choose the alternative?

See Answer

Q: How does a public offering of debt or equity securities issued by

How does a public offering of debt or equity securities issued by a public firm differ from a private placement?

See Answer

Q: What are the net proceeds, gross proceeds, and underwriter’s spread

What are the net proceeds, gross proceeds, and underwriter’s spread? How does each affect the funds received by a public firm when debt or equity securities are issued?

See Answer

Q: Why would an investment bank use a syndicate to assist in underwriting

Why would an investment bank use a syndicate to assist in underwriting debt or equity securities?

See Answer

Q: What is the difference between a prospectus and a red herring prospectus

What is the difference between a prospectus and a red herring prospectus?

See Answer

Q: What is a shelf registration? Why would a public firm want

What is a shelf registration? Why would a public firm want to issue securities using a shelf registration?

See Answer

Q: Describe the various sources of capital funding available to new and small

Describe the various sources of capital funding available to new and small firms.

See Answer

Q: Explain how a country’s import trade limitations and tariffs influence MNC’s foreign

Explain how a country’s import trade limitations and tariffs influence MNC’s foreign direct investment.

See Answer

Q: Imagine that you are a financial manager of a multinational corporation,

Imagine that you are a financial manager of a multinational corporation, like Starbucks Coffee, in charge of determining the impact of exchange rate changes on the firm. Changes in currency exchange a...

See Answer

Q: Use the following financial statements for Garners’ Platoon Mental Health Care,

Use the following financial statements for Garners’ Platoon Mental Health Care, Inc., to calculate and interpret the Altman’s Z-score for this firm. / / /

See Answer

Q: Given these two exchange rates, $1 = 12.268

Given these two exchange rates, $1 = 12.268 Mexican pesos and $1 = €0.7624, compute the cross rate between the Mexican peso and the euro. State this exchange rate in pesos and in euros.

See Answer

Q: Given these two exchange rates, $1 = 0.9952

Given these two exchange rates, $1 = 0.9952 Australian dollars and $1 = £0.6476, compute the cross rate between the Australian dollars and the pound. State this exchange rate in Australian dollars and...

See Answer

Q: In 1997, many East Asian currencies suddenly and dramatically devalued.

In 1997, many East Asian currencies suddenly and dramatically devalued. What is the percentage change in value of a $50 million investment in Indonesia when the exchange rate changes from $1 = 2,000 r...

See Answer

Q: The Russian financial crisis of 1998 caused its currency to be dramatically

The Russian financial crisis of 1998 caused its currency to be dramatically devalued. What is the percentage change in value of a $100 million investment in Russia when the exchange rate changes from...

See Answer

Q: The spot rate between the U.S. dollar and the

The spot rate between the U.S. dollar and the New Zealand dollar is $1 = NZD1.1867. If the interest rate in the United States is 5 percent and in New Zealand is four percent, then what should be the 3...

See Answer

Q: The spot rate between the U.S. dollar and the

The spot rate between the U.S. dollar and the Taiwan dollar is $1 = TWD29.905. If the interest rate in the United States is five percent and in Taiwan is three percent, then what should be the 1-month...

See Answer

Q: The current spot rate between the U.S. dollar and

The current spot rate between the U.S. dollar and the Swedish krona is $1 = 6.5228 krona. If the inflation rate in the United States is four percent and in Sweden is 2 percent, then what is the exp...

See Answer

Q: The current spot rate between the U.S. dollar and

The current spot rate between the U.S. dollar and the Netherland Antilles guilder is $1 = 1.7915 guilder. If the inflation rate in the United States is three percent and in the Netherland Antilles is...

See Answer

Q: Peter’s TV Supplies is considering a merger with Jan’s Radio Supply Stores

Peter’s TV Supplies is considering a merger with Jan’s Radio Supply Stores. Peter’s total operating costs of producing services are $250,000 for a sales volume (SP) of $4.5 million. Jan’s total operat...

See Answer

Q: What are the risks of foreign direct investment into the United States

What are the risks of foreign direct investment into the United States? What does new FDI into the United States mean for firms already operating in that industry in the United States?

See Answer

Q: Classify each of the following as a horizontal merger, a vertical

Classify each of the following as a horizontal merger, a vertical merger, a market extension merger, a conglomerate merger, or a product extension merger.

See Answer

Q: Cindy’s Computer Corp. is considering a merger with Bobby’s Hard Drive

Cindy’s Computer Corp. is considering a merger with Bobby’s Hard Drive, Inc. Cindy’s total operating costs of producing services are $3.4 million for a sales volume (SC) of $16 million. Bobby’s total...

See Answer

Q: Consider a market that has three firms with the following market shares

Consider a market that has three firms with the following market shares: Firm A = 35% Firm B = 41% Firm C = 24% Suppose firm B wants to acquire firm C so that the post-acquisition market would exhibi...

See Answer

Q: Consider a market that has three firms with the following market shares

Consider a market that has three firms with the following market shares: Firm A = 35% Firm B = 41% Firm C = 24% Suppose firm A wants to acquire firm C so that the post-acquisition market would exhi...

See Answer

Q: Suppose that the financial ratios of a potential borrowing firm take the

Suppose that the financial ratios of a potential borrowing firm take the following values: X1 = Net working capital/Total assets = 0.10, X2 = Retained earnings/Total assets = 0.20, X3 = Earnings befor...

See Answer

Q: Suppose that the financial ratios of a potential borrowing firm took the

Suppose that the financial ratios of a potential borrowing firm took the following values: X1 = Net working capital/Total assets = 0.27, X2 = Retained earnings/Total assets = 0.37, X3 = Earnings befor...

See Answer

Q: Suppose a linear probability model you have developed finds there are two

Suppose a linear probability model you have developed finds there are two factors influencing the past bankruptcy behavior of firms: the debt ratio and the profit margin. Based on past bankruptcy expe...

See Answer

Q: A linear probability model you have developed finds there are two factors

A linear probability model you have developed finds there are two factors influencing the past bankruptcy behavior of firms: the equity multiplier and the total asset turnover ratio. Based on past ban...

See Answer

Q: George’s Dry Cleaning is considering a merger with Weezzie’s Laundry Supply Stores

George’s Dry Cleaning is considering a merger with Weezzie’s Laundry Supply Stores. George’s total operating costs of producing services are $550,000 for sales volume (SG) of $4.5 million. Weezzie’s t...

See Answer

Q: Jenny’s Day Care is considering a merger with Lionel’s Diaper Manufacturers.

Jenny’s Day Care is considering a merger with Lionel’s Diaper Manufacturers. Jenny’s total operating costs of producing services are $595,000 for sales volume (SJ) of $2.4 million. Lionel’s total oper...

See Answer

Q: The Justice Department has been asked to review a merger request for

The Justice Department has been asked to review a merger request for a market with the following four firms. Firm Assets A……………..$12 million...

See Answer

Q: Suppose a linear probability model you have developed finds there are two

Suppose a linear probability model you have developed finds there are two factors influencing the past bankruptcy behavior of firms: the debt ratio and the profit margin. Based on past bankruptcy expe...

See Answer

Q: What is synergy and how does it apply to mergers?

What is synergy and how does it apply to mergers?

See Answer

Q: A linear probability model you have developed finds there are two factors

A linear probability model you have developed finds there are two factors influencing the past bankruptcy behavior of firms: the equity multiplier and the total asset turnover ratio. Based on past ban...

See Answer

Q: The Altman’s Z-score model has several weaknesses. What are

The Altman’s Z-score model has several weaknesses. What are they?

See Answer

Q: A survey of a local market has provided the following average cost

A survey of a local market has provided the following average cost data: Johnson Construction Corp. (JCC) has assets of $3 million and an average cost of 20 percent. Anderson Architects (AA) has asset...

See Answer

Q: Describe the three dimensions of revenue synergies that may be achieved in

Describe the three dimensions of revenue synergies that may be achieved in a merger.

See Answer

Q: A survey of a national market has provided the following average cost

A survey of a national market has provided the following average cost data: Jackson County Construction (JCC) has assets of $2.55 million and an average cost of 30 percent. Arkansas Architects (AA) ha...

See Answer

Q: You own stock in Make-UP-Artists, Inc.

You own stock in Make-UP-Artists, Inc. which has just made a bid of $30 million to purchase MHM Corporation. MHM Corp. currently has total cash flows of $2.5 million that are expected to grow by 2 per...

See Answer

Q: Describe the difference between a forward rate selling at a discount and

Describe the difference between a forward rate selling at a discount and selling at a premium. If the spot rate between the U.S. dollar and the Brazilian real is $1 = 2.0875 real and the 3-month forwa...

See Answer

Q: What is the difference between economies of scope and economies of scale

What is the difference between economies of scope and economies of scale? Can two firms involved in a merger benefit from both economies of scale and economies of scope?

See Answer

Q: Cakes, Corp. currently has a 60 percent market share in

Cakes, Corp. currently has a 60 percent market share in banking services, followed by Cookies, Inc., with 20 percent and Dippen Dough with 20 percent. a. What is the concentration ratio as measured by...

See Answer

Q: What is the Herfindahl-Hirschman Index? How is it calculated

What is the Herfindahl-Hirschman Index? How is it calculated and interpreted?

See Answer

Q: Tractor Supply, Corp. currently has a 50 percent market share

Tractor Supply, Corp. currently has a 50 percent market share in banking services, followed by Farm Equipment, Inc., with 30 percent and Plow Mart with 20 percent. a. What is the concentration ratio a...

See Answer

Q: How can managers’ personal incentives result in value-destroying mergers and

How can managers’ personal incentives result in value-destroying mergers and acquisitions?

See Answer

Q: The managers of BSW, Inc. have approached KCMP Corp.

The managers of BSW, Inc. have approached KCMP Corp. about a possible merger. KCMP Corp. is asking a price of $72 million to be purchased by BSW, Inc. KCMP Corp. currently has total cash flows of $6 m...

See Answer

Q: Why is NPV valuation an appropriate tool to use in the evaluation

Why is NPV valuation an appropriate tool to use in the evaluation of a merger target?

See Answer

Q: The managers of State Bank have been approached by City Bank about

The managers of State Bank have been approached by City Bank about a possible merger. State Bank is asking a price of $205 million to be purchased by City Bank. State Bank currently has total cash flo...

See Answer

Q: What is the difference between business failure, economic failure, and

What is the difference between business failure, economic failure, and technical insolvency?

See Answer

Q: Does a Chapter 7 bankruptcy increase the probability that creditors will be

Does a Chapter 7 bankruptcy increase the probability that creditors will be paid in full more so than a Chapter 11 bankruptcy?

See Answer

Q: A linear probability model you have developed finds there are two factors

A linear probability model you have developed finds there are two factors influencing the past bankruptcy behavior of firms: the debt- to-equity ratio and the sales-to-total assets ratio. Based on pas...

See Answer

Q: What is meant by hedging exchange rate risk and what are some

What is meant by hedging exchange rate risk and what are some ways it is done?

See Answer

Q: A firm is experiencing a temporary period of financial distress as the

A firm is experiencing a temporary period of financial distress as the result of a hurricane that has hit its local area. Because many of the firm’s customers have been severely hurt by the hurricane,...

See Answer

Q: A linear probability model you have developed finds there are two factors

A linear probability model you have developed finds there are two factors influencing the past bankruptcy behavior of firms: the debt- to-equity ratio and the profit margin. Based on past bankruptcy e...

See Answer

Q: What is the job of the trustee in an informal liquidation of

What is the job of the trustee in an informal liquidation of a firm’s assets?

See Answer

Q: What is the difference between a Chapter 11 and a Chapter 7

What is the difference between a Chapter 11 and a Chapter 7 bankruptcy?

See Answer

Q: What are the advantages of borrowing money in the country you plan

What are the advantages of borrowing money in the country you plan to invest it in?

See Answer

Q: If a Sony television costs $500 in the United States,

If a Sony television costs $500 in the United States, what do you think it should cost in Japan? What are some reasons that your price might not be right?

See Answer

Q: What happens to a country’s currency over time when it has a

What happens to a country’s currency over time when it has a high inflation rate? What will that mean for the country’s exports and imports?

See Answer

Q: What forces are at work that cause the price of wheat per

What forces are at work that cause the price of wheat per bushel to be the same in most every country of the world?

See Answer

Q: You own $25,000 in subordinated debt of Local Crossings

You own $25,000 in subordinated debt of Local Crossings, Inc. which declared bankruptcy on May 15, 2018 through a Chapter 7 filing. Local Crossings’ balance sheet at the time of the bankruptcy filing...

See Answer

Q: If the spot exchange rate between the U.S. dollar

If the spot exchange rate between the U.S. dollar and the Singapore dollar is $1 = SG$1.5266 and the 3-month expected exchange rate is $1 = SG$1.5305, then what is the expected inflation relationship...

See Answer

Q: Over the past decade, China has acquired hundreds of billions of

Over the past decade, China has acquired hundreds of billions of U.S. dollars because of the trade imbalance between the two countries. They have used many of these dollars to purchase U.S. Treasury b...

See Answer

Q: To what extent are employees of a bankrupt firm paid their wages

To what extent are employees of a bankrupt firm paid their wages and benefits due?

See Answer

Q: Can a U.S. firm experience political risk problems in

Can a U.S. firm experience political risk problems in its overseas projects because of the U.S. government? Give examples.

See Answer

Q: Give some examples of the financial complications that occur when evaluating a

Give some examples of the financial complications that occur when evaluating a capital budgeting project in a foreign country.

See Answer

Q: Below are the Consumer Price Index inflation rates each year for the

Below are the Consumer Price Index inflation rates each year for the United States and Japan. Also shown is the spot exchange rate for the beginning of each year. A. Using PPP (equation 19.3), comput...

See Answer

Q: Convert each of the following direct quotes to dollar indirect quotes:

Convert each of the following direct quotes to dollar indirect quotes: a. 1 Danish krone = $0.170 b. 1 Indian rupee = $0.0184 c. 1 Israeli shekel = $0.2751

See Answer

Q: Convert each of the following direct quotes to dollar indirect quotes:

Convert each of the following direct quotes to dollar indirect quotes: a. 1 Korean won = $0.0009 b. 1 Malaysian ringgit = $0.3238 c. 1 Thai baht = $0.0331

See Answer

Q: Convert each of the following indirect quotes to dollar direct quotes:

Convert each of the following indirect quotes to dollar direct quotes: a. $1 = 20,864 Vietnam dong b. $1 = 6.300 Venezuelan bolivar fuerte c. $1 = 9.175 South African rand

See Answer

Q: Convert each of the following indirect quotes to dollar direct quotes:

Convert each of the following indirect quotes to dollar direct quotes: a. $1 = 3.7497 Saudi Arabian riyal b. $1 = 44.15 Philippine peso c. $1 = 0.5409 Latvian lat

See Answer

Q: What is the order of payment to a firm’s creditors in a

What is the order of payment to a firm’s creditors in a Chapter 7 bankruptcy?

See Answer

Q: Compute the amount of each foreign currency that can be purchased for

Compute the amount of each foreign currency that can be purchased for $500,000: a. 1 Danish krone = $0.170 b. 1 Indian rupee = $0.0184 c. 1 Israeli shekel = $0.2751

See Answer

Q: Compute the amount of each foreign currency that can be purchased for

Compute the amount of each foreign currency that can be purchased for one million dollars: a. 1 Korean won = $0.0009 b. 1 Malaysian ringgit = $0.3238 c. 1 Thai baht = $0.0331

See Answer

Q: Compute the number of dollars that can be bought with two million

Compute the number of dollars that can be bought with two million of each foreign currency units: a. $1 = 20,864 Vietnam dong b. $1 = 6.300 Venezuelan bolivar fuerte c. $1 = 9.175 South African rand

See Answer

Q: What is the difference between a linear discriminant and a linear probability

What is the difference between a linear discriminant and a linear probability credit-scoring model?

See Answer

Q: Compute the number of dollars that can be bought with one million

Compute the number of dollars that can be bought with one million of each foreign currency units: a. $1 = 3.7497 Saudi Arabian riyal b. $1 = 44.150 Philippine peso c. $1 = 0.5409 Latvian lat

See Answer

Q: If the price of silver in England is £15.23

If the price of silver in England is £15.23 per ounce, what is the expected price of silver in the United States if the spot exchange rate is $1 = £0.6535?

See Answer

Q: If the price of copper in Europe is €2.12

If the price of copper in Europe is €2.12 per ounce, what is the expected price of copper in the United States if the spot exchange rate is $1 = €0.7623?

See Answer

Q: A financial manager has determined that the appropriate discount rate for a

A financial manager has determined that the appropriate discount rate for a foreign project is 12 percent. However, that discount rate applies in the United States using dollars. What discount rate sh...

See Answer

Q: A financial manager has determined that the appropriate discount rate for a

A financial manager has determined that the appropriate discount rate for a foreign project is 16 percent. However, that discount rate applies in the United States using dollars. What discount rate sh...

See Answer

Q: A U.S. firm is expecting to pay cash flows

A U.S. firm is expecting to pay cash flows of 15 million Egyptian pounds and 25 million Qatar rials. The current spot exchange rates are: $1 = 5.725 pounds and $1 = 3.639 rials. If these cash fl...

See Answer

Q: WorldGone, Inc. declared bankruptcy on September 25, 2018 through

WorldGone, Inc. declared bankruptcy on September 25, 2018 through a Chapter 7 filing. WorldGone’s balance sheet at the time of the bankruptcy filing is listed below. The accrued wage...

See Answer

Q: The U.S. dollar spot exchange rate with the Canadian

The U.S. dollar spot exchange rate with the Canadian dollar is $1 = CA$1.18. The U.S. dollar and Swiss franc exchange rate is $1 = 1.219 francs. If the cross rate between the franc and Canadian dollar...

See Answer

Q: The U.S. dollar spot exchange rate with the Australian

The U.S. dollar spot exchange rate with the Australian dollar is $1 = AU$1.2697. The U.S. dollar and euro exchange rate is $1 = €0.7559. If the cross rate between the euro and Australian dollar is €1...

See Answer

Q: A U.S. firm is expecting cash flows of 25

A U.S. firm is expecting cash flows of 25 million Mexican pesos and 35 million Indian rupees. The current spot exchange rates are: $1 = 12.268 pesos and $1 = 45.204 rupees. If these cash flows are not...

See Answer

Q: You have approached your local bank for a start-up loan

You have approached your local bank for a start-up loan commitment for $250,000 needed to open a computer repair store. You have requested that the term of the loan be one year. Your bank has offered...

See Answer

Q: A firm has an Altman’s Z-score of 1.76

A firm has an Altman’s Z-score of 1.76. What does this mean?

See Answer

Q: Calculate the total fees a firm would have to pay when its

Calculate the total fees a firm would have to pay when its bank offers the firm the following loan commitment: A loan commitment of $4.25 million with an up-front fee of 75 basis points and a back-end...

See Answer

Q: Husker’s Tuxedo’s, Inc. needs to raise $250 million to

Husker’s Tuxedo’s, Inc. needs to raise $250 million to finance its plan for nationwide expansion. In discussions with its investment bank, Husker’s learns that the bankers recommend an offer price (or...

See Answer

Q: Don’s Captain Morgan, Inc. needs to raise $12.

Don’s Captain Morgan, Inc. needs to raise $12.5 million to finance plant expansion. In discussions with its investment bank, Don’s learns that the bankers recommend an offer price (or gross proceeds)...

See Answer

Q: The Fitness Studio, Inc., with the help of its investment

The Fitness Studio, Inc., with the help of its investment bank, recently issued $43.125 million of new debt. The offer price (and face value) on the debt was $1,000 per bond and the underwriter’s spre...

See Answer

Q: Harper’s Dog Pens, Inc., with the help of its investment

Harper’s Dog Pens, Inc., with the help of its investment bank recently issued $191.5 million of new debt. The offer price on the debt was $1,000 per bond and the underwriter’s spread was 5 percent of...

See Answer

Q: Describe the difference between a horizontal merger and a vertical merger.

Describe the difference between a horizontal merger and a vertical merger.

See Answer

Q: You have approached your local bank for a start-up loan

You have approached your local bank for a start-up loan commitment for $250,000 needed to open a computer repair store. You have requested that the term of the loan be one year. Your bank has offered...

See Answer

Q: Casey’s One Stop has been approved for a $127,500

Casey’s One Stop has been approved for a $127,500 loan commitment from its local bank. The bank has offered the following terms: term = one year, up-front fee = 85 basis points, back-end fee = 35 basi...

See Answer

Q: DiPitro’s Paint and Wallpaper, Inc. needs to raise $1

DiPitro’s Paint and Wallpaper, Inc. needs to raise $1 million to finance plant expansion. In discussions with its investment bank, DiPitro’s learns that the bankers recommend a debt issue with gross p...

See Answer

Q: Renee’s Boutique, Inc. needs to raise $58 million to

Renee’s Boutique, Inc. needs to raise $58 million to finance firm expansion. In discussions with its investment bank, Renee’s learns that the bankers recommend a debt issue with an offer price of $1,0...

See Answer

Q: The Fitness Studio, Inc., with the help of its investment

The Fitness Studio, Inc., with the help of its investment bank, recently issued 2.5 million shares of new stock. The offer price on the stock was $20.50 per share and The Fitness Studio received a tot...

See Answer

Q: A linear probability model you have developed finds that a firm has

A linear probability model you have developed finds that a firm has a PD of .16. What does this mean?

See Answer

Q: Harper’s Dog Pens, Inc., with the help of its investment

Harper’s Dog Pens, Inc., with the help of its investment bank, recently issued 8.5 million shares of new stock. The offer price on the stock was $12.00 per share and Harper’s received a total of $97.7...

See Answer

Q: Zimba Technology Corp. recently went public with an initial public offering

Zimba Technology Corp. recently went public with an initial public offering of 2.5 million shares of stock. The underwriter used a firm commitment offering in which the net proceeds was $8.05 per shar...

See Answer

Q: Howett Pockett, Inc. plans to issue 10 million new shares

Howett Pockett, Inc. plans to issue 10 million new shares of its stock. In discussions with its investment bank, Howett Pocket learns that the bankers recommend a net proceed of $33.80 per share and t...

See Answer

Q: During the last year, you have had a loan commitment from

During the last year, you have had a loan commitment from your bank to fund inventory purchases for your small business. The total line available was $500,000, of which you took down $400,000. It is n...

See Answer

Q: Use the following financial statements for Lake of Egypt Marina to calculate

Use the following financial statements for Lake of Egypt Marina to calculate and interpret the Altman’s Z-score for this firm as of 2018. / /

See Answer

Q: During the last year, you have had a loan commitment from

During the last year, you have had a loan commitment from your bank to fund working capital for your business. The total line available was $17 million, of which you took down $13 million. It is now t...

See Answer

Q: DiPitro’s Paint and Wallpaper, Inc. needs to raise $1

DiPitro’s Paint and Wallpaper, Inc. needs to raise $1 million to finance plant expansion. In discussions with its investment bank, DiPitro’s learns that the bankers recommend a gross price of $25 per...

See Answer

Q: Renee’s Boutique, Inc. needs to raise $58 million to

Renee’s Boutique, Inc. needs to raise $58 million to finance firm expansion. In discussions with its investment bank, Renee’s learns that the bankers recommend an offer price of $33.75 per share and t...

See Answer

Q: Hughes Technology Corp. recently went public with an initial public offering

Hughes Technology Corp. recently went public with an initial public offering in which they received a total of $60 million in new capital funding. The underwriter used a firm commitment offering in wh...

See Answer

Q: Howett Pockett, Inc. needs to raise $12 million in

Howett Pockett, Inc. needs to raise $12 million in new capital funding from a seasoned equity offering. In discussions with its investment bank, Howett Pocket learns that the bankers recommend a gross...

See Answer

Q: Nuran Security Systems, Inc. needs to raise $150 million

Nuran Security Systems, Inc. needs to raise $150 million for asset expansion. As it raises the capital funding, Nuran wants to maintain its current debt ratio of 60 percent. Nuran has been approved fo...

See Answer

Q: What do global organizations like the World Trade Organization and the International

What do global organizations like the World Trade Organization and the International Monetary Fund do?

See Answer

Q: What process do banks use to evaluate bank loans to small versus

What process do banks use to evaluate bank loans to small versus midmarket business firms?

See Answer

Q: What is the difference between a spot loan and a loan commitment

What is the difference between a spot loan and a loan commitment?

See Answer

Q: Why do banks charge up-front fees and back-end

Why do banks charge up-front fees and back-end fees on loan commitments?

See Answer

Q: What is meant when it is said that the U.S

What is meant when it is said that the U.S. dollar is strengthening? How would it impact your vacation abroad and foreign visitors to the United States?

See Answer

Q: What is the difference between a fixed-rate and a floating

What is the difference between a fixed-rate and a floating-rate loan?

See Answer

Q: What types of programs does the Small Business Administration offer to new

What types of programs does the Small Business Administration offer to new and small businesses? Under what conditions would a new or small firm use each program?

See Answer

Q: What is venture capital?

What is venture capital?

See Answer

Q: What are the different types of venture capital firms? How do

What are the different types of venture capital firms? How do institutional venture capital firms differ from angel venture capital firms?

See Answer

Q: What are the advantages and disadvantages to a new or small firm

What are the advantages and disadvantages to a new or small firm of getting capital funding from a venture capital firm?

See Answer

Q: As a new or small firm considers going public what must the

As a new or small firm considers going public what must the owners consider?

See Answer

Q: Describe the various sources of capital funding available to public firms.

Describe the various sources of capital funding available to public firms.

See Answer

Q: What is the purpose of trading zones? What are some of

What is the purpose of trading zones? What are some of the most important zones for world trade?

See Answer

Q: What is the difference between a direct and an indirect placement of

What is the difference between a direct and an indirect placement of commercial paper?

See Answer

Q: Can a public firm with a lower-than-prime credit

Can a public firm with a lower-than-prime credit rating issue commercial paper?

See Answer

Q: Consider the same 3-year oil swap. Suppose a dealer

Consider the same 3-year oil swap. Suppose a dealer is paying the fixed price and receiving floating. What position in oil forward contracts will hedge oil price risk in this position? Verify that the...

See Answer

Q: Use a change of numeraire and measure to verify that the value

Use a change of numeraire and measure to verify that the value of a claim paying ST if ST

See Answer

Q: Let S = $100, σ = 30%, r =

Let S = $100, σ = 30%, r = 0.08, t = 1, and δ = 0. Suppose the true expected return on the stock is 15%. Set n = 10. Compute European call prices, ∆, and B for strikes of $70, $80, $90, $100, $110, $1...

See Answer

Q: Make the same assumptions as in the previous problem. a

Make the same assumptions as in the previous problem. a. What is the 9-month forward price for the stock? b. Compute the price of a 95-strike 9-month call option on a futures contract. c. What is the...

See Answer

Q: Repeat the previous problem for a 40-strike 180-day

Repeat the previous problem for a 40-strike 180-day put. Repeat the previous problem Consider a 40-strike 180-day call with S = $40. Compute a delta-gamma-theta approximation for the value of the call...

See Answer

Q: Repeat the previous problem for up-and-out puts assuming

Repeat the previous problem for up-and-out puts assuming a barrier of $44. Previous Problem Let S = $40, K = $45, σ = 0.30, r = 0.08, δ = 0, and T = {0.25, 0.5, 1, 2, 3, 4, 5, 100}. a. Compute the pr...

See Answer

Q: Consider the equity-linked CD in Section 15.3.

Consider the equity-linked CD in Section 15.3. Assuming that profit for the issuing bank is zero, draw a graph showing how the participation rate, γ , varies with the coupon, c. Repeat assuming the is...

See Answer

Q: Now suppose the firm finances the project by issuing debt that has

Now suppose the firm finances the project by issuing debt that has lower priority than existing debt. How much must a $1, $10, or $25 project be worth if the shareholders are willing to fund it?

See Answer

Q: Consider the widget investment problem of Section 17.1 with the

Consider the widget investment problem of Section 17.1 with the following modification. The expected growth rate of the widget price is zero. (This means there is no reason to consider project delay.)...

See Answer

Q: What is Pr(St > $105) for t =

What is Pr(St > $105) for t = 1? How does this probability change when you change t? How does it change when you change σ?

See Answer

Q: Assume that the market index is 100. Show that if the

Assume that the market index is 100. Show that if the expected return on the market is 15%, the dividend yield is zero, and volatility is 20%, then the probability of the index falling below 95 over a...

See Answer

Q: Suppose that ln(S) and ln(Q) have

Suppose that ln(S) and ln(Q) have correlation ρ =−0.3 and that S(0) = $100, Q(0) = $100, r = 0.06, σS = 0.4, and σQ = 0.2. Neither stock pays dividends. Use equation (20.38) to find the price today of...

See Answer

Q: Verify that equation (23.7) satisfies the appropriate boundary

Verify that equation (23.7) satisfies the appropriate boundary conditions for Pr(ST≤ H and ST >K).

See Answer

Q: Consider Joe and Sarah’s bet in Examples 21.2 and 21

Consider Joe and Sarah’s bet in Examples 21.2 and 21.3. a. In this bet, note that $106.184 is the forward price. A bet paying $1 if the share price is above the forward price is worth less than a bet...

See Answer

Q: Suppose that the stock price follows a jump-diffusion process as

Suppose that the stock price follows a jump-diffusion process as outlined in Section 20.7. Let the jump intensity be λ = 0.75, the expected jump exp(αJ), with αJ =−0.15, and let the jump volatility be...

See Answer

Q: The quanto forward price can be computed using the risk-neutral

The quanto forward price can be computed using the risk-neutral distribution as E(Yx−1). Use Proposition 20.4 to derive the quanto forward price given by equation (23.30).

See Answer

Q: Use the following inputs to compute the price of a European call

Use the following inputs to compute the price of a European call option: S = $50, K = $100, r = 0.06, σ = 0.30, T = 0.01, δ = 0. a. Verify that the Black-Scholes price is zero. b. Verify that the vega...

See Answer

Q: Verify that the 4-year zero-coupon bond price generated

Verify that the 4-year zero-coupon bond price generated by the tree in Figure 25.5 is $0.6243. Figure 25.5

See Answer

Q: Compute the 95% 10-day VaR for a written strangle

Compute the 95% 10-day VaR for a written strangle (sell an out-of-the-money call and an out-of-the-money put) on 100,000 shares of stock A. Assume the options have strikes of $90 and $110 and have 1 y...

See Answer

Q: Repeat the previous problem, except that the time to maturity can

Repeat the previous problem, except that the time to maturity can be 1, 2, 3, 4, 5, 10, or 20 years. How does the bond yield change with time to maturity?

See Answer

Q: When you open a brokerage account, you typically sign an agreement

When you open a brokerage account, you typically sign an agreement giving the broker the right to lend your shares without notifying or compensating you. Why do brokers want you to sign this agreement...

See Answer

Q: An off-market forward contract is a forward where either you

An off-market forward contract is a forward where either you have to pay a premium or you receive a premium for entering into the contract. (With a standard forward contract, the premium is zero.) Sup...

See Answer

Q: Compute estimated profit in 1 year if Telco sells a put option

Compute estimated profit in 1 year if Telco sells a put option with a strike of $0.95, $1.00, or $1.05. Draw a graph of profit in each case.

See Answer

Q: For the period 1999–2004, using daily data, compute

For the period 1999–2004, using daily data, compute the following: a. An EWMA estimate, with b = 0.95, of IBM’s volatility using all data. b. An EWMA estimate, with b = 0.95, of IBM’s volatility, at...

See Answer

Q: Suppose that 10 years from now it becomes possible for money managers

Suppose that 10 years from now it becomes possible for money managers to engage in time travel. In particular, suppose that a money manager could travel to January 1981, when the 1-year Treasury bill...

See Answer

Q: Using the same information as the previous problem, suppose the interest

Using the same information as the previous problem, suppose the interest rate on the borrowing date is 7.5%. Determine the dollar settlement of the FRA assuming a. Settlement occurs on the date the lo...

See Answer

Q: Given an 8-quarter oil swap price of $20.

Given an 8-quarter oil swap price of $20.43, construct the implicit loan balance for each quarter over the life of the swap.

See Answer

Q: Suppose call and put prices are given by /

Suppose call and put prices are given by What no-arbitrage property is violated? What spread position would you use to effect arbitrage? Demonstrate that the spread position is an arbitrage.

See Answer

Q: Repeat the option price calculation in the previous question for stock prices

Repeat the option price calculation in the previous question for stock prices of $80, $90, $110, $120, and $130, keeping everything else fixed. What happens to the initial put ∆ as the stock price inc...

See Answer

Q: Repeat the previous problem, except that for each strike price,

Repeat the previous problem, except that for each strike price, compute the expected return on the option for times to expiration of 3 months, 6 months, 1 year, and 2 years. What effect does time to m...

See Answer

Q: Assume K = $40, σ = 30%, r =

Assume K = $40, σ = 30%, r = 0.08, T = 0.5, and the stock is to pay a single dividend of $2 tomorrow, with no dividends thereafter. a. Suppose S = $50. What is the price of a European call option? Con...

See Answer

Q: Let S = $40, K = $45, σ

Let S = $40, K = $45, σ = 0.30, r = 0.08, and δ = 0. Compute the value of knockout calls with a barrier of $60 and times to expiration of 1 month, 2 months, and so on, up to 1 year. As you increase ti...

See Answer

Q: Now suppose the firm finances the project by issuing debt that has

Now suppose the firm finances the project by issuing debt that has higher priority than existing debt. How much must a $10 or $25 project be worth if the shareholders are willing to fund it?

See Answer

Q: To answer this question, use the assumptions of Example 17.

To answer this question, use the assumptions of Example 17.1 and the risk-neutral valuation method (and risk-neutral probability) described in Example 17.2. a. Compute the value of a claim that pays t...

See Answer

Q: Assuming a $10m investment that is 40% stock A and

Assuming a $10m investment that is 40% stock A and 60% stock B, compute the 95% and 99% VaR for the position over 1-day, 10-day, and 20-day horizons.

See Answer

Q: What is E(St|St > $105) for

What is E(St|St > $105) for t = 1? How does this expectation change when you change t, σ, and r?

See Answer

Q: Suppose that on any given day the annualized continuously compounded stock return

Suppose that on any given day the annualized continuously compounded stock return has a volatility of either 15%, with a probability of 80%, or 30%, with a probability of 20%. This is a mixture of nor...

See Answer

Q: Consider again the bet in Example 21.3. Suppose the

Consider again the bet in Example 21.3. Suppose the bet is S − $106.184 if the price is above $106.184, and $106.184 − S if the price is below $106.184. What is the...

See Answer

Q: Suppose that S1 and S2 are correlated, non-dividend-

Suppose that S1 and S2 are correlated, non-dividend-paying assets that follow geometric Brownian motion. Specifically, let S1 (0) = S2(0) = $100, r = 0.06, σ1 = 0.35, σ2 = 0.25, ρ = 0.40 and T = 1. Ve...

See Answer

Q: In this problem we use the lognormal approximation (see equation (

In this problem we use the lognormal approximation (see equation (11.14)) to draw one-step binomial trees from the perspective of a yen-based investor. Use the information in Table 23.4. a. Construct...

See Answer

Q: Compute January 12 2004 bid and ask volatilities (using the Black

Compute January 12 2004 bid and ask volatilities (using the Black-Scholes implied volatility function) for IBM options expiring January 17. For which options are you unable to compute a plausible impl...

See Answer

Q: Using Monte Carlo, compute the 95% and 99% 1

Using Monte Carlo, compute the 95% and 99% 1-, 10-, and 20-day tail VaRs for the position in Problem 26.2. Problem 26.2. Assuming a $10m investment in one stock, compute the 95% and 99% VaR for stock...

See Answer

Q: Consider a firm with an F rating. a. What

Consider a firm with an F rating. a. What is the probability that after 4 years it will still have an F rating? b. What is the probability that after 4 years it will have an FF or FFF rating? c. From...

See Answer

Q: Suppose a stock pays a quarterly dividend of $3. You

Suppose a stock pays a quarterly dividend of $3. You plan to hold a short position in the stock across the dividend ex-date. What is your obligation on that date? If you are a taxable investor, what w...

See Answer

Q: For Figure 2.6, verify the following:

For Figure 2.6, verify the following: a. The S&R index price at which the call option diagram intersects the x-axis is $1095.68. b. The S&R index price at which the call option and forward c...

See Answer

Q: Suppose the firm issues a single zero-coupon bond with time

Suppose the firm issues a single zero-coupon bond with time to maturity 3 years and maturity value $110. a. Compute the price, yield to maturity, default probability, and expected recovery (E[BT |Defa...

See Answer

Q: Compute estimated profit in 1 year if Telco sells collars with the

Compute estimated profit in 1 year if Telco sells collars with the following strikes: a. $0.95 for the put and $1.00 for the call. b. $0.975 for the put and $1.025 for the call. c. $0.95 for the put a...

See Answer

Q: The S&R index spot price is 1100 and the continuously

The S&R index spot price is 1100 and the continuously compounded risk-free rate is 5%. You observe a 9-month forward price of 1129.257. a. What dividend yield is implied by this forward price? b. Supp...

See Answer

Q: What is the rate on a synthetic FRA for a 90-

What is the rate on a synthetic FRA for a 90-day loan commencing on day 90? A 180-day loan commencing on day 90? A 270-day loan commencing on day 90?

See Answer

Q: Suppose call and put prices are given by / Find

Suppose call and put prices are given by Find the convexity violations. What spread would you use to effect arbitrage? Demonstrate that the spread position is an arbitrage.

See Answer

Q: Let S = $100, K = $95, σ

Let S = $100, K = $95, σ = 30%, r = 8%, T = 1, and δ = 0. Let u = 1.3, d = 0.8, and n = 2. Construct the binomial tree for an American put option. At each node provide the premium, ∆, and B.

See Answer

Q: Let S = $100, σ = 30%, r =

Let S = $100, σ = 30%, r = 0.08, t = 1, and δ = 0. Suppose the true expected return on the stock is 15%. Set n = 10. Compute European put prices, ∆, and B for strikes of $70, $80, $90, $100, $110, $12...

See Answer

Q: “Time decay is greatest for an option close to expiration.”

“Time decay is greatest for an option close to expiration.” Use the spreadsheet functions to evaluate this statement. Consider both the dollar change in the option value and the percentage change in t...

See Answer

Q: Examine the prices of up-and-out puts with strikes

Examine the prices of up-and-out puts with strikes of $0.9 and $1.0 in Table 14.3. With barriers of $1 and $1.05, the 0.90-strike up-and-outs appear to have the same premium as the ordinary put. Howev...

See Answer

Q: Consider a project that in one year pays $50 if the

Consider a project that in one year pays $50 if the economy performs well (the stock market goes up) and that pays $100 if the economy performs badly (the stock market goes down). The probability of t...

See Answer

Q: The formula for an infinitely lived call is given in equation (

The formula for an infinitely lived call is given in equation (12.18). Suppose that S follows equation (20.20), with α replaced by r, and that E∗(dV ) = rV dt. Use It&E...

See Answer

Q: ABC stock has a bid price of $40.95 and

ABC stock has a bid price of $40.95 and an ask price of $41.05. Assume there is a $20 brokerage commission. a. What amount will you pay to buy 100 shares? b. What amount will you receive for selling 1...

See Answer

Q: Suppose that a derivative claim makes continuous payments at the rate.

Suppose that a derivative claim makes continuous payments at the rate. Show that the Black-Scholes equation becomes For the following four problems, assume that S follows equation (21.5) and Q follow...

See Answer

Q: Repeat the previous problem assuming that δ1= 0.05 and

Repeat the previous problem assuming that δ1= 0.05 and δ2 = 0.12. Verify that both procedures give a price of approximately $15.850. Previous Problem Suppose that S1 and S2 are correlated, non-divide...

See Answer

Q: Suppose an option knocks in at H1> S, and knocks

Suppose an option knocks in at H1> S, and knocks out at H2 >H1. Suppose that K H1, it is not possible to hit H2 without hitting H1): What is the value of this option?

See Answer

Q: Verify that the price of the 12% interest rate cap in

Verify that the price of the 12% interest rate cap in Figure 25.6 is $3.909.

See Answer

Q: Compute the 95% 10-day tail VaR for the position

Compute the 95% 10-day tail VaR for the position in Problem 26.8. Problem 26.8. Compute the 95% 10-day VaR for a written strangle (sell an out-of-the-money call and an out-of-the-money put) on 100,00...

See Answer

Q: Consider two firms, one with an FF rating and one with

Consider two firms, one with an FF rating and one with an FFF rating. What is the probability that after 4 years each will have retained its rating? What is the probability that each will have moved t...

See Answer

Q: Short interest is a measure of the aggregate short positions on a

Short interest is a measure of the aggregate short positions on a stock. Check an online brokerage or other financial service for the short interest on several stocks of your choice. Can you guess whi...

See Answer

Q: For Figure 2.8, verify the following:

For Figure 2.8, verify the following: a. The S&R index price at which the put option diagram intersects the x-axis is $924.32. b. The S&R index price at which the put option and forward cont...

See Answer

Q: Compute estimated profit in 1 year if Telco buys paylater calls as

Compute estimated profit in 1 year if Telco buys paylater calls as follows (the net premium may not be exactly zero): a. Sell one 0.975-strike call and buy two 1.034-strike calls. b. Sell two 1.00-str...

See Answer

Q: Suppose the S&P 500 index futures price is currently 1200

Suppose the S&P 500 index futures price is currently 1200. You wish to purchase four futures contracts on margin. a. What is the notional value of your position? b. Assuming a 10% initial margin, what...

See Answer

Q: a. Suppose you enter into a long 6-month forward

a. Suppose you enter into a long 6-month forward position at a forward price of $50. What is the payoff in 6 months for prices of $40, $45, $50, $55, and $60? b. Suppose you buy a 6-month call option...

See Answer

Q: What is the rate on a synthetic FRA for a 180-

What is the rate on a synthetic FRA for a 180-day loan commencing on day 180? Suppose you are the counterparty for a borrower who uses the FRA to hedge the interest rate on a $10m loan. What positions...

See Answer

Q: Using the zero-coupon bond prices and natural gas swap prices

Using the zero-coupon bond prices and natural gas swap prices in Table 8.9, what are gas forward prices for each of the 8 quarters?

See Answer

Q: Suppose call and put prices are given by / Find

Suppose call and put prices are given by Find the convexity violations. What spread would you use to effect arbitrage? Demonstrate that the spread position is an arbitrage.

See Answer

Q: Suppose S0 = $100, K = $50, r

Suppose S0 = $100, K = $50, r = 7.696% (continuously compounded), δ = 0, and T = 1. a. Suppose that for h = 1, we have u = 1.2 and d = 1.05. What is the binomial option price for a call option that li...

See Answer

Q: Repeat the previous problem, except that for each strike price,

Repeat the previous problem, except that for each strike price, compute the expected return on the option for times to expiration of 3 months, 6 months, 1 year, and 2 years. What effect does time to m...

See Answer

Q: In the absence of an explicit formula, we can estimate the

In the absence of an explicit formula, we can estimate the change in the option price due to a change in an input—such as σ—by computing the following fo...

See Answer

Q: Suppose S = $40, K = $40, σ

Suppose S = $40, K = $40, σ = 0.30, r = 0.08, and δ = 0. a. What is the price of a standard European call with 2 years to expiration? b. Suppose you have a compound call giving you the right to pay $2...

See Answer

Q: A firm has outstanding a bond with a 5-year maturity

A firm has outstanding a bond with a 5-year maturity and maturity value of $50, convertible into 10 shares. There are also 20 shares outstanding. What is the price of the warrant? The share price? Sup...

See Answer

Q: Verify the binomial calculations in Figure 17.3.

Verify the binomial calculations in Figure 17.3.

See Answer

Q: Let t = 1. What is E (St |St

Let t = 1. What is E (St |St < $98)? What is E (St |St < $120)? How do both expectations change when you vary t from 0.05 to 5? Let σ = 0.1. Does either answer change? How?

See Answer

Q: Compute estimated profit in 1 year if XYZ sells a call option

Compute estimated profit in 1 year if XYZ sells a call option with a strike of $0.95, $1.00, or $1.05. Draw a graph of profit in each case.

See Answer

Q: Assume S0 = $100, r = 0.05,

Assume S0 = $100, r = 0.05, σ = 0.25, δ = 0, and T = 1. Use Monte Carlo valuation to compute the price of a claim that pays $1 if ST > $100, and 0 otherwise. (This is called a cash-or-nothing call an...

See Answer

Q: Suppose that the processes for S1 and S2 are given by these

Suppose that the processes for S1 and S2 are given by these two equations:  Note that the diffusions dZ1 and dZ2 are different. In this problem we want to find the expected return on Q, αQ, where Q f...

See Answer

Q: What is the value of a claim paying ? Check your answer

What is the value of a claim paying ? Check your answer using Proposition 20.4.  (20.4)

See Answer

Q: Suppose there are 1-, 2-, and 3-year zero

Suppose there are 1-, 2-, and 3-year zero-coupon bonds, with prices given by P1, P2, and P3. The implied forward interest rate from year 1 to 2 is r0(1, 2) = P1/P2 − 1, and from year 2 to 3 is r0(2, 3...

See Answer

Q: Suppose the stock price is $50, but that we plan

Suppose the stock price is $50, but that we plan to buy 100 shares if and when the stock reaches $45. Suppose further that σ = 0.3, r = 0.08, T − t = 1, and δ = 0. This is a noncancellable limit order...

See Answer

Q: Verify that the 1-year forward rate 3 years hence in

Verify that the 1-year forward rate 3 years hence in Figure 25.5 is 14.0134%. For the next four problems, here are two BDT interest rate trees with effective annual interest rates at each node.

See Answer

Q: Suppose you write a 1-year cash-or-nothing

Suppose you write a 1-year cash-or-nothing put with a strike of $50 and a 1-year cash-or-nothing call with a strike of $215, both on stock A. a. What is the 1-year 99% VaR for each option separately?...

See Answer

Q: Suppose that in Figure 27.6 the tranches have promised payments

Suppose that in Figure 27.6 the tranches have promised payments of $160 (senior), $50 (mezzanine), and $90 (subordinated). Reproduce the table for this case, assuming zero default correlation.

See Answer

Q: Suppose that you go to a bank and borrow $100.

Suppose that you go to a bank and borrow $100. You promise to repay the loan in 90 days for $102. Explain this transaction using the terminology of short-sales.

See Answer

Q: For each entry in Table 2.5, explain the circumstances

For each entry in Table 2.5, explain the circumstances in which the maximum gain or loss occurs.

See Answer

Q: Consider the 3-year swap in the previous example. Suppose

Consider the 3-year swap in the previous example. Suppose you are the fixed-rate payer in the swap. How much have you overpaid relative to the forward price after the first swap settlement? What is th...

See Answer

Q: Suppose that Wirco does nothing to manage the risk of copper price

Suppose that Wirco does nothing to manage the risk of copper price changes. What is its profit 1 year from now, per pound of copper? Suppose that Wirco buys copper forward at $1. What is its profit 1...

See Answer

Q: Suppose the S&P 500 index is currently 950 and the

Suppose the S&P 500 index is currently 950 and the initial margin is 10%. You wish to enter into 10 S&P 500 futures contracts. a. What is the notional value of your position? What is the margin? b. Su...

See Answer

Q: Suppose you are the counterparty for a lender who enters into an

Suppose you are the counterparty for a lender who enters into an FRA to hedge the lending rate on $10m for a 90-day loan commencing on day 270. What positions in zero-coupon bonds would you use to hed...

See Answer

Q: Using the zero-coupon bond prices and natural gas swap prices

Using the zero-coupon bond prices and natural gas swap prices in Table 8.9, what is the implicit loan amount in each quarter in an 8-quarter natural gas swap?

See Answer

Q: In each case identify the arbitrage and demonstrate how you would make

In each case identify the arbitrage and demonstrate how you would make money by creating a table showing your payoff. a. Consider two European options on the same stock with the same time to expiratio...

See Answer

Q: Let S = $100, K = $95, r

Let S = $100, K = $95, r = 8% (continuously compounded), σ = 30%, δ = 0, T = 1 year, and n = 3. a. Verify that the binomial option price for an American call option is $18.283. Verify that there is ne...

See Answer

Q: Let S = $100, σ = 0.30,

Let S = $100, σ = 0.30, r = 0.08, t = 1, and δ = 0. Using equation (11.12) to compute the probability of reaching a terminal node and Suidn−i to compute the price at that node, plot the risk-neutral d...

See Answer

Q: Make the same assumptions as in the previous problem. a

Make the same assumptions as in the previous problem. a. What is the price of a standard European put with 2 years to expiration? b. Suppose you have a compound call giving you the right to pay $2 1 y...

See Answer

Q: Suppose a firm has 20 shares of equity, a 10-

Suppose a firm has 20 shares of equity, a 10-year zero-coupon debt with a maturity value of $200, and warrants for 8 shares with a strike price of $25. What is the value of the debt, the share price,...

See Answer

Q: A project costing $100 will produce perpetual net cash flows that

A project costing $100 will produce perpetual net cash flows that have an annual volatility of 35% with no expected growth. If the project existed, net cash flows today would be $8. The project beta i...

See Answer

Q: Obtain at least 5 years’ worth of daily or weekly stock price

Obtain at least 5 years’ worth of daily or weekly stock price data for a stock of your choice. 1. Compute annual volatility using all the data. 2. Compute annual volatility for each calendar year in y...

See Answer

Q: Let KT= S0erT. Compute Pr(ST

Let KT= S0erT. Compute Pr(ST

See Answer

Q: Let h = 1/52. Simulate both the continuously compounded

Let h = 1/52. Simulate both the continuously compounded actual return and the actual stock price, St+h. What are the mean, standard deviation, skewness, and kurtosis of both the continuously compounde...

See Answer

Q: Suppose that S1 follows equation (20.26) with δ

Suppose that S1 follows equation (20.26) with δ = 0. Consider an asset that follows the process dS2 = α2S2 dt − σ2S2 dZ Show that (Î&plusm...

See Answer

Q: What is the value of a claim paying Q(T )−

What is the value of a claim paying Q(T )−1S(T )? Check your answer using Proposition 20.4.  (20.4)

See Answer

Q: Covered call writers often plan to buy back the written call if

Covered call writers often plan to buy back the written call if the stock price drops sufficiently. The logic is that the written call at that point has little “upside,â€&...

See Answer

Q: In this problem you will compute January 12 2004 bid and ask

In this problem you will compute January 12 2004 bid and ask volatilities (using the Black-Scholes implied volatility function) for 1-year IBM options expiring the following January. Note that IBM pay...

See Answer

Q: What are the 1-, 2-, 3-, 4-, and

What are the 1-, 2-, 3-, 4-, and 5-year zero-coupon bond prices implied by the two trees?

See Answer

Q: Suppose the 7-year zero-coupon bond has a yield

Suppose the 7-year zero-coupon bond has a yield of 6% and yield volatility of 10% and the 10-year zero-coupon bond has a yield of 6.5% and yield volatility of 9.5%. The correlation between the 7-year...

See Answer

Q: Repeat the previous problem, only assuming that defaults are perfectly correlated

Repeat the previous problem, only assuming that defaults are perfectly correlated. Repeat the previous problem, Suppose that in Figure 27.6 the tranches have promised payments of $160 (senior), $50 (...

See Answer

Q: Suppose your bank’s loan officer tells you that if you take out

Suppose your bank’s loan officer tells you that if you take out a mortgage (i.e., you borrow money to buy a house), you will be permitted to borrow no more than 80% of the value of the house. Describe...

See Answer

Q: Suppose the S&R index is 800, the continuously compounded

Suppose the S&R index is 800, the continuously compounded risk-free rate is 5%, and the dividend yield is 0%. A 1-year 815-strike European call costs $75 and a 1- year 815-strike European put costs $4...

See Answer

Q: Consider a one-period binomial model with h = 1,

Consider a one-period binomial model with h = 1, where S = $100, r = 0.08, σ = 30%, and δ = 0. Compute American put option prices for K = $100, $110, $120, and $130. a. At which strike(s) does early e...

See Answer

Q: Suppose the stock price is $40 and the effective annual interest

Suppose the stock price is $40 and the effective annual interest rate is 8%. a. Draw on a single graph payoff and profit diagrams for the following options: (i) 35-strike call with a premium of $9.12....

See Answer

Q: What happens to the variability of Wirco’s profit if Wirco undertakes any

What happens to the variability of Wirco’s profit if Wirco undertakes any strategy (buying calls, selling puts, collars, etc.) to lock in the price of copper next year? You can use your answer to the...

See Answer

Q: Verify that going long a forward contract and lending the present value

Verify that going long a forward contract and lending the present value of the forward price creates a payoff of one share of stock when a. The stock pays no dividends. b. The stock pays discrete divi...

See Answer

Q: Using the information in Table 7.1, suppose you buy

Using the information in Table 7.1, suppose you buy a 3-year par coupon bond and hold it for 2 years, after which time you sell it. Assume that interest rates are certain not to change and that you re...

See Answer

Q: What is the fixed rate in a 5-quarter interest rate

What is the fixed rate in a 5-quarter interest rate swap with the first settlement in quarter 2?

See Answer

Q: Suppose the interest rate is 0% and the stock of XYZ

Suppose the interest rate is 0% and the stock of XYZ has a positive dividend yield. Is there any circumstance in which you would early-exercise an American XYZ call? Is there any circumstance in which...

See Answer

Q: Repeat the previous problem assuming that the stock pays a continuous dividend

Repeat the previous problem assuming that the stock pays a continuous dividend of 8% per year (continuously compounded). Calculate the prices of the American and European puts and calls. Which options...

See Answer

Q: Repeat the previous problem for n = 50. What is the

Repeat the previous problem for n = 50. What is the risk-neutral probability that S1< $80? S1> $120? Previous Problem Let S = $100, σ = 0.30, r = 0.08, t = 1, and δ = 0. Using equation (11.12) to comp...

See Answer

Q: Consider the hedging example using gap options, in particular the assumptions

Consider the hedging example using gap options, in particular the assumptions and prices in Table 14.4. a. Implement the gap pricing formula. Reproduce the numbers in Table 14.4. b. Consider the optio...

See Answer

Q: A project has certain cash flows today of $1, growing

A project has certain cash flows today of $1, growing at 5% per year for 10 years, after which the cash flow is constant. The risk-free rate is 5%. The project costs $20 and cash flows begin 1 year af...

See Answer

Q: Let S = $120, K = $100, σ

Let S = $120, K = $100, σ = 30%, r = 0, and δ = 0.08. a. Compute the Black-Scholes call price for 1 year to maturity and for a variety of very long times to maturity. What happens to the price as T →∞...

See Answer

Q: Consider Pr(St

Consider Pr(St

See Answer

Q: Suppose that S and Q follow equations (20.36)

Suppose that S and Q follow equations (20.36) and (20.37). Derive the value of a claim paying S(T )aQ(T )b by each of the following methods: a. Compute the expected value of the claim and discounting...

See Answer

Q: You are offered the opportunity to receive for free the payoff

You are offered the opportunity to receive for free the payoff [Q(T ) − F0,T (Q)]× max [0, S(T ) − K] (Note that this payoff can be negative.) Should you accept the offer?

See Answer

Q: The box on page 282 discusses the following result: If the

The box on page 282 discusses the following result: If the strike price of a European put is set to equal the forward price for the stock, the put premium increases with maturity. a. How is this resul...

See Answer

Q: For the lookback call: a. What is the value

For the lookback call: a. What is the value of a lookback call as St approaches zero? Verify that the formula gives you the same answer. b. Verify that at maturity the value of the call is ST − ST .

See Answer

Q: For this problem, use the implied volatilities for the options expiring

For this problem, use the implied volatilities for the options expiring in January 2005, computed in the preceding problem. Compare the implied volatilities for calls and puts. Where is the difference...

See Answer

Q: What volatilities were used to construct each tree? (You computed

What volatilities were used to construct each tree? (You computed zero-coupon bond prices in the previous problem; now you have to compute the year-1 yield volatility for 1-, 2-, 3-, and 4-year bonds....

See Answer

Q: Using the same assumptions as in Problem 26.12, compute

Using the same assumptions as in Problem 26.12, compute the 10-day 95% VaR for a claim that pays $3m each year in years 7–10. Problem 26.12 Suppose the 7-year zero-coupon bond has a yield of 6% and y...

See Answer

Q: Using Monte Carlo simulation, reproduce Tables 27.10 and 27

Using Monte Carlo simulation, reproduce Tables 27.10 and 27.11. Produce a similar table assuming a default correlation of 25%.

See Answer

Q: Pick a derivatives exchange such as CME Group, Eurex, or

Pick a derivatives exchange such as CME Group, Eurex, or the Chicago Board Options Exchange. Go to that exchange’s website and try to determine the following: a. What products the exchange trades. b....

See Answer

Q: Suppose you enter into a put ratio spread where you buy a

Suppose you enter into a put ratio spread where you buy a 45-strike put and sell two 40-strike puts. If you delta-hedge this position, what investment is required? What is your overnight profit if the...

See Answer

Q: Suppose the stock price is $40 and the effective annual interest

Suppose the stock price is $40 and the effective annual interest rate is 8%. Draw payoff and profit diagrams for the following options: a. 35-strike put with a premium of $1.53. b. 40-strike put with...

See Answer

Q: Golddiggers has zero net income if it sells gold for a price

Golddiggers has zero net income if it sells gold for a price of $380. However, by shorting a forward contract it is possible to guarantee a profit of $40/oz. Suppose a manager decides not to hedge and...

See Answer

Q: Verify that when there are transaction costs, the lower no-

Verify that when there are transaction costs, the lower no-arbitrage bound is given by equation (5.12).

See Answer

Q: As in the previous problem, consider holding a 3-year

As in the previous problem, consider holding a 3-year bond for 2 years. Now suppose that interest rates can change, but that at time 0 the rates in Table 7.1 prevail. What transactions could you under...

See Answer

Q: Using the zero-coupon bond yields in Table 8.9

Using the zero-coupon bond yields in Table 8.9, what is the fixed rate in a 4-quarter interest rate swap? What is the fixed rate in an 8-quarter interest rate swap?

See Answer

Q: In the following, suppose that neither stock pays a dividend.

In the following, suppose that neither stock pays a dividend. a. Suppose you have a call option that permits you to receive one share of Apple by giving up one share of AOL. In what circumstance might...

See Answer

Q: Let S = $40, K = $40, r

Let S = $40, K = $40, r = 8% (continuously compounded), σ = 30%, δ = 0, T =0.5 year, and n = 2. a. Construct the binomial tree for the stock. What are u and d? b. Show that the call price is $4.110. c...

See Answer

Q: We sawin Section 10.1 that the undiscounted risk-neutral

We sawin Section 10.1 that the undiscounted risk-neutral expected stock price equals the forward price. We will verify this using the binomial tree in Figure 11.4. a. Using S = $100, r = 0.08, and &Ic...

See Answer

Q: Consider a bull spread where you buy a 40-strike call

Consider a bull spread where you buy a 40-strike call and sell a 45-strike call. Suppose σ = 0.30, r = 0.08, δ = 0, and T = 0.5. a. Suppose S = $40. What are delta, gamma, vega, theta, and rho? b. Sup...

See Answer

Q: Consider a put for which T = 0.5 and K

Consider a put for which T = 0.5 and K = $45. Compute the Greeks and verify that equation (13.9) is zero.

See Answer

Q: Using the information in the previous problem, compute the prices of

Using the information in the previous problem, compute the prices of a. An Asian arithmetic average strike call. b. An Asian geometric average strike call. Previous Problem Suppose that S = $100, K =...

See Answer

Q: Problem 12.11 showed how to compute approximate Greek measures for

Problem 12.11 showed how to compute approximate Greek measures for an option. Use this technique to compute delta for the gap option in Figure 14.3, for stock prices ranging from $90 to $110 and for t...

See Answer

Q: Using the information in Table 15.5, suppose we have

Using the information in Table 15.5, suppose we have a bond that pays one barrel of oil in 2 years. a. Suppose the bond pays a fractional barrel of oil as an interest payment after 1 year and after 2...

See Answer

Q: Using the assumptions of Example 16.4, and the stock

Using the assumptions of Example 16.4, and the stock price derived in Example 16.5 suppose you were to perform a “naive” valuation of the convertible as a risk free...

See Answer

Q: Consider the oil project with a single barrel, in which S

Consider the oil project with a single barrel, in which S = $15, r = 5%, δ = 4%, and X = $13.60. Suppose that, in addition, the land can be sold for the residual value of R = $1 after the barrel of oi...

See Answer

Q: Assume that one stock follows the process dS/S =

Assume that one stock follows the process dS/S = αdt + σdZ (20.44) Another stock follows the process  (20.45) (Note that the σdZ terms for S and Q are identical.) Neither stock pays dividends. dq...

See Answer

Q: An agricultural producer wishes to insure the value of a crop.

An agricultural producer wishes to insure the value of a crop. Let Q represent the quantity of production in bushels and S the price of a bushel. The insurance payoff is therefore Q(T ) × V [S(T ), T...

See Answer

Q: Under the social security system in the United States, workers pay

Under the social security system in the United States, workers pay taxes and receive a monthly annuity after retirement. Some have argued that the United States should invest the social security tax p...

See Answer

Q: For the lookback put: a. What is the value

For the lookback put: a. What is the value of a lookback put if St= 0? Verify that the formula gives you the same answer. b. Verify that at maturity the value of the put is 

See Answer

Q: Suppose S = $100, r = 8%, σ =

Suppose S = $100, r = 8%, σ = 30%, T = 1, and δ = 0. Use the Black-Scholes formula to generate call and put prices with the strikes ranging from $40 to $250, with increments of $5. Compute the implied...

See Answer

Q: For years 2–5, compute the following: a

For years 2–5, compute the following: a. The forward interest rate, rf, for a forward rate agreement that settles at the time borrowing is repaid. That is, if you borrow at t − 1 at the 1-year rate ˜r...

See Answer

Q: Assume that the volatility of the S&P index is 30

Assume that the volatility of the S&P index is 30%. a. What is the price of a bond that after 2 years pays S2 + max (0, S2 − S0)? b. Suppose the bond pays S2 + [λ × max (0, S2 − S0)]. For what λ will...

See Answer

Q: Following Table 27.10, compute the prices of first,

Following Table 27.10, compute the prices of first, second, and Nth-to-default bonds assuming that defaults are uncorrelated and that there are 5, 10, 20, and 50 bonds in the portfolio. How are the Nt...

See Answer

Q: Consider the widget exchange. Suppose that each widget contract has a

Consider the widget exchange. Suppose that each widget contract has a market value of $0 and a notional value of $100. There are three traders, A, B, and C. Over one day, the following trades occur: A...

See Answer

Q: Consider the example in Table 4.6. Suppose that losses

Consider the example in Table 4.6. Suppose that losses are fully tax-deductible. What is the expected after-tax profit in this case? Table 4.6

See Answer

Q: Suppose the S&R index is 800, and that the

Suppose the S&R index is 800, and that the dividend yield is 0. You are an arbitrageur with a continuously compounded borrowing rate of 5.5% and a continuously compounded lending rate of 5%. Assume th...

See Answer

Q: Consider the implied forward rate between year 1 and year 2,

Consider the implied forward rate between year 1 and year 2, based on Table 7.1. a. Suppose that r0 (1, 2) = 6.8%. Show how buying the 2-year zero-coupon bond and borrowing at the 1-year rate and impl...

See Answer

Q: What 8-quarter dollar annuity is equivalent to an 8-

What 8-quarter dollar annuity is equivalent to an 8-quarter annuity of =C1?

See Answer

Q: The price of a non-dividend-paying stock is $

The price of a non-dividend-paying stock is $100 and the continuously compounded risk-free rate is 5%. A 1-year European call option with a strike price of $100 × e0.05×1= $105.127 has a premium of $1...

See Answer

Q: Use the same data as in the previous problem, only suppose

Use the same data as in the previous problem, only suppose that the call price is $5 instead of $4.110. Data from Previous Problem: Let S = $40, K = $40, r = 8% (continuously compounded), σ = 30%, δ...

See Answer

Q: Compute the 1-year forward price using the 50-step

Compute the 1-year forward price using the 50-step binomial tree in Problem 11.13. Problem 11.13 Repeat the previous problem for n = 50. What is the risk-neutral probability that S1< $80? S1> $120? Pr...

See Answer

Q: Consider a bull spread where you buy a 40-strike put

Consider a bull spread where you buy a 40-strike put and sell a 45-strike put. Suppose σ = 0.30, r = 0.08, δ = 0, and T = 0.5. a. Suppose S = $40. What are delta, gamma, vega, theta, and rho? b. Suppo...

See Answer

Q: Suppose there is a single 5-year zero-coupon debt

Suppose there is a single 5-year zero-coupon debt issue with a maturity value of $120. The expected return on assets is 12%. What is the expected return on equity? The volatility of equity? What happe...

See Answer

Q: You own one 45-strike call with 180 days to expiration

You own one 45-strike call with 180 days to expiration. Compute and graph the 1-day holding period profit if you delta- and gamma-hedge this position using a 40-strike call with 180 days to expiration...

See Answer

Q: Consider the gap put in Figure 14.4. Using the

Consider the gap put in Figure 14.4. Using the technique in Problem 12.11, compute vega for this option at stock prices of $90, $95, $99, $101, $105, and $110, and for times to expiration of 1 week, 3...

See Answer

Q: Using the information in Table 15.5, suppose we have

Using the information in Table 15.5, suppose we have a bond that after 2 years pays one barrel of oil plus λ × max(0, S2 − 20.90), where S2 is the year-2 sp...

See Answer

Q: Consider Panels B and D in Figure 16.4. Using

Consider Panels B and D in Figure 16.4. Using the information in each panel, compute the share price at each node for each bond issue.

See Answer

Q: Verify in Figure 17.2 that if volatility were 30%

Verify in Figure 17.2 that if volatility were 30% instead of 50%, immediate exercise would be optimal.

See Answer

Q: Refer to Table 19.1. a. Verify the

Refer to Table 19.1. a. Verify the regression coefficients in equation (19.12). b. Perform the analysis for t = 1, verifying that exercise is optimal on paths 4, 6, 7, and 8, and not on path 1.

See Answer

Q: Warren Buffett stated in the 2009 Letter to Shareholders: “Our

Warren Buffett stated in the 2009 Letter to Shareholders: “Our derivatives dealings require our counterparties to make payments to us when contracts are initiated. Berkshire therefore always holds th...

See Answer

Q: A European shout option is an option for which the payoff at

A European shout option is an option for which the payoff at expiration is max(0, S − K, G − K), where G is the price at which you shouted. (Suppose you have an XYZ shout call with a strike price of $...

See Answer

Q: Explain why the VIX formula in equation (24.29)

Explain why the VIX formula in equation (24.29) overestimates implied volatility if options are American. The following three problems use the Merton jump formula. As a base case, assume S = $100, r =...

See Answer

Q: You are going to borrow $250m at a floating rate for

You are going to borrow $250m at a floating rate for 5 years. You wish to protect yourself against borrowing rates greater than 10.5%. Using each tree, what is the price of a 5-year interest rate cap?...

See Answer

Q: Consider the widget investment problem outlined in Section 17.1.

Consider the widget investment problem outlined in Section 17.1. Show the following in a spreadsheet. a. Compute annual widget prices for the next 50 years. b. For each year, compute the net present v...

See Answer

Q: Repeat the previous problem, assuming that default correlations are 0.

Repeat the previous problem, assuming that default correlations are 0.25. Repeat the previous problem, Following Table 27.10, compute the prices of first, second, and Nth-to-default bonds assuming th...

See Answer

Q: The profit calculation in the chapter assumes that you borrow at a

The profit calculation in the chapter assumes that you borrow at a fixed interest rate to finance investments. An alternative way to borrow is to short-sell stock. What complications would arise in ca...

See Answer

Q: Suppose the S&P 500 currently has a level of 875

Suppose the S&P 500 currently has a level of 875. The continuously compounded return on a 1-year T-bill is 4.75%. You wish to hedge an $800,000 portfolio that has a beta of 1.1 and a correlation of 1....

See Answer

Q: Suppose the September Eurodollar futures contract has a price of 96.

Suppose the September Eurodollar futures contract has a price of 96.4. You plan to borrow $50m for 3 months in September at LIBOR, and you intend to use the Eurodollar contract to hedge your borrowing...

See Answer

Q: Suppose that to buy either a call or a put option you

Suppose that to buy either a call or a put option you pay the quoted ask price, denoted Ca(K, T ) and Pa(K, T ), and to sell an option you receive the bid, Cb(K, T ) and Pb(K, T ). Similarly, the ask...

See Answer

Q: Suppose that the exchange rate is $0.92/=C

Suppose that the exchange rate is $0.92/=C. Let r$ = 4%, and r=C = 3%, u = 1.2, d = 0.9, T = 0.75, n = 3, and K = $0.85. a. What is the price of a 9-month European call? b. What is the price of a 9-mo...

See Answer

Q: Suppose S = $100, K = $95, r

Suppose S = $100, K = $95, r = 8% (continuously compounded), t = 1, σ = 30%, and δ = 5%. Explicitly construct an eight-period binomial tree using the Cox-Ross- Rubinstein expressions for u and d: 

See Answer

Q: Assume r = 8%, σ = 30%, δ = 0

Assume r = 8%, σ = 30%, δ = 0. In doing the following calculations, use a stock price range of $60–$140, stock price increments of $5, and two different times to expiration: 1 year and 1 day. Consider...

See Answer

Q: You have sold one 45-strike put with 180 days to

You have sold one 45-strike put with 180 days to expiration. Compute and graph the 1-day holding period profit if you delta- and gamma-hedge this position using the stock and a 40-strike call with 180...

See Answer

Q: Let S = $40, σ = 0.30,

Let S = $40, σ = 0.30, r = 0.08, T = 1, and δ = 0. Also let Q = $60, σQ= 0.50, δQ= 0.04, and ρ = 0.5. What is the price of a standard 40-strike call with S as the underlying asset? What is the price o...

See Answer

Q: Suppose x1∼ N (2, 0.5) and

Suppose x1∼ N (2, 0.5) and x2 ∼ N (8, 14). The correlation between x1 and x2 is −0.3. What is the distribution of x1+ x2? What is the distribution of x1+ x2?

See Answer

Q: Using the information in Table 15.5, assume that the

Using the information in Table 15.5, assume that the volatility of oil is 15%. a. Show that a bond that pays one barrel of oil in 1 year sells today for $19.2454. b. Consider a bond that in 1 year has...

See Answer

Q: As discussed in the text, compensation options are prematurely exercised or

As discussed in the text, compensation options are prematurely exercised or canceled for a variety of reasons. Suppose that compensation options both vest and expire in 3 years and that the probabilit...

See Answer

Q: Consider the last row of Table 17.1. What is

Consider the last row of Table 17.1. What is the solution for S∗ and S∗ when ks = kr = 0? (This answer does not require calculation.) In the following five problems, assume that the spot price of gold...

See Answer

Q: Refer to Figure 19.2. a. Verify that

Refer to Figure 19.2. a. Verify that the price of a European put option is $0.0564. b. Verify that the price of an American put option is $0.1144. Be sure to allow for the possibility of exercise at t...

See Answer

Q: Consider the Level 3 outperformance option with a multiplier, discussed in

Consider the Level 3 outperformance option with a multiplier, discussed in Section 16.2. This can be valued binomially using the single state variable SLevel 3/SS&P, and multiplying the resulting valu...

See Answer

Q: Using the Merton jump formula, generate an implied volatility plot for

Using the Merton jump formula, generate an implied volatility plot for K = 50, 55, . . . 150. a. How is the implied volatility plot affected by changing αJ to−0.40 or−0.10? b. How is the implied volat...

See Answer

Q: Suppose that the yield curve is given by y(t)

Suppose that the yield curve is given by y(t) = 0.10 − 0.07e−0.12t, and that the short-term interest rate process is dr(t) = (θ(t) − 0.15r(t)) + 0.01dZ. Compute the calibrated Hull-White tree for 5 ye...

See Answer

Q: Suppose that firms face a 40% income tax rate on all

Suppose that firms face a 40% income tax rate on all profits. In particular, losses receive full credit. Firm A has a 50% probability of a $1000 profit and a 50% probability of a $600 loss each year....

See Answer

Q: Suppose you know nothing about widgets. You are going to approach

Suppose you know nothing about widgets. You are going to approach a widget merchant to borrow one in order to short-sell it. (That is, you will take physical possession of the widget, sell it, and ret...

See Answer

Q: In this problem we will use Monte Carlo to simulate the behavior

In this problem we will use Monte Carlo to simulate the behavior of the martingale St/Pt, with Pt as numeraire. Let x0 = S0/P0(0, T ). Simulate the process  Let h be approximately 1 day. a. Evaluate...

See Answer

Q: Suppose the current exchange rate between Germany and Japan is 0.

Suppose the current exchange rate between Germany and Japan is 0.02 =C/¥. The euro-denominated annual continuously compounded risk-free rate is 4% and the yen-denominated annual continuously compounde...

See Answer

Q: Use the following inputs to compute the price of a European call

Use the following inputs to compute the price of a European call option: S = $100, K = $50, r = 0.06, σ = 0.30, T = 0.01, δ = 0. a. Verify that the Black-Scholes price is $50.0299. b. Verify that the...

See Answer

Q: Consider two zero-coupon bonds with 2 years and 10 years

Consider two zero-coupon bonds with 2 years and 10 years to maturity. Let a =0.2, b = 0.1, r = 0.05, σVasicek = 10%, and σCIR = 44.721%. The interest rate risk premium is zero in each case. We will co...

See Answer

Q: Using the delta-approximation method and assuming a $10m investment

Using the delta-approximation method and assuming a $10m investment in stock A, compute the 95% and 99% 1-, 10-, and 20-day VaRs for a position consisting of stock A plus one 105-strike put option for...

See Answer

Q: There are four debt issues with different priorities, each promising $

There are four debt issues with different priorities, each promising $30 at maturity. a. Compute the yield on each debt issue assuming that all four mature in 1 year, 2 years, 5 years, or 10 years. b....

See Answer

Q: Suppose that there is a 3%per year chance that the

Suppose that there is a 3%per year chance that the firm’s asset value can jump to zero. Assume that the firm issues 5-year zero-coupon debt with a promised payment of $110. Using the Merton jump model...

See Answer

Q: Suppose you short-sell 300 shares of XYZ stock at $

Suppose you short-sell 300 shares of XYZ stock at $30.19 with a commission charge of 0.5%. Supposing you pay commission charges for purchasing the security to cover the short-sale, how much profit hav...

See Answer

Q: Suppose XYZ stock pays no dividends and has a current price of

Suppose XYZ stock pays no dividends and has a current price of $50. The forward price for delivery in 1 year is $55. Suppose the 1-year effective annual interest rate is 10%. a. Graph the payoff and p...

See Answer

Q: If Telco does nothing to manage copper price risk, what is

If Telco does nothing to manage copper price risk, what is its profit 1 year from now, per pound of copper that it buys? If it hedges the price of wire by buying copper forward, what is its estimated...

See Answer

Q: Draw profit diagrams for the following positions: a. 1050

Draw profit diagrams for the following positions: a. 1050-strike S&R straddle. b. Written 950-strike S&R straddle. c. Simultaneous purchase of a 1050-strike straddle and sale of a 950-strike S&R strad...

See Answer

Q: The S&R index spot price is 1100, the risk

The S&R index spot price is 1100, the risk-free rate is 5%, and the dividend yield on the index is 0. a. Suppose you observe a 6-month forward price of 1135. What arbitrage would you undertake? b. Sup...

See Answer

Q: Using the information in Table 7.1, a.

Using the information in Table 7.1, a. Compute the implied forward rate from time 1 to time 3. b. Compute the implied forward price of a par 2-year coupon bond that will be issued at time 1.

See Answer

Q: Suppose you are selecting a futures contract with which to hedge a

Suppose you are selecting a futures contract with which to hedge a portfolio. You have a choice of six contracts, each of which has the same variability, but with correlations of −0.95, −0.75, −0.50,...

See Answer

Q: Consider the same facts as the previous problem, only now consider

Consider the same facts as the previous problem, only now consider hedging with the 3-month Eurodollar futures. Suppose the Eurodollar futures contract that matures 60 days from today has a price on d...

See Answer

Q: Using the information about zero-coupon bond prices and oil forward

Using the information about zero-coupon bond prices and oil forward prices in Table 8.9, construct the set of swap prices for oil for 1 through 8 quarters.

See Answer

Q: Suppose the dollar-denominated interest rate is 5%, the yen

Suppose the dollar-denominated interest rate is 5%, the yen-denominated interest rate is 1% (both rates are continuously compounded), the spot exchange rate is 0.009 $/¥, and the price of a dollar-den...

See Answer

Q: Repeat the option price calculation in the previous question for stock prices

Repeat the option price calculation in the previous question for stock prices of $80, $90, $110, $120, and $130, keeping everything else fixed. What happens to the initial option ∆ as the stock price...

See Answer

Q: Let S = $100, K = $100, σ

Let S = $100, K = $100, σ = 30%, r = 0.08, t = 1, and δ = 0. Let n = 10. Suppose the stock has an expected return of 15%. a. What is the expected return on a European call option? A European put optio...

See Answer

Q: Use Itˆo’s Lemma to evaluate dS−1. For the

Use Itˆo’s Lemma to evaluate dS−1. For the following four problems, use Itˆo’s Lemma to determine the process followed by th...

See Answer

Q: Suppose S = $100, K = $95, σ

Suppose S = $100, K = $95, σ = 30%, r = 0.08, δ = 0.03, and T = 0.75. a. Compute the Black-Scholes price of a call. b. Compute the Black-Scholes price of a call for which S = $100 × e−0.03×0.75, K = $...

See Answer

Q: Suppose you buy a 950-strike S&R call,

Suppose you buy a 950-strike S&R call, sell a 1000-strike S&R call, sell a 950-strike S&R put, and buy a 1000-strike S&R put. a. Verify that there is no S&R price risk in this transaction. b. What is...

See Answer

Q: Let S = $40, K = $45, σ

Let S = $40, K = $45, σ = 0.30, r = 0.08, δ = 0, and T = {0.25, 0.5, 1, 2, 3, 4, 5, 100}. a. Compute the prices of knock-out calls with a barrier of $38. b. Compute the ratio of the knock-out call pri...

See Answer

Q: Explain how to synthetically create the equity-linked CD in Section

Explain how to synthetically create the equity-linked CD in Section 15.3 by using a forward contract on the S&P index and a put option instead of a call option. (Hint: Use put-call parity. Remembe...

See Answer

Q: The firm is considering an investment project costing $1. What

The firm is considering an investment project costing $1. What is the amount by which the project’s value must exceed its cost in order for shareholders to be willing to pay for it? Repeat for project...

See Answer

Q: Suppose you observe the following month-end stock prices for stocks

Suppose you observe the following month-end stock prices for stocks A and B: For each stock: a. Compute the mean monthly continuously compounded return. What is the annual return? b. Compute the mean...

See Answer

Q: Using the information in Table 8.9, verify that it

Using the information in Table 8.9, verify that it is possible to derive the 8-quarter dollar interest swap rate from the 8-quarter euro interest swap rate by using equation (8.13). Equation (8.13)....

See Answer

Q: The spot price of a widget is $70.00 per

The spot price of a widget is $70.00 per unit. Forward prices for 3, 6, 9, and 12 months are $70.70, $71.41, $72.13, and $72.86. Assuming a 5% continuously compounded annual risk-free rate, what are t...

See Answer

Q: a. Suppose that you want to borrow a widget beginning in

a. Suppose that you want to borrow a widget beginning in December of Year 0 and ending in March of Year 1. What payment will be required to make the transaction fair to both parties? b. Suppose that y...

See Answer

Q: Assume that the volatility of the S&P index is 30

Assume that the volatility of the S&P index is 30% and consider a bond with the payoff S2 + λ × [max (0, S2 − S0) – max (0, S2 − K)]. a. If λ = 1 and K = $1500, what is the price of the bond? b. Suppo...

See Answer

Q: An option has a gold futures contract as the underlying asset.

An option has a gold futures contract as the underlying asset. The current 1-year gold futures price is $300/oz, the strike price is $290, the risk-free rate is 6%, volatility is 10%, and time to expi...

See Answer

Q: The stock price of XYZ is $100. One million shares

The stock price of XYZ is $100. One million shares of XYZ (a negligible fraction of the shares outstanding) are buried on a tiny, otherwise worthless plot of land in a vault that would cost $50 millio...

See Answer

Q: Suppose S0 = 100, r = 0.06, σS

Suppose S0 = 100, r = 0.06, σS= 0.4 and δ = 0. Use Monte Carlo to compute prices for claims that pay the following: a.  b. c. 

See Answer

Q: Verify that e−r(T−t)N(

Verify that e−r(T−t)N(d2) satisfies the Black-Scholes equation.

See Answer

Q: Use the answers to the previous two problems to verify that the

Use the answers to the previous two problems to verify that the Black-Scholes formula, equation (12.1), satisfies the Black-Scholes equation. Verify that the boundary condition V [S(T ), T ]= max[0, S...

See Answer

Q: We now use Monte Carlo to simulate the behavior of the martingale

We now use Monte Carlo to simulate the behavior of the martingale Pt/St, with St as numeraire. Let x0 = P0(0, T )/S0. Simulate the process xt+h= (1+ σ√hZt+h)xt Let h be approximately 1 day. a. Evaluat...

See Answer

Q: In the previous problem we saw that a ratio spread can have

In the previous problem we saw that a ratio spread can have zero initial premium. Can a bull spread or bear spread have zero initial premium? A butterfly spread? Why or why not? Previous Problem Comp...

See Answer

Q: Verify that equation (23.14) (for both cases

Verify that equation (23.14) (for both cases K >H andK

See Answer

Q: Consider a perpetual put option with S = $50, K

Consider a perpetual put option with S = $50, K = $60, r = 0.06, σ = 0.40, and δ = 0.03. a. What is the price of the option and at what stock price should it be exercised? b. Suppose δ = 0.04 with all...

See Answer

Q: XYZ Corp. compensates executives with 10-year European call options

XYZ Corp. compensates executives with 10-year European call options, granted at the money. If there is a significant drop in the share price, the company’s board will reset the strike price of the opt...

See Answer

Q: Consider the June 165, 170, and 175 call option prices

Consider the June 165, 170, and 175 call option prices in Table 9.1. a. Does convexity hold if you buy a butterfly spread, buying at the ask price and selling at the bid? b. Does convexity hold if yo...

See Answer

Q: Suppose that the exchange rate is 1 dollar for 120 yen.

Suppose that the exchange rate is 1 dollar for 120 yen. The dollar interest rate is 5% (continuously compounded) and the yen rate is 1% (continuously compounded). Consider an at-the-money American dol...

See Answer

Q: Construct an asymmetric butterfly using the 950-, 1020-, and 1050

Construct an asymmetric butterfly using the 950-, 1020-, and 1050-strike options. How many of each option do you hold? Draw a profit diagram for the position.

See Answer

Q: Use Itˆo’s Lemma to evaluate d(√S). For the

Use Itˆo’s Lemma to evaluate d(√S). For the following four problems, use Itˆo’s Lemma to determine the process followed by t...

See Answer

Q: You have purchased a 40-strike call with 91 days to

You have purchased a 40-strike call with 91 days to expiration. You wish to delta-hedge, but you are also concerned about changes in volatility; thus, you want to vega-hedge your position as well. a....

See Answer

Q: XYZ wants to hedge against depreciations of the euro and is also

XYZ wants to hedge against depreciations of the euro and is also concerned about the price of oil, which is a significant component of XYZ’s costs. However, there is a positive correlation between the...

See Answer

Q: Suppose that S = $100, σ = 30%, r

Suppose that S = $100, σ = 30%, r = 6%, t = 1, and δ = 0. XYZ writes a European put option on one share with strike price K = $90. a. Construct a two-period binomial tree for the stock and price the p...

See Answer

Q: Using the CEV option pricing model, set β = 1and generate

Using the CEV option pricing model, set β = 1and generate option prices for strikes from 60 to 140, in increments of 5, for times to maturity of 0.25, 0.5, 1.0, and 2.0. Plot the resulting...

See Answer

Q: In this problem you will price various options with payoffs based on

In this problem you will price various options with payoffs based on the Eurostoxx index and the dollar/euro exchange rate. Assume that Q= 2750 (the index), x = 1.25 ($/=C), s = 0.08 (the exchange rat...

See Answer

Q: Suppose that LMN Investment Bank wishes to sell Auric a zero-

Suppose that LMN Investment Bank wishes to sell Auric a zero-cost collar of width 30 without explicit premium (i.e., there will be no cash payment from Auric to LMN). Also suppose that on every option...

See Answer

Q: Let c be consumption. Under what conditions on the parameters λ0

Let c be consumption. Under what conditions on the parameters λ0 and λ1 could the following functions serve as utility functions for a risk-averse investor? (Remember that marginal utility must be pos...

See Answer

Q: Suppose we wish to borrow $10 million for 91 days beginning

Suppose we wish to borrow $10 million for 91 days beginning next June, and that the quoted Eurodollar futures price is 93.23. a. What 3-month LIBOR rate is implied by this price? b. How much will be n...

See Answer

Q: Compute Macaulay and modified durations for the following bonds: a

Compute Macaulay and modified durations for the following bonds: a. A 5-year bond paying annual coupons of 4.432% and selling at par. b. An 8-year bond paying semiannual coupons with a coupon rate of...

See Answer

Q: Suppose the S&P 500 futures price is 1000, σ

Suppose the S&P 500 futures price is 1000, σ = 30%, r = 5%, δ = 5%, T = 1, and n = 3. a. What are the prices of European calls and puts for K = $1000? Why do you find the prices to be equal? b. What a...

See Answer

Q: Suppose you sell a 45-strike call with 91 days to

Suppose you sell a 45-strike call with 91 days to expiration. What is delta? If the option is on 100 shares, what investment is required for a delta-hedged portfolio? What is your overnight profit if...

See Answer

Q: Suppose XYZ stock pays no dividends and has a current price of

Suppose XYZ stock pays no dividends and has a current price of $50. The forward price for delivery in one year is $53. If there is no advantage to buying either the stock or the forward contract, what...

See Answer

Q: Let S = $100, K = $90, σ

Let S = $100, K = $90, σ = 30%, r = 8%, δ = 5%, and T = 1. a. What is the Black-Scholes call price? b. Now price a put where S = $90, K = $100, σ = 30%, r = 5%, δ = 8%, and T = 1. c. What is the link...

See Answer

Q: Verify that the butterfly spread in Figure 3.14 can be

Verify that the butterfly spread in Figure 3.14 can be duplicated by the following transactions (use the option prices in Table 3.4): a. Buy 35 call, sell two 40 calls, buy 45 call. b. Buy 35 put, s...

See Answer

Q: Estimate a GARCH (1,1) for the S&

Estimate a GARCH (1,1) for the S&P 500 index, using data from January 1999 to December 2003.

See Answer

Q: Using the information in Table 8.9, what are the

Using the information in Table 8.9, what are the euro-denominated fixed rates for 4- and 8-quarter swaps?

See Answer

Q: Use the same inputs as in the previous problem. Suppose that

Use the same inputs as in the previous problem. Suppose that you observe a bid option price of $50 and an ask price of $50.10. a. Explain why you cannot compute an implied volatility for the bid price...

See Answer

Q: You have been asked to construct an oil contract that has the

You have been asked to construct an oil contract that has the following characteristics: The initial cost is zero. Then in each period, the buyer pays S −, with a cap of $21.90 −  and a floor of $19...

See Answer

Q: Repeat the previous problem, except that instead of hedging volatility risk

Repeat the previous problem, except that instead of hedging volatility risk, you wish to hedge interest rate risk, i.e., to rho-hedge. In addition to delta-, gamma-, and rho-hedging, can you delta-gam...

See Answer

Q: A chooser option (also known as an as-you-

A chooser option (also known as an as-you-like-it option) becomes a put or call at the discretion of the owner. For example, consider a chooser on the S&R index for which both the call, with value C (...

See Answer

Q: Consider again the Netscape PEPS discussed in this chapter and assume the

Consider again the Netscape PEPS discussed in this chapter and assume the following: the price of Netscape is $39.25, Netscape is not expected to pay dividends, the interest rate is 7%, and the 5-year...

See Answer

Q: Firm A has a stock price of $40 and has made

Firm A has a stock price of $40 and has made an offer for firm B where A promises to pay $60/share for B, as long as A’s stock price remains between $35 and $45. If the price of A is below $35, A will...

See Answer

Q: Repeat Problem 17.18 assuming that the volatility of gold is

Repeat Problem 17.18 assuming that the volatility of gold is 20% and that once opened, the mine can be costlessly shut down forever. What is the value of the mine? What is the price at which the mine...

See Answer

Q: Consider Example 6.1. Suppose the February forward price had

Consider Example 6.1. Suppose the February forward price had been $2.80. What would the arbitrage be? Suppose it had been $2.65. What would the arbitrage be? In each case, specify the transactions and...

See Answer

Q: Using the CEV option pricing model, set β = 3 and

Using the CEV option pricing model, set β = 3 and generate option prices for strikes from 60 to 140, in increments of 5, for times to maturity of 0.25, 0.5, 1.0, and 2.0. Plot the resulting implied vo...

See Answer

Q: Compute estimated profit in 1 year if Telco buys a call option

Compute estimated profit in 1 year if Telco buys a call option with a strike of $0.95, $1.00, or $1.05. Draw a graph of profit in each case.

See Answer

Q: Using Monte Carlo, simulate the process dr = a(b

Using Monte Carlo, simulate the process dr = a(b − r)dt + σ, assuming that r = 6%, a = 0.2, b = 0.08, φ = 0 and σ = 0.02. Compute the prices of 1-, 2-, and 3-year zero coupon bonds, and verify that y...

See Answer

Q: Use the same assumptions as in the preceding problem, without the

Use the same assumptions as in the preceding problem, without the bid-ask spread. Suppose that we want to construct a paylater strategy using a ratio spread. Instead of buying a 440-strike call, Auric...

See Answer

Q: A collect-on-delivery call (COD) costs zero

A collect-on-delivery call (COD) costs zero initially, with the payoff at expiration being 0 if S

See Answer

Q: Let S = $40, σ = 0.30,

Let S = $40, σ = 0.30, r = 0.08, T = 1, and δ = 0. Also let Q = $60, σQ= 0.50,δQ= 0, and ρ = 0.5. In this problem we will compute prices of exchange calls with S as the price of the underlying asset a...

See Answer

Q: Here is a quote from an investment website about an investment strategy

Here is a quote from an investment website about an investment strategy using options: One strategy investors are applying to the XYZ options is using “synthetic stock.”Asynthetic stock is created whe...

See Answer

Q: Consider the following two bonds which make semiannual coupon payments: a

Consider the following two bonds which make semiannual coupon payments: a 20- year bond with a 6% coupon and 20% yield, and a 30-year bond with a 6% coupon and a 20% yield. a. For each bond, compute t...

See Answer

Q: For a stock index, S = $100, σ =

For a stock index, S = $100, σ = 30%, r = 5%, δ = 3%, and T = 3. Let n = 3. a. What is the price of a European call option with a strike of $95? b. What is the price of a European put option with a st...

See Answer

Q: In this problem we consider whether parity is violated by any of

In this problem we consider whether parity is violated by any of the option prices in Table 9.1. Suppose that you buy at the ask and sell at the bid, and that your continuously compounded lending r...

See Answer

Q: Suppose the yield curve is flat at 8%. Consider 3-

Suppose the yield curve is flat at 8%. Consider 3- and 6-year zero-coupon bonds. You buy one 3-year bond and sell an appropriate quantity of the 6-year bond to duration hedge the position. Any additio...

See Answer

Q: Repeat the previous problem, but this time for perpetual options.

Repeat the previous problem, but this time for perpetual options. What do you notice about the prices? What do you notice about the exercise barriers? Previous Problem Let S = $100, K = $90, σ = 30%,...

See Answer

Q: Consider a 5-year equity-linked note that pays one

Consider a 5-year equity-linked note that pays one share of XYZ at maturity. The price of XYZ today is $100, and XYZ is expected to pay its annual dividend of $1 at the end of this year, increasing by...

See Answer

Q: Suppose that S = $100, σ = 30%, r

Suppose that S = $100, σ = 30%, r = 8%, and δ = 0. Today you buy a contract which, 6 months from today, will give you one 3-month to expiration at-the-money call option. (This is called a forward star...

See Answer

Q: A DECS contract pays two shares if ST < 27.875

A DECS contract pays two shares if ST < 27.875, 1.667 shares if the price is above ST > 33.45, and $27.875 and $55.75 otherwise. The quarterly dividend is $0.87. Value this DECS assuming that S = $26....

See Answer

Q: Suppose that S = $50, K = $45,

Suppose that S = $50, K = $45, σ = 0.30, r = 0.08, and t = 1. The stock will pay a $4 dividend in exactly 3 months. Compute the price of European and American call options using a four-step binomial t...

See Answer

Q: The S&R index spot price is 1100, the risk

The S&R index spot price is 1100, the risk-free rate is 5%, and the continuous dividend yield on the index is 2%. a. Suppose you observe a 6-month forward price of 1120. What arbitrage would you under...

See Answer

Q: Use the Black-Scholes equation to verify the solution in Chapter

Use the Black-Scholes equation to verify the solution in Chapter 20, given by Proposition 20.3, for the value of a claim paying Sa.

See Answer

Q: Firm A has a stock price of $40, and has

Firm A has a stock price of $40, and has made an offer for firm B where A promises to pay 1.5 shares for each share of B, as long as A’s stock price remains between $35 and $45. If the price of A is b...

See Answer

Q: Repeat Problem 17.18 assuming that the volatility of gold is

Repeat Problem 17.18 assuming that the volatility of gold is 20% and that once opened, the mine can be costlessly shut down once, and then costlessly reopened once. What is the value of the mine? What...

See Answer

Q: Using the information in Table 4.11, verify that a

Using the information in Table 4.11, verify that a regression of revenue on price gives a regression slope coefficient of about 100,000. Table 4.11:

See Answer

Q: Compute estimated profit in 1 year if XYZ buys collars with the

Compute estimated profit in 1 year if XYZ buys collars with the following strikes: a. $0.95 for the put and $1.00 for the call. b. $0.975 for the put and $1.025 for the call. c. $1.05 for the put and...

See Answer

Q: An 8-year bond with 6% annual coupons and a

An 8-year bond with 6% annual coupons and a 5.004% yield sells for $106.44 with a Macaulay duration of 6.631864. A 9-year bond has 7% annual coupons with a 5.252% yield and sells for $112.29 with a Ma...

See Answer

Q: Repeat the previous problem calculating prices for American options instead of European

Repeat the previous problem calculating prices for American options instead of European. What happens? Previous Problem For a stock index, S = $100, σ = 30%, r = 5%, δ = 3%, and T = 3. Let n = 3. a. W...

See Answer

Q: You wish to insure a portfolio for 1 year. Suppose that

You wish to insure a portfolio for 1 year. Suppose that S = $100, σ = 30%, r = 8%, and δ = 0. You are considering two strategies. The simple insurance strategy entails buying one put option with a 1-y...

See Answer

Q: Suppose you are a market-maker in S&R index

Suppose you are a market-maker in S&R index forward contracts. The S&R index spot price is 1100, the risk-free rate is 5%, and the dividend yield on the index is 0. a. What is the no-arbitrage forward...

See Answer

Q: Using weekly price data (constructed Wednesday to Wednesday), compute historical

Using weekly price data (constructed Wednesday to Wednesday), compute historical annual volatilities for IBM, Xerox, and the S&P 500 index for 1991 through 2004. Annualize your answer by multiplying b...

See Answer

Q: Use the same inputs as in the previous problem, except that

Use the same inputs as in the previous problem, except that K = $1.00. a. What is the price of a 9-month European put? b. What is the price of a 9-month American put?

See Answer

Q: A stock purchase contract with a zero initial premium calls for you

A stock purchase contract with a zero initial premium calls for you to pay for one share of stock in 3 years. The stock price is $100 and the 3-year interest rate is 3%. a. If you expect the stock to...

See Answer

Q: Suppose S (0) = $100, r = 0

Suppose S (0) = $100, r = 0.06, σS= 0.4, and δ = 0. Use equation (20.32) to compute prices for claims that pay the following: a. S2 b. √S c. Sâˆ...

See Answer

Q: What are 95% and 99% 1-, 10-, and

What are 95% and 99% 1-, 10-, and 20-dayVaRs for a portfolio that has $4m invested in stock A, $3.5m in stock B, and $2.5m in stock C?

See Answer

Q: Suppose that in order to hedge interest rate risk on your borrowing

Suppose that in order to hedge interest rate risk on your borrowing, you enter into an FRA that will guarantee a 6%effective annual interest rate for 1 year on $500,000.00. On the date you borrow the...

See Answer

Q: The strike price of a compensation option is generally set on the

The strike price of a compensation option is generally set on the day the option is issued. On November 10, 2000, the CEO of Analog Devices, Jerald Fishman, received 600,000 options. The stock price w...

See Answer

Q: Using the information in Table 4.9 about Scenario C:

Using the information in Table 4.9 about Scenario C: a. Compute total revenue when correlation between price and quantity is positive. b. What is the correlation between price and revenue?

See Answer

Q: A 6-year bond with a 4% coupon sells for

A 6-year bond with a 4% coupon sells for $102.46 with a 3.5384% yield. The conversion factor for the bond is 0.90046. An 8-year bond with 5.5% coupons sells for $113.564 with a conversion factor of 0....

See Answer

Q: Suppose that u < e(r−δ)h.

Suppose that u < e(r−δ)h. Show that there is an arbitrage opportunity. Now suppose that d >e(r−δ)h. Show again that there is an arbitrage opportunity.

See Answer

Q: You have written a 35–40–45 butterfly spread with

You have written a 35–40–45 butterfly spread with 91 days to expiration. Compute and graph the 1-day holding period profit if you delta- and gamma-hedge this position using the stock and a 40-strike c...

See Answer

Q: a. What is the 1-year bond forward price in

a. What is the 1-year bond forward price in year 1? b. What is the price of a call option that expires in 1 year, giving you the right to pay $0.9009 to buy a bond expiring in 1 year? c. What is the p...

See Answer

Q: Construct a spreadsheet for which you can input up to five strike

Construct a spreadsheet for which you can input up to five strike prices and quantities of put and call options bought or sold at those strikes, and which will automatically construct the total expira...

See Answer

Q: Suppose you have a project that will produce a single widget.

Suppose you have a project that will produce a single widget. Widgets today cost $1 and the project costs $0.90. The risk-free rate is 5%. Under what circumstances would you invest immediately in the...

See Answer

Q: Value the M&I stock purchase contract assuming that the 3

Value the M&I stock purchase contract assuming that the 3-year interest rate is 3% and the M&I volatility is 15%. How does your answer change if volatility is 35%?

See Answer

Q: Construct a four-period, three-step (eight terminal

Construct a four-period, three-step (eight terminal node) binomial interest rate tree where the initial interest rate is 10% and rates can move up or down by 2%; model your tree after that in Figure 2...

See Answer

Q: Four years after the option grant, the stock price for Analog

Four years after the option grant, the stock price for Analog Devices was about $40. Using the same input as in the previous problem, compute the market value of the options granted in 2000, assuming...

See Answer

Q: Using the information in Table 4.9 about Scenario C:

Using the information in Table 4.9 about Scenario C: a. Using your answer to the previous question, use equation (4.7) to compute the variance-minimizing hedge ratio. b. Run a regression of revenue o...

See Answer

Q: Using the information in Table 4.9 about Scenario C:

Using the information in Table 4.9 about Scenario C: a. What is the expected quantity of production? b. Suppose you short the expected quantity of corn. What is the standard deviation of hedged reve...

See Answer

Q: Compute profit diagrams for the following ratio spreads: a.

Compute profit diagrams for the following ratio spreads: a. Buy 950-strike call, sell two 1050-strike calls. b. Buy two 950-strike calls, sell three 1050-strike calls. c. Consider buying n 950-strike...

See Answer

Q: Suppose S = $100, K = $95, r

Suppose S = $100, K = $95, r = 8% (continuously compounded), t = 1, σ = 30%, and δ = 5%. Explicitly construct an eight-period binomial tree using the lognormal expressions for u and d:  Compute the p...

See Answer

Q: Suppose a 10-year zero-coupon bond with a face

Suppose a 10-year zero-coupon bond with a face value of $100 trades at $69.20205. a. What is the yield to maturity and modified duration of the zero-coupon bond? b. Calculate the approximate bond pric...

See Answer

Q: A default-free zero-coupon bond costs $91 and

A default-free zero-coupon bond costs $91 and will pay $100 at maturity in 1 year. What is the effective annual interest rate? What is the payoff diagram for the bond? The profit diagram?

See Answer

Q: Suppose that firms face a 40% income tax rate on positive

Suppose that firms face a 40% income tax rate on positive profits and that net losses receive no credit. (Thus, if profits are positive, after-tax income is (1− 0.4) × profit, while if there is a loss...

See Answer

Q: Suppose the spot $/¥ exchange rate is 0.008, the

Suppose the spot $/¥ exchange rate is 0.008, the 1-year continuously compounded dollar-denominated rate is 5% and the 1-year continuously compounded yen-denominated rate is 1%. Suppose the 1-year forw...

See Answer

Q: a. What are some possible explanations for the shape of this

a. What are some possible explanations for the shape of this forward curve? b. What annualized rate of return do you earn on a cash-and-carry entered into in December of Year 0 and closed in March of...

See Answer

Q: Suppose the yield curve is flat at 6%. Consider a 4

Suppose the yield curve is flat at 6%. Consider a 4-year 5%-coupon bond and an 8-year 7%-coupon bond. All coupons are annual. a. What are the prices and durations of both bonds? b. Consider buying one...

See Answer

Q: Let S = $40, σ = 0.30,

Let S = $40, σ = 0.30, r = 0.08, T = 1, and δ = 0. Also let Q = $40, σQ = 0.30, δQ = 0, and ρ = 1. Consider an exchange call with S as the price of the underlying asset and Q as the price of the strik...

See Answer

Q: Suppose a security has a bid price of $100 and an

Suppose a security has a bid price of $100 and an ask price of $100.12. At what price can the market-maker purchase a security? At what price can a market-maker sell a security? What is the spread in...

See Answer

Q: Suppose you observe the following zero-coupon bond prices per $

Suppose you observe the following zero-coupon bond prices per $1 of maturity payment: 0.96154 (1-year), 0.91573 (2-year), 0.87630 (3-year), 0.82270 (4-year), 0.77611 (5-year). For each maturity year c...

See Answer

Q: Let S = $100, K = $95, σ

Let S = $100, K = $95, σ = 30%, r = 8%, T = 1, and δ = 0. Let u = 1.3, d = 0.8, and n = 2. Construct the binomial tree for a call option. At each node provide the premium, ∆, and B.

See Answer

Q: A lender plans to invest $100m for 150 days, 60

A lender plans to invest $100m for 150 days, 60 days from today. (That is, if today is day 0, the loan will be initiated on day 60 and will mature on day 210.) The implied forward rate over 150 days,...

See Answer

Q: Consider a one-period binomial model with h = 1,

Consider a one-period binomial model with h = 1, where S = $100, r = 0, σ = 30%, and δ = 0.08. Compute American call option prices for K = $70, $80, $90, and $100. a. At which strike(s) does early exe...

See Answer

Q: Repeat Problem 11.4, only set r = 0 and

Repeat Problem 11.4, only set r = 0 and δ = 0.08. What is the lowest strike price (if there is one) at which early exercise will occur? If early exercise never occurs, explain why not. For the followi...

See Answer

Q: Suppose that S = $100, K = $100,

Suppose that S = $100, K = $100, r = 0.08, σ = 0.30, δ = 0, and T = 1. Construct a standard two-period binomial stock price tree using the method in Chapter 10. a. Consider stock price averages comput...

See Answer

Q: Suppose XYZ is a non-dividend-paying stock. Suppose

Suppose XYZ is a non-dividend-paying stock. Suppose S = $100, σ = 40%, δ = 0, and r = 0.06. a. What is the price of a 105-strike call option with 1 year to expiration? b. What is the 1-year forward pr...

See Answer

Q: Let S = $40, K = $45, σ

Let S = $40, K = $45, σ = 0.30, r = 0.08, T = 1, and δ = 0. a. What is the price of a standard call? b. What is the price of a knock-in call with a barrier of $44? Why? c. What is the price of a knock...

See Answer

Q: Suppose call and put prices are given by / What

Suppose call and put prices are given by What no-arbitrage property is violated? What spread position would you use to effect arbitrage? Demonstrate that the spread position is an arbitrage.

See Answer

Q: Let S = $100, K = $120, σ

Let S = $100, K = $120, σ = 30%, r = 0.08, and δ = 0. a. Compute the Black-Scholes call price for 1 year to maturity and for a variety of very long times to maturity. What happens to the option price...

See Answer

Q: Suppose you observe the following par coupon bond yields: 0.

Suppose you observe the following par coupon bond yields: 0.03000 (1-year), 0.03491 (2-year), 0.03974 (3-year), 0.04629 (4-year), 0.05174 (5-year). For each maturity year compute the zero-coupon bond...

See Answer

Q: Use a spreadsheet to verify the option prices in Examples 12.

Use a spreadsheet to verify the option prices in Examples 12.1 and 12.2.

See Answer

Q: Consider the same 3-year swap. Suppose you are a

Consider the same 3-year swap. Suppose you are a dealer who is paying the fixed oil price and receiving the floating price. Suppose that you enter into the swap and immediately thereafter all interest...

See Answer

Q: Suppose the 1-year copper forward price were $0.

Suppose the 1-year copper forward price were $0.80 instead of $1. If XYZ were to sell forward its expected copper production, what is its estimated profit 1 year from now? Should XYZ produce copper? W...

See Answer

Q: Consider AAAPI, the Nikkei ADR in disguise. To answer this

Consider AAAPI, the Nikkei ADR in disguise. To answer this question, use the information in Table 23.4. a. What is the volatility of Y, the price of AAAPI? b. What is the covariance between Y and x,...

See Answer

Q: Consider a perpetual call option with S = $50, K

Consider a perpetual call option with S = $50, K = $60, r = 0.06, σ = 0.40, and δ = 0.03. a. What is the price of the option and at what stock price should it be exercised? b. Suppose δ = 0.04 with al...

See Answer

Q: A $50 stock pays a $1 dividend every 3 months

A $50 stock pays a $1 dividend every 3 months, with the first dividend coming 3 months from today. The continuously compounded risk-free rate is 6%. a. What is the price of a prepaid forward contract...

See Answer

Q: Verify that equation (21.12) satisfies the Black-

Verify that equation (21.12) satisfies the Black-Scholes equation. What is the boundary condition for which this is a solution?

See Answer

Q: Using the information in the previous problem, find the price of

Using the information in the previous problem, find the price of a 5-year coupon bond that has a par payment of $1,000.00 and annual coupon payments of $60.00.

See Answer

Q: Suppose you short the S&R index for $1000 and

Suppose you short the S&R index for $1000 and buy a 950-strike call. Construct payoff and profit diagrams for this position. Verify that you obtain the same payoff and profit diagram by borrowing $931...

See Answer

Q: Suppose that oil forward prices for 1 year, 2 years,

Suppose that oil forward prices for 1 year, 2 years, and 3 years are $20, $21, and $22. The 1-year effective annual interest rate is 6.0%, the 2-year interest rate is 6.5%, and the 3-year interest rat...

See Answer

Q: Using Table 6.6, what is your best guess about

Using Table 6.6, what is your best guess about the current price of gold per ounce?

See Answer

Q: Supposing the effective quarterly interest rate is 1.5%, what

Supposing the effective quarterly interest rate is 1.5%, what are the per-barrel swap prices for 4-quarter and 8-quarter oil swaps? (Use oil forward prices in Table 8.9.) What is the total cost of pre...

See Answer

Q: Let S = $100,K = $95, r

Let S = $100,K = $95, r = 8%, T = 0.5, and δ = 0. Let u = 1.3, d = 0.8, and n = 1. a. Verify that the price of a European call is $16.196. b. Suppose you observe a call price of $17. What is the arbit...

See Answer

Q: a. Suppose the March Year 1 forward price were $3

a. Suppose the March Year 1 forward price were $3.10. Describe two different transactions you could use to undertake arbitrage. b. Suppose the September Year 1 forward price fell to $2.70 and subseque...

See Answer

Q: Suppose that top executives of XYZ are told they will receive at

Suppose that top executives of XYZ are told they will receive at-the-money call options on 10,000 shares each year for the next 3 years. When granted, the options have 5 years to maturity. XYZâ&...

See Answer

Q: Suppose you sell a 40-strike put with 91 days to

Suppose you sell a 40-strike put with 91 days to expiration. What is delta? If the option is on 100 shares, what investment is required for a delta-hedged portfolio? What is your overnight profit if t...

See Answer

Q: Suppose you observe the prices {5, 4, 5,

Suppose you observe the prices {5, 4, 5, 6, 5}. What are the arithmetic and geometric averages? Now you observe {3, 4, 5, 6, 7}. What are the two averages? What happens to the difference between the t...

See Answer

Q: Suppose the effective semiannual interest rate is 3%. a.

Suppose the effective semiannual interest rate is 3%. a. What is the price of a bond that pays one unit of the S&P index in 3 years? b. What semiannual dollar coupon is required if the bond is to sell...

See Answer

Q: There is a single debt issue. Compute the yield on this

There is a single debt issue. Compute the yield on this debt assuming that it matures in 1 year and has a maturity value of $127.42, 2 years with a maturity value of $135.30, 5 years with a maturity v...

See Answer

Q: : Suppose that copper costs $3.00 today and the

Suppose that copper costs $3.00 today and the continuously compounded lease rate for copper is 5%. The continuously compounded interest rate is 10%. The copper price in 1 year is uncertain and copper...

See Answer

Q: Suppose the gold spot price is $300/oz, the

Suppose the gold spot price is $300/oz, the 1-year forward price is 310.686, and the continuously compounded risk-free rate is 5%. a. What is the lease rate? b. What is the return on a cash-and-carry...

See Answer

Q: You have a project costing $1.50 that will produce

You have a project costing $1.50 that will produce two widgets, one each the first and second years after project completion. Widgets today cost $0.80 each, with the price growing at 2% per year. The...

See Answer

Q: Let S = $100,K = $95, r

Let S = $100,K = $95, r = 8%, T = 0.5, and δ = 0. Let u = 1.3, d = 0.8, and n = 1. a. Verify that the price of a European put is $7.471. b. Suppose you observe a put price of $8. What is the arbitrage...

See Answer

Q: The price of a 6-month dollar-denominated call option

The price of a 6-month dollar-denominated call option on the euro with a $0.90 strike is $0.0404. The price of an otherwise equivalent put option is $0.0141. The annual continuously compounded dollar...

See Answer

Q: The premium of a 100-strike yen-denominated put on

The premium of a 100-strike yen-denominated put on the euro is ¥8.763. The current exchange rate is 95 ¥/=C. What is the strike of the corresponding euro-denominated yen call, and what is its premium?...

See Answer

Q: Assuming that the stock price satisfies equation (20.20),

Assuming that the stock price satisfies equation (20.20), verify that Ke−r(T−t) +S(t)e−δ(T−t) satisfies the Black-Scholes equation, where K is a constant. What is the boundary condition for which this...

See Answer

Q: You draw these five numbers from a standard normal distribution: {−

You draw these five numbers from a standard normal distribution: {−1.7, 0.55, −0.3, −0.02, .85}. What are the equivalent draws from a normal distribution with mean 0.8 and variance 25?

See Answer

Q: Let ui∼ U (0, 1). Compute  −

Let ui∼ U (0, 1). Compute  − 6, 1000 times. (This will use 12,000 random numbers.) Construct a histogram and compare it to a theoretical standard normal density. What are the mean and standard deviat...

See Answer

Q: A mine costing $1000 will produce 1 ounce of gold per

A mine costing $1000 will produce 1 ounce of gold per year forever at a marginal extraction cost of $250, with production commencing 1 year after the mine opens. Gold volatility is zero. What is the v...

See Answer

Q: Construct payoff and profit diagrams for the purchase of a 950-

Construct payoff and profit diagrams for the purchase of a 950-strike S&R call and sale of a 1000-strike S&R call. Verify that you obtain exactly the same profit diagram for the purchase of a 950-stri...

See Answer

Q: Assume r = 8%, σ = 30%, δ = 0

Assume r = 8%, σ = 30%, δ = 0. Using 1-year-to-expiration European options, construct a position where you sell two 80-strike puts, buy one 95-strike put, buy one 105-strike call, and sell two 120-str...

See Answer

Q: Suppose you invest in the S&R index for $1000

Suppose you invest in the S&R index for $1000, buy a 950-strike put, and sell a 1050- strike call. Draw a profit diagram for this position. What is the net option premium? If you wanted to construct a...

See Answer

Q: Use Itˆo’s Lemma to evaluate dS2. For the following four

Use Itˆo’s Lemma to evaluate dS2. For the following four problems, use Itˆo’s Lemma to determine the process followed by the specified equatio...

See Answer

Q: Verify that you earn the same profit and payoff by (a

Verify that you earn the same profit and payoff by (a) buying the S&R index for $1000 and (b) buying a 950-strike S&R call, selling a 950-strike S&R put, and lending $931.37.

See Answer

Q: Verify that ASaeγt satisfies the Black-Scholes PDE for 

Verify that ASaeγt satisfies the Black-Scholes PDE for 

See Answer

Q: Verify that you earn the same profit and payoff by (a

Verify that you earn the same profit and payoff by (a) shorting the S&R index for $1000 and (b) selling a 1050-strike S&R call, buying a 1050-strike put, and borrowing $1029.41.

See Answer

Q: Use a change of numeraire and measure to verify that the value

Use a change of numeraire and measure to verify that the value of a claim paying K if ST

See Answer

Q: A barrier COD option is like a COD except that payment for

A barrier COD option is like a COD except that payment for the option occurs whenever a barrier is struck. Price a barrier COD put for the same values as in the previous problem, with a barrier of $95...

See Answer

Q: Compute daily volatilities for 1991 through 2004 for IBM, Xerox,

Compute daily volatilities for 1991 through 2004 for IBM, Xerox, and the S&P 500 index. Annualize by multiplying by √252. How do your answers compare to those in Problem 24.1? Answer Problem 24.1 Her...

See Answer

Q: Suppose that S1 and S2 follow geometric Brownian motion and pay continuous

Suppose that S1 and S2 follow geometric Brownian motion and pay continuous proportional dividends at the rates δ1 and δ2. Use the martingale argument to show that the value of a claim paying S1(T ) if...

See Answer

Q: a. What is the 2-year forward price for a

a. What is the 2-year forward price for a 1-year bond? b. What is the price of a call option that expires in 2 years, giving you the right to pay $0.90 to buy a bond expiring in 1 year? c. What is the...

See Answer

Q: Suppose you short the S&R index for $1000 and

Suppose you short the S&R index for $1000 and buy a 1050-strike call. Construct payoff and profit diagrams for this position. Verify that you obtain the same payoff and profit diagram by borrowing $10...

See Answer

Q: Suppose the firm issues a single zero-coupon bond with maturity

Suppose the firm issues a single zero-coupon bond with maturity value $100. a. Compute the yield, probability of default, and expected loss given default for times to maturity of 1, 2, 3, 4, 5, 10, an...

See Answer

Q: Repeat the previous problem, but set φ = 0.05

Repeat the previous problem, but set φ = 0.05. Be sure that you simulate the risk neutral process, obtained by including the risk premium in the interest rate process. Repeat the previous problem Usi...

See Answer

Q: Suppose the businesses in the previous problem use futures contracts to hedge

Suppose the businesses in the previous problem use futures contracts to hedge their temperature-related risk. Who do you think might accept the opposite risk?

See Answer

Q: Construct Table 5.1 from the perspective of a seller,

Construct Table 5.1 from the perspective of a seller, providing a descriptive name for each of the transactions.

See Answer

Q: Repeat the previous problem supposing that the brokerage fee is quoted as

Repeat the previous problem supposing that the brokerage fee is quoted as 0.3% of the bid or ask price. Previous Problem ABC stock has a bid price of $40.95 and an ask price of $41.05. Assume there i...

See Answer

Q: Consider the bonds in Example 7.8. What hedge ratio

Consider the bonds in Example 7.8. What hedge ratio would have exactly hedged the portfolio if interest rates had decreased by 25 basis points? Increased by 25 basis points? Repeat assuming a 50-basis...

See Answer

Q: Using the base case parameters, plot the implied volatility curve you

Using the base case parameters, plot the implied volatility curve you obtain for the base case against that for the case where there is a jump to zero, with the same λ.

See Answer

Q: Obtain at least 5 years of daily data for at least three

Obtain at least 5 years of daily data for at least three stocks and, if you can, one currency. Estimate annual volatility for each year for each asset in your data. What do you observe about the patte...

See Answer

Q: a. Suppose you enter into a short 6-month forward

a. Suppose you enter into a short 6-month forward position at a forward price of $50. What is the payoff in 6 months for prices of $40, $45, $50, $55, and $60? b. Suppose you buy a 6-month put option...

See Answer

Q: What position is the opposite of a purchased call? The opposite

What position is the opposite of a purchased call? The opposite of a purchased put?

See Answer

Q: Suppose you desire to short-sell 400 shares of JKI stock

Suppose you desire to short-sell 400 shares of JKI stock, which has a bid price of $25.12 and an ask price of $25.31. You cover the short position 180 days later when the bid price is $22.87 and the a...

See Answer

Q: Compute estimated profit in 1 year if XYZ buys a put option

Compute estimated profit in 1 year if XYZ buys a put option with a strike of $0.95, $1.00, or $1.05. Draw a graph of profit in each case.

See Answer

Q: A $50 stock pays an 8% continuous dividend. The

A $50 stock pays an 8% continuous dividend. The continuously compounded risk free rate is 6%. a. What is the price of a prepaid forward contract that expires 1 year from today? b. What is the price of...

See Answer

Q: The exchange rate is ¥95/=C, the yen-

The exchange rate is ¥95/=C, the yen-denominated interest rate is 1.5%, the euro-denominated interest rate is 3.5%, and the exchange rate volatility is 10%. a. What is the price of a 90-strike yen-den...

See Answer

Q: Suppose you enter into a put ratio spread where you buy a

Suppose you enter into a put ratio spread where you buy a 45-strike put and sell two 40-strike puts, both with 91 days to expiration. Compute and graph the 1-day holding period profit if you delta- an...

See Answer

Q: Consider the example of Auric. a. Suppose that Auric

Consider the example of Auric. a. Suppose that Auric insures against a price increase by purchasing a 440-strike call. Verify by drawing a profit diagram that simultaneously selling a 400- strike put...

See Answer

Q: Repeat the previous problem for debt instead of equity. Previous

Repeat the previous problem for debt instead of equity. Previous Problem Suppose there is a single 5-year zero-coupon debt issue with a maturity value of $120. The expected return on assets is 12%. Wh...

See Answer

Q: Suppose you invest in the S&R index for $1000

Suppose you invest in the S&R index for $1000, buy a 950-strike put, and sell a 1107- strike call. Draw a profit diagram for this position. How close is this to a zero-cost collar?

See Answer

Q: If XYZ does nothing to manage copper price risk, what is

If XYZ does nothing to manage copper price risk, what is its profit 1 year from now, per pound of copper? If on the other hand XYZ sells forward its expected copper production, what is its estimated p...

See Answer

Q: Again consider the widget investment problem in Section 17.1.

Again consider the widget investment problem in Section 17.1. Verify that with S = $50, K = $30, r = 0.04879, σ = 0, and δ = 0.009569, the perpetual call price is $30.597 and exercise optimally occurs...

See Answer

Q: Swaps often contain caps or floors. In this problem, you

Swaps often contain caps or floors. In this problem, you are to construct an oil contract that has the following characteristics: The initial cost is zero. Then in each period, the buyer pays the mark...

See Answer

Q: Heating degree-day and cooling degree-day futures contracts make

Heating degree-day and cooling degree-day futures contracts make payments based on whether the temperature is abnormally hot or cold. Explain why the following businesses might be interested in such a...

See Answer

Q: Verify that S(t)e−δ(T−

Verify that S(t)e−δ(T−t)N(d1) satisfies the Black-Scholes equation.

See Answer

Q: Under the same assumptions as the previous problem, show that the

Under the same assumptions as the previous problem, show that the value of a claim paying is  where σ2, δ1, and δ2 are defined as in the previous problem. In the next set of problems you will use Mo...

See Answer

Q: Assume that S = $45, K = $40,

Assume that S = $45, K = $40, r = 0.05, δ = 0.02, and σ = 0.30. Using the up rebate formula (equation (23.21)), find the value of H that maximizes (H − K) × UR(S, σ, r , T , δ), for T = 1, 10, 100, 10...

See Answer

Q: Using Monte Carlo, simulate the process dr = a(b

Using Monte Carlo, simulate the process dr = a(b − r)dt + σdZ, assuming that r = 6%, a = 0.2, b = 0.08, φ = 0, and σ = 0.02. Compute the prices of 1-, 2-, and 3-year zero-coupon bonds, and verify that...

See Answer

Q: Consider production ratios of 2:1:1, 3:

Consider production ratios of 2:1:1, 3:2:1, and 5:3:2 for oil, gasoline, and heating oil. Assume that other costs are the same per gallon of processed oil. a. Which ratio maximizes the per-gallon prof...

See Answer

Q: Replicate the GARCH(1,1) estimation in Example 24

Replicate the GARCH(1,1) estimation in Example 24.2, using daily returns from on IBM from January 1999 to December 2003. Compare your estimates with and without the four largest returns.

See Answer

Q: Suppose you buy a 40–45 bull spread with 91 days

Suppose you buy a 40–45 bull spread with 91 days to expiration. If you delta-hedge this position, what investment is required? What is your overnight profit if the stock tomorrow is $39? What if the s...

See Answer

Q: If XYZ does nothing to manage copper price risk, what is

If XYZ does nothing to manage copper price risk, what is its profit 1 year from now, per pound of copper? If on the other hand XYZ sells forward its expected copper production, what is its estimated p...

See Answer

Q: Using the same information as the previous question, draw payoff and

Using the same information as the previous question, draw payoff and profit diagrams for a short position in the stock. Verify that profit is 0 at a price in 1 year of $55.

See Answer

Q: Suppose the firm issues a single zero-coupon bond.

Suppose the firm issues a single zero-coupon bond. a. Suppose the maturity value of the bond is $80. Compute the yield and default probability for times to maturity of 1, 2, 3, 4, 5, 10, and 20 years....

See Answer

Q: Suppose that you buy the S&R index for $1000

Suppose that you buy the S&R index for $1000, buy a 1000-strike put, and borrow $980.39. Perform a payoff and profit calculation mimicking Table 3.1. Graph the resulting payoff and profit diagrams...

See Answer

Q: Use Itˆo’s Lemma to evaluate d[ln(S)].

Use Itˆo’s Lemma to evaluate d[ln(S)]. For the following four problems, use Itˆo’s Lemma to determine the process followed by the specified eq...

See Answer

Q: Repeat the previous problem, only use Monte Carlo simulation.

Repeat the previous problem, only use Monte Carlo simulation. Repeat the previous problem Using the delta-approximation method and assuming a $10m investment in stock A, compute the 95% and 99% 1-, 1...

See Answer

Q: Suppose that you short the S&R index for $1000

Suppose that you short the S&R index for $1000 and sell a 1000-strike put. Construct a table mimicking Table 3.1 that summarizes the payoff and profit of this position. Verify that your table matc...

See Answer

Q: The current price of oil is $32.00 per barrel

The current price of oil is $32.00 per barrel. Forward prices for 3, 6, 9, and 12 months are $31.37, $30.75, $30.14, and $29.54. Assuming a 2% continuously compounded annual risk-free rate, what is th...

See Answer

Q: Suppose you buy theS&Rindex for $1000 and buy a

Suppose you buy theS&Rindex for $1000 and buy a 950-strike put. Construct payoff and profit diagrams for this position. Verify that you obtain the same payoff and profit diagram by investing $931.37 i...

See Answer

Q: Given a continuously compounded risk-free rate of 3% annually

Given a continuously compounded risk-free rate of 3% annually, at what lease rate will forward prices equal the current commodity price? (Recall the copper example in Section 6.3.) If the lease rate w...

See Answer

Q: The firm has a single outstanding debt issue with a promised maturity

The firm has a single outstanding debt issue with a promised maturity payment of $120 in 5 years. What is the probability of bankruptcy? What is the credit spread?

See Answer

Q: Suppose the premium on a 6-month S&R call

Suppose the premium on a 6-month S&R call is $109.20 and the premium on a put with the same strike price is $60.18. What is the strike price?

See Answer

Q: Repeat the previous problem, assuming that the dividend yield is 1

Repeat the previous problem, assuming that the dividend yield is 1.5%. Previous Problem Suppose you are a market-maker in S&R index forward contracts. The S&R index spot price is 1100, the risk-free...

See Answer

Q: Suppose that price and quantity are positively correlated as in this table

Suppose that price and quantity are positively correlated as in this table: There is a 50% chance of either price. The futures price is $2.50. Demonstrate the effect of hedging if we do the following:...

See Answer

Q: Compute estimated profit in 1 year if XYZ buys paylater puts as

Compute estimated profit in 1 year if XYZ buys paylater puts as follows (the net premium may not be exactly zero): a. Sell one 1.025-strike put and buy two 0.975-strike puts. b. Sell two 1.034-strike...

See Answer

Q: Look back to the numerical example graphed in Figure 1A.

Look back to the numerical example graphed in Figure 1A.1. Suppose the interest rate is 20%. What would the ant (A) and grasshopper (G) do if they both start with $100,000? Would they invest in their...

See Answer

Q: Why might one expect managers to act in shareholders’ interests?

Why might one expect managers to act in shareholders’ interests? Give some reasons.

See Answer

Q: Mexican Motors’ market cap is 200 billion pesos. Next year’s free

Mexican Motors’ market cap is 200 billion pesos. Next year’s free cash flow is 8.5 billion pesos. Security analysts are forecasting that free cash flow will grow by 7.5% per year for the next five yea...

See Answer

Q: Phoenix Corp. faltered in the recent recession but is recovering.

Phoenix Corp. faltered in the recent recession but is recovering. Free cash flow has grown rapidly. Forecasts made in 2016 are as follows. Phoenix’s recovery will be complete by 2021...

See Answer

Q: The constant-growth DCF formula: P0 = DIV1 /

The constant-growth DCF formula: P0 = DIV1 /r − g is sometimes written as: P0 = ROE (1 − b) BVPS / r − b ROE where BVPS is book equity value per share, b is the plowback ratio, and ROE is the r...

See Answer

Q: Portfolio managers are frequently paid a proportion of the funds under management

Portfolio managers are frequently paid a proportion of the funds under management. Suppose you manage a $100 million equity portfolio offering a dividend yield (DIV1/P0) of 5%. Dividends and portfolio...

See Answer

Q: Company X is expected to pay an end-of-year

Company X is expected to pay an end-of-year dividend of $5 a share. After the dividend its stock is expected to sell at $110. If the market capitalization rate is 8%, what is the current stock price?...

See Answer

Q: Company Y does not plow back any earnings and is expected to

Company Y does not plow back any earnings and is expected to produce a level dividend stream of $5 a share. If the current stock price is $40, what is the market capitalization rate?

See Answer

Q: Company Z’s earnings and dividends per share are expected to grow indefinitely

Company Z’s earnings and dividends per share are expected to grow indefinitely by 5% a year. If next year’s dividend is $10 and the market capitalization rate is 8%, what is the current stock price?

See Answer

Q: If company Z (see Problem 5) were to distribute all

If company Z (see Problem 5) were to distribute all its earnings, it could maintain a level dividend stream of $15 a share. How much is the market actually paying per share for growth opportunities?...

See Answer

Q: Under what conditions does r, a stock’s market capitalization rate,

Under what conditions does r, a stock’s market capitalization rate, equal its earnings–price ratio EPS1/P0?

See Answer

Q: What do financial managers mean by “free cash flow”? How

What do financial managers mean by “free cash flow”? How is free cash flow calculated? Briefly explain.

See Answer

Q: Many firms have devised defenses that make it more difficult or costly

Many firms have devised defenses that make it more difficult or costly for other firms to take them over. How might such defenses affect the firm’s agency problems? Are managers of firms with formidab...

See Answer

Q: What is meant by the “horizon value” of a business

What is meant by the “horizon value” of a business? How can it be estimated?

See Answer

Q: Suppose the horizon date is set at a time when the firm

Suppose the horizon date is set at a time when the firm will run out of positive-NPV investment opportunities. How would you calculate the horizon value?

See Answer

Q: Respond briefly to the following statement: “You say stock price

Respond briefly to the following statement: “You say stock price equals the present value of future dividends? That’s crazy! All the investors I know are looking for capital gains.”

See Answer

Q: Vegetron’s chief financial officer (CFO) is wondering how to analyze

Vegetron’s chief financial officer (CFO) is wondering how to analyze a proposed $1 million investment in a new venture code-named project X. He asks what you think. Your response sho...

See Answer

Q: Write down the equation defining a project’s internal rate of return (

Write down the equation defining a project’s internal rate of return (IRR). In practice how is IRR calculated?

See Answer

Q: Calculate the net present value of the following project for discount rates

Calculate the net present value of the following project for discount rates of 0, 50, and 100%: What is the IRR of the project?

See Answer

Q: You have the chance to participate in a project that produces the

You have the chance to participate in a project that produces the following cash flows: The internal rate of return is 13%. If the opportunity cost of capital is 10%, would you accept the offer?

See Answer

Q: Consider a project with the following cash flows: /

Consider a project with the following cash flows: a. How many internal rates of return does this project have? b. Which of the following numbers is the project IRR: (i) –50%; (ii)...

See Answer

Q: Consider projects Alpha and Beta: / The opportunity cost

Consider projects Alpha and Beta: The opportunity cost of capital is 8%. Suppose you can undertake Alpha or Beta, but not both. Use the IRR rule to make the choice.

See Answer

Q: Suppose you have the following investment opportunities, but only $90

Suppose you have the following investment opportunities, but only $90,000 available for investment. Which projects should you take?

See Answer

Q: Read the following passage: “Companies usually buy (a

Read the following passage: “Companies usually buy (a) assets. These include both tangible assets such as (b) and intangible assets such as (c). To pay for these assets, they sell (d) assets suc...

See Answer

Q: Consider the following projects: / a. If the

Consider the following projects: a. If the opportunity cost of capital is 10%, which projects have a positive NPV? b. Calculate the payback period for each project. c. Which project(s) would a firm us...

See Answer

Q: Which of the following are real assets, and which are financial

Which of the following are real assets, and which are financial? a. A share of stock. b. A personal IOU. c. A trademark. d. A factory. e. Undeveloped land. f. The balance in the firm’s checking...

See Answer

Q: An investment costs $1,548 and pays $138 in

An investment costs $1,548 and pays $138 in perpetuity. If the interest rate is 9%, what is the NPV?

See Answer

Q: The Borstal Company has to choose between two machines that do the

The Borstal Company has to choose between two machines that do the same job but have different lives. The two machines have the following costs: These costs are expressed in real terms. a. Suppose yo...

See Answer

Q: Explain why we refer to the opportunity cost of capital, instead

Explain why we refer to the opportunity cost of capital, instead of just “cost of capital” or “discount rate.” While you’re at it, also explain the following statement: “The opportunity cost of capita...

See Answer

Q: Look again at your calculations for Problem 29. Suppose that technological

Look again at your calculations for Problem 29. Suppose that technological change is expected to reduce costs by 10% per year. There will be new machines in year 1 that cost 10% less to buy and operat...

See Answer

Q: The president’s executive jet is not fully utilized. You judge that

The president’s executive jet is not fully utilized. You judge that its use by other officers would increase direct operating costs by only $20,000 a year and would save $100,000 a year in airline bil...

See Answer

Q: One measure of the effective tax rate is the difference between the

One measure of the effective tax rate is the difference between the IRRs of pretax and after-tax cash flows, divided by the pretax IRR. Consider, for example, an investment I generating a perpetual st...

See Answer

Q: We warned that equivalent annual costs should be calculated in real terms

We warned that equivalent annual costs should be calculated in real terms. We did not fully explain why. This problem will show you. Look back to the cash flows for machines A and B (in “The Choice be...

See Answer

Q: Which of the following should be treated as incremental cash flows when

Which of the following should be treated as incremental cash flows when deciding whether to invest in a new manufacturing plant? The site is already owned by the company, but existing buildings would...

See Answer

Q: In 1898, Simon North announced plans to construct a funeral home

In 1898, Simon North announced plans to construct a funeral home on land he owned and rented out as a storage area for railway carts. (A local newspaper commended Mr. North for not putting the car...

See Answer

Q: IRR rule The Titanic Shipbuilding Company has a noncancelable contract to build

IRR rule The Titanic Shipbuilding Company has a noncancelable contract to build a small cargo vessel. Construction involves a cash outlay of $250,000 at the end of each of the next two years. At the e...

See Answer

Q: In the early 1990s, the California Air Resources Board (CARB

In the early 1990s, the California Air Resources Board (CARB) started planning its “Phase 2” requirements for reformulated gasoline (RFG). RFG is gasoline blended to tight specifications designed to r...

See Answer

Q: In most large corporations, ownership and management are separated. What

In most large corporations, ownership and management are separated. What are the main implications of this separation?

See Answer

Q: The following table shows the nominal returns on the U.S

The following table shows the nominal returns on the U.S. stocks and the rate of inflation. a. What was the standard deviation of the nominal market returns? b. Calculate the arithmetic average real...

See Answer

Q: During the boom years of 2010–2014, ace mutual fund

During the boom years of 2010–2014, ace mutual fund manager Diana Sauros produced the following percentage rates of return. Rates of return on the market are given for comparison.  Calculate the ave...

See Answer

Q: True or false? a. Investors prefer diversified companies

True or false? a. Investors prefer diversified companies because they are less risky. b. If stocks were perfectly positively correlated, diversification would not reduce risk. c. Diversification o...

See Answer

Q: A factory costs $400,000. It will produce

A factory costs $400,000. It will produce an inflow after operating costs of $100,000 in year 1, $200,000 in year 2, and $300,000 in year 3. The opportunity cost of capital is 12%. Show your calculat...

See Answer

Q: In which of the following situations would you get the largest reduction

In which of the following situations would you get the largest reduction in risk by spreading your investment across two stocks? a. The two shares are perfectly correlated. b. There is no correlatio...

See Answer

Q: To calculate the variance of a three-stock portfolio, you

To calculate the variance of a three-stock portfolio, you need to add nine boxes:  Use the same symbols that we used in this chapter; for example, x1 = proportion invested in stock 1 and σ12 = covar...

See Answer

Q: Suppose the standard deviation of the market return is 20%.

Suppose the standard deviation of the market return is 20%. a. What is the standard deviation of returns on a well-diversified portfolio with a beta of 1.3? b. What is the standard deviation of retu...

See Answer

Q: A portfolio contains equal investments in 10 stocks. Five have

A portfolio contains equal investments in 10 stocks. Five have a beta of 1.2; the remainder have a beta of 1.4. What is the portfolio beta? a. 1.3. b. Greater than 1.3 because the portfolio is not co...

See Answer

Q: What is the beta of each of the stocks shown in table

What is the beta of each of the stocks shown in table 

See Answer

Q: Look again at projects D and E in Section 5-3

Look again at projects D and E in Section 5-3. Assume that the projects are mutually exclusive and that the opportunity cost of capital is 10%. a. Calculate the profitability index for each project. b...

See Answer

Q: Consider the following capital rationing problem: / Set up

Consider the following capital rationing problem: Set up this problem as a linear program and solve it. You can allow partial investments, that is, 0 ≤ x ≤ 1. C...

See Answer

Q: Here are inflation rates and U.S. stock market and

Here are inflation rates and U.S. stock market and Treasury bill returns between 1929 and 1933:  a. What was the real return on the stock market in each year? b. What was the arithmatic average real...

See Answer

Q: Each of the following statements is dangerous or misleading. Explain why

Each of the following statements is dangerous or misleading. Explain why. a. A long-term United States government bond is always absolutely safe. b. All investors should prefer stocks to bonds becau...

See Answer

Q: Hippique s.a., which owns a stable of racehorses,

Hippique s.a., which owns a stable of racehorses, has just invested in a mysterious black stallion with great form but disputed bloodlines. Some experts in horseflesh predict the horse will win the co...

See Answer

Q: Lonesome Gulch Mines has a standard deviation of 42% per year

Lonesome Gulch Mines has a standard deviation of 42% per year and a beta of +.10. Amalgamated Copper has a standard deviation of 31% a year and a beta of +.66. Explain why Lonesome Gulch is the safer...

See Answer

Q: Hyacinth Macaw invests 60% of her funds in stock I and

Hyacinth Macaw invests 60% of her funds in stock I and the balance in stock J. The standard deviation of returns on I is 10%, and on J it is 20%. Calculate the variance of portfolio returns, assuming...

See Answer

Q: Halcyon Lines is considering the purchase of a new bulk carrier

Halcyon Lines is considering the purchase of a new bulk carrier for $8 million. The forecasted revenues are $5 million a year and operating costs are $4 million. A major refit costing $2 million will...

See Answer

Q: a. How many variance terms and how many different covariance terms

a. How many variance terms and how many different covariance terms do you need to calculate the risk of a 100-share portfolio? b. Suppose all stocks had a standard deviation of 30% and a correlation...

See Answer

Q: Your eccentric Aunt Claudia has left you $50,000 in

Your eccentric Aunt Claudia has left you $50,000 in BP shares plus $50,000 cash. Unfortunately her will requires that the BP stock not be sold for one year and the $50,000 cash must be entirely invest...

See Answer

Q: There are few, if any, real companies with negative betas

There are few, if any, real companies with negative betas. But suppose you found one with β = –.25. a. How would you expect this stock’s rate of return to change if the overall market rose by an extr...

See Answer

Q: You can form a portfolio of two assets, A and B

You can form a portfolio of two assets, A and B, whose returns have the following characteristics:  If you demand an expected return of 12%, what are the portfolio weights? What is the portfolio’s st...

See Answer

Q: A common stock will pay a cash dividend of $4 next

A common stock will pay a cash dividend of $4 next year. After that, the dividends are expected to increase indefinitely at 4% per year. If the discount rate is 14%, what is the PV of the stream of di...

See Answer

Q: Borgia Pharmaceuticals has $1 million allocated for capital expenditures. Which

Borgia Pharmaceuticals has $1 million allocated for capital expenditures. Which of the following projects should the company accept to stay within the $1 million budget? How much does the budget limit...

See Answer

Q: Here are some historical data on the risk characteristics of Bank of

Here are some historical data on the risk characteristics of Bank of America and Starbucks:  Assume the standard deviation of the return on the market was 23.0%. a. The correlation coefficient of Ba...

See Answer

Q: Suppose that Treasury bills offer a return of about 6% and

Suppose that Treasury bills offer a return of about 6% and the expected market risk premium is 8.5%. The standard deviation of Treasury-bill returns is zero and the standard deviation of market return...

See Answer

Q: Calculate the beta of each of the stocks in Table 7.

Calculate the beta of each of the stocks in Table 7.9 relative to a portfolio with equal investments in each stock. 

See Answer

Q: Table 7.9 shows standard deviations and correlation coefficients for eight

Table 7.9 shows standard deviations and correlation coefficients for eight stocks from different countries. Calculate the variance of a portfolio with equal investments in each stock. 

See Answer

Q: A game of chance offers the following odds and payoffs. Each

A game of chance offers the following odds and payoffs. Each play of the game costs $100, so the net profit per play is the payoff less $100.  What are the expected cash payoff and expected rate of...

See Answer

Q: As winner of a breakfast cereal competition, you can choose one

As winner of a breakfast cereal competition, you can choose one of the following prizes: a. $100,000 now. b. $180,000 at the end of five years. c. $11,400 a year forever. d. $19,000 for each of 10 ye...

See Answer

Q: Answer this question by drawing graphs like Figure 1A.1.

Answer this question by drawing graphs like Figure 1A.1. Casper Milktoast has $200,000 on hand to support consumption in periods 0 (now) and 1 (next year). He wants to consume exactly the same amount...

See Answer

Q: Siegfried Basset is 65 years of age and has a life expectancy

Siegfried Basset is 65 years of age and has a life expectancy of 12 more years. He wishes to invest $20,000 in an annuity that will make a level payment at the end of each year until his death. If the...

See Answer

Q: David and Helen Zhang are saving to buy a boat at the

David and Helen Zhang are saving to buy a boat at the end of five years. If the boat costs $20,000 and they can earn 10% a year on their savings, how much do they need to put aside at the end of years...

See Answer

Q: The interest rate is 10%. a. What is

The interest rate is 10%. a. What is the PV of an asset that pays $1 a year in perpetuity? b. The value of an asset that appreciates at 10% per annum approximately doubles in seven years. What is t...

See Answer

Q: Kangaroo Autos is offering free credit on a new $10

Kangaroo Autos is offering free credit on a new $10,000 car. You pay $1,000 down and then $300 a month for the next 30 months. Turtle Motors next door does not offer free credit but will give you $1,...

See Answer

Q: Some people believe firmly, even passionately, that ranking projects on

Some people believe firmly, even passionately, that ranking projects on IRR is OK if each project’s cash flows can be reinvested at the project’s IRR. They also say that the NPV rule “assumes that cas...

See Answer

Q: Recalculate the NPV of the office building venture in Example 2.

Recalculate the NPV of the office building venture in Example 2.1 at interest rates of 5, 10, and 15%. Plot the points on a graph with NPV on the vertical axis and the discount rates on the horizontal...

See Answer

Q: If the interest rate is 7%, what is the value of

If the interest rate is 7%, what is the value of the following three investments? a. An investment that offers you $100 a year in perpetuity with the payment at the end of each year. b. A similar in...

See Answer

Q: Refer to Sections 2-3 through 2-4. If

Refer to Sections 2-3 through 2-4. If the rate of interest is 8% rather than 10%, how much would you need to set aside to provide each of the following? a. $1 billion at the end of each year in perpe...

See Answer

Q: How much will you have at the end of 20 years

How much will you have at the end of 20 years if you invest $100 today at 15% annually compounded? How much will you have if you invest at 15% continuously compounded?

See Answer

Q: You have just read an advertisement stating, “Pay us $

You have just read an advertisement stating, “Pay us $100 a year for 10 years and we will pay you $100 a year thereafter in perpetuity.” If this is a fair deal, what is the rate of interest?

See Answer

Q: Which would you prefer? a. An investment paying interest

Which would you prefer? a. An investment paying interest of 12% compounded annually. b. An investment paying interest of 11.7% compounded semiannually. c. An investment paying 11.5% compounded contin...

See Answer

Q: A leasing contract calls for an immediate payment of $100,

A leasing contract calls for an immediate payment of $100,000 and nine subsequent $100,000 semiannual payments at six-month intervals. What is the PV of these payments if the annual discount rate is 8...

See Answer

Q: Vocabulary test. Explain the differences between: a. Real

Vocabulary test. Explain the differences between: a. Real and financial assets. b. Capital budgeting and financing decisions. c. Closely held and public corporations. d. Limited and unlimited liab...

See Answer

Q: The following statements are true. Explain why. a.

The following statements are true. Explain why. a. If a bond’s coupon rate is higher than its yield to maturity, then the bond will sell for more than face value. b. If a bond’s coupon rate is lower...

See Answer

Q: Several years ago The Wall Street Journal reported that the winner of

Several years ago The Wall Street Journal reported that the winner of the Massachusetts State Lottery prize had the misfortune to be both bankrupt and in prison for fraud. The prize was $9,420,713, to...

See Answer

Q: A mortgage requires you to pay $70,000 at the

A mortgage requires you to pay $70,000 at the end of each of the next eight years. The interest rate is 8%. a. What is the present value of these payments? b. Calculate for each year the loan balance...

See Answer

Q: Look again at the project cash flows in Problem 10. Calculate

Look again at the project cash flows in Problem 10. Calculate the modified IRR as defined in Footnote 5 in Section 5-3. Assume the cost of capital is 12%. Now try the following variation on the MIRR c...

See Answer

Q: You estimate that by the time you retire in 35 years,

You estimate that by the time you retire in 35 years, you will have accumulated savings of $2 million. If the interest rate is 8% and you live 15 years after retirement, what annual level of expenditu...

See Answer

Q: The annually compounded discount rate is 5.5%. You

The annually compounded discount rate is 5.5%. You are asked to calculate the present value of a 12-year annuity with payments of $50,000 per year. Calculate PV for each of the following cases. a. T...

See Answer

Q: Dear Financial Adviser, My spouse and I are each

Dear Financial Adviser, My spouse and I are each 62 and hope to retire in three years. After retirement we will receive $7,500 per month after taxes from our employers’ pension plans and $1,500 per...

See Answer

Q: Your firm’s geologists have discovered a small oil field in New York’s

Your firm’s geologists have discovered a small oil field in New York’s Westchester County. The field is forecasted to produce a cash flow of C1 = $2 million in the first year. You estimate that you co...

See Answer

Q: Suppose that you take out a $200,000, 20

Suppose that you take out a $200,000, 20-year mortgage loan to buy a condo. The interest rate on the loan is 6%, and payments on the loan are made annually at the end of each year. a. What is your an...

See Answer

Q: Here are two useful rules of thumb. The “Rule

Here are two useful rules of thumb. The “Rule of 72” says that with discrete compounding the time it takes for an investment to double in value is roughly 72/interest rate (in percent). The “Rule of...

See Answer

Q: You own an oil pipeline that will generate a $2 million

You own an oil pipeline that will generate a $2 million cash return over the coming year. The pipeline’s operating costs are negligible, and it is expected to last for a very long time. Unfortunately,...

See Answer

Q: True or false? a. All stocks in an equivalent

True or false? a. All stocks in an equivalent-risk class are priced to offer the same expected rate of return. b. The value of a share equals the PV of future dividends per share.

See Answer

Q: Each of the following formulas for determining shareholders’ required rate of return

Each of the following formulas for determining shareholders’ required rate of return can be right or wrong depending on the circumstances: a. r = DIV1 / P0 + g b. r = EPS1 /P0 For each formula const...

See Answer

Q: Alpha Corp’s earnings and dividends are growing at 15% per year

Alpha Corp’s earnings and dividends are growing at 15% per year. Beta Corp’s earnings and dividends are growing at 8% per year. The companies’ assets, earnings, and dividends per share are now (at dat...

See Answer

Q: True or false? a. A project’s depreciation tax shields

True or false? a. A project’s depreciation tax shields depend on the actual future rate of inflation. b. Project cash flows should take account of interest paid on any borrowing undertaken to finance...

See Answer

Q: Look again at the financial forecasts for Growth-Tech given in

Look again at the financial forecasts for Growth-Tech given in Table 4.4. This time assume you know that the opportunity cost of capital is r = .12 (discard the .09...

See Answer

Q: Compost Science, Inc. (CSI), is in the business

Compost Science, Inc. (CSI), is in the business of converting Boston’s sewage sludge into fertilizer. The business is not in itself very profitable. However, to induce CSI to remain in business, the M...

See Answer

Q: If you invest $100 at an interest rate of 15

If you invest $100 at an interest rate of 15%, how much will you have at the end of eight years?

See Answer

Q: Machines A and B are mutually exclusive and are expected to produce

Machines A and B are mutually exclusive and are expected to produce the following real cash flows: The real opportunity cost of capital is 10%. a. Calculate the NPV of each machine. b. Calculate the e...

See Answer

Q: A project requires an initial investment of $100,000 and

A project requires an initial investment of $100,000 and is expected to produce a cash inflow before tax of $26,000 per year for five years. Company A has substantial accumulated tax losses and is...

See Answer

Q: Construct a new version of Table 4.7, assuming that

Construct a new version of Table 4.7, assuming that competition drives down profitability (on existing assets as well as new investment) to 11.5% in year 6, 11% in year 7, 10.5% in year 8,...

See Answer

Q: Consider the following two mutually exclusive projects: / a

Consider the following two mutually exclusive projects: a. Calculate the NPV of each project for discount rates of 0%, 10%, and 20%. Plot these on a graph with NPV on the vertical axis and disco...

See Answer

Q: Go to the Excel spreadsheet versions of Tables 6.1,

Go to the Excel spreadsheet versions of Tables 6.1, 6.5, and 6.6 and answer the following questions. a. How does the guano project’s NPV change if IM&C is forced to use the seven-year MACRS tax...

See Answer

Q: A widget manufacturer currently produces 200,000 units a year.

A widget manufacturer currently produces 200,000 units a year. It buys widget lids from an outside supplier at a price of $2 a lid. The plant manager believes that it would be cheaper to make these li...

See Answer

Q: A machine costs $380,000 and is expected to

A machine costs $380,000 and is expected to produce the following cash flows:  If the cost of capital is 12%, what is the machine’s NPV?

See Answer

Q: If the cost of capital is 9%, what is the PV

If the cost of capital is 9%, what is the PV of $374 paid in year 9?

See Answer

Q: Reliable Electric is considering a proposal to manufacture a new type of

Reliable Electric is considering a proposal to manufacture a new type of industrial electric motor that would replace most of its existing product line. A research breakthrough has given Reliable a tw...

See Answer

Q: Marsha Jones has bought a used Mercedes horse transporter for her Connecticut

Marsha Jones has bought a used Mercedes horse transporter for her Connecticut estate. It cost $35,000. The object is to save on horse transporter rentals. Marsha had been renting a transporter every o...

See Answer

Q: a. If the one-year discount factor is .905

a. If the one-year discount factor is .905, what is the one-year interest rate? b. If the two-year interest rate is 10.5%, what is the two-year discount factor? c. Given these one- and two-year disc...

See Answer

Q: Machine C was purchased five years ago for $200,000

Machine C was purchased five years ago for $200,000 and produces an annual real cash flow of $80,000. It has no salvage value but is expected to last another five years. The company can replace machin...

See Answer

Q: United Pigpen is considering a proposal to manufacture high-protein hog

United Pigpen is considering a proposal to manufacture high-protein hog feed. The project would make use of an existing warehouse, which is currently rented out to a neighboring firm. The next year’s...

See Answer

Q: Hindustan Motors has been producing its Ambassador car in India since 1948

Hindustan Motors has been producing its Ambassador car in India since 1948. As the company’s website explains, the Ambassador’s “dependability, spaciousness, and comfort factor have made it the most p...

See Answer

Q: How does the PV of depreciation tax shields vary across the recovery

How does the PV of depreciation tax shields vary across the recovery-period classes shown in Table 6.4? Give a general answer; then check it by calculating the PVs of depreciation tax shields in the...

See Answer

Q: In the International Mulch and Compost example (Section 6-2

In the International Mulch and Compost example (Section 6-2), we assumed that losses on the project could be used to offset taxable profits elsewhere in the corporation. Suppose that the losses had to...

See Answer

Q: As a result of improvements in product engineering, United Automation is

As a result of improvements in product engineering, United Automation is able to sell one of its two milling machines. Both machines perform the same function but differ in age. The newer machine cou...

See Answer

Q: Low-energy lightbulbs typically cost $3.60, have

Low-energy lightbulbs typically cost $3.60, have a life of nine years, and use about $2.00 of electricity a year. Conventional lightbulbs are cheaper to buy, for they cost only $.60. On the other hand...

See Answer

Q: Mr. Cyrus Clops, the president of Giant Enterprises, has

Mr. Cyrus Clops, the president of Giant Enterprises, has to make a choice between two possible investments: The opportunity cost of capital is 9%. Mr. Clops is tempted to take B, which has the higher...

See Answer

Q: Hayden Inc. has a number of copiers that were bought four

Hayden Inc. has a number of copiers that were bought four years ago for $20,000. Currently maintenance costs $2,000 a year, but the maintenance agreement expires at the end of two years and thereafter...

See Answer

Q: Look at the last question where you calculated the equivalent annual cost

Look at the last question where you calculated the equivalent annual cost of producing reformulated gasoline in California. Capital investment was $400 million. Suppose this amount can be depreciated...

See Answer

Q: If the PV of $139 is $125, what

If the PV of $139 is $125, what is the discount factor?

See Answer

Q: What is the PV of $100 received in: a

What is the PV of $100 received in: a. Year 10 (at a discount rate of 1%)? b. Year 10 (at a discount rate of 13%)? c. Year 15 (at a discount rate of 25%)? d. Each of years 1 through 3 (at a discou...

See Answer

Q: a. An 8%, five-year bond yields 6%.

a. An 8%, five-year bond yields 6%. If this yield to maturity remains unchanged, what will be its price one year hence? Assume annual coupon payments and a face value of $100. b. What is the total re...

See Answer

Q: Restate the net cash flows in Table 6.6 in real

Restate the net cash flows in Table 6.6 in real terms. Discount the restated cash flows at a real discount rate. Assume a 20% nominal rate and 10% expected inflation. NPV should be unchanged at +3,802...

See Answer

Q: a. Longer-maturity bonds necessarily have longer durations.

a. Longer-maturity bonds necessarily have longer durations. b. The longer a bond’s duration, the lower its volatility. c. Other things equal, the lower the bond coupon, the higher its volatility....

See Answer

Q: Calculate the durations and volatilities of securities A, B, and

Calculate the durations and volatilities of securities A, B, and C. Their cash flows are shown below. The interest rate is 8%.

See Answer

Q: What is the payback period on each of the following projects?

What is the payback period on each of the following projects? Given that you wish to use the payback rule with a cutoff period of two years, which projects would you accept? c. If you use a cutoff per...

See Answer

Q: True or false? Explain. a. The value of

True or false? Explain. a. The value of a share equals the discounted stream of future earnings per share. b. The value of a share equals the PV of earnings per share assuming the firm does not grow...

See Answer

Q: Company Z-prime is like Z in all respects save one

Company Z-prime is like Z in all respects save one: Its growth will stop after year 4. In year 5 and afterward, it will pay out all earnings as dividends. What is Z-prime’s stock price? Assume next ye...

See Answer

Q: F&H Corp. continues to invest heavily in a declining

F&H Corp. continues to invest heavily in a declining industry. Here is an excerpt from a recent speech by F&H’s CFO: We at F&H have of course noted the complaints of a few spineless investors and unin...

See Answer

Q: A 10-year U.S. Treasury bond with a

A 10-year U.S. Treasury bond with a face value of $1,000 pays a coupon of 5.5% (2.75% of face value every six months). The reported yield to maturity is 5.2% (a six-month discount rate of 5.2/2 = 2.6%...

See Answer

Q: A project produces a cash flow of $432 in year 1

A project produces a cash flow of $432 in year 1, $137 in year 2, and $797 in year 3. If the cost of capital is 15%, what is the project’s PV? If the project requires an investment of $1,200, what is...

See Answer

Q: A six-year government bond makes annual coupon payments of 5

A six-year government bond makes annual coupon payments of 5% and offers a yield of 3% annually compounded. Suppose that one year later the bond still yields 3%. What return has the bondholder earned...

See Answer

Q: A 6% six-year bond yields 12% and a

A 6% six-year bond yields 12% and a 10% six-year bond yields 8%. Calculate the six-year spot rate. Assume annual coupon payments.

See Answer

Q: You have estimated spot rates as follows: r1 = 5.

You have estimated spot rates as follows: r1 = 5.00%, r2 = 5.40%, r3 = 5.70%, r4 = 5.90%, r5 = 6.00%. a. What are the discount factors for each date (that is, the present value of $1 paid in yea...

See Answer

Q: CSC is evaluating a new project to produce encapsulators. The initial

CSC is evaluating a new project to produce encapsulators. The initial investment in plant and equipment is $500,000. Sales of encapsulators in year 1 are forecasted at $200,000 and costs at $100,000....

See Answer

Q: Calculate durations and modified durations for the 3% bonds in Table 

Calculate durations and modified durations for the 3% bonds in Table 3.2. You can follow the procedure set out in Table 3.4 for the 9% coupon bonds. Confirm that modified durat...

See Answer

Q: The continuously compounded interest rate is 12%. a. You

The continuously compounded interest rate is 12%. a. You invest $1,000 at this rate. What is the investment worth after five years? b. What is the PV of $5 million to be received in eight years? c....

See Answer

Q: Find the spreadsheet for Table 3.4. in Connect.

Find the spreadsheet for Table 3.4. in Connect. Show how duration and volatility change if (a) the bond’s coupon is 8% of face value and (b) the bond’s yield is 6%. Explain your finding.

See Answer

Q: The formula for the duration of a perpetual bond that makes an

The formula for the duration of a perpetual bond that makes an equal payment each year in perpetuity is (1 + yield)/yield. If each bond yields 5%, which has the longer duration—a perpetual bond or a 1...

See Answer

Q: Look again at Table 3.5. Suppose the spot interest

Look again at Table 3.5. Suppose the spot interest rates change to the following downward-sloping term structure: r1 = 4.6%, r2 =Â&nbs...

See Answer

Q: When appraising mutually exclusive investments in plant and equipment, financial managers

When appraising mutually exclusive investments in plant and equipment, financial managers calculate the investments’ equivalent annual costs and rank the investments on this basis. Why is this necessa...

See Answer

Q: The Beyond the Page feature, “Goldman Sachs Causes a Ruckus

The Beyond the Page feature, “Goldman Sachs Causes a Ruckus,” describes the controversial involvement of Goldman Sachs in a mortgage-backed securities deal in 2006. When this involvement was revealed,...

See Answer

Q: Consider the following three stocks: a. Stock A is

Consider the following three stocks: a. Stock A is expected to provide a dividend of $10 a share forever. b. Stock B is expected to pay a dividend of $5 next year. Thereafter, dividend growth is exp...

See Answer

Q: Look at the spot interest rates shown in Problem 25. Suppose

Look at the spot interest rates shown in Problem 25. Suppose that someone told you that the five-year spot interest rate was 2.5%. Why would you not believe him? How could you make money if he was rig...

See Answer

Q: New Economy Transport (A) The New Economy Transport Company

New Economy Transport (A) The New Economy Transport Company (NETCO) was formed in 1959 to carry cargo and passengers between ports in the Pacific Northwest and Alaska. By 2015 its fleet had grown to f...

See Answer

Q: Look again at the spot interest rates shown in Problem 25.

Look again at the spot interest rates shown in Problem 25. What can you deduce about the one-year spot interest rate in three years if. . . a. The expectations theory of term...

See Answer

Q: Each of the following statements is true. Use an example to

Each of the following statements is true. Use an example to explain why they are consistent. When a company introduces a new product, or expands production of an existing product, investment in net wo...

See Answer

Q: We can imagine the financial manager doing several things on behalf of

We can imagine the financial manager doing several things on behalf of the firm’s stockholders. For example, the manager might: a. Make shareholders as wealthy as possible by investing in real assets...

See Answer

Q: Suppose that you buy a two-year 8% bond at

Suppose that you buy a two-year 8% bond at its face value. a. What will be your total nominal return over the two years if inflation is 3% in the first year and 5% in the second? What will be your re...

See Answer

Q: If a bond’s yield to maturity does not change, the return

If a bond’s yield to maturity does not change, the return on the bond each year will be equal to the yield to maturity. Confirm this with a simple example of a four-year bond selling at a premium to f...

See Answer

Q: Find the arbitrage opportunity (opportunities?). Assume for simplicity that coupons

Find the arbitrage opportunity (opportunities?). Assume for simplicity that coupons are paid annually. In each case the face value of the bond is $1,000.

See Answer

Q: You are quoted an interest rate of 6% on an investment

You are quoted an interest rate of 6% on an investment of $10 million. What is the value of your investment after four years if interest is compounded: a. Annually? b. Monthly? or c. Continuously?...

See Answer

Q: Which of the following statements always apply to corporations? a

Which of the following statements always apply to corporations? a. Unlimited liability. b. Limited life. c. Ownership can be transferred without affecting operations. d. Managers can be fired with...

See Answer

Q: The duration of a bond that makes an equal payment each year

The duration of a bond that makes an equal payment each year in perpetuity is (1 + yield)/yield. Prove it.

See Answer

Q: Calculate the IRR (or IRRs) for the following project:

Calculate the IRR (or IRRs) for the following project: For what range of discount rates does the project have positive NPV?

See Answer

Q: What spot interest rates are implied by the following Treasury bonds?

What spot interest rates are implied by the following Treasury bonds? Assume for simplicity that the bonds pay annual coupons. The price of a one-year strip is 97.56%, and the price of a four-year str...

See Answer

Q: Pharmecology just paid an annual dividend of $1.35 per

Pharmecology just paid an annual dividend of $1.35 per share. It’s a mature company, but future EPS and dividends are expected to grow with inflation, which is forecasted at 2.75% per year. a. What i...

See Answer

Q: Look one more time at Table 3.5. /

Look one more time at Table 3.5. a. Suppose you knew the bond prices but not the spot interest rates. Explain how you would calculate the spot rates. b. Suppose that you could buy bond...

See Answer

Q: Consider three investors: a. Mr. Single invests for

Consider three investors: a. Mr. Single invests for one year. b. Ms. Double invests for two years. c. Mrs. Triple invests for three years. Assume each invests in company Z (see Problem 5). Show tha...

See Answer

Q: A 10-year bond is issued with a face value of

A 10-year bond is issued with a face value of $1,000, paying interest of $60 a year. If yields to maturity increase shortly after the T-bond is issued, what happens to the bond’s a. Coupon rate? b....

See Answer

Q: In February 2015 Treasury 4¾s of 2041 offered a semi

In February 2015 Treasury 4¾s of 2041 offered a semi-annually compounded yield to maturity of 2.70%. Recognizing that coupons are paid semi-annually, calculate the bond’s price.

See Answer

Q: In December 2005, Mid-American Energy brought online one of

In December 2005, Mid-American Energy brought online one of the largest wind farms in the world. It cost an estimated $386 million and the 257 turbines have a total capacity of 360.5 megawatts (mW). W...

See Answer

Q: A 10-year German government bond (bund) has a

A 10-year German government bond (bund) has a face value of €100 and a coupon rate of 5% paid annually. Assume that the interest rate (in euros) is equal to 6% per year. What is the bond’s PV?

See Answer

Q: The following table tracks the main components of working capital over the

The following table tracks the main components of working capital over the life of a four-year project. Calculate net working capital and the cash inflows and outflows due to investment in working ca...

See Answer

Q: Look again at Table 3.5. Suppose that spot interest

Look again at Table 3.5. Suppose that spot interest rates all change to 4%—a “flat” term structure of interest rates. a. What is the...

See Answer

Q: Ms. Espinoza is retired and depends on her investments for her

Ms. Espinoza is retired and depends on her investments for her income. Mr. Liu is a young executive who wants to save for the future. Both are stockholders in Scaled Composites, LLC, which is building...

See Answer

Q: The one-year spot interest rate is r1 = 5%

The one-year spot interest rate is r1 = 5% and the two-year rate is r2 = 6%. If the expectations theory is correct, what is the expected one-year interest rate in one year’s time?

See Answer

Q: Air conditioning for a college dormitory will cost $1.5

Air conditioning for a college dormitory will cost $1.5 million to install and $200,000 per year to operate. The system should last 25 years. The real cost of capital is 5%, and the college pays no ta...

See Answer

Q: a. The cost of a new automobile is $10,

a. The cost of a new automobile is $10,000. If the interest rate is 5%, how much would you have to set aside now to provide this sum in five years? b. You have to pay $12,000 a year in school fees at...

See Answer

Q: The two-year interest rate is 10% and the expected

The two-year interest rate is 10% and the expected annual inflation rate is 5%. a. What is the expected real interest rate? b. If the expected rate of inflation suddenly rises to 7%, what does Fishe...

See Answer

Q: Ms. T. Potts, the treasurer of Ideal China,

Ms. T. Potts, the treasurer of Ideal China, has a problem. The company has just ordered a new kiln for $400,000. Of this sum, $50,000 is described by the supplier as an installation cost. Ms. Potts do...

See Answer

Q: Company Q’s current return on equity (ROE) is 14%.

Company Q’s current return on equity (ROE) is 14%. It pays out one-half of earnings as cash dividends (pay-out ratio = .5). Current book value per share is $50. Book value per share will grow as Q rei...

See Answer

Q: Here are the prices of three bonds with 10-year maturities

Here are the prices of three bonds with 10-year maturities: Bond Coupon (%)………………Price (%) 2%......................................81.62% 4…………………………………..98.39 8………………………………...133.42 If coupons are...

See Answer

Q: Is the yield on high-coupon bonds more likely to be

Is the yield on high-coupon bonds more likely to be higher than that on low-coupon bonds when the term structure is upward-sloping or when it is downward-sloping? Explain.

See Answer

Q: Which of the following statements are true? The opportunity cost

Which of the following statements are true? The opportunity cost of capital: a. Equals the interest rate at which the company can borrow. b. Depends on the risk of the cash flows to be valued. c....

See Answer

Q: A bond’s credit rating provides a guide to its price. As

A bond’s credit rating provides a guide to its price. As we write this in early 2015, Aaa bonds yield 3.4% and Baa bonds yield 4.4%. If some bad news causes a 10% five-year bond to be unexpectedly dow...

See Answer

Q: Ten years ago, in 2007, George Reeby founded a small

Ten years ago, in 2007, George Reeby founded a small mail-order company selling high-quality sports equipment. Since those early days Reeby Sports has grown steadily and been consistently profitable....

See Answer

Q: Construct a new version of Table 4.7, assuming that

Construct a new version of Table 4.7, assuming that the concatenator division grows at 20%, 12%, and 6%, instead of 12%, 9%, and 6%. You will get negative early free cash flows. a. Reca...

See Answer

Q: Mr. Art Deco will be paid $100,000 one

Mr. Art Deco will be paid $100,000 one year hence. This is a nominal flow, which he discounts at an 8% nominal discount rate: PV =_10__0_,0_0_0_ = $92,593 1.08 The inflation rate is 4%. Calculate the...

See Answer

Q: A factory costs $800,000. You reckon that it

A factory costs $800,000. You reckon that it will produce an inflow after operating costs of $170,000 a year for 10 years. If the opportunity cost of capital is 14%, what is the net present value of t...

See Answer

Q: After spending $3 million on research, Better Mousetraps has developed

After spending $3 million on research, Better Mousetraps has developed a new trap. The project requires an initial investment in plant and equipment of $6 million. This investment will be depreciated...

See Answer

Q: Which comes first in the market for U.S. Treasury

Which comes first in the market for U.S. Treasury bonds: a. Spot interest rates or yields to maturity? b. Bond prices or yields to maturity?

See Answer

Q: Assume annual coupons. a. What is the formula for

Assume annual coupons. a. What is the formula for the value of a two-year, 5% bond in terms of spot rates? b. What is the formula for its value in terms of yield to maturity? c. If the two-year spot...

See Answer

Q: The following table shows the prices of a sample of U.

The following table shows the prices of a sample of U.S. Treasury strips in February 2012. Each strip makes a single payment of $1,000 at maturity. a. Calculate the annually compounded, spot interest...

See Answer

Q: True or false? a. U.S. CEOs

True or false? a. U.S. CEOs are paid much more than CEOs in other countries. b. A large fraction of compensation for U.S. CEOs comes from stock-option grants. c. Stock-option grants give the manager a...

See Answer

Q: Who monitors the top management of public U.S. corporations

Who monitors the top management of public U.S. corporations? (We have mentioned several types of monitoring in this chapter.)

See Answer

Q: People often convey the idea behind MM’s proposition 1 by various supermarket

People often convey the idea behind MM’s proposition 1 by various supermarket analogies, for example, “The value of a pie should not depend on how it is sliced,” or, “The cost of a whole chicken shoul...

See Answer

Q: Gaucho Services starts life with all-equity financing and a cost

Gaucho Services starts life with all-equity financing and a cost of equity of 14%. Suppose it refinances to the following market-value capital structure:  Use MM’s proposition 2 to calculate the new...

See Answer

Q: Can you invent any new kinds of debt that might be attractive

Can you invent any new kinds of debt that might be attractive to investors? Why do you think they have not been issued?

See Answer

Q: The present value of interest tax shields is often written as TcD

The present value of interest tax shields is often written as TcD, where D is the amount of debt and Tc is the marginal corporate tax rate. Under what assumptions is this present value correct?

See Answer

Q: Here are book and market value balance sheets of the United Frypan

Here are book and market value balance sheets of the United Frypan Company (UF): Assume that MM’s theory holds with taxes. There is no growth, and the $40 of debt is expected to be permanent. Assume a...

See Answer

Q: What is the relative tax advantage of corporate debt if the corporate

What is the relative tax advantage of corporate debt if the corporate tax rate is Tc = .35, the personal tax rate is Tp = .35, but all equity income is received as capital gains and escapes tax entire...

See Answer

Q: “The firm can’t use interest tax shields unless it has (

“The firm can’t use interest tax shields unless it has (taxable) income to shield.” What does this statement imply for debt policy? Explain briefly.

See Answer

Q: On February 29, 2015, when PDQ Computers announced bankruptcy,

On February 29, 2015, when PDQ Computers announced bankruptcy, its share price fell from $3.00 to $.50 per share. There were 10 million shares outstanding. Does that imply bankruptcy costs of 10 × (3....

See Answer

Q: The traditional theory of optimal capital structure states that firms trade off

The traditional theory of optimal capital structure states that firms trade off corporate interest tax shields against the possible costs of financial distress due to borrowing. What does this theory...

See Answer

Q: Rajan and Zingales identified four variables that seemed to explain differences in

Rajan and Zingales identified four variables that seemed to explain differences in debt ratios in several countries. What are the four variables?

See Answer

Q: How would your answer to Problem 10 change if technological improvements reduce

How would your answer to Problem 10 change if technological improvements reduce the cost of new BG production facilities by 3% per year? Thus a new plant built in year 1 would cost only 25 (1 – .03) =...

See Answer

Q: Fill in the blanks: According to the pecking-order theory

Fill in the blanks: According to the pecking-order theory, a. The firm’s debt ratio is determined by ________. b. Debt ratios depend on past profitability, because ______.

See Answer

Q: For what kinds of companies is financial slack most valuable? Are

For what kinds of companies is financial slack most valuable? Are there situations in which financial slack should be reduced by borrowing and paying out the proceeds to the stockholders? Explain.

See Answer

Q: The trade-off theory relies on the threat of financial distress

The trade-off theory relies on the threat of financial distress. But why should a public corporation ever have to land in financial distress? According to the theory, the firm should operate at the to...

See Answer

Q: Why does asymmetric information push companies to raise external funds by borrowing

Why does asymmetric information push companies to raise external funds by borrowing rather than by issuing common stock?

See Answer

Q: Compute the present value of interest tax shields generated by these three

Compute the present value of interest tax shields generated by these three debt issues. Consider corporate taxes only. The marginal tax rate is Tc = .35. a. A $1,000, one-year loan at 8%. b. A five-ye...

See Answer

Q: Suppose that Congress sets the top personal tax rate on interest and

Suppose that Congress sets the top personal tax rate on interest and dividends at 35% and the top rate on realized capital gains at 15%. The corporate tax rate stays at 35%. Compute the difference bet...

See Answer

Q: “The trouble with MM’s argument is that it ignores the fact

“The trouble with MM’s argument is that it ignores the fact that individuals cannot deduct interest for personal income tax.” Show why this is not an objection if personal tax rates on interest and eq...

See Answer

Q: Look back at the Johnson & Johnson example in Section 18-

Look back at the Johnson & Johnson example in Section 18-1. Suppose Johnson & Johnson increases its long-term debt to $30 billion. It uses the additional debt to repurchase shares. Reconstruct...

See Answer

Q: In Section 18-3, we briefly referred to three games

In Section 18-3, we briefly referred to three games: playing for time, cash in and run, and bait and switch. For each game, construct a simple numerical example (like the example for the risk-shifting...

See Answer

Q: Let us go back to Circular File’s market value balance sheet:

Let us go back to Circular File’s market value balance sheet:  Who gains and who loses from the following maneuvers? a. Circular scrapes up $5 in cash and pays a cash dividend. b. Circular halts ope...

See Answer

Q: Reevaluate the NPV of the proposed polyzone project under each of the

Reevaluate the NPV of the proposed polyzone project under each of the following assumptions. What’s the right management decision in each case? a. Spread in year 4 holds at $1.20 per pound. b. The U.S...

See Answer

Q: a. Who benefits from the fine print in bond contracts when

a. Who benefits from the fine print in bond contracts when the firm gets into financial trouble? Give a one-sentence answer. b. Who benefits from the fine print when the bonds are issued? Suppose the...

See Answer

Q: he Salad Oil Storage (SOS) Company has financed a large

he Salad Oil Storage (SOS) Company has financed a large part of its facilities with long-term debt. There is a significant risk of default, but the company is not on the ropes yet. Explain: a. Why SOS...

See Answer

Q: “I was amazed to find that the announcement of a stock

“I was amazed to find that the announcement of a stock issue drives down the value of the issuing firm by 30%, on average, of the proceeds of the issue. That issue cost dwarfs the underwriter’s spread...

See Answer

Q: Ronald Masulis analyzed the stock price impact of exchange offers of debt

Ronald Masulis analyzed the stock price impact of exchange offers of debt for equity or vice versa.35 In an exchange offer, the firm offers to trade freshly issued securities for seasoned securities i...

See Answer

Q: The possible payoffs from Ms. Ketchup’s projects (see Example 18

The possible payoffs from Ms. Ketchup’s projects (see Example 18.1) have not changed but there is now a 40% chance that Project 2 will pay off $24 and a 60% chance that it will pay off $0. a. Recalcul...

See Answer

Q: Some corporations’ debt–equity targets are expressed not as a debt

Some corporations’ debt–equity targets are expressed not as a debt ratio but as a target debt rating on the firm’s outstanding bonds. What are the pros and cons of setting a target rating rather than...

See Answer

Q: Most financial managers measure debt ratios from their companies’ book balance sheets

Most financial managers measure debt ratios from their companies’ book balance sheets. Many financial economists emphasize ratios from market-value balance sheets. Which is the right measure in princi...

See Answer

Q: You are considering a five-year lease of office space for

You are considering a five-year lease of office space for R&D personnel. Once signed, the lease cannot be canceled. It would commit your firm to six annual $100,000 payments, with the first payment du...

See Answer

Q: : The U.S. government has settled a dispute with

The U.S. government has settled a dispute with your company for $16 million. The government is committed to pay this amount in exactly 12 months. However, your company will have to pay tax on the awa...

See Answer

Q: To finance the Madison County project, Wishing Well needs to arrange

To finance the Madison County project, Wishing Well needs to arrange an additional $80 million of long-term debt and make a $20 million equity issue. Underwriting fees, spreads, and other costs of thi...

See Answer

Q: Photographic laboratories recover and recycle the silver used in photographic film.

Photographic laboratories recover and recycle the silver used in photographic film. Stikine River Photo is considering purchase of improved equipment for their laboratory at Telegraph Creek. Here is t...

See Answer

Q: Calculate the weighted-average cost of capital (WACC) for

Calculate the weighted-average cost of capital (WACC) for Federated Junkyards of America, using the following information: Debt: $75,000,000 book value outstanding. The debt is trading at 90% of book...

See Answer

Q: True or false? Use of the WACC formula assumes a

True or false? Use of the WACC formula assumes a. A project supports a fixed amount of debt over the project’s economic life. b. The ratio of the debt supported by a project to project value is consta...

See Answer

Q: What is meant by the flow-to-equity valuation method

What is meant by the flow-to-equity valuation method? What discount rate is used in this method? What assumptions are necessary for this method to give an accurate valuation?

See Answer

Q: True or false? The APV method a. Starts with

True or false? The APV method a. Starts with a base-case value for the project. b. Calculates the base-case value by discounting project cash flows, forecasted assuming all equity financing, at the WA...

See Answer

Q: A project costs $1 million and has a base-case

A project costs $1 million and has a base-case NPV of exactly zero (NPV = 0). What is the project’s APV in the following cases? a. If the firm invests, it has to raise $500,000 by a stock issue. Issue...

See Answer

Q: Whispering Pines, Inc., is all-equity-financed.

Whispering Pines, Inc., is all-equity-financed. The expected rate of return on the company’s shares is 12%. a. What is the opportunity cost of capital for an average-risk Whispering Pines investment?...

See Answer

Q: Consider a project lasting one year only. The initial outlay is

Consider a project lasting one year only. The initial outlay is $1,000 and the expected inflow is $1,200. The opportunity cost of capital is r = .20. The borrowing rate is rD = .10, and the tax shield...

See Answer

Q: The WACC formula seems to imply that debt is “cheaper”

The WACC formula seems to imply that debt is “cheaper” than equity—that is, that a firm with more debt could use a lower discount rate. Does this make sense? Explain briefly.

See Answer

Q: Suppose Federated Junkyards decides to move to a more conservative debt policy

Suppose Federated Junkyards decides to move to a more conservative debt policy. A year later its debt ratio is down to 15% (D/V = .15). The interest rate has dropped to 8.6%. Recalculate Federated’s W...

See Answer

Q: Suppose KCS Corp. buys out Patagonia Trucking, a privately owned

Suppose KCS Corp. buys out Patagonia Trucking, a privately owned business, for $50 million. KCS has only $5 million cash in hand, so it arranges a $45 million bank loan. A normal debt-to-value ratio f...

See Answer

Q: The Cambridge Opera Association has come up with a unique door prize

The Cambridge Opera Association has come up with a unique door prize for its December 2019 fund-raising ball: Twenty door prizes will be distributed, each one a ticket entitling the bearer to receive...

See Answer

Q: Table 19.3 shows a book balance sheet for the Wishing

Table 19.3 shows a book balance sheet for the Wishing Well Motel chain. The company’s long-term debt is secured by its real estate assets, but it also uses short-term bank loans as a permanent source...

See Answer

Q: Table 19.4 shows a simplified balance sheet for Rensselaer Felt

Table 19.4 shows a simplified balance sheet for Rensselaer Felt. Calculate this company’s weighted-average cost of capital. The debt has just been refinanced at an interest rate of 6% (short term) and...

See Answer

Q: How will Rensselaer Felt’s WACC and cost of equity change if it

How will Rensselaer Felt’s WACC and cost of equity change if it issues $50 million in new equity and uses the proceeds to retire long-term debt? Assume the company’s borrowing rates are unchanged. Use...

See Answer

Q: Digital Organics (DO) has the opportunity to invest $1

Digital Organics (DO) has the opportunity to invest $1 million now (t = 0) and expects after-tax returns of $600,000 in t = 1 and $700,000 in t = 2. The project will last for two years only. The appro...

See Answer

Q: Consider another perpetual project like the crusher described in Section 19-

Consider another perpetual project like the crusher described in Section 19-1. Its initial investment is $1,000,000, and the expected cash inflow is $95,000 a year in perpetuity. The opportunity cost...

See Answer

Q: Suppose the project described in Problem 17 is to be undertaken by

Suppose the project described in Problem 17 is to be undertaken by a university. Funds for the project will be withdrawn from the university’s endowment, which is invested in a widel...

See Answer

Q: Consider a project to produce solar water heaters. It requires a

Consider a project to produce solar water heaters. It requires a $10 million investment and offers a level after-tax cash flow of $1.75 million per year for 10 years. The opportunity cost of capital i...

See Answer

Q: The Bunsen Chemical Company is currently at its target debt ratio of

The Bunsen Chemical Company is currently at its target debt ratio of 40%. It is contemplating a $1 million expansion of its existing business. This expansion is expected to produce a cash inflow of $1...

See Answer

Q: Chiara Company’s management has made the projections shown in Table 19.

Chiara Company’s management has made the projections shown in Table 19.5. Use this table as a starting point to value the company as a whole. The WACC for Chiara is 12% and the long-...

See Answer

Q: In footnote 15 we referred to the Miles–Ezzell discount rate

In footnote 15 we referred to the Miles–Ezzell discount rate formula, which assumes that debt is not rebalanced continuously, but at one-year intervals. Derive this formula. Then use it to unlever San...

See Answer

Q: You are asked to value a large building in northern New Jersey

You are asked to value a large building in northern New Jersey. The valuation is needed for a railroad bankruptcy settlement. Here are the facts: a. The settlement requires that the building’s value e...

See Answer

Q: The WACC formula assumes that debt is rebalanced to maintain a constant

The WACC formula assumes that debt is rebalanced to maintain a constant debt ratio D/V. Rebalancing ties the level of future interest tax shields to the future value of the company. This makes the tax...

See Answer

Q: Modify Table 19.1 on the assumption that competition eliminates any

Modify Table 19.1 on the assumption that competition eliminates any opportunities to earn more than WACC on new investment after year 7 (PVGO = 0). How does the valuation of Rio change? Table 19.1:

See Answer

Q: Modify Table 19.1 on the assumption that competition eliminates any

Modify Table 19.1 on the assumption that competition eliminates any opportunities to earn more than WACC on new investment after year 7 (PVGO = 0). How does the valuation of Rio change? Table 19.6 is...

See Answer

Q: Sulphur Ridge Mining is considering the development of a new calonium mine

Sulphur Ridge Mining is considering the development of a new calonium mine at Moose Bend in northern Alberta. The mine would require an upfront investment of $110 million and would produce 100,000 ton...

See Answer

Q: The manufacture of polysyllabic acid is a competitive industry. Most plants

The manufacture of polysyllabic acid is a competitive industry. Most plants have an annual output of 100,000 tons. Operating costs are $.90 a ton, and the sales price is $1 a ton. A 100,000-ton plant...

See Answer

Q: The world airline system is composed of the routes X and Y

The world airline system is composed of the routes X and Y, each of which requires 10 aircraft. These routes can be serviced by three types of aircraft—A, B, and C. There are 5 type A aircraft availab...

See Answer

Q: Taxes are a cost, and, therefore, changes in tax

Taxes are a cost, and, therefore, changes in tax rates can affect consumer prices, project lives, and the value of existing firms. The following problem illustrates this. It also illustrates that tax...

See Answer

Q: Define the following: (a) Agency costs in capital

Define the following: (a) Agency costs in capital investment, (b) private benefits, (c) empire building, (d) entrenching investment, (e) delegated monitoring.

See Answer

Q: Use the Beyond the Page feature to access the Excel program for

Use the Beyond the Page feature to access the Excel program for measuring the profitability of the Nodhead project. Reconstruct Table 12.4 assuming a steady-state growth rate of 10% per year. Your ans...

See Answer

Q: Ohio Building Products (OBP) is considering the launch of a

Ohio Building Products (OBP) is considering the launch of a new product which would require an initial investment in equipment of $30,800 (no investment in working capital is required). The forecast p...

See Answer

Q: Consider an asset with the following cash flows: 

Consider an asset with the following cash flows:  The firm uses straight-line depreciation. Thus, for this project, it writes off $4 million per year in years 1, 2, and 3. The discount rate is 10%....

See Answer

Q: In our Nodhead example, true depreciation was decelerated. That is

In our Nodhead example, true depreciation was decelerated. That is not always the case. For instance, Table 12.6 shows how on average the market value of a Boeing 737 has varied with its age26 and the...

See Answer

Q: We noted that management compensation must in practice rely on results rather

We noted that management compensation must in practice rely on results rather than on effort. Why? What problems are introduced by not rewarding effort?

See Answer

Q: Here are a few questions about compensation schemes that tie top management’s

Here are a few questions about compensation schemes that tie top management’s compensation to the rate of return earned on the company’s common stock. a. Today’s stock price depends on investors’ expe...

See Answer

Q: Calculate the year-by-year book and economic profitability for

Calculate the year-by-year book and economic profitability for investment in polyzone production, as described in Chapter 11. Use the cash flows and competitive spreads shown in Table 11.2, and assume...

See Answer

Q: Use the Beyond the Page feature to access the Excel program for

Use the Beyond the Page feature to access the Excel program for calculating the profitability of the Nodhead project. Now suppose that the cash flows from Nodhead’s new supermarket are as follows: ...

See Answer

Q: Table 12.5 shows a condensed income statement and balance sheet

Table 12.5 shows a condensed income statement and balance sheet for Androscoggin Copper’s Rumford smelting plant. a. Calculate the plant’s EVA. Assume the cost of capital is 9%. b. As Table 12.5 shows...

See Answer

Q: Herbal Resources is a small but profitable producer of dietary supplements for

Herbal Resources is a small but profitable producer of dietary supplements for pets. This is not a high-tech business, but Herbal’s earnings have averaged around $1.2 million after tax, largely on the...

See Answer

Q: Monitoring alone can never completely eliminate agency costs in capital investment.

Monitoring alone can never completely eliminate agency costs in capital investment. Briefly explain why.

See Answer

Q: True or false? Explain briefly. a. Book profitability

True or false? Explain briefly. a. Book profitability measures are biased measures of true profitability for individual assets. However, these biases “wash out” when firms hold a balanced mix of old a...

See Answer

Q: Consider the following project:  The internal rate

Consider the following project:  The internal rate of return is 20%. The NPV, assuming a 20% opportunity cost of capital, is exactly zero. Calculate the expected economic income and economic depreci...

See Answer

Q: Explain what each of the following terms or phrases means:

Explain what each of the following terms or phrases means: a. Venture capital b. Book building c. Underwriting spread d. Registration statement e. Winner’s curse

See Answer

Q: True or false? a. Venture capitalists typically provide first

True or false? a. Venture capitalists typically provide first-stage financing sufficient to cover all development expenses. Second-stage financing is provided by stock issued in an IPO. b. Underpric...

See Answer

Q: You need to choose between making a public offering and arranging a

You need to choose between making a public offering and arranging a private placement. In each case the issue involves $10 million face value of 10-year debt. You have the following data for each: A...

See Answer

Q: Associated Breweries is planning to market alcohol-free beer. To

Associated Breweries is planning to market alcohol-free beer. To finance the venture it proposes to make a rights issue at $10 of one new share for each two shares held. (The company currently has out...

See Answer

Q: Here is a further vocabulary quiz. Briefly explain each of the

Here is a further vocabulary quiz. Briefly explain each of the following: a. Zero-stage vs. first- or second-stage financing b. Carried interest c. Rights issue d. Road show e. Best-efforts offer...

See Answer

Q: a. “A signal is credible only if it is costly

a. “A signal is credible only if it is costly.” Explain why management’s willingness to invest in Marvin’s equity was a credible signal. Was its willingness to accept only part of the venture capital...

See Answer

Q: In some U.K. IPOs any investor may be able

In some U.K. IPOs any investor may be able to apply to buy shares. Mr. Bean has observed that on average these stocks are underpriced by about 9% and for some years has followed a policy of applying f...

See Answer

Q: Why are the costs of debt issues less than those of equity

Why are the costs of debt issues less than those of equity issues? List the possible reasons.

See Answer

Q: Here are several questions about economic value added or EVA.

Here are several questions about economic value added or EVA. a. Is EVA expressed as a percentage or a dollar amount? b. Write down the formula for calculating EVA. c. What is the difference, if any,...

See Answer

Q: There are three reasons that a common stock issue might cause a

There are three reasons that a common stock issue might cause a fall in price: (a) the price fall is needed to absorb the extra supply, (b) the issue causes temporary price pressure until it has been...

See Answer

Q: Construct a simple example to show the following: a.

Construct a simple example to show the following: a. Existing shareholders are made worse off when a company makes a cash offer of new stock below the market price. b. Existing shareholders are not m...

See Answer

Q: In 2012, the Pandora Box Company made a rights issue at

In 2012, the Pandora Box Company made a rights issue at €5 a share of one new share for every four shares held. Before the issue there were 10 million shares outstanding and the share price was €6. a....

See Answer

Q: Problem 14 contains details of a rights offering by Pandora Box.

Problem 14 contains details of a rights offering by Pandora Box. Suppose that the company had decided to issue new stock at €4. How many new shares would it have needed to sell to raise the same sum o...

See Answer

Q: Suppose that instead of having a rights issue of new stock at

Suppose that instead of having a rights issue of new stock at €4 (see Problem 15), Pandora decided to make a general cash offer at €4. Would existing shareholders still be just as well off? Explain....

See Answer

Q: Suppose that in April 2019 Van Dyck Exponents offered 100 shares for

Suppose that in April 2019 Van Dyck Exponents offered 100 shares for sale in an IPO. Half of the shares were sold by the company and the other half by existing shareholders, each of whom sold exactly...

See Answer

Q: Refer to the Marvin Prospectus Appendix at the end of this chapter

Refer to the Marvin Prospectus Appendix at the end of this chapter to answer the following questions. a. If there is unexpectedly heavy demand for the issue, how many extra shares can the underwriter...

See Answer

Q: a. Why do venture capital companies prefer to advance money in

a. Why do venture capital companies prefer to advance money in stages? If you were the management of Marvin Enterprises, would you have been happy with such an arrangement? With the benefit of hindsig...

See Answer

Q: Here is recent financial data on Pisa Construction, Inc.

Here is recent financial data on Pisa Construction, Inc. Pisa has not performed spectacularly to date. However, it wishes to issue new shares to obtain $80,000 to finance expansion into a promising m...

See Answer

Q: After each of the following issue methods, we have listed two

After each of the following issue methods, we have listed two types of issue. Choose the one more likely to employ that method. a. Rights issue (initial public offer/further sale of an already public...

See Answer

Q: The Modern Language Corporation earned $1.6 million on net

The Modern Language Corporation earned $1.6 million on net assets of $20 million. The cost of capital is 11.5%. Calculate the net ROI and EVA.

See Answer

Q: For each of the following pairs of issues, which is likely

For each of the following pairs of issues, which is likely to involve the lower proportionate underwriting and administrative costs? a. A large issue/a small issue b. A bond issue/a common stock iss...

See Answer

Q: Explain the difference between a uniform-price auction and a discriminatory

Explain the difference between a uniform-price auction and a discriminatory auction. Why might you prefer to sell securities by one method rather than another?

See Answer

Q: In 2014, Entergy paid a regular quarterly dividend of $.83

In 2014, Entergy paid a regular quarterly dividend of $.83 per share. a. Match each of the following dates. (A1) Friday, July 25 (B1) Record date (A2) Monday, August 11 (B2) Payment date (A3) Tuesday...

See Answer

Q: Here are several “facts” about typical corporate dividend policies.

Here are several “facts” about typical corporate dividend policies. Which are true and which false? a. Companies decide each year’s dividend by looking at their capital expenditure requirements and t...

See Answer

Q: Seashore Salt Co. has surplus cash. Its CFO decides to

Seashore Salt Co. has surplus cash. Its CFO decides to pay back $4 per share to investors by initiating a regular dividend of $1 per quarter or $4 per year. The stock price jumps to $90 when the pay-o...

See Answer

Q: Look again at Problem 3. Assume instead that the CFO announces

Look again at Problem 3. Assume instead that the CFO announces a stock repurchase of $4 per share instead of a cash dividend. a. What happens to the stock price when the repurchase is announced? Woul...

See Answer

Q: Go back to the first Rational Demiconductor balance sheet. Now assume

Go back to the first Rational Demiconductor balance sheet. Now assume that Rational wins a lawsuit and is paid $1 million in cash. Its market capitalization rises by that amount. It decides to pay out...

See Answer

Q: Go back to the first Rational Demiconductor balance sheet one more time

Go back to the first Rational Demiconductor balance sheet one more time. Assume that Rational does not win the lawsuit (see Problem 5) and is left with only $1 million in surplus cash. Nevertheless Ra...

See Answer

Q: Surf & Turf Hotels is a mature business, although it pays

Surf & Turf Hotels is a mature business, although it pays no cash dividends. Next year’s earnings are forecasted at $56 million. There are 10 million outstanding shares. The company has traditionally...

See Answer

Q: Some types of investors prefer dividend-paying stocks because dividends provide

Some types of investors prefer dividend-paying stocks because dividends provide a regular, convenient source of income. Does demand from these investors necessarily lift the prices of dividend-paying...

See Answer

Q: Fill in the blanks: “A project’s economic income for

Fill in the blanks: “A project’s economic income for a given year equals the project’s _____ less its _____ depreciation. New projects may take several years to reach full profitability. In these case...

See Answer

Q: Half shell Seafood is still generating good profits, but growth is

Half shell Seafood is still generating good profits, but growth is slowing down. How should its CFO decide when to start up a program of paying out cash to stockholders? What questions should the CFO...

See Answer

Q: Investors and financial managers focus more on changes in cash dividends than

Investors and financial managers focus more on changes in cash dividends than on the level of cash dividends. Why?

See Answer

Q: Does the good news conveyed by the announcement of a dividend increase

Does the good news conveyed by the announcement of a dividend increase mean that a firm can increase its stock price in the long run simply by paying cash dividends? Explain.

See Answer

Q: Little Oil has outstanding one million shares with a total market value

Little Oil has outstanding one million shares with a total market value of $20 million. The firm is expected to pay $1 million of dividends next year, and thereafter the amount paid out is expected to...

See Answer

Q: We stated in Section 16-3 that MM’s proof of dividend

We stated in Section 16-3 that MM’s proof of dividend irrelevance assumes that new shares are sold at a fair price. Look back at Problem 17. Assume that new shares are issued in year 1 at $10 a share....

See Answer

Q: Look back one last time at Problem 17. How would you

Look back one last time at Problem 17. How would you value Little Oil if it paid out $500,000 in cash dividends year in and year out, with no expected growth or decline? Remaining free cash flow will...

See Answer

Q: House of Haddock has 5,000 shares outstanding and the stock

House of Haddock has 5,000 shares outstanding and the stock price is $140. The company is expected to pay a dividend of $20 per share next year and thereafter the dividend is expected to grow indefini...

See Answer

Q: Here are key financial data for House of Herring, Inc.:

Here are key financial data for House of Herring, Inc.: Earnings per share for 2018 …………..….. $5.50 Number of shares outstanding…….40 million Target payout ratio……………………………..50% Planned dividend per s...

See Answer

Q: Respond to the following comment: “It’s all very well saying

Respond to the following comment: “It’s all very well saying that I can sell shares to cover cash needs, but that may mean selling at the bottom of the market. If the company pays a regular cash divi...

See Answer

Q: Hors d’Age Cheeseworks has been paying a regular cash dividend of $

Hors d’Age Cheeseworks has been paying a regular cash dividend of $4 per share each year for over a decade. The company is paying out all its earnings as dividends and is not expected to grow. There a...

See Answer

Q: How in practice do managers of public firms meet short-run

How in practice do managers of public firms meet short-run earnings targets? By creative accounting?

See Answer

Q: An article on stock repurchase in the Los Angeles Times noted:

An article on stock repurchase in the Los Angeles Times noted: “An increasing number of companies are finding that the best investment they can make these days is in themselves.” Discuss this view. Ho...

See Answer

Q: Comment briefly on each of the following statements: a.

Comment briefly on each of the following statements: a. “Unlike American firms, which are always being pressured by their shareholders to increase dividends, Japanese companies pay out a much smaller...

See Answer

Q: Generous dividend payouts and high price–earnings multiples are correlated positively

Generous dividend payouts and high price–earnings multiples are correlated positively. Does this imply that paying out cash as dividends instead of repurchases increases share price?

See Answer

Q: “Many companies use stock repurchases to increase earnings per share.

“Many companies use stock repurchases to increase earnings per share. For example, suppose that a company is in the following position: Net profit………………………………………..……$10 million Number of shares before...

See Answer

Q: The middle-of-the-road party holds that dividend

The middle-of-the-road party holds that dividend policy doesn’t matter because the supply of high-, medium-, and low-payout stocks has already adjusted to satisfy investors’ demands. Investors who lik...

See Answer

Q: Consider the following two statements: “Dividend policy is irrelevant,”

Consider the following two statements: “Dividend policy is irrelevant,” and “Stock price is the present value of expected future dividends.” (See Chapter 4.) They sound contradictory. This question is...

See Answer

Q: Suppose that there are just three types of investors with the following

Suppose that there are just three types of investors with the following tax rates: The remaining stock is held by the institutions. All three groups simply seek to maximize their after-tax income. The...

See Answer

Q: Mr. Milquetoast admires Warren Buffet and believes that Berkshire Hathaway is

Mr. Milquetoast admires Warren Buffet and believes that Berkshire Hathaway is a good investment. He wants to invest $100,000, but hesitates because Berkshire Hathaway has never paid a dividend. He nee...

See Answer

Q: Which of the following U.S. investors have tax reasons

Which of the following U.S. investors have tax reasons to prefer companies that pay out cash by repurchases instead of cash dividends? Which should not care? a. A pension fund b. An individual inves...

See Answer

Q: What is meant by “the information content of dividends”? Explain

What is meant by “the information content of dividends”? Explain.

See Answer

Q: Compare typical compensation and incentive arrangements for (a) top management

Compare typical compensation and incentive arrangements for (a) top management, for example, the CEO or CFO, and (b) plant or division managers. What are the chief differences? Can you explain them?

See Answer

Q: MM insisted that payout policy should be analyzed holding debt and investment

MM insisted that payout policy should be analyzed holding debt and investment policy constant. Why? Explain.

See Answer

Q: Suppose that new security designs could be patented.13 The patent

Suppose that new security designs could be patented.13 The patent holder could restrict use of the new design or charge other firms royalties for using it. What effect would such patents have on MM’s...

See Answer

Q: Ms. Kraft owns 50,000 shares of the common stock

Ms. Kraft owns 50,000 shares of the common stock of Copperhead Corporation with a market value of $2 per share, or $100,000 overall. The company is currently financed as follows: Market Value...

See Answer

Q: Spam Corp. is financed entirely by common stock and has a

Spam Corp. is financed entirely by common stock and has a beta of 1.0. The firm is expected to generate a level, perpetual stream of earnings and dividends. The stock has a price–earnings ratio of 8 a...

See Answer

Q: The common stock and debt of Northern Sludge are valued at $

The common stock and debt of Northern Sludge are valued at $50 million and $30 million, respectively. Investors currently require a 16% return on the common stock and an 8% return on the debt. If Nort...

See Answer

Q: Suppose that Macbeth Spot Removers issues only $2,500 of

Suppose that Macbeth Spot Removers issues only $2,500 of debt and uses the proceeds to repurchase 250 shares. a. Rework Table 17.2 to show how earnings per share and share return now vary with operati...

See Answer

Q: True or false? a. MM’s propositions assume perfect financial

True or false? a. MM’s propositions assume perfect financial markets, with no distorting taxes or other imperfections. b. MM’s proposition 1 says that corporate borrowing increases earnings per share...

See Answer

Q: Refer to Section 17-1. Suppose that Ms. Macbeth’s

Refer to Section 17-1. Suppose that Ms. Macbeth’s investment bankers have informed her that since the new issue of debt is risky, debtholders will demand a return of 12.5%, which is 2.5% above the ris...

See Answer

Q: Companies A and B differ only in their capital structure. A

Companies A and B differ only in their capital structure. A is financed 30% debt and 70% equity; B is financed 10% debt and 90% equity. The debt of both companies is risk-free. a. Rosencrantz owns 1%...

See Answer

Q: Here is a limerick: There once was a man named

Here is a limerick: There once was a man named Carruthers, Who kept cows with miraculous udders. He said, “Isn’t this neat? They give cream from one teat, And skim milk from each of the others!” What...

See Answer

Q: Suppose all plant and division managers were paid only a fixed salary

Suppose all plant and division managers were paid only a fixed salary—no other incentives or bonuses. a. Describe the agency problems that would appear in capital investment decisions. b. How would ty...

See Answer

Q: Executive Chalk is financed solely by common stock and has outstanding 25

Executive Chalk is financed solely by common stock and has outstanding 25 million shares with a market price of $10 a share. It now announces that it intends to issue $160 million of debt and to use t...

See Answer

Q: Executive Cheese has issued debt with a market value of $100

Executive Cheese has issued debt with a market value of $100 million and has outstanding 15 million shares with a market price of $10 a share. It now announces that it intends to issue a further $60 m...

See Answer

Q: Indicate what’s wrong with the following arguments: a. “

Indicate what’s wrong with the following arguments: a. “As the firm borrows more and debt becomes risky, both stockholders and bondholders demand higher rates of return. Thus by reducing the debt rati...

See Answer

Q: Each of the following statements is false or at least misleading.

Each of the following statements is false or at least misleading. Explain why in each case. a. “A capital investment opportunity offering a 10% DCF rate of return is an attractive project if it can be...

See Answer

Q: Imagine a firm that is expected to produce a level stream of

Imagine a firm that is expected to produce a level stream of operating profits. As leverage is increased, what happens to a. The ratio of the market value of the equity to income after interest? b. Th...

See Answer

Q: Archimedes Levers is financed by a mixture of debt and equity.

Archimedes Levers is financed by a mixture of debt and equity. You have the following information about its cost of capital:  Can you fill in the blanks?

See Answer

Q: Consider the following three tickets: Ticket A pays $10 if

Consider the following three tickets: Ticket A pays $10 if is elected as president, ticket B pays $10 if is elected, and ticket C pays $10 if neither is elected. (Fill in the blanks yourself.) Could t...

See Answer

Q: Look back at Problem 19. Suppose now that Archimedes repurchases debt

Look back at Problem 19. Suppose now that Archimedes repurchases debt and issues equity so that D/V = .3. The reduced borrowing causes rD to fall to 11%. How do the other variables change?

See Answer

Q: Omega Corporation has 10 million shares outstanding, now trading at $

Omega Corporation has 10 million shares outstanding, now trading at $55 per share. The firm has estimated the expected rate of return to shareholders at about 12%. It has also issued $200 million of l...

See Answer

Q: Gamma Airlines has an asset beta of 1.5. The

Gamma Airlines has an asset beta of 1.5. The risk-free interest rate is 6%, and the market risk premium is 8%. Assume the capital asset pricing model is correct. Gamma pays taxes at a marginal rate of...

See Answer

Q: Complete the following passage: A ______ option gives its owner

Complete the following passage: A ______ option gives its owner the opportunity to buy a stock at a specified price that is generally called the _____ price. A ____ option gives its owner the opportun...

See Answer

Q: Pintail’s stock price is currently $200. A one-year

Pintail’s stock price is currently $200. A one-year American call option has an exercise price of $50 and is priced at $75. How would you take advantage of this great opportunity? Now suppose the opti...

See Answer

Q: It is possible to buy three-month call options and three

It is possible to buy three-month call options and three-month puts on stock Q. Both options have an exercise price of $60 and both are worth $10. If the interest rate is 5% a year, what is the stock...

See Answer

Q: Which one of the following statements is correct? a.

Which one of the following statements is correct? a. Value of put + present value of exercise price = value of call + share price b. Value of put + share price = value of call + present value of exerc...

See Answer

Q: Three six-month call options are traded on Hogswill stock:

Three six-month call options are traded on Hogswill stock:  How would you make money by trading in Hogswill options?

See Answer

Q: You’ve just completed a month-long study of energy markets and

You’ve just completed a month-long study of energy markets and conclude that energy prices will be much more volatile in the next year than historically. Assuming you’re right, what types of option st...

See Answer

Q: Suppose that you hold a share of stock and a put option

Suppose that you hold a share of stock and a put option on that share. What is the payoff when the option expires if (a) the stock price is below the exercise price? (b) the stock price is above the...

See Answer

Q: What is put–call parity and why does it hold?

What is put–call parity and why does it hold? Could you apply the parity formula to a call and put with different exercise prices?

See Answer

Q: There is another strategy involving calls and borrowing or lending that gives

There is another strategy involving calls and borrowing or lending that gives the same payoffs as the strategy described in Problem 3. What is the alternative strategy? Problem#3: Suppose that you ho...

See Answer

Q: Suppose you buy a one-year European call option on Wombat

Suppose you buy a one-year European call option on Wombat stock with an exercise price of $100 and sell a one-year European put option with the same exercise price. The current stock price is $100, an...

See Answer

Q: Look again at Figure 20.13. It appears that the

Look again at Figure 20.13. It appears that the investor in panel (b) can’t lose and the investor in panel (a) can’t win. Is that correct? Explain. 

See Answer

Q: Discuss briefly the risks and payoffs of the following positions:

Discuss briefly the risks and payoffs of the following positions: a. Buy stock and a put option on the stock. b. Buy stock. c. Buy call. d. Buy stock and sell call option on the stock. e. Buy bond. f....

See Answer

Q: “The buyer of the call and the seller of the put

“The buyer of the call and the seller of the put both hope that the stock price will rise. Therefore the two positions are identical.” Is the speaker correct? Illustrate with a position diagram.

See Answer

Q: FX Bank has succeeded in hiring ace foreign exchange trader Lucinda Cable

FX Bank has succeeded in hiring ace foreign exchange trader Lucinda Cable. Her remuneration package reportedly includes an annual bonus of 20% of the profits that she generates in excess of $100 milli...

See Answer

Q: A European call and put option have the same maturity and both

A European call and put option have the same maturity and both are at the money. The stock does not pay a dividend. Which option should sell for the higher price? Explain.

See Answer

Q: Suppose that Mr. Colleoni borrows the present value of $100

Suppose that Mr. Colleoni borrows the present value of $100, buys a six month put option on stock Y with an exercise price of $150, and sells a six-month put option on Y with an exercise price of $50....

See Answer

Q: a. If you can’t sell a share short, you can

a. If you can’t sell a share short, you can achieve exactly the same final payoff by a combination of options and borrowing or lending. What is this combination? b. Now work out the mixture of stock a...

See Answer

Q: The common stock of Triangular File Company is selling at $90

The common stock of Triangular File Company is selling at $90. A 26-week call option written on Triangular File’s stock is selling for $8. The call’s exercise price is $100. The risk-free interest rat...

See Answer

Q: Is it more valuable to own an option to buy a portfolio

Is it more valuable to own an option to buy a portfolio of stocks or to own a portfolio of options to buy each of the individual stocks? Say briefly why.

See Answer

Q: In December 2014, a 13-month call on the stock

In December 2014, a 13-month call on the stock of Amazon.com, with an exercise price of $305, sold for $42.50. The stock price was $305. The risk-free interest rate was 1%. How much would you be willi...

See Answer

Q: Table 20.4 lists some prices of options on common stocks

Table 20.4 lists some prices of options on common stocks (prices are quoted to the nearest dollar). The interest rate is 10% a year. Can you spot any mispricing? What would you do to take advantage of...

See Answer

Q: Figure 20.14 shows some complicated position diagrams. Work out

Figure 20.14 shows some complicated position diagrams. Work out the combination of stocks, bonds, and options that produces each of these positions. 

See Answer

Q: Some years ago the Australian firm Bond Corporation sold a share in

Some years ago the Australian firm Bond Corporation sold a share in some land that it owned near Rome for $110 million and as a result boosted its annual earnings by $74 million. A television program...

See Answer

Q: Look again at the valuation in Table 22.2 of the

Look again at the valuation in Table 22.2 of the option to invest in the Mark II project. Consider a change in each of the following inputs. Would the change increase or decrease the value of the expa...

See Answer

Q: Suppose a stock price can go up by 15% or down

Suppose a stock price can go up by 15% or down by 13% over the next year. You own a one-year put on the stock. The interest rate is 10%, and the current stock price is $60. a. What exercise price lea...

See Answer

Q: Magna Charter has been asked to operate a Beaver bush plane for

Magna Charter has been asked to operate a Beaver bush plane for a mining company exploring north and west of Fort Liard. Magna will have a firm one-year contract with the mining company and expects th...

See Answer

Q: Reconstruct Table 25.2 as a leveraged lease, assuming

Reconstruct Table 25.2 as a leveraged lease, assuming that the lessor borrows $80,000, 80% of the cost of the bus, nonrecourse at an interest rate of 11%. All lease payments are devoted to debt servi...

See Answer

Q: Suppose that the Greymare lease gives the company the option to purchase

Suppose that the Greymare lease gives the company the option to purchase the bus at the end of the lease period for $1. How would this affect the tax treatment of the lease? Recalculate its value to G...

See Answer

Q: The following terms are often used to describe leases: a

The following terms are often used to describe leases: a. Direct b. Full-service c. Operating d. Financial e. Rental f. Net g. Leveraged h. Sale and lease-back i. Full-payout Match one or more of thes...

See Answer

Q: Explain why the following statements are true: a. In

Explain why the following statements are true: a. In a competitive leasing market, the annual operating lease payment equals the lessor’s equivalent annual cost. b. Operating leases are attractive to...

See Answer

Q: What happens if a bankrupt lessee affirms the lease? What happens

What happens if a bankrupt lessee affirms the lease? What happens if the lease is rejected?

See Answer

Q: How does a leveraged lease differ from an ordinary, long-

How does a leveraged lease differ from an ordinary, long-term financial lease? List the key differences.

See Answer

Q: Acme has branched out to rentals of office furniture to start-

Acme has branched out to rentals of office furniture to start-up companies. Consider a $3,000 desk. Desks last for six years and can be depreciated on a five-year MACRS schedule (see Table 6.4). What...

See Answer

Q: Refer again to Problem 8. Suppose a blue-chip company

Refer again to Problem 8. Suppose a blue-chip company requests a six year financial lease for a $3,000 desk. The company has just issued five-year notes at an interest rate of 6% per year. What is the...

See Answer

Q: In Problem 8 we assumed identical lease rates for old and new

In Problem 8 we assumed identical lease rates for old and new desks. a. How does the initial break-even lease rate change if the expected inflation rate is 5% per year? Assume that the real cost of ca...

See Answer

Q: The price of Moria Mining stock is $100. During each

The price of Moria Mining stock is $100. During each of the next two six-month periods the price may either rise by 25% or fall by 20% (equivalent to a standard deviation of 31.5% a year). At month 6...

See Answer

Q: Look at Table 25.1. 

Look at Table 25.1.  How would the initial break-even operating lease rate change if rapid technological change in limo manufacturing reduces the costs of new limos by 5% per year?

See Answer

Q: Suppose that National Waferonics has before it a proposal for a four

Suppose that National Waferonics has before it a proposal for a four-year financial lease. The firm constructs a table like Table 25.2. The bottom line of its table shows the lease cash flows:  Thes...

See Answer

Q: Look again at the bus lease described in Table 25.2

Look again at the bus lease described in Table 25.2.  a. What is the value of the lease if Greymare’s marginal tax rate is Tc = .20? b. What would the lease value be if, for tax purposes, the initia...

See Answer

Q: Buffelhead’s stock price is $220 and could halve or double in

Buffelhead’s stock price is $220 and could halve or double in each six month period (equivalent to a standard deviation of 98%). A one-year call option on Buffelhead has an exercise price of $165. The...

See Answer

Q: Suppose that you own an American put option on Bufflehead stock (

Suppose that you own an American put option on Bufflehead stock (see Problem 12) with an exercise price of $220. a. Would you ever want to exercise the put early? b. Calculate the value of the put....

See Answer

Q: Recalculate the value of the Buffelhead call option (see Problem 12

Recalculate the value of the Buffelhead call option (see Problem 12), assuming that the option is American and that at the end of the first six months the company pays a dividend of $25. (Thus the pri...

See Answer

Q: Suppose that you have an option that allows you to sell Buffelhead

Suppose that you have an option that allows you to sell Buffelhead stock (see Problem 12) in month 6 for $165 or to buy it in month 12 for $165. What is the value of this unusual option? Problem 12: B...

See Answer

Q: The current price of the stock of Mont Tremblant Air is C

The current price of the stock of Mont Tremblant Air is C$100. During each six-month period it will either rise by 11.1% or fall by 10% (equivalent to an annual standard deviation of 14.9%). The inter...

See Answer

Q: The current price of United Carbon (UC) stock is $

The current price of United Carbon (UC) stock is $200. The standard deviation is 22.3% a year, and the interest rate is 21% a year. A one-year call option on UC has an exercise price of $180. a. Use...

See Answer

Q: Suppose you construct an option hedge by buying a levered position in

Suppose you construct an option hedge by buying a levered position in delta shares of stock and selling one call option. As the share price changes, the option delta changes, and you will need to adju...

See Answer

Q: a. In Section 21-3 we calculated the risk (

a. In Section 21-3 we calculated the risk (beta) of a six-month call option on Google stock with an exercise price of $530. Now repeat the exercise for a similar option with an exercise price of $450....

See Answer

Q: a. Can the delta of a call option be greater than

a. Can the delta of a call option be greater than 1.0? Explain. b. Can it be less than zero? c. How does the delta of a call change if the stock price rises? d. How does it change if the risk of th...

See Answer

Q: Other things equal, which of these American options are you most

Other things equal, which of these American options are you most likely to want to exercise early? a. A put option on a stock with a large dividend or a call on the same stock. b. A put option on a...

See Answer

Q: Is it better to exercise a call option on the with-

Is it better to exercise a call option on the with-dividend date or on the ex-dividend date? How about a put option? Explain.

See Answer

Q: Use the Black–Scholes program from the Beyond the Page feature

Use the Black–Scholes program from the Beyond the Page feature to value the Owens Corning warrants described in Section 21-4. The standard deviation of Owens Corning stock was 41% a year and the inter...

See Answer

Q: A start-up company is moving into its first offices and

A start-up company is moving into its first offices and needs desks, chairs, filing cabinets, and other furniture. It can buy the furniture for $25,000 or rent it for $1,500 per month. The founders ar...

See Answer

Q: Use the put-call parity formula (see Section 20-

Use the put-call parity formula (see Section 20-2) and the one-period binomial model to show that the option delta for a put option is equal to the option delta for a call option minus 1. Section 20-2...

See Answer

Q: Show how the option delta changes as the stock price rises relative

Show how the option delta changes as the stock price rises relative to the exercise price. Explain intuitively why this is the case. (What happens to the option delta if the exercise price of an optio...

See Answer

Q: Your company has just awarded you a generous stock option scheme.

Your company has just awarded you a generous stock option scheme. You suspect that the board will either decide to increase the dividend or announce a stock repurchase program. Which do you secretly h...

See Answer

Q: Flip back to Tables 6.2 and 6.6,

Flip back to Tables 6.2 and 6.6, where we assumed an economic life of seven years for IM&€™s guano plant. What’s wrong with that assumption? How would you undertake a m...

See Answer

Q: In Section 21-1 we used a simple one-step

In Section 21-1 we used a simple one-step model to value two Google options each with an exercise price of $530. We showed that the call option could be replicated by borrowing $233.22 and investing $...

See Answer

Q: Some corporations have issued perpetual warrants. Warrants are call options issued

Some corporations have issued perpetual warrants. Warrants are call options issued by a firm, allowing the warrant holder to buy the firm’s stock. a. What does the Black–Scholes formula predict for t...

See Answer

Q: Take another look at our two-step binomial trees for Google

Take another look at our two-step binomial trees for Google, for example, in Figure 21.2. Use the replicating-portfolio or risk-neutral method to value six-month call and put options with...

See Answer

Q: You own a parcel of vacant land. You can develop it

You own a parcel of vacant land. You can develop it now, or wait. a. What is the advantage of waiting? b. Why might you decide to develop the property immediately?

See Answer

Q: Gas turbines are among the least efficient ways to produce electricity,

Gas turbines are among the least efficient ways to produce electricity, much less thermally efficient than coal or nuclear plants. Why do gas-turbine generating stations exist? What’s the option?

See Answer

Q: Why is quantitative valuation of real options often difficult in practice?

Why is quantitative valuation of real options often difficult in practice? List the reasons briefly.

See Answer

Q: Alert financial managers can create real options. Give three or four

Alert financial managers can create real options. Give three or four possible examples.

See Answer

Q: Josh Kidding, who has only read part of Chapter 10,

Josh Kidding, who has only read part of Chapter 10, decides to value a real option by (1) setting out a decision tree, with cash flows and probabilities forecasted for each future outcome; (2) decid...

See Answer

Q: Redo the example in Figure 22.8, assuming that the

Redo the example in Figure 22.8, assuming that the real option is a put option allowing the company to abandon the R&D program if commercial prospects are sufficiently poor at year 2. Use put&acir...

See Answer

Q: In Chapter 4, we expressed the value of a share of

In Chapter 4, we expressed the value of a share of stock as P0 = EPS1 /r + PVGO where EPS1 is earnings per share from existing assets, r is the expected rate of return required by investors, and P...

See Answer

Q: True or false? a. Real-options analysis sometimes

True or false? a. Real-options analysis sometimes tells firms to make negative-NPV investments to secure future growth opportunities. b. Using the Black–Scholes formula to value options to invest is...

See Answer

Q: Describe each of the following situations in the language of options:

Describe each of the following situations in the language of options: a. Drilling rights to undeveloped heavy crude oil in Northern Alberta. Development and production of the oil is a negative-NPV en...

See Answer

Q: Look again at Table 22.2. How does the value

Look again at Table 22.2. How does the value in 1982 of the option to invest in the Mark II change if a. The investment required for the Mark II is $800 million (vs. $900 million)? b. The present val...

See Answer

Q: Imagine that Google’s stock price will either rise by 33.3

Imagine that Google’s stock price will either rise by 33.3% or fall by 25% over the next six months (see Section 21-1). Recalculate the value of the call option (exercise price = $530) using (a) the...

See Answer

Q: Look back at the Malted Herring option in Section 22-2

Look back at the Malted Herring option in Section 22-2. How did the company’s analysts estimate the present value of the project? It turns out that they assumed that the probability...

See Answer

Q: You own a one-year call option to buy one acre

You own a one-year call option to buy one acre of Los Angeles real estate. The exercise price is $2 million, and the current, appraised market value of the land is $1.7 million. The land is currently...

See Answer

Q: A variation on Problem 12: Suppose the land is occupied by

A variation on Problem 12: Suppose the land is occupied by a warehouse generating rents of $150,000 after real estate taxes and all other out-of-pocket costs. The present value of the land plus wareho...

See Answer

Q: You have an option to purchase all of the assets of the

You have an option to purchase all of the assets of the Overland Railroad for $2.5 billion. The option expires in nine months. You estimate Overland’s current (month 0) present value...

See Answer

Q: In Section 10-4 we considered two production technologies for a

In Section 10-4 we considered two production technologies for a new Wankel-engined outboard motor. Technology A was the most efficient but had no salvage value if the new outboards failed to sell. Tec...

See Answer

Q: Respond to the following comments. a. “You don’t

Respond to the following comments. a. “You don’t need option pricing theories to value flexibility. Just use a decision tree. Discount the cash flows in the tree at the company cost of capital.” b. “...

See Answer

Q: In binomial trees, risk-neutral probabilities are set to generate

In binomial trees, risk-neutral probabilities are set to generate an expected rate of return equal to the risk-free interest rate in each branch of the tree. What do you think of the following stateme...

See Answer

Q: Suppose you expect to need a new plant that will be ready

Suppose you expect to need a new plant that will be ready to produce turbo-encabulators in 36 months. If design A is chosen, construction must begin immediately. Design B is more expensive, but you ca...

See Answer

Q: You have an A-rated bond. Is a rise in

You have an A-rated bond. Is a rise in rating more likely than a fall? Would your answer be the same if the bond were B-rated?

See Answer

Q: Why is it more difficult to estimate the value at risk for

Why is it more difficult to estimate the value at risk for a portfolio of loans rather than for a single loan? Why did this pose a problem for rating agencies that needed to assess the risk of package...

See Answer

Q: Over the coming year Ragwort’s stock price will halve to $50

Over the coming year Ragwort’s stock price will halve to $50 from its current level of $100 or it will rise to $200. The one-year interest rate is 10%. a. What is the delta of a one-year call option...

See Answer

Q: Other things equal, would you expect the difference between the price

Other things equal, would you expect the difference between the price of a Treasury bond and a corporate bond to increase or decrease with a. The company’s business risk? b. The degree of leverage?

See Answer

Q: The difference between the value of a government bond and a simple

The difference between the value of a government bond and a simple corporate bond is equal to the value of an option. What is this option and what is its exercise price?

See Answer

Q: What variables are required to use a market-based approach to

What variables are required to use a market-based approach to calculate the probability that a company will default on its debt?

See Answer

Q: Use the Black–Scholes model and redraw Figures 23.5

Use the Black–Scholes model and redraw Figures 23.5 and 23.6 assuming that the standard deviation of the return on the firm’s assets is 40% a year. Do the calculations for 60% leverage only. (Hint: It...

See Answer

Q: You own a 5% bond maturing in two years and priced

You own a 5% bond maturing in two years and priced at 87%. Suppose that there is a 10% chance that at maturity the bond will default and you will receive only 40% of the promised payment. What is the...

See Answer

Q: The following table shows some financial data for two companies:

The following table shows some financial data for two companies:  Use the formula shown in Section 23-4 to calculate which has the higher probability of default.

See Answer

Q: Company A has issued a single zero-coupon bond maturing in

Company A has issued a single zero-coupon bond maturing in 10 years. Company B has issued a coupon bond maturing in 10 years. Explain why it is more complicated to value B’s debt than A’s.

See Answer

Q: Company X has borrowed $150 maturing this year and $50

Company X has borrowed $150 maturing this year and $50 maturing in 10 years. Company Y has borrowed $200 maturing in five years. In both cases asset value is $140. Sketch a scenario in which X does no...

See Answer

Q: Discuss the problems with developing a numerical credit scoring system for evaluating

Discuss the problems with developing a numerical credit scoring system for evaluating personal loans. You can only test your system using data for applicants who have in the past been granted credit....

See Answer

Q: What problems are you likely to encounter when using a market-

What problems are you likely to encounter when using a market-based approach for estimating the probability that a company will default?

See Answer

Q: Use the Black–Scholes formula to value the following options:

Use the Black–Scholes formula to value the following options: a. A call option written on a stock selling for $60 per share with a $60 exercise price. The stock’s standard deviation is 6% per month....

See Answer

Q: How much would it cost you to insure the bonds of Backwoods

How much would it cost you to insure the bonds of Backwoods Chemical against default? (See Section 23-1.) Section 23-1: (In 2009, Caesars Entertainment issued $3.7 billion of second lien notes maturin...

See Answer

Q: Digital Organics has 10 million outstanding shares trading at $25 per

Digital Organics has 10 million outstanding shares trading at $25 per share. It also has a large amount of debt outstanding, all coming due in one year. The debt pays interest at 8%. It has a par (fac...

See Answer

Q: Look back at the first Backwoods Chemical example at the start of

Look back at the first Backwoods Chemical example at the start of Section 23-1. Suppose that the firm’s book balance sheet is  The debt has a one-year maturity and a promised interest payment of 9%....

See Answer

Q: It was one of Morse’s most puzzling cases. That morning

It was one of Morse’s most puzzling cases. That morning Rupert Thorndike, the autocratic CEO of Thorndike Oil, was found dead in a pool of blood on his bedroom floor. He had been sh...

See Answer

Q: Select the most appropriate term from within the parentheses: a

Select the most appropriate term from within the parentheses: a. (High-grade utility bonds/Low-grade industrial bonds) generally have only light sinking fund requirements. b. Collateral trust bonds ar...

See Answer

Q: For each of the following sinking funds, state whether the fund

For each of the following sinking funds, state whether the fund increases or decreases the value of the bond at the time of issue (or whether it is impossible to say): a. An optional sinking fund oper...

See Answer

Q: Use Table 24.1 (but not the text) to

Use Table 24.1 (but not the text) to answer the following questions: a. Who are the principal underwriters for the J.C. Penney bond issue? b. Who is the trustee for the issue? c. How many dollars does...

See Answer

Q: Look at Table 24.1:  a

Look at Table 24.1:  a. Suppose the debenture was issued on September 1, 1992, at 99.489%. How much would you have to pay to buy one bond delivered on September 15? Don’t forget to include accrued i...

See Answer

Q: Explain the three principal ways in which the terms of private placement

Explain the three principal ways in which the terms of private placement bonds commonly differ from those of public issues.

See Answer

Q: True or false? a. Convertible bonds are usually senior

True or false? a. Convertible bonds are usually senior claims on the firm. b. The higher the conversion ratio, the more valuable the convertible. c. The higher the conversion price, the more valuable...

See Answer

Q: “A call option is always riskier than the stock it is

“A call option is always riskier than the stock it is written on.” True or false? How does the risk of an option change when the stock price changes?

See Answer

Q: A puttable bond is a bond that may be repaid before maturity

A puttable bond is a bond that may be repaid before maturity at the investor’s option. Sketch a diagram similar to Figure 24.3 showing the relationship between the value of a straight bond and that of...

See Answer

Q: Alpha Corp. is prohibited from issuing more senior debt unless net

Alpha Corp. is prohibited from issuing more senior debt unless net tangible assets exceed 200% of senior debt. Currently the company has outstanding $100 million of senior debt and has net tangible as...

See Answer

Q: Iota Microsystems’ 10% convertible is about to mature. The conversion

Iota Microsystems’ 10% convertible is about to mature. The conversion ratio is 27. a. What is the conversion price? b. The stock price is $47. What is the conversion value? c. Should you convert?

See Answer

Q: In 1996, Marriott International made an issue of unusual bonds called

In 1996, Marriott International made an issue of unusual bonds called liquid yield option notes, or LYONS. The bond matured in 2011, had a zero coupon, and was issued at $532.15. It could have been co...

See Answer

Q: Zenco, Inc. is financed by 3 million shares of common

Zenco, Inc. is financed by 3 million shares of common stock and by $5 million face value of 8% convertible debt maturing in 2026. Each bond has a face value of $1,000 and a conversion ratio of 200. Wh...

See Answer

Q: Dorlcote Milling has outstanding a $1 million 3% mortgage bond

Dorlcote Milling has outstanding a $1 million 3% mortgage bond maturing in 10 years. The coupon on any new debt issued by the company is 10%. The finance director, Mr. Tulliver, cannot decide whether...

See Answer

Q: This question illustrates that when there is scope for the firm to

This question illustrates that when there is scope for the firm to vary its risk, lenders may be more prepared to lend if they are offered a piece of the action through the issue of a convertible bond...

See Answer

Q: Occasionally it is said that issuing convertible bonds is better than issuing

Occasionally it is said that issuing convertible bonds is better than issuing stock when the firm’s shares are undervalued. Suppose that the financial manager of the Butternut Furniture Company does h...

See Answer

Q: a. As a senior bondholder, would you like the company

a. As a senior bondholder, would you like the company to issue more junior debt to finance its investment program, would you prefer it not to do so, or would you not care? b. You hold debt secured on...

See Answer

Q: True or false? Briefly explain in each case. a

True or false? Briefly explain in each case. a. It is better to hold unsecured bonds than secured bonds in the event of default. b. Many new and exotic debt securities are triggered by government poli...

See Answer

Q: Question: For which of the following options might it be

For which of the following options might it be rational to exercise before maturity? Explain briefly why or why not. a. American put on a non-dividend-paying stock. b. American call—the dividend...

See Answer

Q: Maple Aircraft has issued a 4¾% convertible subordinated debenture due 2020

Maple Aircraft has issued a 4¾% convertible subordinated debenture due 2020. The conversion price is $47.00 and the debenture is callable at 102.75% of face value. The market price of the convertible...

See Answer

Q: Suppose that the J.C. Penney bond was issued at

Suppose that the J.C. Penney bond was issued at face value and that investors continue to demand a yield of 8.25%. Sketch what you think would happen to the bond price as the first interest payment da...

See Answer

Q: Bond prices can fall either because of a change in the general

Bond prices can fall either because of a change in the general level of interest rates or because of an increased risk of default. To what extent do floating-rate bonds and puttable bonds protect the...

See Answer

Q: Proctor Power has fixed assets worth $200 million and net working

Proctor Power has fixed assets worth $200 million and net working capital worth $100 million. It is financed partly by equity and partly by three issues of debt. These consist of $250 million of First...

See Answer

Q: Elixir Corporation has just filed for bankruptcy. Elixir is a holding

Elixir Corporation has just filed for bankruptcy. Elixir is a holding company whose assets consist of real estate worth $80 million and 100% of the equity of its two operating subsidiaries. It is fina...

See Answer

Q: a. Residential mortgages may stipulate either a fixed rate or a

a. Residential mortgages may stipulate either a fixed rate or a variable rate. As a borrower, what considerations might cause you to prefer one rather than the other? b. Why might holders of mortgage...

See Answer

Q: After a sharp change in interest rates, newly issued bonds generally

After a sharp change in interest rates, newly issued bonds generally sell at yields different from those of outstanding bonds of the same quality. One suggested explanation is that there is a differen...

See Answer

Q: Suppose that a company simultaneously issues a zero-coupon bond and

Suppose that a company simultaneously issues a zero-coupon bond and a coupon bond with identical maturities. Both are callable at any time at their face values. Other things equal, which is likely to...

See Answer

Q: a. If interest rates rise, will callable or noncallable bonds

a. If interest rates rise, will callable or noncallable bonds fall more in price? b. Sometimes you encounter bonds that can be repaid after a fixed interval at the option of either the issuer or the b...

See Answer

Q: Explain carefully why bond indentures may place limitations on the following actions

Explain carefully why bond indentures may place limitations on the following actions: a. Sale of the company’s assets. b. Payment of dividends to shareholders. c. Issue of additional senior debt.

See Answer

Q: Johnny Jones’s high school derivatives homework asks for a binomial valuation of

Johnny Jones’s high school derivatives homework asks for a binomial valuation of a 12-month call option on the common stock of the Overland Railroad. The stock is now selling for $45...

See Answer

Q: The Surplus Value Company had $10 million (face value)

The Surplus Value Company had $10 million (face value) of convertible bonds outstanding in 2015. Each bond has the following features. Face value....................................$1,000 Conver...

See Answer

Q: Piglet Pies has issued a zero-coupon 10-year bond

Piglet Pies has issued a zero-coupon 10-year bond that can be converted into 10 Piglet shares. Comparable straight bonds are yielding 8%. Piglet stock is priced at $50 a share. a. Suppose that you had...

See Answer

Q: n Section 25-4 we showed that the lease offered to

n Section 25-4 we showed that the lease offered to Greymare Bus Lines had a positive NPV of $820 if Greymare paid no tax and a +$700 NPV to a lessor paying 35% tax. What is the minimum lease payment t...

See Answer

Q: In Section 25-5 we listed four circumstances in which there

In Section 25-5 we listed four circumstances in which there are potential gains from leasing. Check them out by conducting a sensitivity analysis on the Greymare Bus Lines lease, assuming that Greymar...

See Answer

Q: In Section 25-5 we stated that if the interest rate

In Section 25-5 we stated that if the interest rate were zero, there would be no advantage in postponing tax and therefore no advantage in leasing. Value the Greymare Bus Lines lease with an interest...

See Answer

Q: A lease with a varying rental schedule is known as a structured

A lease with a varying rental schedule is known as a structured lease. Try structuring the Greymare Bus Lines lease to increase value to the lessee while preserving the value to the lessor. Assume tha...

See Answer

Q: Nodhead College needs a new computer. It can either buy it

Nodhead College needs a new computer. It can either buy it for $250,000 or lease it from Compulease. The lease terms require Nodhead to make six annual payments (prepaid) of $62,000. Nodhead pays no t...

See Answer

Q: The Safety Razor Company has a large tax-loss carry forward

The Safety Razor Company has a large tax-loss carry forward and does not expect to pay taxes for another 10 years. The company is therefore proposing to lease $100,000 of new machinery. The lease term...

See Answer

Q: How does the position of an equipment lessor differ from the position

How does the position of an equipment lessor differ from the position of a secured lender when a firm falls into bankruptcy? Assume that the secured loan would have the leased equipment as collateral....

See Answer

Q: How would the lessee in Figure 25.1 evaluate the NPV

How would the lessee in Figure 25.1 evaluate the NPV of the lease?  Sketch the correct valuation procedure. Then suppose that the equity lessor wants to evaluate the lease. Again sketch the correct...

See Answer

Q: Define the following terms: a. Spot price b

Define the following terms: a. Spot price b. Forward vs. futures contract c. Long vs. short position d. Basis risk e. Mark to market f. Net convenience yield

See Answer

Q: What is basis risk? In which of the following cases would

What is basis risk? In which of the following cases would you expect basis risk to be serious? a. A broker owning a large block of Disney common stock hedges by selling index futures. b. An lowa corn...

See Answer

Q: Construct a new model for Dynamic Mattress based on your answer to

Construct a new model for Dynamic Mattress based on your answer to Problem 21. Does your model generate a feasible financial plan for 2016? Problem 21: The balancing item in the Dynamic long-term pla...

See Answer

Q: a. Use the Dynamic Mattress model (Tables 29.9

a. Use the Dynamic Mattress model (Tables 29.9 to 29.11) and the spreadsheets to produce pro forma income statements, balance sheets, and statements of cash flows for 2016 and 2017. Assume business as...

See Answer

Q: The financial statements of Eagle Sport Supply are shown in Table 29

The financial statements of Eagle Sport Supply are shown in Table 29.18. For simplicity, “Costs” include interest. Assume that Eagle’s...

See Answer

Q: a. What is the internal growth rate of Eagle Sport (

a. What is the internal growth rate of Eagle Sport (see Problem 24) if the dividend payout ratio is fixed at 60% and the equity-to-asset ratio is fixed at two-thirds? b. What is the sustainable growth...

See Answer

Q: Bio-Plasma Corp. is growing at 30% per year

Bio-Plasma Corp. is growing at 30% per year. It is all-equity-financed and has total assets of $1 million. Its return on equity is 20%. Its plowback ratio is 40%. a. What is the internal growth rate?...

See Answer

Q: Table 29.19 shows the 2016 financial statements for the Executive

Table 29.19 shows the 2016 financial statements for the Executive Cheese Company. Annual depreciation is 10% of fixed assets at the beginning of the year, plus 10% of new investment. The c...

See Answer

Q: The lag between the purchase date and the date on which payment

The lag between the purchase date and the date on which payment is due is known as the terms lag. The lag between the due date and the date on which the buyer actually pays is the due lag, and the lag...

See Answer

Q: The Branding Iron Company sells its irons for $50 apiece wholesale

The Branding Iron Company sells its irons for $50 apiece wholesale. Production cost is $40 per iron. There is a 25% chance that wholesaler Q will go bankrupt within the next year. Q orders 1,000 irons...

See Answer

Q: Look back at the discussion in Section 30-2 of credit

Look back at the discussion in Section 30-2 of credit decisions with repeat orders. If p1 = .8, what is the minimum level of p2 at which Cast Iron is justified in extending credit?

See Answer

Q: True or false? a. Exporters who require greater certainty

True or false? a. Exporters who require greater certainty of payment arrange for the customers to sign a bill of lading in exchange for a sight draft. b. It makes sense to monitor the credit manager...

See Answer

Q: You own a $1 million portfolio of aerospace stocks with a

You own a $1 million portfolio of aerospace stocks with a beta of 1.2. You are very enthusiastic about aerospace but uncertain about the prospects for the overall stock market. Explain how you could h...

See Answer

Q: How should your willingness to grant credit be affected by differences in

How should your willingness to grant credit be affected by differences in (a) the profit margin, (b) the interest rate, (c) the probability of repeat orders? In each case illustrate your answer wit...

See Answer

Q: Complete the passage that follows by choosing the appropriate terms from the

Complete the passage that follows by choosing the appropriate terms from the following list: lockbox banking, Fedwire, CHIPS, concentration banking. Firms can increase their cash resources by speeding...

See Answer

Q: In October 2008, six-month (182-day)

In October 2008, six-month (182-day) Treasury bills were issued at a discount of 1.4%. What was the annual yield?

See Answer

Q: For each item below, choose the investment that best fits the

For each item below, choose the investment that best fits the accompanying description: a. Maturity often overnight (repurchase agreements/bankers’ acceptances) b. Maturity never more than 270 days...

See Answer

Q: Some of the items in the previous problem involve a cash discount

Some of the items in the previous problem involve a cash discount. For each of these, calculate the rate of interest paid by customers who pay on the due date instead of taking the cash discount.

See Answer

Q: As treasurer of the Universal Bed Corporation, Aristotle Procrustes is worried

As treasurer of the Universal Bed Corporation, Aristotle Procrustes is worried about his bad debt ratio, which is currently running at 6%. He believes that imposing a more stringent credit policy migh...

See Answer

Q: Look at the previous problem. Assume that the change in credit

Look at the previous problem. Assume that the change in credit terms results in a 2% increase in sales. Recalculate the effect of the changed credit terms. Previous problem: Until recently, Augean Cle...

See Answer

Q: Look again at the previous problem. Suppose another month has passed

Look again at the previous problem. Suppose another month has passed, so the bill has only one month left to run. It is now selling at a discount of 3%. What is the yield? What was your realized retur...

See Answer

Q: Knob, Inc., is a nationwide distributor of furniture hardware.

Knob, Inc., is a nationwide distributor of furniture hardware. The company now uses a central billing system for credit sales of $180 million annually. First National, Knob’s principal bank, offers to...

See Answer

Q: Anne Teak, the financial manager of a furniture manufacturer, is

Anne Teak, the financial manager of a furniture manufacturer, is considering operating a lockbox system. She forecasts that 300 payments a day will be made to lockboxes, with an average payment size o...

See Answer

Q: a. Marshall Arts has just invested $1 million in long

a. Marshall Arts has just invested $1 million in long-term Treasury bonds. Marshall is concerned about increasing volatility in interest rates. He decides to hedge using bond futures contracts. Should...

See Answer

Q: A parent company settles the collection account balances of its subsidiaries once

A parent company settles the collection account balances of its subsidiaries once a week. (That is, each week it transfers any balances in the accounts to a central account.) The cost of a wire transf...

See Answer

Q: Lockboxes The financial manager of JAC Cosmetics is considering opening a lockbox

Lockboxes The financial manager of JAC Cosmetics is considering opening a lockbox in Pittsburgh. Checks cleared through the lockbox will amount to $10,000 per day. The lockbox will make cash available...

See Answer

Q: A three-month Treasury bill and a six-month bill

A three-month Treasury bill and a six-month bill both sell at a discount of 10%. Which offers the higher annual yield?

See Answer

Q: In Section 30-4 we described a three-month bill

In Section 30-4 we described a three-month bill that was issued on an annually compounded yield of 5.16%. Suppose that one month has passed and the investment still offers the same annually compounded...

See Answer

Q: In 2006 agency bonds sold at a yield of 5.32

In 2006 agency bonds sold at a yield of 5.32%, while high-grade tax-exempts of comparable maturity offered 3.7% annually. If an investor receives the same after-tax return from corporates and tax-exem...

See Answer

Q: The IRS prohibits companies from borrowing money to buy tax-exempts

The IRS prohibits companies from borrowing money to buy tax-exempts and deducting the interest payments on the borrowing from taxable income. Should the IRS prohibit such activity? If it didn’t, would...

See Answer

Q: Suppose you are a wealthy individual paying 35% tax on income

Suppose you are a wealthy individual paying 35% tax on income. What is the expected after-tax yield on each of the following investments? a. A municipal note yielding 7.0% pretax. b. A Treasury bill...

See Answer

Q: Reliant Umbrellas has been approached by Plumpton Variety Stores of Nevada.

Reliant Umbrellas has been approached by Plumpton Variety Stores of Nevada. Plumpton has expressed interest in an initial purchase of 5,000 umbrellas at $10 each on Reliant’s standard terms of 2/30, n...

See Answer

Q: Galenic, Inc., is a wholesaler for a range of pharmaceutical

Galenic, Inc., is a wholesaler for a range of pharmaceutical products. Before deducting any losses from bad debts, Galenic operates on a profit margin of 5%. For a long time the firm has employed a nu...

See Answer

Q: What are the trade-offs involved in the decision of how

What are the trade-offs involved in the decision of how much inventory the firm should carry?

See Answer

Q: Large businesses spend millions of dollars annually on insurance. Why?

Large businesses spend millions of dollars annually on insurance. Why? Should they insure against all risks or does insurance make more sense for some risks than others?

See Answer

Q: Company X sells on a 1/30, net 60 basis

Company X sells on a 1/30, net 60 basis. Customer Y buys goods invoiced at $1,000. a. How much can Y deduct from the bill if Y pays on day 30? b. What is the effective annual rate of interest if Y p...

See Answer

Q: Look back at Section 30-2. Cast Iron’s costs have

Look back at Section 30-2. Cast Iron’s costs have increased from $1,000 to $1,050. Assuming there is no possibility of repeat orders, answer the following: a. When should Cast Iron grant or refuse cre...

See Answer

Q: Consider three securities: a. A floating-rate bond

Consider three securities: a. A floating-rate bond b. A preferred share paying a fixed dividend c. A floating-rate preferred If you were responsible for short-term investment of your firm’s excess...

See Answer

Q: Listed below are some common terms of sale. Can you explain

Listed below are some common terms of sale. Can you explain what each means? a. 2/30, net 60 b. 2/5, EOM, net 30 c. COD

See Answer

Q: Phoenix Lambert currently sells its goods cash-on-delivery.

Phoenix Lambert currently sells its goods cash-on-delivery. However, the financial manager believes that by offering credit terms of 2/10 net 30 the company can increase sales by 4%, without significa...

See Answer

Q: Jim Khana, the credit manager of Velcro Saddles, is reappraising

Jim Khana, the credit manager of Velcro Saddles, is reappraising the company’s credit policy. Velcro sells on terms of net 30. Cost of goods sold is 85% of sales, and fixed costs are...

See Answer

Q: Look again at the last problem. Suppose (a)

Look again at the last problem. Suppose (a) that it costs $95 to classify each new credit applicant and (b) that an almost equal proportion of new applicants falls into each of the four categories....

See Answer

Q: Until recently, Augean Cleaning Products sold its products on terms of

Until recently, Augean Cleaning Products sold its products on terms of net 60, with an average collection period of 75 days. In an attempt to induce customers to pay more promptly, it has changed its...

See Answer

Q: Look again at Table 31.3. Suppose that B Corporation’s

Look again at Table 31.3. Suppose that B Corporation’s fixed assets are reexamined and found to be worth $12 million instead of $9 million. How would this affect the AB Corporation’s balance sheet und...

See Answer

Q: Which of the following motives for mergers make economic sense?

Which of the following motives for mergers make economic sense? a. Merging to achieve economies of scale. b. Merging to reduce risk by diversification. c. Merging to redeploy cash generated by a firm...

See Answer

Q: On some catastrophe bonds, payments are reduced if the claims against

On some catastrophe bonds, payments are reduced if the claims against the issuer exceed a specified sum. In other cases payments are reduced only if claims against the entire industry exceed some sum....

See Answer

Q: Velcro Saddles is contemplating the acquisition of Pogo Ski Sticks, Inc

Velcro Saddles is contemplating the acquisition of Pogo Ski Sticks, Inc. The values of the two companies as separate entities are $20 million and $10 million, respectively. Velcro Saddles estimates th...

See Answer

Q: True or false? a. Sellers almost always gain in

True or false? a. Sellers almost always gain in mergers. b. Buyers usually gain more than sellers in acquisitions. c. Firms that do unusually well tend to be acquisition targets. d. Merger activity in...

See Answer

Q: Briefly define the following terms: a. Purchase accounting

Briefly define the following terms: a. Purchase accounting b. Tender offer c. Poison pill d. Golden parachute e. Synergy

See Answer

Q: Respond to the following comments. a. “Our cost

Respond to the following comments. a. “Our cost of debt is too darn high, but our banks won’t reduce interest rates as long as we’re stuck in this volatile widget-trading business. We’ve got to acquir...

See Answer

Q: Sometimes the stock price of a possible target company rises in anticipation

Sometimes the stock price of a possible target company rises in anticipation of a merger bid. Explain how this complicates the bidder’s evaluation of the target company.

See Answer

Q: Suppose you obtain special information—information unavailable to investors—indicating

Suppose you obtain special information—information unavailable to investors—indicating that Backwoods Chemical’s stock price is 40% undervalued. Is that a reason to launch a takeover bid for Backwoods...

See Answer

Q: As treasurer of Leisure Products, Inc., you are investigating the

As treasurer of Leisure Products, Inc., you are investigating the possible acquisition of Plastitoys. You have the following basic data:  You estimate that investors currently expect a steady growth...

See Answer

Q: The Muck and Slurry merger has fallen through (see Section 31

The Muck and Slurry merger has fallen through (see Section 31-2). But World Enterprises is determined to report earnings per share of $2.67. It therefore acquires the Wheelrim and Axle Company. You ar...

See Answer

Q: Explain the distinction between a tax-free and a taxable merger

Explain the distinction between a tax-free and a taxable merger. Are there circumstances in which you would expect buyer and seller to agree to a taxable merger?

See Answer

Q: What is meant by dual-class equity? Do you think

What is meant by dual-class equity? Do you think it should be allowed or outlawed?

See Answer

Q: Why may market-based financial systems be better in supporting innovation

Why may market-based financial systems be better in supporting innovation and in releasing capital from declining industries?

See Answer

Q: Are the following hypothetical mergers horizontal, vertical, or conglomerate?

Are the following hypothetical mergers horizontal, vertical, or conglomerate? a. IBM acquires Dell Computer. b. Dell Computer acquires Walmart. c. Walmart acquires Tyson Foods. d. Tyson Foods acquires...

See Answer

Q: List some of the commodity futures contracts that are traded on exchanges

List some of the commodity futures contracts that are traded on exchanges. Who do you think could usefully reduce risk by buying each of these contracts? Who do you think might wish to sell each contr...

See Answer

Q: Which of the following transactions are not likely to be classed as

Which of the following transactions are not likely to be classed as tax-free? a. A cash acquisition of assets. b. A merger in which payment is entirely in the form of voting stock.

See Answer

Q: What are the government’s motives in a privatization?

What are the government’s motives in a privatization?

See Answer

Q: List the disadvantages of traditional U.S. conglomerates.

List the disadvantages of traditional U.S. conglomerates.

See Answer

Q: What is the difference between Chapter 7 and Chapter 11 bankruptcies?

What is the difference between Chapter 7 and Chapter 11 bankruptcies?

See Answer

Q: Explain why equity can sometimes have a positive value even when companies

Explain why equity can sometimes have a positive value even when companies file for bankruptcy.

See Answer

Q: For what kinds of firm would an LBO or MBO transaction not

For what kinds of firm would an LBO or MBO transaction not be productive?

See Answer

Q: We described several problems with Chapter 11 bankruptcy. Which of these

We described several problems with Chapter 11 bankruptcy. Which of these problems could be mitigated by negotiating a prepackaged bankruptcy?

See Answer

Q: Define the following terms: a. LBO b.

Define the following terms: a. LBO b. MBO c. Spin-off d. Carve-out e. Asset sale f. Privatization g. Leveraged restructuring

See Answer

Q: Table 26.4 contains spot and six-month futures prices

Table 26.4 contains spot and six-month futures prices for several commodities and financial instruments. There may be some money-making opportunities. See if you can find them, and explain how you wou...

See Answer

Q: True or false? a. One of the first tasks

True or false? a. One of the first tasks of an LBO’s financial manager is to pay down debt. b. Once an LBO or MBO goes private, it almost always stays private. c. Targets for LBOs in the 1980s tend...

See Answer

Q: What advantages have been claimed for public conglomerates?

What advantages have been claimed for public conglomerates?

See Answer

Q: Phoenix Motors wants to lock in the cost of 10,000

Phoenix Motors wants to lock in the cost of 10,000 ounces of platinum to be used in next quarter’s production of catalytic converters. It buys three-month futures contracts for 10,000 ounces at a pric...

See Answer

Q: Private-equity partnerships have a limited term. What are the

Private-equity partnerships have a limited term. What are the advantages of this arrangement?

See Answer

Q: True or false? a. When a company becomes bankrupt

True or false? a. When a company becomes bankrupt, it is usually in the interests of stockholders to seek a liquidation rather than a reorganization. b. In Chapter 11 a reorganization plan must be p...

See Answer

Q: True, false, or “It depends on. . .”?

True, false, or “It depends on. . .”? a. Carve-out or spin-off of a division improves incentives for the division’s managers. b. Private-equity partnerships have limited lives. The main purpose is t...

See Answer

Q: The Sealed Air leveraged restructuring is described in the Chapter 18 Beyond

The Sealed Air leveraged restructuring is described in the Chapter 18 Beyond the Page feature. Outline the similarities and differences between the RJR Nabisco LBO and the Sealed Air restructuring. We...

See Answer

Q: Read Barbarians at the Gate (Further Reading). What agency costs

Read Barbarians at the Gate (Further Reading). What agency costs can you identify? Do you think the LBO was well-designed to reduce these costs?

See Answer

Q: Explain the structure of a private-equity partnership. Pay particular

Explain the structure of a private-equity partnership. Pay particular attention to incentives and compensation. What types of investment were such partnerships designed to make?

See Answer

Q: We described carried interest as an option. What kind of option

We described carried interest as an option. What kind of option? How does this option change incentives in a private-equity partnership? Can you think of circumstances where these incentive changes wo...

See Answer

Q: True or false? a. Hedging transactions in an active

True or false? a. Hedging transactions in an active futures market have zero or slightly negative NPVs. b. When you buy a futures contract, you pay now for delivery at a future date. c. The holder of...

See Answer

Q: “Privatization appears to bring efficiency gains because public companies are better

“Privatization appears to bring efficiency gains because public companies are better able to reduce agency costs.” Why do you think this may (or may not) be true?

See Answer

Q: What are some of the advantages and disadvantages of Japanese keiretsu’s?

What are some of the advantages and disadvantages of Japanese keiretsu’s?

See Answer

Q: Which countries have a. The largest stock markets?

Which countries have a. The largest stock markets? b. The largest bond markets? c. The smallest direct holdings of shares by individual investors? d. The largest holdings of bank deposits by indiv...

See Answer

Q: In December 2014, 6-month futures on the Australian S

In December 2014, 6-month futures on the Australian S&P/ASX 200 Index traded at 5,376. Spot was 5,442. The interest rate was 2.5%, and the dividend yield was about 4.7%. Were the futures fairly priced...

See Answer

Q: What is meant by the German system of codetermination?

What is meant by the German system of codetermination?

See Answer

Q: What is tunneling? Why does the threat of tunneling impede the

What is tunneling? Why does the threat of tunneling impede the development of financial markets?

See Answer

Q: Agency problems are inevitable. That is, we can never expect

Agency problems are inevitable. That is, we can never expect managers to give 100% weight to shareholders’ interests and none to their own. a. Why not? b. List the mechanisms that are used around th...

See Answer

Q: Banks are not the only financial intermediary from which corporations can obtain

Banks are not the only financial intermediary from which corporations can obtain financing. What are the other intermediaries? How much financing do they supply, relative to banks, in the United Kingd...

See Answer

Q: Why is transparency important in a market-based financial system?

Why is transparency important in a market-based financial system? Why is it less important in a bank-based system?

See Answer

Q: The following table shows 2014 gold futures prices for varying contract lengths

The following table shows 2014 gold futures prices for varying contract lengths. Gold is predominantly an investment good, not an industrial commodity. Investors hold gold because it diversifies their...

See Answer

Q: In September 2020 swap dealers were quoting a rate for five-

In September 2020 swap dealers were quoting a rate for five-year euro interest rate swaps of 4.5% against Euribor (the short-term interest rate for euro loans). Euribor at the time was 4.1%. Suppose t...

See Answer

Q: Securities A, B, and C have the following cash flows

Securities A, B, and C have the following cash flows:  a. Calculate their durations if the interest rate is 8%. b. Suppose that you have an investment of $10 million in A. What combination of B and...

See Answer

Q: What is meant by “delta” (δ) in the

What is meant by “delta” (δ) in the context of hedging? Give examples of how delta can be estimated or calculated.

See Answer

Q: A gold-mining firm is concerned about short-term volatility

A gold-mining firm is concerned about short-term volatility in its revenues. Gold currently sells for $1,300 an ounce, but the price is extremely volatile and could fall as low as $1,220 or rise as hi...

See Answer

Q: Legs Diamond owns shares in a Vanguard Index 500 mutual fund worth

Legs Diamond owns shares in a Vanguard Index 500 mutual fund worth $1 million on July 15. (This is an index fund that tracks the Standard and Poor’s 500 Index.) He wants to cash in now, but his accoun...

See Answer

Q: Price changes of two gold-mining stocks have shown strong positive

Price changes of two gold-mining stocks have shown strong positive correlation. Their historical relationship is Average percentage change in A = .001 + .75 (percentage change in B) Changes in B expla...

See Answer

Q: What kind of industries do you think should thrive in a market

What kind of industries do you think should thrive in a market-based financial system? In a bank-based system?

See Answer

Q: Petrochemical Parfum (PP) is concerned about a possible increase in

Petrochemical Parfum (PP) is concerned about a possible increase in the price of heavy fuel oil, which is one of its major inputs. Show how PP can use either options or futures contracts to protect it...

See Answer

Q: Consider the commodities and financial assets listed in Table 26.5

Consider the commodities and financial assets listed in Table 26.5. The risk-free interest rate is 6% a year, and the term structure is flat. a. Calculate the six-month futures price for each case. b....

See Answer

Q: Is a total return swap on a bond the same as a

Is a total return swap on a bond the same as a credit default swap? Why or why not?

See Answer

Q: Yesterday you sold six-month futures on the German DAX stock

Yesterday you sold six-month futures on the German DAX stock market index at a price of 9,120. Today the DAX closed at 9,100 and DAX futures closed at 9,140. You get a call from your broker, who remin...

See Answer

Q: “Speculators want futures contracts to be incorrectly priced; hedgers want

“Speculators want futures contracts to be incorrectly priced; hedgers want them to be correctly priced.” Why?

See Answer

Q: Your investment bank has an investment of $100 million in the

Your investment bank has an investment of $100 million in the stock of the Swiss Roll Corporation and a short position in the stock of the Frankfurter Sausage Company. Here is the recent price history...

See Answer

Q: Phillip’s Screwdriver Company has borrowed $20 million from a bank at

Phillip’s Screwdriver Company has borrowed $20 million from a bank at a floating interest rate of 2 percentage points above three-month Treasury bills, which now yield 5%. Assume that interest payment...

See Answer

Q: An importer in the United States is due to take delivery of

An importer in the United States is due to take delivery of clothing from Mexico in six months. The price is fixed in Mexican pesos. Which of the following transactions could eliminate the importer’s...

See Answer

Q: Suppose that in 2023 one- and two-year interest rates

Suppose that in 2023 one- and two-year interest rates are 5.2% in the United States and 1.0% in Japan. The spot exchange rate is ¥120.22/$. Suppose that one year later interest rates are 3% in both co...

See Answer

Q: It is the year 2021 and Pork Barrels Inc. is considering

It is the year 2021 and Pork Barrels Inc. is considering construction of a new barrel plant in Spain. The forecasted cash flows in millions of euros are as follows: The spot exchange rate is $1.2&Acir...

See Answer

Q: Why are pyramids common in many countries but not in the United

Why are pyramids common in many countries but not in the United States or United Kingdom?

See Answer

Q: Table 27.1 shows the 90-day forward rate on

Table 27.1 shows the 90-day forward rate on the South African rand. a. Is the dollar at a forward discount or premium on the rand? b. What is the annual percentage discount or premium?...

See Answer

Q: Look at Table 27.1. If the three-month

Look at Table 27.1. If the three-month interest rate on dollars is 0.2%, what do you think is the three-month interest rate on the Brazilian real? Explain what would happen if the rate wer...

See Answer

Q: Penny Farthing, the treasurer of International Bicycles, Inc., has

Penny Farthing, the treasurer of International Bicycles, Inc., has noticed that the interest rate in Japan is below the rates in most other countries. She is, therefore, suggesting that the company sh...

See Answer

Q: Suppose you are the treasurer of Lufthansa, the German international airline

Suppose you are the treasurer of Lufthansa, the German international airline. How is company value likely to be affected by exchange rate changes? What policies would you adopt to reduce exchange rate...

See Answer

Q: Exacta, s.a. Exacta, s.a

Exacta, s.a. Exacta, s.a., is a major French producer, based in Lyons, of precision machine tools. About two thirds of its output is exported. The majority of these sales is within the European Union....

See Answer

Q: Companies may be affected by changes in the nominal exchange rate or

Companies may be affected by changes in the nominal exchange rate or in the real exchange rate. Explain how this can occur. Which changes are easiest to hedge against?

See Answer

Q: A Ford dealer in the United States may be exposed to a

A Ford dealer in the United States may be exposed to a devaluation of the yen if this leads to a cut in the price of Japanese cars. Suppose that the dealer estimates that a 1% decline in the value of...

See Answer

Q: You have bid for a possible export order that would provide a

You have bid for a possible export order that would provide a cash inflow of €1 million in six months. The spot exchange rate is $1.3549 = €1 and the six-month forward rate is $1.3620 = €1. There are...

See Answer

Q: In November 2014, an American investor buys 1,000 shares

In November 2014, an American investor buys 1,000 shares in a Mexican company at a price of 500 pesos each. The share does not pay any dividend. A year later she sells the shares for 550 pesos each. T...

See Answer

Q: Table 27.5 shows the annual interest rate (annually compounded

Table 27.5 shows the annual interest rate (annually compounded) and exchange rates against the dollar for different currencies. Are there any arbitrage opportunities? If so, how would you...

See Answer

Q: What is a keiretsu? Give a brief description.

What is a keiretsu? Give a brief description.

See Answer

Q: “Last year we had a substantial income in sterling, which

“Last year we had a substantial income in sterling, which we hedged by selling sterling forward. In the event sterling appreciated. So our decision to sell forward cost us a lot of money. I think that...

See Answer

Q: Carpet Baggers, Inc., is proposing to construct a new bagging

Carpet Baggers, Inc., is proposing to construct a new bagging plant in a country in Europe. The two prime candidates are Germany and Switzerland. The forecasted cash flows from the proposed plants are...

See Answer

Q: Alpha and Omega are U.S. corporations. Alpha has

Alpha and Omega are U.S. corporations. Alpha has a plant in Hamburg that imports components from the United States, assembles them, and then sells the finished product in Germany. Omega is at the oppo...

See Answer

Q: Look at Table 27.1. a. How many

Look at Table 27.1. a. How many Japanese yen do you get for your dollar? b. What is the one-month forward rate for yen? c. Is the yen at a forward discount or premium on the dollar? d....

See Answer

Q: Define each of the following theories in a sentence or simple equation

Define each of the following theories in a sentence or simple equation: a. Interest rate parity. b. Expectations theory of forward rates. c. Purchasing power parity. d. International capital marke...

See Answer

Q: “Northern Refineries does not avoid risk by selling oil futures.

“Northern Refineries does not avoid risk by selling oil futures. If prices stay above $2.40 a gallon, then it will actually have lost by selling oil futures at that price.” Is this a fair comment?

See Answer

Q: In March 1997, the exchange rate for the Indonesian rupiah was

In March 1997, the exchange rate for the Indonesian rupiah was R2,419 = $1. Inflation in the year to March 1998 was about 30% in Indonesia and 2% in the United States. a. If purchasing power parity h...

See Answer

Q: A U.S. company has committed to pay 10 million

A U.S. company has committed to pay 10 million kronor to a Swedish company in one year. What is the cost (in present value) of covering this liability by buying kronor forward? The Swedish interest ra...

See Answer

Q: A firm in the United States is due to receive payment of

A firm in the United States is due to receive payment of €1 million in eight years’ time. It would like to protect itself against a decline in the value of the euro, but finds it difficult to get forw...

See Answer

Q: Airlux Antarctica has current assets of $300 million, current liabilities

Airlux Antarctica has current assets of $300 million, current liabilities of $200 million and a crash—sorry—cash ratio of .05. How much cash and marketable securities does it hold?

See Answer

Q: Do Japanese investors play an important role in corporate financial policy and

Do Japanese investors play an important role in corporate financial policy and governance? If not, could they?

See Answer

Q: Question: On average, it takes Microlimp’s customers 60 days

On average, it takes Microlimp’s customers 60 days to pay their bills. If Microlimp has annual sales of $500 million, what is the average value of unpaid bills?

See Answer

Q: True or false? a. A company’s debt–equity

True or false? a. A company’s debt–equity ratio is always less than 1. b. The quick ratio is always less than the current ratio. c. The return on equity is always less than the return on assets.

See Answer

Q: Describe some alternative measures of a firm’s overall performance. What are

Describe some alternative measures of a firm’s overall performance. What are their advantages and disadvantages? In each case discuss what benchmarks you might use to judge whether performance is sati...

See Answer

Q: Discuss alternative measures of financial leverage. Should the market value of

Discuss alternative measures of financial leverage. Should the market value of equity be used or the book value? Is it better to use the market value of debt or the book value? How should you treat of...

See Answer

Q: Suppose that a firm has both fixed-rate and floating-

Suppose that a firm has both fixed-rate and floating-rate debt outstanding. What effect will a decline in interest rates have on the firm’s times-interest-earned ratio? What about the ratio of the mar...

See Answer

Q: How would the following actions affect a firm’s current ratio?

How would the following actions affect a firm’s current ratio? a. Inventory is sold. b. The firm takes out a bank loan to pay its suppliers. c. The firm arranges a line of credit with a bank that al...

See Answer

Q: The following table shows interest rates and exchange rates for the U

The following table shows interest rates and exchange rates for the U.S. dollar and the Lilliputian nano. The spot exchange rate is 15 nanos = $1. Complete the miss...

See Answer

Q: Sara Togas sells all its output to Federal Stores. The following

Sara Togas sells all its output to Federal Stores. The following table shows selected financial data, in millions, for the two firms: Calculate the sales-to-assets ratio, the operating profit margin,...

See Answer

Q: As you can see, someone has spilled ink over some of

As you can see, someone has spilled ink over some of the entries in the balance sheet and income statement of Transylvania Railroad (Table 28.11). Can you use the following information to...

See Answer

Q: Here are some data for five companies in the same industry:

Here are some data for five companies in the same industry: You have been asked to calculate a measure of times-interest-earned for the industry. Discuss the possible ways that you might calculate suc...

See Answer

Q: If you buy a nine-month T-bill future,

If you buy a nine-month T-bill future, you undertake to buy a $1 million three-month bill in nine months’ time. Suppose that Treasury bills and notes currently offer the following yields: Mo...

See Answer

Q: How would rapid inflation affect the accuracy and relevance of a manufacturing

How would rapid inflation affect the accuracy and relevance of a manufacturing company’s balance sheet and income statement? Does your answer depend on how much debt the firm has issued?

See Answer

Q: Suppose that you wish to use financial ratios to estimate the risk

Suppose that you wish to use financial ratios to estimate the risk of a company’s stock. Which of those that we have described in this chapter are likely to be helpful? Can you think of other accounti...

See Answer

Q: We noted that, when calculating EVA, you should calculate income

We noted that, when calculating EVA, you should calculate income as the sum of the after-tax interest payment and net income. Why do you need to deduct the tax shield? Would an alternative be to use a...

See Answer

Q: Sometimes analysts use the average of capital at the start and end

Sometimes analysts use the average of capital at the start and end of the year to calculate return on capital. Provide some examples to illustrate when this does and does not make sense.

See Answer

Q: Take another look at Geomorph Trading’s balance sheet in Problem 10 and

Take another look at Geomorph Trading’s balance sheet in Problem 10 and consider the following additional information: The “R&R reserve” cover...

See Answer

Q: Construct a balance sheet for Galactic Enterprises given the following data:

Construct a balance sheet for Galactic Enterprises given the following data: Cash balances………….…………..……………$25,000 Inventories……………………………………… $30,000 Net plant and equipment…………………..$140,000 Accounts r...

See Answer

Q: Table 28.10 gives abbreviated balance sheets and income statements for

Table 28.10 gives abbreviated balance sheets and income statements for Starbucks. Calculate the following using balance-sheet figures from the start of the year: a. Return on assets. b....

See Answer

Q: Residents of the northeastern United States suffered record-setting low temperatures

Residents of the northeastern United States suffered record-setting low temperatures throughout November and December 2024. Spot prices of heating oil rose 25% to over $7 a gallon. a. What effect did...

See Answer

Q: Look again at Table 28.10. Calculate a common-

Look again at Table 28.10. Calculate a common-size balance sheet and income statement for Starbucks. Table 28.10:

See Answer

Q: Look again at Table 28.10. At the end of

Look again at Table 28.10. At the end of fiscal 2014, Starbucks had 748 million shares outstanding with a share price of $81.25. The company’s weighted average cost of ca...

See Answer

Q: German banks often control a large fraction of the shareholder votes for

German banks often control a large fraction of the shareholder votes for German businesses. How do they get that voting power?

See Answer

Q: There are no universally accepted definitions of financial ratios, but five

There are no universally accepted definitions of financial ratios, but five of the following ratios are clearly incorrect. Substitute the correct definitions. a. Debt–equity ratio = (long-term debt +...

See Answer

Q: Keller Cosmetics maintains an operating profit margin of 8% and a

Keller Cosmetics maintains an operating profit margin of 8% and a sales-to-assets ratio of 3. It has assets of $500,000 and equity of $300,000. Interest payments are $30,000 and the tax rate is 35%....

See Answer

Q: A firm has a long-term debt–equity ratio of

A firm has a long-term debt–equity ratio of .4. Shareholders’ equity is $1 million. Current assets are $200,000, and total assets are $1.5 million. If the current ratio is 2.0, what is the ratio of de...

See Answer

Q: Magic Flutes has total receivables of $3,000, which

Magic Flutes has total receivables of $3,000, which represent 20 days’ sales. Total assets are $75,000. The firm’s operating profit margin is 5%. Find the firm’s sales-to-assets ratio and return on as...

See Answer

Q: Consider this simplified balance sheet for Geomorph Trading: /

Consider this simplified balance sheet for Geomorph Trading: a. Calculate the ratio of debt to equity. b. What are Geomorph’s net working capital and total long-term capital? Calcu...

See Answer

Q: Look again at the balance sheet for Geomorph in Problem 10.

Look again at the balance sheet for Geomorph in Problem 10. Suppose that at year-end Geomorph had $30 in cash and marketable securities. Immediately after the year-end it used a line of credit to borr...

See Answer

Q: This question reviews some of the difficulties encountered in interpreting accounting numbers

This question reviews some of the difficulties encountered in interpreting accounting numbers. a. Give four examples of important assets, liabilities, or transactions that may not be shown on the com...

See Answer

Q: In fiscal 2012 and 2013, Caterpillar’s financial statements included the following

In fiscal 2012 and 2013, Caterpillar’s financial statements included the following items. What was Caterpillar’s cash cycle?

See Answer

Q: After a record harvest, grain silos are full to the brim

After a record harvest, grain silos are full to the brim. Are storage costs likely to be high or low? What does this imply for the net convenience yield?

See Answer

Q: Abbreviated financial statements for Archimedes Levers are shown in Table 29.

Abbreviated financial statements for Archimedes Levers are shown in Table 29.13 on the next page. If sales increase by 10% in 2017 and all other items, including debt, increase correspondi...

See Answer

Q: What is the most common form of ownership of corporations worldwide?

What is the most common form of ownership of corporations worldwide?

See Answer

Q: A firm is considering several policy changes to increase sales. It

A firm is considering several policy changes to increase sales. It plans to increase the variety of goods it keeps in inventory, but this will increase inventory by $100,000. It will offer more libera...

See Answer

Q: If a firm pays its bills with a 30-day delay

If a firm pays its bills with a 30-day delay, what fraction of its purchases will be paid in the current quarter? In the following quarter? What if the delay is 60 days?

See Answer

Q: What effect will each of the following have on the cash cycle

What effect will each of the following have on the cash cycle? a. The inventory turnover falls from 80 to 60 days. b. Customers are given a larger discount for cash transactions. c. The firm adopts...

See Answer

Q: Listed below are six transactions that Dynamic Mattress might make. Indicate

Listed below are six transactions that Dynamic Mattress might make. Indicate how each transaction would affect (1) cash and (2) working capital. The transactions are a. Pay out an extra $10 million c...

See Answer

Q: State how each of the following events would affect the firm’s balance

State how each of the following events would affect the firm’s balance sheet. State whether each change is a source or use of cash. a. An automobile manufacturer increases production in response to a...

See Answer

Q: Here is a forecast of sales by National Bromide for the first

Here is a forecast of sales by National Bromide for the first four months of 2016 (figures in $ thousands): On the average 50% of credit sales are paid for in the current month, 30% are paid in the ne...

See Answer

Q: Dynamic Futon forecasts the following purchases from suppliers: /

Dynamic Futon forecasts the following purchases from suppliers: a. Forty percent of goods are supplied cash-on-delivery. The remainder are paid with an average delay of one month. If Dynamic Futon sta...

See Answer

Q: Each of the following events affects one or more tables in Sections

Each of the following events affects one or more tables in Sections 29-2 to 29-3. Show the effects of each event by adjusting the tables listed in parentheses: a. Dynamic repays only $10 million of s...

See Answer

Q: True or false? a. Financial planning should attempt to

True or false? a. Financial planning should attempt to minimize risk. b. The primary aim of financial planning is to obtain better forecasts of future cash flows and earnings. c. Financial planning...

See Answer

Q: A year ago a bank entered into a $50 million five

A year ago a bank entered into a $50 million five-year interest rate swap. It agreed to pay company A each year a fixed rate of 6% and to receive in return LIBOR. When the bank entered into this swap,...

See Answer

Q: Suppose that a shareholder can gain effective control of a company with

Suppose that a shareholder can gain effective control of a company with 30% of the shares. Explain how a shareholder might gain control of company Z by setting up a holding company X2 that holds share...

See Answer

Q: Table 29.12 summarizes the 2017 income statement and end-

Table 29.12 summarizes the 2017 income statement and end-year balance sheet of Drake’s Bowling Alleys. Drake’s financial manager forecasts a 10% increas...

See Answer

Q: What is the maximum possible growth rate for Archimedes (see Problem

What is the maximum possible growth rate for Archimedes (see Problem 10) if the payout ratio is set at 50% and (a) no external debt or equity is to be issued? (b) the firm maintains a fixed debt rat...

See Answer

Q: Table 29.14 lists data from the budget of Ritewell Publishers

Table 29.14 lists data from the budget of Ritewell Publishers. Half the company’s sales are for cash on the nail; the other half are paid for with a one-month delay. The...

See Answer

Q: Which items in Table 29.7 would be affected by the

Which items in Table 29.7 would be affected by the following events? a. Interest rates rise. b. Suppliers demand interest for late payment. c. Dynamic receives an unexpected bill in the...

See Answer

Q: Table 29.16 on the next page shows Dynamic Mattress’s year

Table 29.16 on the next page shows Dynamic Mattress’s year-end 2013 balance sheet, and Table 29.17 shows its income statement for 2014. Work out the statement...

See Answer

Q: Work out a short-term financing plan for Dynamic Mattress Company

Work out a short-term financing plan for Dynamic Mattress Company, assuming the limit on the line of credit is raised from $100 to $120 million. Otherwise keep to the assumptions used in developing Ta...

See Answer

Q: Dynamic Mattress decides to lease its new mattress-stuffing machines rather

Dynamic Mattress decides to lease its new mattress-stuffing machines rather than buy them. As a result, capital expenditure in the first quarter is reduced by $50 million, but the company must make le...

See Answer

Q: Our long-term planning model of Dynamic Mattress is an example

Our long-term planning model of Dynamic Mattress is an example of a top-down planning model. Some firms use a bottom-up financial planning model, which incorporates forecasts of revenues and costs for...

See Answer

Q: Corporate financial plans are often used as a basis for judging subsequent

Corporate financial plans are often used as a basis for judging subsequent performance. What do you think can be learned from such comparisons? What problems are likely to arise, and how might you cop...

See Answer

Q: The balancing item in the Dynamic long-term planning model is

The balancing item in the Dynamic long-term planning model is borrowing. What is meant by balancing item? How would the model change if dividends were made the balancing item instead? In that case how...

See Answer

Q: What are the four primary disadvantages of the sole proprietorship and partnership

What are the four primary disadvantages of the sole proprietorship and partnership forms of business organization? What benefits are there to these types of business organization as opposed to the cor...

See Answer

Q: A firm’s enterprise value is equal to the market value of its

A firm’s enterprise value is equal to the market value of its debt and equity, less the firm’s holdings of cash and cash equivalents. This figure is particularly relevant to potential purchasers of th...

See Answer

Q: From the previous two questions, prepare a pro forma balance sheet

From the previous two questions, prepare a pro forma balance sheet showing EFN, assuming a 15 percent increase in sales, no new external debt or equity financing, and a constant payout ratio. Data fro...

See Answer

Q: If the Baseball Shoppe has an 8 percent ROA and a 20

If the Baseball Shoppe has an 8 percent ROA and a 20 percent payout ratio, what is its internal growth rate?

See Answer

Q: If the Garnett Corp. has a 15 percent ROE and a

If the Garnett Corp. has a 15 percent ROE and a 25 percent payout ratio, what is its sustainable growth rate?

See Answer

Q: Based on the following information, calculate the sustainable growth rate for

Based on the following information, calculate the sustainable growth rate for Kaleb’s Kickboxing: Profit margin=8.2% Capital intensity ratio=.75 Debt–equity ratio=.40 Net income=$43,000 Dividends=$12,...

See Answer

Q: Assuming the following ratios are constant, what is the sustainable growth

Assuming the following ratios are constant, what is the sustainable growth rate? Total asset turnover=2.50 Profit margin=7.8% Equity multiplier=1.80 Payout ratio=60%

See Answer

Q: Seaweed Mfg., Inc., is currently operating at only 95 percent

Seaweed Mfg., Inc., is currently operating at only 95 percent of fixed asset capacity. Current sales are $550,000. How fast can sales grow before any new fixed assets are needed?

See Answer

Q: For the company in the previous problem, suppose fixed assets are

For the company in the previous problem, suppose fixed assets are $440,000 and sales are projected to grow to $630,000. How much in new fixed assets are required to support this growth in sales? Assum...

See Answer

Q: McCormac Co. wishes to maintain a growth rate of 12 percent

McCormac Co. wishes to maintain a growth rate of 12 percent a year, a debt–equity ratio of 1.20, and a dividend payout ratio of 30 percent. The ratio of total assets to sales is constant at .75. What...

See Answer

Q: A firm wishes to maintain a growth rate of 11.5

A firm wishes to maintain a growth rate of 11.5 percent and a dividend payout ratio of 30 percent. The ratio of total assets to sales is constant at .60, and profit margin is 6.2 percent. If the firm...

See Answer

Q: A firm wishes to maintain an internal growth rate of 7 percent

A firm wishes to maintain an internal growth rate of 7 percent and a dividend payout ratio of 25 percent. The current profit margin is 5 percent, and the firm uses no external financing sources. What...

See Answer

Q: Can our goal of maximizing the value of the stock conflict with

Can our goal of maximizing the value of the stock conflict with other goals, such as avoiding unethical or illegal behavior? In particular, do you think subjects like customer and employee safety, the...

See Answer

Q: Based on the following information, calculate the sustainable growth rate for

Based on the following information, calculate the sustainable growth rate for Hendrix Guitars, Inc.: Profit margin=4.8% Total asset turnover=1.25 Total debt ratio=.65 Payout ratio=30%

See Answer

Q: You’ve collected the following information about St. Pierre, Inc.:

You’ve collected the following information about St. Pierre, Inc.: Sales=$195,000 Net income=$17,500 Dividends=$9,300 Total debt=$86,000 Total equity=$58,000 What is the sustainable growth rate for St...

See Answer

Q: Coheed, Inc., had equity of $135,000 at

Coheed, Inc., had equity of $135,000 at the beginning of the year. At the end of the year, the company had total assets of $250,000. During the year the company sold no new equity. Net income for the...

See Answer

Q: Calculate the internal growth rate for the company in the previous problem

Calculate the internal growth rate for the company in the previous problem. Now calculate the internal growth rate using ROA × b for both beginning of period and end of period total assets. What do yo...

See Answer

Q: The most recent financial statements for Moose Tours, Inc., follow

The most recent financial statements for Moose Tours, Inc., follow. Sales for 2009 are projected to grow by 20 percent. Interest expense will remain constant; the tax rate and the dividend payout rate...

See Answer

Q: In the previous problem, suppose the firm was operating at only

In the previous problem, suppose the firm was operating at only 80 percent capacity in 2008. What is EFN now?

See Answer

Q: In Problem 25, suppose the firm wishes to keep its debt

In Problem 25, suppose the firm wishes to keep its debt– equity ratio constant. What is EFN now?

See Answer

Q: Redo Problem 25 using sales growth rates of 15 and 25 percent

Redo Problem 25 using sales growth rates of 15 and 25 percent in addition to 20 percent. Illustrate graphically the relationship between EFN and the growth rate, and use this graph to determine the re...

See Answer

Q: Redo Problem 27 using sales growth rates of 30 and 35 percent

Redo Problem 27 using sales growth rates of 30 and 35 percent in addition to 20 percent. Illustrate graphically the relationship between EFN and the growth rate, and use this graph to determine the re...

See Answer

Q: Nearside, Inc., wishes to maintain a growth rate of 12

Nearside, Inc., wishes to maintain a growth rate of 12 percent per year and a debt–equity ratio of .30. Profit margin is 6.70 percent, and the ratio of total assets to sales is constant at 1.35. Is th...

See Answer

Q: Would our goal of maximizing the value of the stock be different

Would our goal of maximizing the value of the stock be different if we were thinking about financial management in a foreign country? Why or why not?

See Answer

Q: Define the following: S=Previous year’s sales A

Define the following: S=Previous year’s sales A=Total assets D=Total debt E=Total equity g=Projected growth in sales PM=Profit margin b=Retention (plowback) ratio Show that EFN can be written as follo...

See Answer

Q: Based on the result in Problem 31, show that the internal

Based on the result in Problem 31, show that the internal and sustainable growth rates are as given in the chapter.

See Answer

Q: In the chapter, we discussed the two versions of the sustainable

In the chapter, we discussed the two versions of the sustainable growth rate formula. Derive the formula ROE × b from the formula given in the chapter, where ROE is based on beginning of period equity...

See Answer

Q: Consider the following simplified financial statements for the Phillips Corporation (assuming

Consider the following simplified financial statements for the Phillips Corporation (assuming no income taxes): Phillips has predicted a sales increase of 15 percent. It has predicted that every ite...

See Answer

Q: In the previous question, assume Phillips pays out half of net

In the previous question, assume Phillips pays out half of net income in the form of a cash dividend. Costs and assets vary with sales, but debt and equity do not. Prepare the pro forma statements and...

See Answer

Q: The most recent financial statements for Zoso, Inc., are shown

The most recent financial statements for Zoso, Inc., are shown here (assuming no income taxes): Assets and costs are proportional to sales. Debt and equity are not. No dividends are paid. Next year&...

See Answer

Q: The most recent financial statements for GPS, Inc., are shown

The most recent financial statements for GPS, Inc., are shown here: Assets and costs are proportional to sales. Debt and equity are not. A dividend of $1,400 was paid, and the company wishes to main...

See Answer

Q: The most recent financial statements for Summer Tyme, Inc., are

The most recent financial statements for Summer Tyme, Inc., are shown here: Assets, costs, and current liabilities are proportional to sales. Long-term debt and equity are not. The company maintains...

See Answer

Q: The most recent financial statements for Live Co. are shown here

The most recent financial statements for Live Co. are shown here: Assets and costs are proportional to sales. Debt and equity are not. The company maintains a constant 30 percent dividend payout rat...

See Answer

Q: What happens to a future value if you increase the rate r

What happens to a future value if you increase the rate r ? What happens to a present value?

See Answer

Q: Why might the revenue and cost figures shown on a standard income

Why might the revenue and cost figures shown on a standard income statement not be representative of the actual cash inflows and outflows that occurred during a period?

See Answer

Q: Take a look back at Example 5.7. Is it

Take a look back at Example 5.7. Is it deceptive advertising? Is it unethical to advertise a future value like this without a disclaimer? To answer the next five questions, refer to the TMCC security...

See Answer

Q: The basic present value equation has four parts. What are they

The basic present value equation has four parts. What are they?

See Answer

Q: What is compounding? What is discounting?

What is compounding? What is discounting?

See Answer

Q: You’re trying to save to buy a new $170,000

You’re trying to save to buy a new $170,000 Ferrari. You have $40,000 today that can be invested at your bank. The bank pays 5.3 percent annual interest on its accounts. How long will it be before you...

See Answer

Q: In 1895, the first U.S. Open Golf Championship

In 1895, the first U.S. Open Golf Championship was held. The winner’s prize money was $150. In 2007, the winner’s check was $1,260,000. What was the percentage increase per year in the winner’s check...

See Answer

Q: In 2008, a gold Morgan dollar minted in 1895 sold for

In 2008, a gold Morgan dollar minted in 1895 sold for $43,125. For this to have been true, what rate of return did this coin return for the lucky numismatist?

See Answer

Q: Although appealing to more refined tastes, art as a collectible has

Although appealing to more refined tastes, art as a collectible has not always performed so profitably. During 2003, Sotheby’s sold the Edgar Degas bronze sculpture Petite Danseuse de Quatorze Ans at...

See Answer

Q: Referring to the TMCC security we discussed at the very beginning of

Referring to the TMCC security we discussed at the very beginning of the chapter: a. Based on the $24,099 price, what rate was TMCC paying to borrow money? b. Suppose that, on March 28, 2020, this sec...

See Answer

Q: You expect to receive $10,000 at graduation in two

You expect to receive $10,000 at graduation in two years. You plan on investing it at 11 percent until you have $75,000. How long will you wait from now?

See Answer

Q: Solve for the unknown interest rate in each of the following:

Solve for the unknown interest rate in each of the following:

See Answer

Q: Corporate ownership varies around the world. Historically individuals have owned the

Corporate ownership varies around the world. Historically individuals have owned the majority of shares in public corporations in the United States. In Germany and Japan, however, banks, other large f...

See Answer

Q: Solve for the unknown number of years in each of the following

Solve for the unknown number of years in each of the following:

See Answer

Q: Assume the total cost of a college education will be $290

Assume the total cost of a college education will be $290,000 when your child enters college in 18 years. You presently have $55,000 to invest. What annual rate of interest must you earn on your inves...

See Answer

Q: At 7 percent interest, how long does it take to double

At 7 percent interest, how long does it take to double your money? To quadruple it?

See Answer

Q: In January 2007, the average house price in the United States

In January 2007, the average house price in the United States was $314,600. In January 2000, the average price was $200,300. What was the annual increase in selling price?

See Answer

Q: You have just received notification that you have won the $1

You have just received notification that you have won the $1 million first prize in the Centennial Lottery. However, the prize will be awarded on your 100th birthday (assuming you’re around to collect...

See Answer

Q: Your coin collection contains fifty 1952 silver dollars. If your grandparents

Your coin collection contains fifty 1952 silver dollars. If your grandparents purchased them for their face value when they were new, how much will your collection be worth when you retire in 2057, as...

See Answer

Q: Suppose you are still committed to owning a $170,000

Suppose you are still committed to owning a $170,000 Ferrari (see Problem 9). If you believe your mutual fund can achieve a 12 percent annual rate of return and you want to buy the car in 9 years on t...

See Answer

Q: You have just made your first $4,000 contribution to

You have just made your first $4,000 contribution to your retirement account. Assuming you earn an 11 percent rate of return and make no additional contributions, what will your account be worth when...

See Answer

Q: You are scheduled to receive $20,000 in two years

You are scheduled to receive $20,000 in two years. When you receive it, you will invest it for six more years at 8.4 percent per year. How much will you have in eight years?

See Answer

Q: For each of the following, compute the future value:

For each of the following, compute the future value:

See Answer

Q: Critics have charged that compensation to top managers in the United States

Critics have charged that compensation to top managers in the United States is simply too high and should be cut back. For example, focusing on large corporations, Ray Irani of Occidental Petroleum ha...

See Answer

Q: For each of the following, compute the present value:

For each of the following, compute the present value:

See Answer

Q: Imprudential, Inc. has an unfunded pension liability of $650

Imprudential, Inc. has an unfunded pension liability of $650 million that must be paid in 20 years. To assess the value of the firm’s stock, financial analysts want to discount this liability back to...

See Answer

Q: What do you think about the Tri-State Megabucks lottery discussed

What do you think about the Tri-State Megabucks lottery discussed in the chapter advertising a $500,000 prize when the lump sum option is $250,000? Is it deceptive advertising?

See Answer

Q: If you were an athlete negotiating a contract, would you want

If you were an athlete negotiating a contract, would you want a big signing bonus payable immediately and smaller payments in the future, or vice versa? How about looking at it from the team’s perspec...

See Answer

Q: Should lending laws be changed to require lenders to report EARs instead

Should lending laws be changed to require lenders to report EARs instead of APRs? Why or why not?

See Answer

Q: There are four pieces to an annuity present value. What are

There are four pieces to an annuity present value. What are they?

See Answer

Q: As you increase the length of time involved, what happens to

As you increase the length of time involved, what happens to the present value of an annuity? What happens to the future value?

See Answer

Q: Eligibility for a subsidized Stafford loan is based on current financial need

Eligibility for a subsidized Stafford loan is based on current financial need. However, both subsidized and unsubsidized Stafford loans are repaid out of future income. Given this, do you see a possib...

See Answer

Q: A viatical settlement is a lump sum of money given to a

A viatical settlement is a lump sum of money given to a terminally ill individual in exchange for his life insurance policy. When the insured person dies, the purchaser receives the payout from the li...

See Answer

Q: On subsidized Stafford loans, a common source of financial aid for

On subsidized Stafford loans, a common source of financial aid for college students, interest does not begin to accrue until repayment begins. Who receives a bigger subsidy, a freshman or a senior? Ex...

See Answer

Q: Could a company’s change in NWC be negative in a given year

Could a company’s change in NWC be negative in a given year? Explain how this might come about. What about net capital spending?

See Answer

Q: Ben Bates graduated from college six years ago with a finance undergraduate

Ben Bates graduated from college six years ago with a finance undergraduate degree. Although he is satisfied with his current job, his goal is to become an investment banker. He feels that an MBA degr...

See Answer

Q: You are looking at a one-year loan of $10

You are looking at a one-year loan of $10,000. The interest rate is quoted as 8 percent plus three points. A point on a loan is simply 1 percent (one percentage point) of the loan amount. Quotes simil...

See Answer

Q: The interest rate on a one-year loan is quoted as

The interest rate on a one-year loan is quoted as 11 percent plus two points (see the previous problem). What is the EAR? Is your answer affected by the loan amount? Previous problem: You are looking...

See Answer

Q: Two banks in the area offer 30-year, $240

Two banks in the area offer 30-year, $240,000 mortgages at 6.8 percent and charge a $2,300 loan application fee. However, the application fee charged by Insecurity Bank and Trust is refundable if the...

See Answer

Q: This problem illustrates a deceptive way of quoting interest rates called add

This problem illustrates a deceptive way of quoting interest rates called add-on interest. Imagine that you see an advertisement for Crazy Judy’s Stereo City that reads something like this: “$1,000 In...

See Answer

Q: This is a classic retirement problem. A time line will help

This is a classic retirement problem. A time line will help in solving it. Your friend is celebrating her 35th birthday today and wants to start saving for her anticipated retirement at age 65. She wa...

See Answer

Q: Your Christmas ski vacation was great, but it unfortunately ran a

Your Christmas ski vacation was great, but it unfortunately ran a bit over budget. All is not lost: You just received an offer in the mail to transfer your $10,000 balance from your current credit car...

See Answer

Q: An insurance company is offering a new policy to its customers.

An insurance company is offering a new policy to its customers. Typically, the policy is bought by a parent or grandparent for a child at the child’s birth. The details of the policy are as follows: T...

See Answer

Q: You have just arranged for a $750,000 mortgage to

You have just arranged for a $750,000 mortgage to finance the purchase of a large tract of land. The mortgage has an 8.1 percent APR, and it calls for monthly payments over the next 30 years. However,...

See Answer

Q: A financial planning service offers a college savings program. The plan

A financial planning service offers a college savings program. The plan calls for you to make six annual payments of $9,000 each, with the first payment occurring today, your child’s 12th birthday. Be...

See Answer

Q: Companies often try to keep accounting earnings growing at a relatively steady

Companies often try to keep accounting earnings growing at a relatively steady pace, thereby avoiding large swings in earnings from period to period. They also try to meet earnings targets. To do so,...

See Answer

Q: Your financial planner offers you two different investment plans. Plan X

Your financial planner offers you two different investment plans. Plan X is a $20,000 annual perpetuity. Plan Y is a 20-year, $28,000 annual annuity. Both plans will make their fi rst payment one year...

See Answer

Q: What is the value of an investment that pays $15,

What is the value of an investment that pays $15,000 every other year forever, if the first payment occurs one year from today and the discount rate is 10 percent compounded daily? What is the value t...

See Answer

Q: As discussed in the text, an annuity due is identical to

As discussed in the text, an annuity due is identical to an ordinary annuity except that the periodic payments occur at the beginning of each period and not at the end of the period. Show that the rel...

See Answer

Q: You have 40 years left until retirement and want to retire with

You have 40 years left until retirement and want to retire with $2 million. Your salary is paid annually, and you will receive $40,000 at the end of the current year. Your salary will increase at 3 pe...

See Answer

Q: Earlier, we discussed the Rule of 72, a useful approximation

Earlier, we discussed the Rule of 72, a useful approximation for many interest rates and periods for the time it takes a lump sum to double in value. For a 10 percent interest rate, show that the “Rul...

See Answer

Q: Seaborn Co. has identified an investment project with the following cash

Seaborn Co. has identified an investment project with the following cash flows. If the discount rate is 10 percent, what is the present value of these cash flows? What is the present value at 18 perce...

See Answer

Q: Investment X offers to pay you $6,000 per year

Investment X offers to pay you $6,000 per year for nine years, whereas Investment Y offers to pay you $8,000 per year for six years. Which of these cash flow streams has the higher present value if th...

See Answer

Q: Paradise, Inc., has identified an investment project with the following

Paradise, Inc., has identified an investment project with the following cash flows. If the discount rate is 8 percent, what is the future value of these cash flows in year 4? What is the future value...

See Answer

Q: An investment offers $5,300 per year for 15 years

An investment offers $5,300 per year for 15 years, with the first payment occurring one year from now. If the required return is 7 percent, what is the value of the investment? What would the value be...

See Answer

Q: If you put up $34,000 today in exchange for

If you put up $34,000 today in exchange for a 7.65 percent, 15-year annuity, what will the annual cash flow be?

See Answer

Q: What does liquidity measure? Explain the trade-off a firm

What does liquidity measure? Explain the trade-off a firm faces between high liquidity and low liquidity levels.

See Answer

Q: If you deposit $4,000 at the end of each

If you deposit $4,000 at the end of each of the next 20 years into an account paying 11.2 percent interest, how much money will you have in the account in 20 years? How much will you have if you make...

See Answer

Q: You want to have $90,000 in your savings account

You want to have $90,000 in your savings account 10 years from now, and you’re prepared to make equal annual deposits into the account at the end of each year. If the account pays 6.8 percent interest...

See Answer

Q: Dinero Bank offers you a $50,000, seven-

Dinero Bank offers you a $50,000, seven-year term loan at 7.5 percent annual interest. What will your annual loan payment be?

See Answer

Q: In the previous problem, suppose a sales associate told you the

In the previous problem, suppose a sales associate told you the policy costs $375,000. At what interest rate would this be a fair deal?

See Answer

Q: Find the EAR in each of the following cases:

Find the EAR in each of the following cases:

See Answer

Q: Find the APR, or stated rate, in each of the

Find the APR, or stated rate, in each of the following cases:

See Answer

Q: First National Bank charges 14.2 percent compounded monthly on its

First National Bank charges 14.2 percent compounded monthly on its business loans. First United Bank charges 14.5 percent compounded semiannually. As a potential borrower, which bank would you go to f...

See Answer

Q: Barcain Credit Corp. wants to earn an effective annual return on

Barcain Credit Corp. wants to earn an effective annual return on its consumer loans of 16 percent per year. The bank uses daily compounding on its loans. What interest rate is the bank required by law...

See Answer

Q: What is the future value of $2,100 in 17

What is the future value of $2,100 in 17 years assuming an interest rate of 8.4 percent compounded semiannually?

See Answer

Q: Gold Door Credit Bank is offering 9.3 percent compounded daily

Gold Door Credit Bank is offering 9.3 percent compounded daily on its savings accounts. If you deposit $4,500 today, how much will you have in the account in 5 years? In 10 years? In 20 years?

See Answer

Q: Suppose a company’s cash flow from assets is negative for a particular

Suppose a company’s cash flow from assets is negative for a particular period. Is this necessarily a good sign or a bad sign?

See Answer

Q: An investment will pay you $58,000 in seven years

An investment will pay you $58,000 in seven years. If the appropriate discount rate is 10 percent compounded daily, what is the present value?

See Answer

Q: Big Dom’s Pawn Shop charges an interest rate of 30 percent per

Big Dom’s Pawn Shop charges an interest rate of 30 percent per month on loans to its customers. Like all lenders, Big Dom must report an APR to consumers. What rate should the shop report? What is the...

See Answer

Q: You want to buy a new sports coupe for $68,

You want to buy a new sports coupe for $68,500, and the finance office at the dealership has quoted you a 6.9 percent APR loan for 60 months to buy the car. What will your monthly payments be? What is...

See Answer

Q: One of your customers is delinquent on his accounts payable balance.

One of your customers is delinquent on his accounts payable balance. You’ve mutually agreed to a repayment schedule of $500 per month. You will charge 1.3 percent per month interest on the overdue bal...

See Answer

Q: Friendly’s Quick Loans, Inc., offers you “three for four

Friendly’s Quick Loans, Inc., offers you “three for four or 1 knock on your door.” This means you get $3 today and repay $4 when you get your paycheck in one week (or else). What’s the effective annua...

See Answer

Q: Live Forever Life Insurance Co. is selling a perpetuity contract that

Live Forever Life Insurance Co. is selling a perpetuity contract that pays $1,800 monthly. The contract currently sells for $95,000. What is the monthly return on this investment vehicle? What is the...

See Answer

Q: You are planning to make monthly deposits of $300 into a

You are planning to make monthly deposits of $300 into a retirement account that pays 10 percent interest compounded monthly. If your first deposit will be made one month from now, how large will your...

See Answer

Q: In the previous problem, suppose you make $3,600

In the previous problem, suppose you make $3,600 annual deposits into the same retirement account. How large will your account balance be in 30 years? Data from previous problem: You are planning to...

See Answer

Q: If the appropriate discount rate for the following cash flows is 11

If the appropriate discount rate for the following cash flows is 11 percent compounded quarterly, what is the present value of the cash flows? Year Cash …………….………………Flow 1………………………………………….……….$ 725 2...

See Answer

Q: First Simple Bank pays 7 percent simple interest on its investment accounts

First Simple Bank pays 7 percent simple interest on its investment accounts. If First Complex Bank pays interest on its accounts compounded annually, what rate should the bank set if it wants to match...

See Answer

Q: What is the primary disadvantage of the corporate form of organization?

What is the primary disadvantage of the corporate form of organization? Name at least two advantages of corporate organization.

See Answer

Q: Suppose a company’s operating cash flow has been negative for several years

Suppose a company’s operating cash flow has been negative for several years running. Is this necessarily a good sign or a bad sign?

See Answer

Q: You are looking at an investment that has an effective annual rate

You are looking at an investment that has an effective annual rate of 17 percent. What is the effective semiannual return? The effective quarterly return? The effective monthly return?

See Answer

Q: You receive a credit card application from Shady Banks Savings and Loan

You receive a credit card application from Shady Banks Savings and Loan offering an introductory rate of 1.5 percent per year, compounded monthly for the first six months, increasing thereafter to 18...

See Answer

Q: You are planning to save for retirement over the next 30 years

You are planning to save for retirement over the next 30 years. To do this, you will invest $700 a month in a stock account and $300 a month in a bond account. The return of the stock account is expec...

See Answer

Q: You have an investment that will pay you 1.17 percent

You have an investment that will pay you 1.17 percent per month. How much will you have per dollar invested in one year? In two years?

See Answer

Q: You want to be a millionaire when you retire in 40 years

You want to be a millionaire when you retire in 40 years. How much do you have to save each month if you can earn a 12 percent annual return? How much do you have to save if you wait 10 years before y...

See Answer

Q: You’ve just joined the investment banking firm of Dewey, Cheatum,

You’ve just joined the investment banking firm of Dewey, Cheatum, and Howe. They’ve offered you two different salary arrangements. You can have $95,000 per year for the next two years, or you can have...

See Answer

Q: You have just won the lottery and will receive $1,

You have just won the lottery and will receive $1,000,000 in one year. You will receive payments for 30 years, which will increase 5 percent per year. If the appropriate discount rate is 8 percent, wh...

See Answer

Q: Your job pays you only once a year for all the work

Your job pays you only once a year for all the work you did over the previous 12 months. Today, December 31, you just received your salary of $50,000 and you plan to spend all of it. However, you want...

See Answer

Q: What is the relationship between the value of an annuity and the

What is the relationship between the value of an annuity and the level of interest rates? Suppose you just bought a 15-year annuity of $9,000 per year at the current interest rate of 10 percent per ye...

See Answer

Q: You’re prepared to make monthly payments of $340, beginning at

You’re prepared to make monthly payments of $340, beginning at the end of this month, into an account that pays 6 percent interest compounded monthly. How many payments will you have made when your ac...

See Answer

Q: Could a company’s cash flow to stockholders be negative in a given

Could a company’s cash flow to stockholders be negative in a given year? Explain how this might come about. What about cash flow to creditors?

See Answer

Q: You want to borrow $73,000 from your local bank

You want to borrow $73,000 from your local bank to buy a new sailboat. You can afford to make monthly payments of $1,450, but no more. Assuming monthly compounding, what is the highest rate you can af...

See Answer

Q: You need a 30-year, fixed-rate mortgage to

You need a 30-year, fixed-rate mortgage to buy a new home for $240,000. Your mortgage bank will lend you the money at a 6.35 percent APR for this 360-month loan. However, you can afford monthly paymen...

See Answer

Q: The present value of the following cash flow stream is $6

The present value of the following cash flow stream is $6,550 when discounted at 10 percent annually. What is the value of the missing cash flow? Year ……………………….…………… Cash Flow 1………………………………………………………...

See Answer

Q: You just won the TVM Lottery. You will receive $1

You just won the TVM Lottery. You will receive $1 million today plus another 10 annual payments that increase by $500,000 per year. Thus, in one year, you receive $1.5 million. In two years you get $2...

See Answer

Q: You have just purchased a new warehouse. To finance the purchase

You have just purchased a new warehouse. To finance the purchase, you’ve arranged for a 30-year mortgage loan for 80 percent of the $2,900,000 purchase price. The monthly payment on this loan will be...

See Answer

Q: Consider a firm with a contract to sell an asset for $

Consider a firm with a contract to sell an asset for $165,000 four years from now. The asset costs $94,000 to produce today. Given a relevant discount rate on this asset of 13 percent per year, will t...

See Answer

Q: What is the present value of $4,000 per year

What is the present value of $4,000 per year, at a discount rate of 10 percent, if the first payment is received 8 years from now and the last payment is received 25 years from now?

See Answer

Q: A 15-year annuity pays $1,500 per month

A 15-year annuity pays $1,500 per month, and payments are made at the end of each month. If the interest rate is 11 percent compounded monthly for the first seven years, and 7 percent compounded month...

See Answer

Q: You have your choice of two investment accounts. Investment A is

You have your choice of two investment accounts. Investment A is a 15-year annuity that features end-of-month $1,200 payments and has an interest rate of 8.5 percent compounded monthly. Investment B i...

See Answer

Q: Given an interest rate of 6.2 percent per year,

Given an interest rate of 6.2 percent per year, what is the value at date t =7 of a perpetual stream of $3,500 payments that begins at date t= 15?

See Answer

Q: Referring back to the General Motors example used at the beginning of

Referring back to the General Motors example used at the beginning of the chapter, note that we suggested that General Motors’ stockholders probably didn’t suffer as a result of the reported loss. Wha...

See Answer

Q: A local finance company quotes a 16 percent interest rate on one

A local finance company quotes a 16 percent interest rate on one-year loans. So, if you borrow $25,000, the interest for the year will be $4,000. Because you must repay a total of $29,000 in one year,...

See Answer

Q: A 5-year annuity of ten $7,000 semiannual

A 5-year annuity of ten $7,000 semiannual payments will begin 8 years from now, with the first payment coming 8.5 years from now. If the discount rate is 10 percent compounded monthly, what is the val...

See Answer

Q: Suppose you are going to receive $10,000 per year

Suppose you are going to receive $10,000 per year for five years. The appropriate interest rate is 11 percent. a. What is the present value of the payments if they are in the form of an ordinary annui...

See Answer

Q: You want to buy a new sports car from Muscle Motors for

You want to buy a new sports car from Muscle Motors for $68,000. The contract is in the form of a 60-month annuity due at an 7.85 percent APR. What will your monthly payment be?

See Answer

Q: Prepare an amortization schedule for a five-year loan of $

Prepare an amortization schedule for a five-year loan of $42,000. The interest rate is 8 percent per year, and the loan calls for equal annual payments. How much interest is paid in the third year? Ho...

See Answer

Q: Rework Problem 55 assuming that the loan agreement calls for a principal

Rework Problem 55 assuming that the loan agreement calls for a principal reduction of $8,400 every year instead of equal annual payments. Data from problem 55: Prepare an amortization schedule for a...

See Answer

Q: Bilbo Baggins wants to save money to meet three objectives. First

Bilbo Baggins wants to save money to meet three objectives. First, he would like to be able to retire 30 years from now with retirement income of $20,000 per month for 25 years, with the first payment...

See Answer

Q: After deciding to buy a new car, you can either lease

After deciding to buy a new car, you can either lease the car or purchase it on a three-year loan. The car you wish to buy costs $32,000. The dealer has a special leasing arrangement where you pay $99...

See Answer

Q: An All-Pro defensive lineman is in contract negotiations. The

An All-Pro defensive lineman is in contract negotiations. The team has offered the following salary structure: Time ……………………………………………….Salary 0…………………………………………………………$7,000,000 1…………………………………………………………...

See Answer

Q: This question illustrates what is known as discount interest. Imagine you

This question illustrates what is known as discount interest. Imagine you are discussing a loan with a somewhat unscrupulous lender. You want to borrow $25,000 for one year. The interest rate is 15 pe...

See Answer

Q: In preparing a balance sheet, why do you think standard accounting

In preparing a balance sheet, why do you think standard accounting practice focuses on historical cost rather than market value?

See Answer

Q: You are serving on a jury. A plaintiff is suing the

You are serving on a jury. A plaintiff is suing the city for injuries sustained after a freak street sweeper accident. In the trial, doctors testified that it will be five years before the plaintiff i...

See Answer

Q: The Maybe Pay Life Insurance Co. is trying to sell you

The Maybe Pay Life Insurance Co. is trying to sell you an investment policy that will pay you and your heirs $25,000 per year forever. If the required return on this investment is 7.2 percent, how muc...

See Answer

Q: Beginning three months from now, you want to be able to

Beginning three months from now, you want to be able to withdraw $2,300 each quarter from your bank account to cover college expenses over the next four years. If the account pays .65 percent interest...

See Answer

Q: If the appropriate discount rate for the following cash flows is 8

If the appropriate discount rate for the following cash flows is 8.45 percent per year, what is the present value of the cash flows? Year …………………………………… Cash Flow 1…………………………………………………………$1,650 2……………...

See Answer

Q: Suppose an investment offers to triple your money in 12 months (

Suppose an investment offers to triple your money in 12 months (don’t believe it). What rate of return per quarter are you being offered?

See Answer

Q: Your company will generate $73,000 in annual revenue each

Your company will generate $73,000 in annual revenue each year for the next eight years from a new information database. If the appropriate interest rate is 8.5 percent, what is the present value of t...

See Answer

Q: Why is it that municipal bonds are not taxed at the federal

Why is it that municipal bonds are not taxed at the federal level, but are taxable across state lines? Why are U.S. Treasury bonds not taxable at the state level? (You may need to dust off the history...

See Answer

Q: Mark Sexton and Todd Story, the owners of S&S

Mark Sexton and Todd Story, the owners of S&S Air, have decided to expand their operations. They instructed their newly hired financial analyst, Chris Guthrie, to enlist an underwriter to help sell $3...

See Answer

Q: If Treasury bills are currently paying 7 percent and the inflation rate

If Treasury bills are currently paying 7 percent and the inflation rate is 3.8 percent, what is the approximate real rate of interest? The exact real rate?

See Answer

Q: Locate the Treasury issue in Figure 7.4 maturing in November

Locate the Treasury issue in Figure 7.4 maturing in November 2027. Is this a note or a bond? What is its coupon rate? What is its bid price? What was the previous day’s asked price?...

See Answer

Q: In comparing accounting net income and operating cash flow, name two

In comparing accounting net income and operating cash flow, name two items you typically find in net income that are not in operating cash flow. Explain what each is and why it is excluded in operatin...

See Answer

Q: Locate the Treasury bond in Figure 7.4 maturing in November

Locate the Treasury bond in Figure 7.4 maturing in November 2024. Is this a premium or a discount bond? What is its current yield? What is its yield to maturity? What is the bid–ask...

See Answer

Q: Bond X is a premium bond making annual payments. The bond

Bond X is a premium bond making annual payments. The bond pays an 8 percent coupon, has a YTM of 6 percent, and has 13 years to maturity. Bond Y is a discount bond making annual payments. This bond pa...

See Answer

Q: You purchase a bond with an invoice price of $968.

You purchase a bond with an invoice price of $968. The bond has a coupon rate of 7.4 percent, and there are four months to the next semiannual coupon date. What is the clean price of the bond?

See Answer

Q: You purchase a bond with a coupon rate of 6.8

You purchase a bond with a coupon rate of 6.8 percent and a clean price of $1,073. If the next semiannual coupon payment is due in two months, what is the invoice price?

See Answer

Q: Suppose the following bond quotes for IOU Corporation appear in the financial

Suppose the following bond quotes for IOU Corporation appear in the financial page of today’s newspaper. Assume the bond has a face value of $1,000 and the current date is April 15,...

See Answer

Q: Sunset Boards is a small company that manufactures and sells surfboards in

Sunset Boards is a small company that manufactures and sells surfboards in Malibu. Tad Marks, the founder of the company, is in charge of the design and sale of the surfboards, but his background is i...

See Answer

Q: Suppose your company needs to raise $30 million and you want

Suppose your company needs to raise $30 million and you want to issue 30-year bonds for this purpose. Assume the required return on your bond issue will be 8 percent, and you’re evaluating two issue a...

See Answer

Q: You want to have $1.5 million in real dollars

You want to have $1.5 million in real dollars in an account when you retire in 40 years. The nominal return on your investment is 11 percent and the inflation rate is 3.8 percent. What real amount mus...

See Answer

Q: Bond P is a premium bond with a 12 percent coupon.

Bond P is a premium bond with a 12 percent coupon. Bond D is a 6 percent coupon bond currently selling at a discount. Both bonds make annual payments, have a YTM of 9 percent, and have five years to m...

See Answer

Q: The McKeegan Corporation has two different bonds currently outstanding. Bond M

The McKeegan Corporation has two different bonds currently outstanding. Bond M has a face value of $20,000 and matures in 20 years. The bond makes no payments for the first six years, then pays $1,100...

See Answer

Q: Consider the prices in the following three Treasury issues as of May

Consider the prices in the following three Treasury issues as of May 15, 2007: The bond in the middle is callable in February 2008. What is the implied value of the call feature?

See Answer

Q: When Marilyn Monroe died, ex-husband Joe DiMaggio vowed to

When Marilyn Monroe died, ex-husband Joe DiMaggio vowed to place fresh flowers on her grave every Sunday as long as he lived. The week after she died in 1962, a bunch of fresh flowers that the former...

See Answer

Q: You are planning to save for retirement over the next 30 years

You are planning to save for retirement over the next 30 years. To save for retirement, you will invest $900 a month in a stock account in real dollars and $450 a month in a bond account in real dolla...

See Answer

Q: Is the yield to maturity on a bond the same thing as

Is the yield to maturity on a bond the same thing as the required return? Is YTM the same thing as the coupon rate? Suppose today a 10 percent coupon bond sells at par. Two years from now, the require...

See Answer

Q: Suppose you buy a 7 percent coupon, 20-year bond

Suppose you buy a 7 percent coupon, 20-year bond today when it’s first issued. If interest rates suddenly rise to 15 percent, what happens to the value of your bond? Why?

See Answer

Q: Staind, Inc., has 7.5 percent coupon bonds on

Staind, Inc., has 7.5 percent coupon bonds on the market that have 10 years left to maturity. The bonds make annual payments. If the YTM on these bonds is 8.75 percent, what is the current bond price?...

See Answer

Q: Jetson Spacecraft Corp. shows the following information on its 2009 income

Jetson Spacecraft Corp. shows the following information on its 2009 income statement: sales=$196,000; costs $104,000; other expenses=$6,800; depreciation expense=$9,100; interest expense=$14,800; taxe...

See Answer

Q: Ackerman Co. has 9 percent coupon bonds on the market with

Ackerman Co. has 9 percent coupon bonds on the market with nine years left to maturity. The bonds make annual payments. If the bond currently sells for $934, what is its YTM?

See Answer

Q: Kiss the Sky Enterprises has bonds on the market making annual payments

Kiss the Sky Enterprises has bonds on the market making annual payments, with 13 years to maturity, and selling for $1,045. At this price, the bonds yield 7.5 percent. What must the coupon rate be on...

See Answer

Q: Grohl Co. issued 11-year bonds a year ago at

Grohl Co. issued 11-year bonds a year ago at a coupon rate of 6.9 percent. The bonds make semiannual payments. If the YTM on these bonds is 7.4 percent, what is the current bond price?

See Answer

Q: Ngata Corp. issued 12-year bonds 2 years ago at

Ngata Corp. issued 12-year bonds 2 years ago at a coupon rate of 8.4 percent. The bonds make semiannual payments. If these bonds currently sell for 105 percent of par value, what is the YTM?

See Answer

Q: Ashes Divide Corporation has bonds on the market with 14.5

Ashes Divide Corporation has bonds on the market with 14.5 years to maturity, a YTM of 6.8 percent, and a current price of $924. The bonds make semiannual payments. What must the coupon rate be on the...

See Answer

Q: An investment offers a 14 percent total return over the coming year

An investment offers a 14 percent total return over the coming year. Bill Bernanke thinks the total real return on this investment will be only 9 percent. What does Bill believe the inflation rate wil...

See Answer

Q: Say you own an asset that had a total return last year

Say you own an asset that had a total return last year of 11.4 percent. If the inflation rate last year was 4.8 percent, what was your real return?

See Answer

Q: Suppose the real rate is 3 percent and the inflation rate is

Suppose the real rate is 3 percent and the inflation rate is 4.7 percent. What rate would you expect to see on a Treasury bill?

See Answer

Q: Some companies, such as Reader’s Digest, have created classes of

Some companies, such as Reader’s Digest, have created classes of stock with no voting rights at all. Why would investors buy such stock?

See Answer

Q: Referring to the previous questions, under what circumstances might a company

Referring to the previous questions, under what circumstances might a company choose not to pay dividends?

See Answer

Q: Given the following information for Rosato Pizza Co., calculate the depreciation

Given the following information for Rosato Pizza Co., calculate the depreciation expense: sales=$41,000; costs=$19,500; addition to retained earnings=$5,100; dividends paid=$1,500; interest expense=$4...

See Answer

Q: When it comes to voting in elections, what are the differences

When it comes to voting in elections, what are the differences between U.S. political democracy and U.S. corporate democracy?

See Answer

Q: One of the assumptions of the two stage growth model is that

One of the assumptions of the two stage growth model is that the dividends drop immediately from the high growth rate to the perpetual growth rate. What do you think about this assumption? What happen...

See Answer

Q: A substantial percentage of the companies listed on the NYSE and NASDAQ

A substantial percentage of the companies listed on the NYSE and NASDAQ don’t pay dividends, but investors are nonetheless willing to buy shares in them. How is this possible given your answer to the...

See Answer

Q: Ragan, Inc., was founded nine years ago by brother and

Ragan, Inc., was founded nine years ago by brother and sister Carrington and Genevieve Ragan. The company manufactures and installs commercial heating, ventilation, and cooling (HVAC) units. Ragan, In...

See Answer

Q: Antiques R Us is a mature manufacturing fi rm. The company

Antiques R Us is a mature manufacturing fi rm. The company just paid a $10.46 dividend, but management expects to reduce the payout by 4 percent per year indefi nitely. If you require an 11.5 percent...

See Answer

Q: Teder Corporation stock currently sells for $64 per share. The

Teder Corporation stock currently sells for $64 per share. The market requires a 10 percent return on the firm’s stock. If the company maintains a constant 4.5 percent growth rate in dividends, what w...

See Answer

Q: E-Eyes.com Bank just issued some new preferred stock

E-Eyes.com Bank just issued some new preferred stock. The issue will pay a $20 annual dividend in perpetuity, beginning 20 years from now. If the market requires a 6.4 percent return on this investmen...

See Answer

Q: You have found the following stock quote for RJW Enterprises, Inc

You have found the following stock quote for RJW Enterprises, Inc., in the financial pages of today’s newspaper. What was the closing price for this stock that appeared in yesterday&...

See Answer

Q: Chartreuse County Choppers Inc. is experiencing rapid growth. The company

Chartreuse County Choppers Inc. is experiencing rapid growth. The company expects dividends to grow at 25 percent per year for the next 11 years before leveling off at 6 percent into perpetuity. The r...

See Answer

Q: Consider four different stocks, all of which have a required return

Consider four different stocks, all of which have a required return of 19 percent and a most recent dividend of $4.50 per share. Stocks W, X, and Y are expected to maintain constant growth rates in di...

See Answer

Q: Prepare a 2009 balance sheet for Bertinelli Corp. based on the

Prepare a 2009 balance sheet for Bertinelli Corp. based on the following information: cash =$195,000; patents and copyrights =$780,000; accounts payable =$405,000; accounts receivable =$137,000; tangi...

See Answer

Q: Storico Co. just paid a dividend of $2.45

Storico Co. just paid a dividend of $2.45 per share. The company will increase its dividend by 20 percent next year and will then reduce its dividend growth rate by 5 percentage points per year until...

See Answer

Q: This one’s a little harder. Suppose the current share price for

This one’s a little harder. Suppose the current share price for the firm in the previous problem is $63.82 and all the dividend information remains the same. What required return must investors be dem...

See Answer

Q: Assume a stock has dividends that grow at a constant rate forever

Assume a stock has dividends that grow at a constant rate forever. If you value the stock using the constant dividend growth model, how many years worth of dividends constitute one-half of the stock’s...

See Answer

Q: Regarding the two-stage dividend growth model in the chapter,

Regarding the two-stage dividend growth model in the chapter, show that the price of a share of stock today can be written as follows: Can you provide an intuitive interpretation of this expression?...

See Answer

Q: The chapter shows that in the two-stage dividend growth model

The chapter shows that in the two-stage dividend growth model, the growth rate in the first stage, g1 , can be greater than or less than the discount rate, R . Can they be exactly equal?

See Answer

Q: The Jackson–Timberlake Wardrobe Co. just paid a dividend of

The Jackson–Timberlake Wardrobe Co. just paid a dividend of $1.95 per share on its stock. The dividends are expected to grow at a constant rate of 6 percent per year indefinitely. If investors require...

See Answer

Q: Keenan Co. is expected to maintain a constant 5.2

Keenan Co. is expected to maintain a constant 5.2 percent growth rate in its dividends indefinitely. If the company has a dividend yield of 6.3 percent, what is the required return on the company’s st...

See Answer

Q: Suppose you know that a company’s stock currently sells for $47

Suppose you know that a company’s stock currently sells for $47 per share and the required return on the stock is 11 percent. You also know that the total return on the stock is evenly divided between...

See Answer

Q: Resnor, Inc., has an issue of preferred stock outstanding that

Resnor, Inc., has an issue of preferred stock outstanding that pays a $5.50 dividend every year in perpetuity. If this issue currently sells for $108 per share, what is the required return?

See Answer

Q: Red, Inc., Yellow Corp., and Blue Company each will

Red, Inc., Yellow Corp., and Blue Company each will pay a dividend of $2.35 next year. The growth rate in dividends for all three companies is 5 percent. The required return for each company’s stock i...

See Answer

Q: Dimeback, Inc., is obligated to pay its creditors $7

Dimeback, Inc., is obligated to pay its creditors $7,300 during the year. a. What is the market value of the shareholders’ equity if assets have a market value of $8,400? b. What if assets equal $6,70...

See Answer

Q: Great Pumpkin Farms just paid a dividend of $3.50

Great Pumpkin Farms just paid a dividend of $3.50 on its stock. The growth rate in dividends is expected to be a constant 5 percent per year indefinitely. Investors require a 14 percent return on the...

See Answer

Q: Metallica Bearings, Inc., is a young start-up company

Metallica Bearings, Inc., is a young start-up company. No dividends will be paid on the stock over the next nine years because the fi rm needs to plow back its earnings to fuel growth. The company wil...

See Answer

Q: Bread, Inc., has an odd dividend policy. The company

Bread, Inc., has an odd dividend policy. The company has just paid a dividend of $6 per share and has announced that it will increase the dividend by $4 per share for each of the next fi ve years, and...

See Answer

Q: Far Side Corporation is expected to pay the following dividends over the

Far Side Corporation is expected to pay the following dividends over the next four years: $11, $8, $5, and $2. Afterward, the company pledges to maintain a constant 5 percent growth rate in dividends...

See Answer

Q: Marcel Co. is growing quickly. Dividends are expected to grow

Marcel Co. is growing quickly. Dividends are expected to grow at a 30 percent rate for the next three years, with the growth rate falling off to a constant 6 percent thereafter. If the required return...

See Answer

Q: Eva Corp. is experiencing rapid growth. Dividends are expected to

Eva Corp. is experiencing rapid growth. Dividends are expected to grow at 25 percent per year during the next three years, 15 percent over the following year, and then 8 percent per year indefinitely....

See Answer

Q: Metroplex Corporation will pay a $3.04 per share dividend

Metroplex Corporation will pay a $3.04 per share dividend next year. The company pledges to increase its dividend by 3.8 percent per year indefinitely. If you require an 11 percent return on your inve...

See Answer

Q: Apocalyptica Corp. pays a constant $9.75 dividend on

Apocalyptica Corp. pays a constant $9.75 dividend on its stock. The company will maintain this dividend for the next 11 years and will then cease paying dividends forever. If the required return on th...

See Answer

Q: Thirsty Cactus Corp. just paid a dividend of $1.

Thirsty Cactus Corp. just paid a dividend of $1.25 per share. The dividends are expected to grow at 28 percent for the next eight years and then level off to a 6 percent growth rate indefinitely. If t...

See Answer

Q: The next dividend payment by Hot Wings, Inc., will be

The next dividend payment by Hot Wings, Inc., will be $2.10 per share. The dividends are anticipated to maintain a 5 percent growth rate forever. If the stock currently sells for $48 per share, what i...

See Answer

Q: In response to the Sarbanes–Oxley Act, many small firms

In response to the Sarbanes–Oxley Act, many small firms in the United States have opted to “go dark” and delist their stock. Why might a company choose this route? What are the costs of “going dark”?...

See Answer

Q: (Refer to Table 2.3 .) Corporation Growth has $

(Refer to Table 2.3 .) Corporation Growth has $88,000 in taxable income, and Corporation Income has $8,800,000 in taxable income. a. What is the tax bill for each firm? b. Suppose both firms have iden...

See Answer

Q: Concerning the profitability index: a. Describe how the profitability

Concerning the profitability index: a. Describe how the profitability index is calculated, and describe the information this measure provides about a sequence of cash flows. What is the profitability...

See Answer

Q: In January 2008, automobile manufacturer Volkswagen announced plans to build an

In January 2008, automobile manufacturer Volkswagen announced plans to build an automatic transmission and engine plant in South Carolina. Volkswagen apparently felt that it would be better able to co...

See Answer

Q: It is sometimes stated that “the net present value approach assumes

It is sometimes stated that “the net present value approach assumes reinvestment of the intermediate cash flows at the required return.” Is this claim correct? To answer, suppose you calculate the NPV...

See Answer

Q: Concerning payback: a. Describe how the payback period is

Concerning payback: a. Describe how the payback period is calculated, and describe the information this measure provides about a sequence of cash flows. What is the payback criterion decision rule? b....

See Answer

Q: Concerning discounted payback: a. Describe how the discounted payback

Concerning discounted payback: a. Describe how the discounted payback period is calculated, and describe the information this measure provides about a sequence of cash flows. What is the discounted pa...

See Answer

Q: Concerning AAR: a. Describe how the average accounting return

Concerning AAR: a. Describe how the average accounting return is usually calculated, and describe the information this measure provides about a sequence of cash flows. What is the AAR criterion decisi...

See Answer

Q: Concerning NPV: a. Describe how NPV is calculated,

Concerning NPV: a. Describe how NPV is calculated, and describe the information this measure provides about a sequence of cash flows. What is the NPV criterion decision rule? b. Why is NPV considered...

See Answer

Q: Concerning IRR: a. Describe how the IRR is calculated

Concerning IRR: a. Describe how the IRR is calculated, and describe the information this measure provides about a sequence of cash flows. What is the IRR criterion decision rule? b. What is the relati...

See Answer

Q: Seth Bullock, the owner of Bullock Gold Mining, is evaluating

Seth Bullock, the owner of Bullock Gold Mining, is evaluating a new gold mine in South Dakota. Dan Dority, the company’s geologist, has just finished his analysis of the mine site. He has estimated th...

See Answer

Q: What is the IRR of the following set of cash flows?

What is the IRR of the following set of cash flows? Year ……………………………… Cash Flow 0………………………………………………..−$19,500 1…………………………………………………….9,800 2…….……………………………..……………..10,300 3…………………………………………………….8,600

See Answer

Q: During 2009, Raines Umbrella Corp. had sales of $730

During 2009, Raines Umbrella Corp. had sales of $730,000. Cost of goods sold, administrative and selling expenses, and depreciation expenses were $580,000, $105,000, and $135,000, respectively. In add...

See Answer

Q: For the cash flows in the previous problem, what is the

For the cash flows in the previous problem, what is the NPV at a discount rate of zero percent? What if the discount rate is 10 percent? If it is 20 percent? If it is 30 percent?

See Answer

Q: Mahjong, Inc., has identified the following two mutually exclusive projects

Mahjong, Inc., has identified the following two mutually exclusive projects: a. What is the IRR for each of these projects? Using the IRR decision rule, which project should the company accept? Is t...

See Answer

Q: Consider the following two mutually exclusive projects: /

Consider the following two mutually exclusive projects: Sketch the NPV profiles for X and Y over a range of discount rates from zero to 25 percent. What is the crossover rate for these two projects?...

See Answer

Q: Light Sweet Petroleum, Inc., is trying to evaluate a generation

Light Sweet Petroleum, Inc., is trying to evaluate a generation project with the following cash flows: Year ……………………………… Cash Flow 0………………………………………−$45,000,000 1……………………………….………….78,000,000 2……………………...

See Answer

Q: What is the profitability index for the following set of cash flows

What is the profitability index for the following set of cash flows if the relevant discount rate is 10 percent? What if the discount rate is 15 percent? If it is 22 percent? Year...………………….. Cash Fl...

See Answer

Q: The Weiland Computer Corporation is trying to choose between the following two

The Weiland Computer Corporation is trying to choose between the following two mutually exclusive design projects: a. If the required return is 10 percent and the company applies the profitability i...

See Answer

Q: Consider the following two mutually exclusive projects: /

Consider the following two mutually exclusive projects: Whichever project you choose, if any, you require a 15 percent return on your investment. a. If you apply the payback criterion, which investm...

See Answer

Q: An investment has an installed cost of $684,680.

An investment has an installed cost of $684,680. The cash flows over the four-year life of the investment are projected to be $263,279, $294,060, $227,604, and $174,356. If the discount rate is zero,...

See Answer

Q: Slow Ride Corp. is evaluating a project with the following cash

Slow Ride Corp. is evaluating a project with the following cash flows: Year ……………………. Cash Flow 0………………………………....−$16,000 1……………………………………..…..6,100 2………………………………….……..7,800 3…………………………..…………….8,400 4...

See Answer

Q: Suppose the company in the previous problem uses an 11 percent discount

Suppose the company in the previous problem uses an 11 percent discount rate and an 8 percent reinvestment rate on all of its projects. Calculate the MIRR of the project using all three methods using...

See Answer

Q: In Problem 19, suppose Raines Umbrella Corp. paid out $

In Problem 19, suppose Raines Umbrella Corp. paid out $25,000 in cash dividends. Is this possible? If spending on net fixed assets and net working capital was zero, and if no new stock was issued duri...

See Answer

Q: A project has an initial cost of I, has a required

A project has an initial cost of I, has a required return of R , and pays C annually for N years. a. Find C in terms of I and N such that the project has a payback period just equal to its life. b. Fi...

See Answer

Q: An investment under consideration has a payback of seven years and a

An investment under consideration has a payback of seven years and a cost of $724,000. If the required return is 12 percent, what is the worst-case NPV? The best-case NPV? Explain. Assume the cash flo...

See Answer

Q: This problem is useful for testing the ability of financial calculators and

This problem is useful for testing the ability of financial calculators and computer software. Consider the following cash flows. How many different IRRs are there? When should we take this project?...

See Answer

Q: The Yurdone Corporation wants to set up a private cemetery business.

The Yurdone Corporation wants to set up a private cemetery business. According to the CFO, Barry M. Deep, business is “looking up.” As a result, the cemetery project will provide a net cash inflow of...

See Answer

Q: A project has the following cash flows: Year ……………………………

A project has the following cash flows: Year …………………………… Cash Flow 0……………………………..……………. $58,000 1…………………………………..………. −34,000 2…………………………………………….−45,000 What is the IRR for this project? If the requi...

See Answer

Q: Anderson International Limited is evaluating a project in Erewhon. The project

Anderson International Limited is evaluating a project in Erewhon. The project will create the following cash flows: Year ……………………………. Cash Flow 0……….……………………….……….−$750,000 1……………………………………….……..205,...

See Answer

Q: What is the payback period for the following set of cash flows

What is the payback period for the following set of cash flows? Year ………………………. Cash Flow 0…………………………………………−$6,400 1………………………………………….…1,600 2…………………………………………….1,900 3…………………………………………….2,300 4……………………...

See Answer

Q: An investment project provides cash inflows of $765 per year for

An investment project provides cash inflows of $765 per year for eight years. What is the project payback period if the initial cost is $2,400? What if the initial cost is $3,600? What if it is $6,500...

See Answer

Q: Buy Coastal, Inc., imposes a payback cutoff of three years

Buy Coastal, Inc., imposes a payback cutoff of three years for its international investment projects. If the company has the following two projects available, should it accept either of them?

See Answer

Q: An investment project has annual cash inflows of $4,200

An investment project has annual cash inflows of $4,200, $5,300, $6,100, and $7,400, and a discount rate of 14 percent. What is the discounted payback period for these cash flows if the initial cost i...

See Answer

Q: Dahlia Industries had the following operating results for 2009: sales =$

Dahlia Industries had the following operating results for 2009: sales =$22,800; cost of goods sold =$16,050; depreciation expense =$4,050; interest expense =$1,830; dividends paid =$1,300. At the begi...

See Answer

Q: An investment project costs $15,000 and has annual cash

An investment project costs $15,000 and has annual cash flows of $4,300 for six years. What is the discounted payback period if the discount rate is zero percent? What if the discount rate is 5 percen...

See Answer

Q: You’re trying to determine whether to expand your business by building a

You’re trying to determine whether to expand your business by building a new manufacturing plant. The plant has an installation cost of $15 million, which will be depreciated straight-line to zero ove...

See Answer

Q: A firm evaluates all of its projects by applying the IRR rule

A firm evaluates all of its projects by applying the IRR rule. If the required return is 16 percent, should the firm accept the following project? Year ……………………………… Cash Flow 0……………………………………………….−$34...

See Answer

Q: For the cash flows in the previous problem, suppose the fi

For the cash flows in the previous problem, suppose the fi rm uses the NPV decision rule. At a required return of 11 percent, should the fi rm accept this project? What if the required return was 30 p...

See Answer

Q: A project that provides annual cash flows of $28,500

A project that provides annual cash flows of $28,500 for nine years costs $138,000 today. Is this a good project if the required return is 8 percent? What if it’s 20 percent? At what discount rate wou...

See Answer

Q: If we define the NPV index as the ratio of NPV to

If we define the NPV index as the ratio of NPV to cost, what is the relationship between this index and the profitability index?

See Answer

Q: In evaluating the Cayenne, would you consider the possible damage to

In evaluating the Cayenne, would you consider the possible damage to Porsche’s reputation erosion?

See Answer

Q: “When evaluating projects, we’re concerned with only the relevant incremental

“When evaluating projects, we’re concerned with only the relevant incremental aftertax cash flows. Therefore, because depreciation is a noncash expense, we should ignore its effects when evaluating pr...

See Answer

Q: A major college textbook publisher has an existing finance textbook. The

A major college textbook publisher has an existing finance textbook. The publisher is debating whether to produce an “essentialized” version, meaning a shorter (and lower-priced) book. What are some o...

See Answer

Q: Conch Republic Electronics is a midsized electronics manufacturer located in Key West

Conch Republic Electronics is a midsized electronics manufacturer located in Key West, Florida. The company president is Shelley Couts, who inherited the company. When it was founded over 70 years ago...

See Answer

Q: Consider the following abbreviated financial statements for Parrothead Enterprises:

Consider the following abbreviated financial statements for Parrothead Enterprises: a. What is owners’ equity for 2008 and 2009? b. What is the change in net working capital for 20...

See Answer

Q: Geary Machine Shop is considering a four-year project to improve

Geary Machine Shop is considering a four-year project to improve its production efficiency. Buying a new machine press for $560,000 is estimated to result in $210,000 in annual pretax cost savings. Th...

See Answer

Q: Dangerfield Industrial Systems Company (DISC) is trying to decide between

Dangerfield Industrial Systems Company (DISC) is trying to decide between two different conveyor belt systems. System A costs $430,000, has a four-year life, and requires $110,000 in pretax annual ope...

See Answer

Q: Suppose in the previous problem that DISC always needs a conveyor belt

Suppose in the previous problem that DISC always needs a conveyor belt system; when one wears out, it must be replaced. Which project should the firm choose now?

See Answer

Q: Consider a project to supply 100 million postage stamps per year to

Consider a project to supply 100 million postage stamps per year to the U.S. Postal Service for the next five years. You have an idle parcel of land available that cost $2,400,000 five years ago; if t...

See Answer

Q: In the previous problem, suppose you were going to use a

In the previous problem, suppose you were going to use a three-year MACRS depreciation schedule for your manufacturing equipment, and you could keep working capital investments down to only $25,000 pe...

See Answer

Q: Vandalay Industries is considering the purchase of a new machine for the

Vandalay Industries is considering the purchase of a new machine for the production of latex. Machine A costs $2,900,000 and will last for six years. Variable costs are 35 percent of sales, and fixed...

See Answer

Q: Compact fluorescent lamps (CFLs) have become more popular in recent

Compact fluorescent lamps (CFLs) have become more popular in recent years, but do they make financial sense? Suppose a typical 60-watt incandescent light bulb costs $.50 and lasts 1,000 hours. A 15-wa...

See Answer

Q: The previous problem suggests that using CFLs instead of incandescent bulbs is

The previous problem suggests that using CFLs instead of incandescent bulbs is a no-brainer. However, electricity costs actually vary quite a bit depending on location and user type (you can get infor...

See Answer

Q: The previous two problems suggest that using CFLs is a good idea

The previous two problems suggest that using CFLs is a good idea from a purely financial perspective unless you live in an area where power is relatively inexpensive, but there is another wrinkle. Sup...

See Answer

Q: The debate regarding CFLs versus incandescent bulbs (see Problems 25–

The debate regarding CFLs versus incandescent bulbs (see Problems 25–27) has even more wrinkles. In no particular order: 1. Incandescent bulbs generate a lot more heat than CFLs. 2. CFL prices will p...

See Answer

Q: Draw up an income statement and balance sheet for this company for

Draw up an income statement and balance sheet for this company for 2008 and 2009.

See Answer

Q: Your small remodeling business has two work vehicles. One is a

Your small remodeling business has two work vehicles. One is a small passenger car used for job-site visits and for other general business purposes. The other is a heavy truck used to haul equipment....

See Answer

Q: You have been hired as a consultant for Pristine Urban-Tech

You have been hired as a consultant for Pristine Urban-Tech Zither, Inc. (PUTZ), manufacturers of fi ne zithers. The market for zithers is growing quickly. The company bought some land three years ago...

See Answer

Q: Aguilera Acoustics, Inc. (AAI), projects unit sales for

Aguilera Acoustics, Inc. (AAI), projects unit sales for a new seven-octave voice emulation implant as follows: Year …………………………..Unit Sales 1………………………………….………… 93,000 2……………………………………………105,000 3………………...

See Answer

Q: A proposed cost-saving device has an installed cost of $

A proposed cost-saving device has an installed cost of $610,000. The device will be used in a five-year project but is classified as three-year MACRS property for tax purposes. The required initial ne...

See Answer

Q: Your company has been approached to bid on a contract to sell

Your company has been approached to bid on a contract to sell 17,500 voice recognition (VR) computer keyboards a year for four years. Due to technological improvements, beyond that time they will be o...

See Answer

Q: Suppose we are thinking about replacing an old computer with a new

Suppose we are thinking about replacing an old computer with a new one. The old one cost us $650,000; the new one will cost $780,000. The new machine will be depreciated straight-line to zero over its...

See Answer

Q: Parker & Stone, Inc., is looking at setting up a

Parker & Stone, Inc., is looking at setting up a new manufacturing plant in South Park to produce garden tools. The company bought some land six years ago for $6 million in anticipation of using it as...

See Answer

Q: Winnebagel Corp. currently sells 30,000 motor homes per year

Winnebagel Corp. currently sells 30,000 motor homes per year at $53,000 each, and 12,000 luxury motor coaches per year at $91,000 each. The company wants to introduce a new portable camper to fill out...

See Answer

Q: A proposed new investment has projected sales of $830,000

A proposed new investment has projected sales of $830,000. Variable costs are 60 percent of sales, and fixed costs are $181,000; depreciation is $77,000. Prepare a pro forma income statement assuming...

See Answer

Q: Consider the following income statement: Sales…………………………… m,……$

Consider the following income statement: Sales…………………………… m,……$824,500 Costs……………………………….….538,900 Depreciation……………..………….126,500 EBIT………………………………………….?….. Taxes (34%)……………………………..…?….. Net income……...

See Answer

Q: A proposed new project has projected sales of $108,000

A proposed new project has projected sales of $108,000, costs of $51,000, and depreciation of $6,800. The tax rate is 35 percent. Calculate operating cash flow using the four different approaches desc...

See Answer

Q: A piece of newly purchased industrial equipment costs $1,080

A piece of newly purchased industrial equipment costs $1,080,000 and is classified as seven-year property under MACRS. Calculate the annual depreciation allowances and end-of-the-year book values for...

See Answer

Q: Consider an asset that costs $548,000 and is depreciated

Consider an asset that costs $548,000 and is depreciated straight-line to zero over its eight-year tax life. The asset is to be used in a five-year project; at the end of the project, the asset can be...

See Answer

Q: An asset used in a four-year project falls in the

An asset used in a four-year project falls in the five-year MACRS class for tax purposes. The asset has an acquisition cost of $7,900,000 and will be sold for $1,400,000 at the end of the project. If...

See Answer

Q: Summer Tyme, Inc., is considering a new three year expansion

Summer Tyme, Inc., is considering a new three year expansion project that requires an initial fixed asset investment of $3.9 million. The fixed asset will be depreciated straight-line to zero over its...

See Answer

Q: In the previous problem, suppose the project requires an initial investment

In the previous problem, suppose the project requires an initial investment in net working capital of $300,000, and the fixed asset will have a market value of $210,000 at the end of the project. What...

See Answer

Q: In the previous problem, suppose the fixed asset actually falls into

In the previous problem, suppose the fixed asset actually falls into the three-year MACRS class. All the other facts are the same. What is the project’s year 1 net cash flow now? Year 2? Year 3? What...

See Answer

Q: Dog Up! Franks is looking at a new sausage system with

Dog Up! Franks is looking at a new sausage system with an installed cost of $560,000. This cost will be depreciated straight-line to zero over the project’s five-year life, at the end of which the sau...

See Answer

Q: Your firm is contemplating the purchase of a new $720,

Your firm is contemplating the purchase of a new $720,000 computer-based order entry system. The system will be depreciated straight-line to zero over its five-year life. It will be worth $75,000 at t...

See Answer

Q: Penguin Pucks, Inc., has current assets of $5,

Penguin Pucks, Inc., has current assets of $5,100, net fixed assets of $23,800, current liabilities of $4,300, and long-term debt of $7,400. What is the value of the shareholders’ equity account for t...

See Answer

Q: In the previous problem, suppose your required return on the project

In the previous problem, suppose your required return on the project is 20 percent and your pretax cost savings are $300,000 per year. Will you accept the project? What if the pretax cost savings are...

See Answer

Q: A five-year project has an initial fixed asset investment of

A five-year project has an initial fixed asset investment of $270,000, an initial NWC investment of $25,000, and an annual OCF of −$42,000. The fixed asset is fully depreciated over the life of the pr...

See Answer

Q: You are evaluating two different silicon wafer milling machines. The Techron

You are evaluating two different silicon wafer milling machines. The Techron I costs $290,000, has a three-year life, and has pretax operating costs of $67,000 per year. The Techron II costs $510,000,...

See Answer

Q: Alson Enterprises needs someone to supply it with 185,000 cartons

Alson Enterprises needs someone to supply it with 185,000 cartons of machine screws per year to support its manufacturing needs over the next five years, and you’ve decided to bid on the contract. It...

See Answer

Q: In the previous problem, suppose you drive the truck x miles

In the previous problem, suppose you drive the truck x miles per year. How many miles would you have to drive the car before upgrading the car would be the better choice?

See Answer

Q: In the previous problem, suppose the required return on the project

In the previous problem, suppose the required return on the project is 12 percent. What is the project’s NPV?

See Answer

Q: What are the three types of financial management decisions? For each

What are the three types of financial management decisions? For each type of decision, give an example of a business transaction that would be relevant.

See Answer

Q: Papa Roach Exterminators, Inc., has sales of $586,

Papa Roach Exterminators, Inc., has sales of $586,000, costs of $247,000, depreciation expense of $43,000, interest expense of $32,000, and a tax rate of 35 percent. What is the net income for this fi...

See Answer

Q: Klingon Widgets, Inc., purchased new cloaking machinery three years ago

Klingon Widgets, Inc., purchased new cloaking machinery three years ago for $7 million. The machinery can be sold to the Romulans today for $4.9 million. Klingon’s current balance sheet shows net fixe...

See Answer

Q: In a large corporation, what are the two distinct groups that

In a large corporation, what are the two distinct groups that report to the chief financial officer? Which group is the focus of corporate finance?

See Answer

Q: So Long, Inc., has sales of $27,500

So Long, Inc., has sales of $27,500, costs of $13,280, depreciation expense of $2,300, and interest expense of $1,105. If the tax rate is 35 percent, what is the operating cash flow, or OCF?

See Answer

Q: The 2008 balance sheet of Maria’s Tennis Shop, Inc., showed

The 2008 balance sheet of Maria’s Tennis Shop, Inc., showed $740,000 in the common stock account and $5.2 million in the additional paid-in surplus account. The 2009 balance sheet showed $815,000 and...

See Answer

Q: Given the information for Maria’s Tennis Shop, Inc., in Problems

Given the information for Maria’s Tennis Shop, Inc., in Problems 11 and 12, suppose you also know that the firm’s net capital spending for 2009 was $940,000, and that the firm reduced its net working...

See Answer

Q: Earnhardt Driving School’s 2008 balance sheet showed net fixed assets of $

Earnhardt Driving School’s 2008 balance sheet showed net fixed assets of $3.4 million, and the 2009 balance sheet showed net fixed assets of $4.2 million. The company’s 2009 income statement showed a...

See Answer

Q: The 2008 balance sheet of Saddle Creek, Inc., showed current

The 2008 balance sheet of Saddle Creek, Inc., showed current assets of $2,100 and current liabilities of $1,380. The 2009 balance sheet showed current assets of $2,250 and current liabilities of $1,71...

See Answer

Q: The 2008 balance sheet of Maria’s Tennis Shop, Inc., showed

The 2008 balance sheet of Maria’s Tennis Shop, Inc., showed long-term debt of $2.6 million, and the 2009 balance sheet showed long-term debt of $2.9 million. The 2009 income statement showed an intere...

See Answer

Q: Suppose the firm in Problem 2 paid out $73,000

Suppose the firm in Problem 2 paid out $73,000 in cash dividends. What is the addition to retained earnings? Information from Problem 2 Papa Roach Exterminators, Inc., has sales of $586,000, costs of...

See Answer

Q: Suppose the firm in Problem 3 had 85,000 shares of

Suppose the firm in Problem 3 had 85,000 shares of common stock outstanding. What is the earnings per share, or EPS, figure? What is the dividends per share figure?

See Answer

Q: The Renata Co. had $236,000 in 2009 taxable

The Renata Co. had $236,000 in 2009 taxable income. Using the rates from Table 2.3 in the chapter, calculate the company’s 2009 income taxes.

See Answer

Q: In Problem 6, what is the average tax rate? What

In Problem 6, what is the average tax rate? What is the marginal tax rate? Problem 6 The Renata Co. had $236,000 in 2009 taxable income. Using the rates from Table 2.3 in the chapter, calculate the...

See Answer

Q: What goal should always motivate the actions of a firm’s financial manager

What goal should always motivate the actions of a firm’s financial manager?

See Answer

Q: Fully explain the kind of information the following financial ratios provide about

Fully explain the kind of information the following financial ratios provide about a firm: a. Quick ratio. b. Cash ratio. c. Total asset turnover. d. Equity multiplier. e. Long-term debt ratio. f. Tim...

See Answer

Q: What types of information do common-size financial statements reveal about

What types of information do common-size financial statements reveal about the firm? What is the best use for these common-size statements? What purpose do common–base year statements have? When would...

See Answer

Q: Explain what peer group analysis is. As a financial manager,

Explain what peer group analysis is. As a financial manager, how could you use the results of peer group analysis to evaluate the performance of your firm? How is a peer group different from an aspira...

See Answer

Q: Why is the Du Pont identity a valuable tool for analyzing the

Why is the Du Pont identity a valuable tool for analyzing the performance of a firm? Discuss the types of information it reveals compared to ROE considered by itself.

See Answer

Q: So-called same-store sales are a very important measure

So-called same-store sales are a very important measure for companies as diverse as McDonald’s and Sears. As the name suggests, examining same-store sales means comparing revenues from the same stores...

See Answer

Q: There are many ways of using standardized financial information beyond those discussed

There are many ways of using standardized financial information beyond those discussed in this chapter. The usual goal is to put firms on an equal footing for comparison purposes. For example, for aut...

See Answer

Q: In recent years, several manufacturing companies have reported the cash flow

In recent years, several manufacturing companies have reported the cash flow from the sale of Treasury securities in the cash from operations section of the statement of cash flows. What is the proble...

See Answer

Q: Suppose a company lengthens the time it takes to pay suppliers.

Suppose a company lengthens the time it takes to pay suppliers. How would this affect the statement of cash flows? How sustainable is the change in cash flows from this practice?

See Answer

Q: What effect would the following actions have on a firm’s current ratio

What effect would the following actions have on a firm’s current ratio? Assume that net working capital is positive. a. Inventory is purchased. b. A supplier is paid. c. A short-term bank loan is repa...

See Answer

Q: Explain what it means for a firm to have a current ratio

Explain what it means for a firm to have a current ratio equal to .50. Would the firm be better off if the current ratio were 1.50? What if it were 15.0? Explain your answers.

See Answer

Q: In early 2003, Doc and Lyn McGee formed the McGee Cake

In early 2003, Doc and Lyn McGee formed the McGee Cake Company. The company produced a full line of cakes, and its specialties included chess cake, lemon pound cake, and doubleiced, double-chocolate c...

See Answer

Q: Specialized ratios are sometimes used in specific industries. For example,

Specialized ratios are sometimes used in specific industries. For example, the so-called book-to-bill ratio is closely watched for semiconductor manufacturers. A ratio of .93 indicates that for every...

See Answer

Q: In recent years, Dixie Co. has greatly increased its current

In recent years, Dixie Co. has greatly increased its current ratio. At the same time, the quick ratio has fallen. What has happened? Has the liquidity of the company improved?

See Answer

Q: Chris Guthrie was recently hired by S&S Air, Inc

Chris Guthrie was recently hired by S&S Air, Inc., to assist the company with its financial planning and to evaluate the company’s performance. Chris graduated from college five...

See Answer

Q: Prepare the 2008 and 2009 common-size balance sheets for Just

Prepare the 2008 and 2009 common-size balance sheets for Just Dew It.

See Answer

Q: Prepare the 2009 combined common-size, common–base year

Prepare the 2009 combined common-size, common–base year balance sheet for Just Dew It.

See Answer

Q: For each account on this company’s balance sheet, show the change

For each account on this company’s balance sheet, show the change in the account during 2009 and note whether this change was a source or use of cash. Do your numbers add up and make...

See Answer

Q: Based on the balance sheets given for Just Dew It, calculate

Based on the balance sheets given for Just Dew It, calculate the following financial ratios for each year: a. Current ratio. b. Quick ratio. c. Cash ratio. d. NWC to total assets ratio. e. Debtâ...

See Answer

Q: Y3K, Inc., has sales of $5,276,

Y3K, Inc., has sales of $5,276, total assets of $3,105, and a debt–equity ratio of 1.40. If its return on equity is 15 percent, what is its net income?

See Answer

Q: A company has net income of $218,000, a

A company has net income of $218,000, a profit margin of 8.70 percent, and an accounts receivable balance of $132,850. Assuming 70 percent of sales are on credit, what is the company’s days’ sales in...

See Answer

Q: The Ashwood Company has a long-term debt ratio of .

The Ashwood Company has a long-term debt ratio of .45 and a current ratio of 1.25. Current liabilities are $875, sales are $5,780, profit margin is 9.5 percent, and ROE is 18.5 percent. What is the am...

See Answer

Q: You’ve probably noticed coverage in the financial press of an initial public

You’ve probably noticed coverage in the financial press of an initial public offering (IPO) of a company’s securities. Is an IPO a primary market transaction or a secondary market transaction?

See Answer

Q: In response to complaints about high prices, a grocery chain runs

In response to complaints about high prices, a grocery chain runs the following advertising campaign: “If you pay your child $3 to go buy $50 worth of groceries, then your child makes twice as much on...

See Answer

Q: Firm A and firm B have debt–total asset ratios of

Firm A and firm B have debt–total asset ratios of 35% and 30% and returns on total assets of 12% and 11%, respectively. Which firm has a greater return on equity?

See Answer

Q: Sherwood Inc.’s net income for the most recent year was

Sherwood Inc.’s net income for the most recent year was $13,168. The tax rate was 34 percent. The firm paid $3,605 in total interest expense and deducted $2,382 in depreciation expense. What was the c...

See Answer

Q: Holliman Corp. has current liabilities of $365,000,

Holliman Corp. has current liabilities of $365,000, a quick ratio of .85, inventory turnover of 5.8, and a current ratio of 1.4. What is the cost of goods sold for the company?

See Answer

Q: Prince Albert Canning PLC had a net loss of £13,

Prince Albert Canning PLC had a net loss of £13,482 on sales of £138,793 (both in thousands of pounds). What was the company’s profit margin? Does the fact that these figures are quoted in a foreign c...

See Answer

Q: Find the following financial ratios for Smolira Golf Corp. (use

Find the following financial ratios for Smolira Golf Corp. (use year-end figures rather than average values where appropriate): Short-term solvency ratios: a. Current ratio...

See Answer

Q: Construct the Du Pont identity for Smolira Golf Corp. /

Construct the Du Pont identity for Smolira Golf Corp.

See Answer

Q: Prepare the 2009 statement of cash flows for Smolira Golf Corp.

Prepare the 2009 statement of cash flows for Smolira Golf Corp.

See Answer

Q: Smolira Golf Corp. has 25,000 shares of common stock

Smolira Golf Corp. has 25,000 shares of common stock outstanding, and the market price for a share of stock at the end of 2009 was $43. What is the price–earnings ratio? What are the...

See Answer

Q: What is Tobin’s Q for Smolira Golf? What assumptions are you

What is Tobin’s Q for Smolira Golf? What assumptions are you making about the book value of debt and the market value of debt? What about the book value of assets and the market valu...

See Answer

Q: What does it mean when we say the New York Stock Exchange

What does it mean when we say the New York Stock Exchange is an auction market? How are auction markets different from dealer markets? What kind of market is NASDAQ?

See Answer

Q: SDJ, Inc., has net working capital of $1,

SDJ, Inc., has net working capital of $1,370, current liabilities of $3,720, and inventory of $1,950. What is the current ratio? What is the quick ratio?

See Answer

Q: Wakers, Inc., has sales of $29 million, total

Wakers, Inc., has sales of $29 million, total assets of $17.5 million, and total debt of $6.3 million. If the profit margin is 8 percent, what is net income? What is ROA? What is ROE?

See Answer

Q: The Blue Moon Corporation has ending inventory of $407,534

The Blue Moon Corporation has ending inventory of $407,534, and cost of goods sold for the year just ended was $4,105,612. What is the inventory turnover? The days’ sales in inventory? How long on ave...

See Answer

Q: Crystal Lake, Inc., has a total debt ratio of .

Crystal Lake, Inc., has a total debt ratio of .63. What is its debt–equity ratio? What is its equity multiplier?

See Answer

Q: Bach Corp. had additions to retained earnings for the year just

Bach Corp. had additions to retained earnings for the year just ended of $430,000. The firm paid out $175,000 in cash dividends, and it has ending total equity of $5.3 million. If the company currentl...

See Answer

Q: Based only on the following information for Bennington Corp., did cash

Based only on the following information for Bennington Corp., did cash go up or down? By how much? Classify each event as a source or use of cash. Decrease in inventory ………………………….$375 Decrease in ac...

See Answer

Q: Tortoise, Inc., had a cost of goods sold of $

Tortoise, Inc., had a cost of goods sold of $28,834. At the end of the year, the accounts payable balance was $6,105. How long on average did it take the company to pay off its suppliers during the ye...

See Answer

Q: Organic Chicken Company has a debt–equity ratio of .65

Organic Chicken Company has a debt–equity ratio of .65. Return on assets is 8.5 percent, and total equity is $540,000. What is the equity multiplier? Return on equity? Net income? Ju...

See Answer

Q: Braam Fire Prevention Corp. has a profit margin of 6.

Braam Fire Prevention Corp. has a profit margin of 6.80 percent, total asset turnover of 1.95, and ROE of 18.27 percent. What is this firm’s debt–equity ratio?

See Answer

Q: For the year just ended, Ypsilanti Yak Yogurt shows an increase

For the year just ended, Ypsilanti Yak Yogurt shows an increase in its net fixed assets account of $835. The company took $148 in depreciation expense for the year. How much did the company spend on n...

See Answer

Q: Suppose you were the financial manager of a not-for-

Suppose you were the financial manager of a not-for-profit business (a not-for-profit hospital, perhaps). What kinds of goals do you think would be appropriate?

See Answer

Q: Ortiz Lumber Yard has a current accounts receivable balance of $431

Ortiz Lumber Yard has a current accounts receivable balance of $431,287. Credit sales for the year just ended were $3,943,709. What is the receivables turnover? The days’ sales in receivables? How lon...

See Answer

Q: If Roten Rooters, Inc., has an equity multiplier of 2

If Roten Rooters, Inc., has an equity multiplier of 2.80, total asset turnover of 1.15, and a profit margin of 5.5 percent, what is its ROE?

See Answer

Q: What are some of the actions that a small company like The

What are some of the actions that a small company like The Grandmother Calendar Company can take if it finds itself in a situation in which growth in sales outstrips production capacity and available...

See Answer

Q: The firm actually priced its product to be about 20 percent less

The firm actually priced its product to be about 20 percent less than that of competitors, even though the Grandmother calendar was more detailed. In retrospect, was this a wise choice?

See Answer

Q: Testaburger, Inc., uses no external financing and maintains a positive

Testaburger, Inc., uses no external financing and maintains a positive retention ratio. When sales grow by 15 percent, the firm has a negative projected EFN. What does this tell you about the firm’s i...

See Answer

Q: After Chris completed the ratio analysis for S&S Air (

After Chris completed the ratio analysis for S&S Air (see Chapter 3), Mark and Todd approached him about planning for next year’s sales. The company had historically used little...

See Answer

Q: For the company in the previous problem, what is the sustainable

For the company in the previous problem, what is the sustainable growth rate?

See Answer

Q: The most recent financial statements for Throwing Copper Co. are shown

The most recent financial statements for Throwing Copper Co. are shown here: Assets and costs are proportional to sales. The company maintains a constant 30 percent dividend payout ratio and a const...

See Answer

Q: Consider the following income statement for the Heir Jordan Corporation:

Consider the following income statement for the Heir Jordan Corporation: A 20 percent growth rate in sales is projected. Prepare a pro forma income statement assuming costs vary with sales and the d...

See Answer

Q: The balance sheet for the Heir Jordan Corporation follows. Based on

The balance sheet for the Heir Jordan Corporation follows. Based on this information and the income statement in the previous problem, supply the missing information using the percentage of sales appr...

See Answer

Q: On the most basic level, if a firm’s WACC is 12

On the most basic level, if a firm’s WACC is 12 percent, what does this mean?

See Answer

Q: In calculating the WACC, if you had to use book values

In calculating the WACC, if you had to use book values for either debt or equity, which would you choose? Why?

See Answer

Q: Stephenson Real Estate Company was founded 25 years ago by the current

Stephenson Real Estate Company was founded 25 years ago by the current CEO, Robert Stephenson. The company purchases real estate, including land and buildings, and rents the property to tenants. The c...

See Answer

Q: Why do we use an after tax figure for cost of debt

Why do we use an after tax figure for cost of debt but not for cost of equity?

See Answer

Q: Why do noninvestment-grade bonds have much higher direct costs than

Why do noninvestment-grade bonds have much higher direct costs than investment-grade issues?

See Answer

Q: What are the advantages of using the DCF model for determining the

What are the advantages of using the DCF model for determining the cost of equity capital? What are the disadvantages? What specific piece of information do you need to fi nd the cost of equity using...

See Answer

Q: Why is underpricing not a great concern with bond offerings?

Why is underpricing not a great concern with bond offerings?

See Answer

Q: Eye tech Pharmaceuticals, Inc., a company that develops treatments for

Eye tech Pharmaceuticals, Inc., a company that develops treatments for eye problems, went public in January 2004. Assisted by the investment bank Merrill Lynch, Eye tech sold 6.5 million shares at $21...

See Answer

Q: Eye tech Pharmaceuticals, Inc., a company that develops treatments for

Eye tech Pharmaceuticals, Inc., a company that develops treatments for eye problems, went public in January 2004. Assisted by the investment bank Merrill Lynch, Eye tech sold 6.5 million shares at $21...

See Answer

Q: Eye tech Pharmaceuticals, Inc., a company that develops treatments for

Eye tech Pharmaceuticals, Inc., a company that develops treatments for eye problems, went public in January 2004. Assisted by the investment bank Merrill Lynch, Eye tech sold 6.5 million shares at $21...

See Answer

Q: Indicate whether the following events might cause stocks in general to change

Indicate whether the following events might cause stocks in general to change price, and whether they might cause Big Widget Corp.’s stock to change price: a. The government announces that inflation...

See Answer

Q: Ren-Stimpy International is planning to raise fresh equity capital by

Ren-Stimpy International is planning to raise fresh equity capital by selling a large new issue of common stock. Ren-Stimpy is currently a publicly traded corporation, and it is trying to choose betwe...

See Answer

Q: In 1980, a certain assistant professor of finance bought 12 initial

In 1980, a certain assistant professor of finance bought 12 initial public offerings of common stock. He held each of these for approximately one month and then sold. The investment rule he followed w...

See Answer

Q: For each of the following scenarios, discuss whether profit opportunities exist

For each of the following scenarios, discuss whether profit opportunities exist from trading in the stock of the firm under the conditions that (1) the market is not weak form efficient, (2) the marke...

See Answer

Q: How is it possible that dividends are so important, but at

How is it possible that dividends are so important, but at the same time, dividend policy is irrelevant?

See Answer

Q: Historically, the U.S. tax code treated dividend payments

Historically, the U.S. tax code treated dividend payments made to shareholders as ordinary income. Thus, dividends were taxed at the investor’s marginal tax rate, which was as high as 38.6 percent in...

See Answer

Q: What is the impact of a stock repurchase on a company’s debt

What is the impact of a stock repurchase on a company’s debt ratio? Does this suggest another use for excess cash?

See Answer

Q: On Tuesday, December 8, Hometown Power Co.’s board

On Tuesday, December 8, Hometown Power Co.’s board of directors declares a dividend of 75 cents per share payable on Wednesday, January 17, to shareholders of record as of Wednesday, January 3. When i...

See Answer

Q: Why are the costs of selling equity so much larger than the

Why are the costs of selling equity so much larger than the costs of selling debt?

See Answer

Q: Some corporations, like one British company that offers its large shareholders

Some corporations, like one British company that offers its large shareholders free crematorium use, pay dividends in kind (that is, offer their services to shareholders at below-market cost). Should...

See Answer

Q: What are the implications of the efficient markets hypothesis for investors who

What are the implications of the efficient markets hypothesis for investors who buy and sell stocks in an attempt to “beat the market”?

See Answer

Q: If increases in dividends tend to be followed by (immediate)

If increases in dividends tend to be followed by (immediate) increases in share prices, how can it be said that dividend policy is irrelevant?

See Answer

Q: Refer to the observed capital structures given in Table 16.7

Refer to the observed capital structures given in Table 16.7 of the text. What do you notice about the types of industries with respect to their average debt–equity ratios? Are certa...

See Answer

Q: Historically, the U.S. tax code treated dividend payments

Historically, the U.S. tax code treated dividend payments made to shareholders as ordinary income. Thus, dividends were taxed at the investor’s marginal tax rate, which was as high as 38.6 percent in...

See Answer

Q: Electronic Timing, Inc. (ETI), is a small company

Electronic Timing, Inc. (ETI), is a small company founded 15 years ago by electronics engineers Tom Miller and Jessica Kerr. ETI manufactures integrated circuits to capitalize on the complex mixed-sig...

See Answer

Q: Loftis Manufacturing, Inc., has recently installed a just-in

Loftis Manufacturing, Inc., has recently installed a just-in-time (JIT) inventory system. Describe the effect this is likely to have on the company’s carrying costs, shortage costs, and operating cycl...

See Answer

Q: It is sometimes argued that excess cash held by a firm can

It is sometimes argued that excess cash held by a firm can aggravate agency problems (discussed in Chapter 1) and, more generally, reduce incentives for shareholder wealth maximization. How would you...

See Answer

Q: In the chapter opening, we discussed the enormous cash positions of

In the chapter opening, we discussed the enormous cash positions of several companies. Why would firms such as these hold such large quantities of cash?

See Answer

Q: Suppose a firm has a book balance of $2 million.

Suppose a firm has a book balance of $2 million. At the automatic teller machine (ATM), the cash manager finds out that the bank balance is $2.5 million. What is the situation here? If this is an ongo...

See Answer

Q: Which of the following most accurately describes a Yankee bond?

Which of the following most accurately describes a Yankee bond? a. A bond issued by General Motors in Japan with the interest payable in U.S. dollars. b. A bond issued by General Motors in Japan with...

See Answer

Q: Given that First Solar was up by over 796 percent for 2007

Given that First Solar was up by over 796 percent for 2007, why didn’t all investors hold this stock?

See Answer

Q: Explain what is meant by business risk and financial risk. Suppose

Explain what is meant by business risk and financial risk. Suppose Firm A has greater business risk than Firm B. Is it true that Firm A also has a higher cost of equity capital? Explain.

See Answer

Q: How are soft rationing and hard rationing different? What are the

How are soft rationing and hard rationing different? What are the implications if a firm is experiencing soft rationing? Hard rationing?

See Answer

Q: Several celebrated investors and stock pickers frequently mentioned in the financial press

Several celebrated investors and stock pickers frequently mentioned in the financial press have recorded huge returns on their investments over the past two decades. Is the success of these particular...

See Answer

Q: As mentioned in the text, Continental Airlines fi led for bankruptcy

As mentioned in the text, Continental Airlines fi led for bankruptcy, at least in part, as a means of reducing labor costs. Whether this move was ethical, or proper, was hotly debated. Give both sides...

See Answer

Q: The DRK Corporation has recently developed a dividend reinvestment plan, or

The DRK Corporation has recently developed a dividend reinvestment plan, or DRIP. The plan allows investors to reinvest cash dividends automatically in DRK in exchange for new shares of stock. Over ti...

See Answer

Q: You have recently been hired by Piepkorn Manufacturing to work in the

You have recently been hired by Piepkorn Manufacturing to work in the newly established treasury department. Piepkorn Manufacturing is a small company that produces cardboard boxes in a variety of siz...

See Answer

Q: For initial public offerings of common stock, 2007 was a relatively

For initial public offerings of common stock, 2007 was a relatively slow year, with about $35.2 billion raised by the process. Relatively few of the 160 firms involved paid cash dividends. Why do you...

See Answer

Q: In the aggregate, debt offerings are much more common than equity

In the aggregate, debt offerings are much more common than equity offerings and typically much larger as well. Why?

See Answer

Q: Airline announced that it would stretch out its bill payments to 45

Airline announced that it would stretch out its bill payments to 45 days from 30 days. The reason given was that the company wanted to “control costs and optimize cash flow.” The increased payables pe...

See Answer

Q: Consider the following information about two alternative credit strategies:

Consider the following information about two alternative credit strategies: The higher cost per unit reflects the expense associated with credit orders, and the higher price per unit reflects the ex...

See Answer

Q: Going all the way back to Chapter 1, recall that we

Going all the way back to Chapter 1, recall that we saw that partnerships and proprietorships can face difficulties when it comes to raising capital. In the context of this chapter, the implication is...

See Answer

Q: Given that Nova star Financial was down by almost 97 percent for

Given that Nova star Financial was down by almost 97 percent for 2007, why did some investors hold the stock? Why didn’t they sell out before the price declined so sharply?

See Answer

Q: What are the different inventory types? How do the types differ

What are the different inventory types? How do the types differ? Why are some types said to have dependent demand whereas other types are said to have independent demand?

See Answer

Q: Suppose the exchange rate for the Swiss franc is quoted as SF

Suppose the exchange rate for the Swiss franc is quoted as SF 1.50 in the spot market and SF 1.53 in the 90-day forward market. a. Is the dollar selling at a premium or a discount relative to the fra...

See Answer

Q: Are the following statements true or false? Explain why.

Are the following statements true or false? Explain why. a. If the general price index in Great Britain rises faster than that in the United States, we would expect the pound to appreciate relative t...

See Answer

Q: Shelley Couts, the owner of Conch Republic Electronics, had received

Shelley Couts, the owner of Conch Republic Electronics, had received the capital budgeting analysis from Jay McCanless for the new PDA the company is considering. Shelley was pleased with the results,...

See Answer

Q: Webb Corporation was founded 20 years ago by its president, Bryan

Webb Corporation was founded 20 years ago by its president, Bryan Webb. The company originally began as a mail-order company, but has grown rapidly in recent years, in large part due to its Web site....

See Answer

Q: Suppose the government announces that, based on a just-completed

Suppose the government announces that, based on a just-completed survey, the growth rate in the economy is likely to be 2 percent in the coming year, as compared to 5 percent for the past year. Will s...

See Answer

Q: Sterling Wyatt, the president of Howlett Industries, has been exploring

Sterling Wyatt, the president of Howlett Industries, has been exploring ways of improving the company’s fi nancial performance. Howlett manufactures and sells office equipment to ret...

See Answer

Q: You recently graduated from college, and your job search led you

You recently graduated from college, and your job search led you to S&S Air. Because you felt the company’s business was taking off, you accepted a job offer. The first day on th...

See Answer

Q: Mark Sexton and Todd Story, the owners of S&S

Mark Sexton and Todd Story, the owners of S&S Air, have been in discussions with a light aircraft dealer in Monaco about selling the company’s planes in Europe. Jarek Jachowicz, the dealer, wants to a...

See Answer

Q: We have seen that over long periods, stock investments have tended

We have seen that over long periods, stock investments have tended to substantially outperform bond investments. However, it is common to observe investors with long horizons holding entirely bonds. A...

See Answer

Q: Mark Sexton and Todd Story have been discussing the future of S

Mark Sexton and Todd Story have been discussing the future of S&S Air. The company has been experiencing fast growth, and the two see only clear skies in the company’s future. However, the fast growth...

See Answer

Q: Joey Moss, a recent finance graduate, has just begun his

Joey Moss, a recent finance graduate, has just begun his job with the investment firm of Covili and Wyatt. Paul Covili, one of the firm’s founders, has been talking to Joey about the firm’s investment...

See Answer

Q: What is forecasting risk? In general, would the degree of

What is forecasting risk? In general, would the degree of forecasting risk be greater for a new product or a cost-cutting proposal? Why?

See Answer

Q: What is the essential difference between sensitivity analysis and scenario analysis?

What is the essential difference between sensitivity analysis and scenario analysis?

See Answer

Q: As indicated by a number of examples in this chapter, earnings

As indicated by a number of examples in this chapter, earnings announcements by companies are closely followed by, and frequently result in, share price revisions. Two issues should come to mind. Firs...

See Answer

Q: A coworker claims that looking at all this marginal this and incremental

A coworker claims that looking at all this marginal this and incremental that is just a bunch of nonsense, saying, “Listen, if our average revenue doesn’t exceed our average cost, then we will have a...

See Answer

Q: At one time at least, many Japanese companies had a “

At one time at least, many Japanese companies had a “no-layoff” policy (for that matter, so did IBM). What are the implications of such a policy for the degree of operating leverage a company faces?

See Answer

Q: Airlines offer an example of an industry in which the degree of

Airlines offer an example of an industry in which the degree of operating leverage is fairly high. Why?

See Answer

Q: As a shareholder of a firm that is contemplating a new project

As a shareholder of a firm that is contemplating a new project, would you be more concerned with the accounting break-even point, the cash break-even point, or the financial break-even point? Why?

See Answer

Q: If a market is semi strong form efficient, is it also

If a market is semi strong form efficient, is it also weak form efficient? Explain.

See Answer

Q: In recent years, it has been common for companies to experience

In recent years, it has been common for companies to experience significant stock price changes in reaction to announcements of massive layoffs. Critics charge that such events encourage companies to...

See Answer

Q: True or false: The most important characteristic in determining the expected

True or false: The most important characteristic in determining the expected return of a well-diversified portfolio is the variance of the individual assets in the portfolio. Explain.

See Answer

Q: If Wilkinson, Inc., has an equity multiplier of 1.

If Wilkinson, Inc., has an equity multiplier of 1.35, total asset turnover of 2.10, and a profit margin of 5.2 percent, what is its ROE?

See Answer

Q: What happens to the future value of an annuity if you increase

What happens to the future value of an annuity if you increase the rate r? What happens to the present value?

See Answer

Q: For 2015, calculate the cash flow from assets, cash flow

For 2015, calculate the cash flow from assets, cash flow to creditors, and cash flow to stockholders.

See Answer

Q: Redo Problem 21 using sales growth rates of 15 and 25 percent

Redo Problem 21 using sales growth rates of 15 and 25 percent in addition to 20 percent. Illustrate graphically the relationship between EFN and the growth rate, and use this graph to determine the re...

See Answer

Q: You are researching Time Manufacturing and have found the following accounting statement

You are researching Time Manufacturing and have found the following accounting statement of cash flows for the most recent year. You also know that the company paid $84 million in current taxes and ha...

See Answer

Q: You’re trying to choose between two different investments, both of which

You’re trying to choose between two different investments, both of which have up-front costs of $75,000. Investment G returns $125,000 in six years. Investment H returns $185,000 in 10 years. Which of...

See Answer

Q: Redo Problem 23 using sales growth rates of 30 and 35 percent

Redo Problem 23 using sales growth rates of 30 and 35 percent in addition to 20 percent. Illustrate graphically the relationship between EFN and the growth rate, and use this graph to determine the re...

See Answer

Q: Shinedown, Inc., wishes to maintain a growth rate of 12 

Shinedown, Inc., wishes to maintain a growth rate of 12 percent per year and a debt–equity ratio of .35. Profit margin is 4.9 percent, and the ratio of total assets to sales is constant at .75. Is thi...

See Answer

Q: Mark Weinstein has been working on an advanced technology in laser eye

Mark Weinstein has been working on an advanced technology in laser eye surgery. His technology will be available in the near term. He anticipates his first annual cash flow from the technology to be $...

See Answer

Q: Refer to the corporate marginal tax rate information in Table 2.

Refer to the corporate marginal tax rate information in Table 2.3(given below): a. Why do you think the marginal tax rate jumps up from 34 percent to 39 percent at a taxable income of $100,001, and t...

See Answer

Q: A prestigious investment bank designed a new security that pays a quarterly

A prestigious investment bank designed a new security that pays a quarterly dividend of $2.75 in perpetuity. The first dividend occurs one quarter from today. What is the price of the security if the...

See Answer

Q: Define the following: S = Previous year’s sales A

Define the following: S = Previous year’s sales A = Total assets E = Total equity g = Projected growth in sales PM = Profit margin b = Retention (plowback) ratio Assuming that all de...

See Answer

Q: So-called “same-store sales” are a very

So-called “same-store sales” are a very important measure for companies as diverse as McDonald’s and Sears. As the name suggests, examining same-store sales means comparing revenues from the same stor...

See Answer

Q: What is the present value of an annuity of $5,

What is the present value of an annuity of $5,500 per year, with the first cash flow received three years from today and the last one received 25 years from today? Use a discount rate of 8 percent.

See Answer

Q: Based on the results in Problem 27, show that the internal

Based on the results in Problem 27, show that the internal and sustainable growth rates can be calculated as shown in Equations 3.24 and 3.25(mentioned below).(Hint: For the internal growth rate, set...

See Answer

Q: What is the value today of a 15-year annuity that

What is the value today of a 15-year annuity that pays $900 a year? The annuity’s first payment occurs six years from today. The annual interest rate is 11 percent for Years 1 through 5, and 13 percen...

See Answer

Q: In the chapter, we discussed one calculation of the sustainable growth

In the chapter, we discussed one calculation of the sustainable growth rate as: In practice, probably the most commonly used calculation of the sustainable growth rate is ROE x b. This equation is id...

See Answer

Q: Audrey Sanborn has just arranged to purchase a $650,000

Audrey Sanborn has just arranged to purchase a $650,000 vacation home in the Bahamas with a 20 percent down payment. The mortgage has a 5.2 percent APR, compounded monthly, and calls for equal monthly...

See Answer

Q: Use the sustainable growth rate equations from the previous problem to answer

Use the sustainable growth rate equations from the previous problem to answer the following questions. I Am Myself, Inc., had total assets of $410,000 and equity of $230,000 at the beginning of the ye...

See Answer

Q: You receive a credit card application from Shady Banks Savings and Loan

You receive a credit card application from Shady Banks Savings and Loan offering an introductory rate of 2.40 percent per year, compounded monthly for the first six months, increasing thereafter to 18...

See Answer

Q: Young Pharmaceuticals is considering a drug project that costs $3.

Young Pharmaceuticals is considering a drug project that costs $3.8 million today and is expected to generate end-of-year annual cash flows of $267,000, forever. At what discount rate would Young be i...

See Answer

Q: Southern California Publishing Company is trying to decide whether to revise its

Southern California Publishing Company is trying to decide whether to revise its popular textbook, Financial Psychoanalysis Made Simple. The company has estimated that the revision will cost $135,000....

See Answer

Q: Your job pays you only once a year for all the work

Your job pays you only once a year for all the work you did over the previous 12 months. Today, December 31, you just received your salary of $72,500, and you plan to spend all of it. However, you wan...

See Answer

Q: Why might the revenue and cost figures shown on a standard income

Why might the revenue and cost figures shown on a standard income statement not represent the actual cash inflows and outflows that occurred during a period?

See Answer

Q: What is the relationship between the value of an annuity and the

What is the relationship between the value of an annuity and the level of interest rates? Suppose you just bought a 15-year annuity of $4,300 per year at the current interest rate of 10 percent per ye...

See Answer

Q: You’re prepared to make monthly payments of $240, beginning at

You’re prepared to make monthly payments of $240, beginning at the end of this month, into an account that pays 10 percent interest compounded monthly. How many payments will you have made when your a...

See Answer

Q: You want to borrow $96,000 from your local bank

You want to borrow $96,000 from your local bank to buy a new sailboat. You can afford to make monthly payments of $1,950, but no more. Assuming monthly compounding, what is the highest APR you can aff...

See Answer

Q: You need a 30-year, fixed-rate mortgage to

You need a 30-year, fixed-rate mortgage to buy a new home for $250,000. Your mortgage bank will lend you the money at an APR of 4.5 percent for this 360-month loan. However, you can only afford monthl...

See Answer

Q: The present value of the following cash flow stream is $7

The present value of the following cash flow stream is $7,300 when discounted at 7.1 percent annually. What is the value of the missing cash flow?

See Answer

Q: You just won the TVM Lottery. You will receive $1

You just won the TVM Lottery. You will receive $1 million today plus another 10 annual payments that increase by $275,000 per year. Thus, in one year you receive $1.275 million. In two years, you get...

See Answer

Q: You have just purchased a new warehouse. To finance the purchase

You have just purchased a new warehouse. To finance the purchase, you’ve arranged for a 30-year mortgage for 80 percent of the $5,200,000 purchase price. The monthly payment on this loan will be $27,5...

See Answer

Q: Consider a firm with a contract to sell an asset for $

Consider a firm with a contract to sell an asset for $135,000 three years from now. The asset costs $89,000 to produce today. Given a relevant discount rate on this asset of 13 percent per year, will...

See Answer

Q: What is the present value of $7,500 per year

What is the present value of $7,500 per year, at a discount rate of 7.1 percent, if the first payment is received 6 years from now and the last payment is received 25 years from now?

See Answer

Q: A 15-year annuity pays $1,750 per month

A 15-year annuity pays $1,750 per month, and payments are made at the end of each month. If the interest rate is 12 percent compounded monthly for the first seven years, and 6 percent compounded month...

See Answer

Q: For each of the following, compute the present value:

For each of the following, compute the present value:

See Answer

Q: You have your choice of two investment accounts. Investment A is

You have your choice of two investment accounts. Investment A is a 15-year annuity that features end-of-month $1,300 payments and has an interest rate of 7.2 percent compounded monthly. Investment B i...

See Answer

Q: Given an interest rate of 5.6 percent per year,

Given an interest rate of 5.6 percent per year, what is the value at Date t = 7 of a perpetual stream of $2,150 annual payments that begins at Date t = 15?

See Answer

Q: A local finance company quotes an interest rate of 17 percent on

A local finance company quotes an interest rate of 17 percent on one-year loans. So, if you borrow $23,000, the interest for the year will be $3,910.Because you must repay a total of $26,910 in one ye...

See Answer

Q: A 5-year annuity of ten $6,175 semiannual

A 5-year annuity of ten $6,175 semiannual payments will begin 9 years from now, with the first payment coming 9.5 years from now. If the discount rate is 11 percent compounded monthly, what is the val...

See Answer

Q: Suppose you are going to receive $16,250 per year

Suppose you are going to receive $16,250 per year for five years. The appropriate interest rate is 7.5 percent. a. What is the present value of the payments if they are in the form of an ordinary annu...

See Answer

Q: You want to buy a new sports car from Muscle Motors for

You want to buy a new sports car from Muscle Motors for $64,000. The contract is in the form of a 60-month annuity due at an APR of 6.15 percent. What will your monthly payment be?

See Answer

Q: You want to lease a set of golf clubs from Pings Ltd

You want to lease a set of golf clubs from Pings Ltd. The lease contract is in the form of 24 equal monthly payments at an APR of 11.2 percent, compounded monthly. Because the clubs cost $2,650 retail...

See Answer

Q: You are saving for the college education of your two children.

You are saving for the college education of your two children. They are two years apart in age; one will begin college 15 years from today and the other will begin 17 years from today. You estimate yo...

See Answer

Q: Tom Adams has received a job offer from a large investment bank

Tom Adams has received a job offer from a large investment bank as a clerk to an associate banker. His base salary will be $63,000. He will receive his first annual salary payment one year from the da...

See Answer

Q: You have recently won the super jackpot in the Washington State Lottery

You have recently won the super jackpot in the Washington State Lottery. On reading the fine print, you discover that you have the following two options: a. You will receive 31 annual payments of $250...

See Answer

Q: Y3K, Inc., has sales of $3,100,

Y3K, Inc., has sales of $3,100, total assets of $1,340, and a debt–equity ratio of 1.20. If its return on equity is 15 percent, what is its net income?

See Answer

Q: You have 30 years left until retirement and want to retire with

You have 30 years left until retirement and want to retire with $2.2 million. Your salary is paid annually, and you will receive $83,000 at the end of the current year. Your salary will increase at 3...

See Answer

Q: On September 1, 2013, Susan Chao bought a motorcycle for

On September 1, 2013, Susan Chao bought a motorcycle for $34,000. She paid $2,000 down and financed the balance with a five-year loan at an annual percentage rate of 7.2 percent, compounded monthly. S...

See Answer

Q: Bilbo Baggins wants to save money to meet three objectives. First

Bilbo Baggins wants to save money to meet three objectives. First, he would like to be able to retire 30 years from now with a retirement income of $20,000 per month for 20 years, with the first payme...

See Answer

Q: After deciding to buy a new car, you can either lease

After deciding to buy a new car, you can either lease the car or purchase it with a three-year loan. The car you wish to buy costs $28,000. The dealer has a leasing arrangement where you pay $2,400 to...

See Answer

Q: An All-Pro defensive lineman is in contract negotiations. The

An All-Pro defensive lineman is in contract negotiations. The team has offered the following salary structure: All salaries are to be paid in a lump sum. The player has asked you as his agent to reneg...

See Answer

Q: This question illustrates what is known as discount interest. Imagine you

This question illustrates what is known as discount interest. Imagine you are discussing a loan with a somewhat unscrupulous lender. You want to borrow $20,000 for one year. The interest rate is 15.7...

See Answer

Q: You are serving on a jury. A plaintiff is suing the

You are serving on a jury. A plaintiff is suing the city for injuries sustained after a freak street sweeper accident. In the trial, doctors testified that it will be five years before the plaintiff i...

See Answer

Q: You are looking at a one-year loan of $10

You are looking at a one-year loan of $10,000. The interest rate is quoted as 8 percent plus three points. A point on a loan is simply 1 percent (one percentage point) of the loan amount. Quotes simil...

See Answer

Q: Two banks in the area offer 30-year, $225

Two banks in the area offer 30-year, $225,000 mortgages at 5.6 percent and charge a $2,900 loan application fee. However, the application fee charged by Insecurity Bank and Trust is refundable if the...

See Answer

Q: This problem illustrates a deceptive way of quoting interest rates called add

This problem illustrates a deceptive way of quoting interest rates called add-on interest. Imagine that you see an advertisement for Crazy Judy’s Stereo City that reads something lik...

See Answer

Q: Suppose two athletes sign 10-year contracts for $80 million

Suppose two athletes sign 10-year contracts for $80 million. In one case, we’re told that the $80 million will be paid in 10 equal installments. In the other case, we’re told that the $80 million will...

See Answer

Q: Your Christmas ski vacation was great, but it unfortunately ran a

Your Christmas ski vacation was great, but it unfortunately ran a bit over budget. All is not lost: You just received an offer in the mail to transfer your $10,000 balance from your current credit car...

See Answer

Q: An insurance company is offering a new policy to its customers.

An insurance company is offering a new policy to its customers. Typically the policy is bought by a parent or grandparent for a child at the child’s birth. The details of the policy are as follows: Th...

See Answer

Q: You have just won the lottery. You will receive $4

You have just won the lottery. You will receive $4,500,000 today, and then receive 40 payments of $1,600,000. These payments will start one year from now and will be paid every six months. A represent...

See Answer

Q: A financial planning service offers a college savings program. The plan

A financial planning service offers a college savings program. The plan calls for you to make six annual payments of $11,000 each, with the first payment occurring today, your child’s 12th birthday. B...

See Answer

Q: Looking at the accounting statement of cash flows, what does the

Looking at the accounting statement of cash flows, what does the bottom line number mean? How useful is this number for analyzing a company?

See Answer

Q: Why do you think most long-term financial planning begins with

Why do you think most long-term financial planning begins with sales forecasts? Put differently, why are future sales the key input?

See Answer

Q: In the chapter, we used Rosengarten Corporation to demonstrate how to

In the chapter, we used Rosengarten Corporation to demonstrate how to calculate EFN. The ROE for Rosengarten is about 7.3 percent, and the plowback ratio is about 67 percent. If you calculate the sust...

See Answer

Q: Solve for the unknown interest rate in each of the following:

Solve for the unknown interest rate in each of the following:

See Answer

Q: First City Bank pays 7.5 percent simple interest on its

First City Bank pays 7.5 percent simple interest on its savings account balances, whereas Second City Bank pays 7.5 percent interest compounded annually. If you made a $7,000 deposit in each bank, how...

See Answer

Q: The most recent financial statements for Heine, Inc., are shown

The most recent financial statements for Heine, Inc., are shown here: Assets and costs are proportional to sales. Debt and equity are not. A dividend of $3,500 was paid, and the company wishes to mai...

See Answer

Q: The Stefani Co. had $198,000 in taxable income

The Stefani Co. had $198,000 in taxable income. Using the rates from Table 2.3 in the chapter, calculate the company’s income taxes. What is the average tax rate? What is the marginal tax rate?

See Answer

Q: Should lending laws be changed to require lenders to report EARs instead

Should lending laws be changed to require lenders to report EARs instead of APRs? Why or why not?

See Answer

Q: How do financial cash flows and the accounting statement of cash flows

How do financial cash flows and the accounting statement of cash flows differ? Which is more useful for analyzing a company?

See Answer

Q: Broslofski Co. maintains a positive retention ratio and keeps its debt

Broslofski Co. maintains a positive retention ratio and keeps its debt–equity ratio constant every year. When sales grow by 20 percent, the firm has a negative projected EFN. What does this tell you a...

See Answer

Q: Solve for the unknown number of years in each of the following

Solve for the unknown number of years in each of the following:

See Answer

Q: The most recent financial statements for Wise Co. are shown here

The most recent financial statements for Wise Co. are shown here: Assets and costs are proportional to sales. The company maintains a constant 30 percent dividend payout ratio and a constant debt&aci...

See Answer

Q: On subsidized Stafford loans, a common source of financial aid for

On subsidized Stafford loans, a common source of financial aid for college students, interest does not begin to accrue until repayment begins. Who receives a bigger subsidy, a freshman or a senior? Ex...

See Answer

Q: Under standard accounting rules, it is possible for a company’s liabilities

Under standard accounting rules, it is possible for a company’s liabilities to exceed its assets. When this occurs, the owners’ equity is negative. Can this happen with market values? Why or why not?...

See Answer

Q: At 6.5 percent interest, how long does it take

At 6.5 percent interest, how long does it take to double your money? To quadruple it?

See Answer

Q: Sankey, Inc., has current assets of $4,900

Sankey, Inc., has current assets of $4,900, net fixed assets of $25,000, current liabilities of $4,100, and long-term debt of $10,300. What is the value of the shareholders’ equity account for this fi...

See Answer

Q: If the Hunter Corp. has a ROE of 12 percent and

If the Hunter Corp. has a ROE of 12 percent and a payout ratio of 15 percent, what is its sustainable growth rate?

See Answer

Q: Toyota Motor Credit Corporation (TMCC), a subsidiary of Toyota Motor

Toyota Motor Credit Corporation (TMCC), a subsidiary of Toyota Motor Corporation, offered some securities for sale to the public on March 28, 2008. Under the terms of the deal, TMCC promised to repay...

See Answer

Q: Why is it not necessarily bad for the cash flow from assets

Why is it not necessarily bad for the cash flow from assets to be negative for a particular period?

See Answer

Q: One tool of financial analysis is common-size financial statements.

One tool of financial analysis is common-size financial statements. Why do you think common-size income statements and balance sheets are used? Note that the accounting statement of cash flows is not...

See Answer

Q: Assuming the following ratios are constant, what is the sustainable growth

Assuming the following ratios are constant, what is the sustainable growth rate? Total asset turnover = 2.85 Profit margin = 5.9% Equity multiplier = 1.70 Payout ratio = 60%

See Answer

Q: The following table presents the long-term liabilities and stockholders’ equity

The following table presents the long-term liabilities and stockholders’ equity of Information Control Corp. one year ago: During the past year, the company issued 5 million shares...

See Answer

Q: One of the implicit assumptions we made in calculating the external funds

One of the implicit assumptions we made in calculating the external funds needed was that the company was operating at full capacity. If the company is operating at less than full capacity, how will t...

See Answer

Q: Why is it not necessarily bad for the operating cash flow to

Why is it not necessarily bad for the operating cash flow to be negative for a particular period?

See Answer

Q: Toyota Motor Credit Corporation (TMCC), a subsidiary of Toyota Motor

Toyota Motor Credit Corporation (TMCC), a subsidiary of Toyota Motor Corporation, offered some securities for sale to the public on March 28, 2008. Under the terms of the deal, TMCC promised to repay...

See Answer

Q: Toyota Motor Credit Corporation (TMCC), a subsidiary of Toyota Motor

Toyota Motor Credit Corporation (TMCC), a subsidiary of Toyota Motor Corporation, offered some securities for sale to the public on March 28, 2008. Under the terms of the deal, TMCC promised to repay...

See Answer

Q: As you increase the length of time involved, what happens to

As you increase the length of time involved, what happens to future values? What happens to present values?

See Answer

Q: Although appealing to more refined tastes, art as a collectible has

Although appealing to more refined tastes, art as a collectible has not always performed so profitably. During 2010, Deutscher-Menzies sold Arkie under the Shower, a painting by renowned Australian pa...

See Answer

Q: The most recent financial statements for Williamson Inc., are shown here

The most recent financial statements for Williamson Inc., are shown here (assuming no income taxes): Assets and costs are proportional to sales. Debt and equity are not. No dividends are paid. Next y...

See Answer

Q: The 2014 balance sheet of Jordan’s Golf Shop, Inc., showed

The 2014 balance sheet of Jordan’s Golf Shop, Inc., showed long-term debt of $1.625 million, and the 2015 balance sheet showed long-term debt of $1.73 million. The 2015 income statement showed an inte...

See Answer

Q: Both ROA and ROE measure profitability. Which one is more useful

Both ROA and ROE measure profitability. Which one is more useful for comparing two companies? Why?

See Answer

Q: Could a company’s change in net working capital be negative in a

Could a company’s change in net working capital be negative in a given year? (Hint: Yes.) Explain how this might come about. What about net capital spending?

See Answer

Q: Consider the ratio EBITD/Assets. What does this ratio tell

Consider the ratio EBITD/Assets. What does this ratio tell us? Why might it be more useful than ROA in comparing two companies?

See Answer

Q: Dahlia Colby, CFO of Charming Florist Ltd., has created the

Dahlia Colby, CFO of Charming Florist Ltd., has created the firm’s pro forma balance sheet for the next fiscal year. Sales are projected to grow by 10 percent to $360 million. Current assets, fixed as...

See Answer

Q: The 2014 balance sheet of Jordan’s Golf Shop, Inc., showed

The 2014 balance sheet of Jordan’s Golf Shop, Inc., showed $510,000 in the common stock account and $3.6 million in the additional paid-in surplus account. The 2015 balance sheet showed $545,000 and $...

See Answer

Q: Toyota Motor Credit Corporation (TMCC), a subsidiary of Toyota Motor

Toyota Motor Credit Corporation (TMCC), a subsidiary of Toyota Motor Corporation, offered some securities for sale to the public on March 28, 2008. Under the terms of the deal, TMCC promised to repay...

See Answer

Q: Could a company’s cash flow to stockholders be negative in a given

Could a company’s cash flow to stockholders be negative in a given year? (Hint: Yes.) Explain how this might come about. What about cash flow to creditors?

See Answer

Q: True or false: All assets are liquid at some price.

True or false: All assets are liquid at some price. Explain.

See Answer

Q: Critics have charged that compensation to top managers in the United States

Critics have charged that compensation to top managers in the United States is simply too high and should be cut back. For example, focusing on large corporations, Larry Ellison of Oracle has been one...

See Answer

Q: Toyota Motor Credit Corporation (TMCC), a subsidiary of Toyota Motor

Toyota Motor Credit Corporation (TMCC), a subsidiary of Toyota Motor Corporation, offered some securities for sale to the public on March 28, 2008. Under the terms of the deal, TMCC promised to repay...

See Answer

Q: A ratio that is becoming more widely used is return on investment

A ratio that is becoming more widely used is return on investment. Return on investment is calculated as net income divided by long- term liabilities plus equity. What do you think return on investmen...

See Answer

Q: The Wintergrass Company has an ROE of 11.4 percent and

The Wintergrass Company has an ROE of 11.4 percent and a payout ratio of 25 percent. a. What is the company’s sustainable growth rate? b. Can the company’s actual growth rate be different from its sus...

See Answer

Q: Compute the future value of $1,900 continuously compounded for

Compute the future value of $1,900 continuously compounded for a. 9 years at an APR of 12 percent. b. 5 years at an APR of 8 percent. c. 17 years at an APR of 5 percent. d. 10 years at an APR of 9 per...

See Answer

Q: Given the information for Jordan’s Golf Shop, Inc., in the

Given the information for Jordan’s Golf Shop, Inc., in the previous two problems, suppose you also know that the firm’s net capital spending for 2015 was $975,000 and that the firm reduced its net wor...

See Answer

Q: Referring back to the Ford example at the beginning of the chapter

Referring back to the Ford example at the beginning of the chapter, note that we suggested that Ford’s stockholders probably didn’t suffer as a result of the reported loss. What do you think was the b...

See Answer

Q: Why is the goal of financial management to maximize the current value

Why is the goal of financial management to maximize the current value of the company’s stock? In other words, why isn’t the goal to maximize the future value?

See Answer

Q: A and Firm B have debt–total asset ratios of 25

A and Firm B have debt–total asset ratios of 25 percent and 40 percent and returns on total assets of 8 percent and 7 percent, respectively. Which firm has a greater return on equity?

See Answer

Q: Wilkinson Co. has identified an investment project with the following cash

Wilkinson Co. has identified an investment project with the following cash flows. If the discount rate is 10 percent, what is the present value of these cash flows? What is the present value at 18 per...

See Answer

Q: A financial ratio by itself tells us little about a company because

A financial ratio by itself tells us little about a company because financial ratios vary a great deal across industries. There are two basic methods for analyzing financial ratios for a company: Time...

See Answer

Q: A small business called The Grandmother Calendar Company began selling personalized photo

A small business called The Grandmother Calendar Company began selling personalized photo calendar kits. The kits were a hit, and sales soon sharply exceeded forecasts. The rush of orders created a hu...

See Answer

Q: Ritter Corporation’s accountants prepared the following financial statements for year-end

Ritter Corporation’s accountants prepared the following financial statements for year-end 2015: a. Explain the change in cash during 2015. b. Determine the change in net working capi...

See Answer

Q: Prince Albert Canning PLC had a net loss of £26,

Prince Albert Canning PLC had a net loss of £26,832 on sales of £294,813. What was the company’s profit margin? Does the fact that these figures are quoted in a foreign currency make any difference? W...

See Answer

Q: Investment X offers to pay you $3,900 per year

Investment X offers to pay you $3,900 per year for nine years, whereas Investment Y offers to pay you $6,100 per year for five years. Which of these cash flow streams has the higher present value if t...

See Answer

Q: The Stancil Corporation provided the following current information: /

The Stancil Corporation provided the following current information: Determine the cash flows from the firm and the cash flows to investors of the firm.

See Answer

Q: A small business called The Grandmother Calendar Company began selling personalized photo

A small business called The Grandmother Calendar Company began selling personalized photo calendar kits. The kits were a hit, and sales soon sharply exceeded forecasts. The rush of orders created a hu...

See Answer

Q: The Optical Scam Company has forecast a sales growth rate of 15

The Optical Scam Company has forecast a sales growth rate of 15 percent for next year. The current financial statements are shown here: a. Using the equation from the chapter, calculate the external...

See Answer

Q: An investment offers $5,650 per year for 15 years

An investment offers $5,650 per year for 15 years, with the first payment occurring one year from now. If the required return is 8 percent, what is the value of the investment? What would the value be...

See Answer

Q: A small business called The Grandmother Calendar Company began selling personalized photo

A small business called The Grandmother Calendar Company began selling personalized photo calendar kits. The kits were a hit, and sales soon sharply exceeded forecasts. The rush of orders created a hu...

See Answer

Q: During the year, the Senbet Discount Tire Company had gross sales

During the year, the Senbet Discount Tire Company had gross sales of $925,000. The firm’s cost of goods sold and selling expenses were $490,000 and $220,000, respectively. Senbet also had notes payabl...

See Answer

Q: Synovec Company has a debt–equity ratio of .70.

Synovec Company has a debt–equity ratio of .70. Return on assets is 8.4 percent, and total equity is $840,000. What is the equity multiplier? Return on equity? Net income?

See Answer

Q: A company has net income of $314,000 a profit

A company has net income of $314,000 a profit margin of 8.9 percent, and an accounts receivable balance of $152,800. Assuming 80 percent of sales are on credit, what is the company’s days’ sales in re...

See Answer

Q: The Perpetual Life Insurance Co. is trying to sell you an

The Perpetual Life Insurance Co. is trying to sell you an investment policy that will pay you and your heirs $12,000 per year forever. If the required return on this investment is 4.7 percent, how muc...

See Answer

Q: A small business called The Grandmother Calendar Company began selling personalized photo

A small business called The Grandmother Calendar Company began selling personalized photo calendar kits. The kits were a hit, and sales soon sharply exceeded forecasts. The rush of orders created a hu...

See Answer

Q: Schwert Corp. shows the following information on its 2015 income statement

Schwert Corp. shows the following information on its 2015 income statement: sales = $215,000; costs = $117,000; other expenses = $6,700; depreciation expense = $18,400; interest expense = $10,000; tax...

See Answer

Q: The Whisenhunt Company has a ratio of long-term debt to

The Whisenhunt Company has a ratio of long-term debt to long-term debt and equity of .29 and a current ratio of 1.20. Current liabilities are $1,280, sales are $6,140, profit margin is 8.9 percent, an...

See Answer

Q: Find the EAR in each of the following cases: /

Find the EAR in each of the following cases:

See Answer

Q: A small business called The Grandmother Calendar Company began selling personalized photo

A small business called The Grandmother Calendar Company began selling personalized photo calendar kits. The kits were a hit, and sales soon sharply exceeded forecasts. The rush of orders created a hu...

See Answer

Q: Given the following information for O’Hara Marine Co., calculate the depreciation

Given the following information for O’Hara Marine Co., calculate the depreciation expense: sales 5 $44,000; costs 5 $27,500; addition to retained earnings 5 $5,200; dividends paid 5 $1,670; interest e...

See Answer

Q: Panda Inc.’s net income for the most recent year was

Panda Inc.’s net income for the most recent year was $9,620. The tax rate was 34 percent. The firm paid $2,380 in total interest expense and deducted $3,170 in depreciation expense. What was the compa...

See Answer

Q: Find the APR, in each of the following cases:

Find the APR, in each of the following cases:

See Answer

Q: Compute the future value of $1,000 compounded annually for

Compute the future value of $1,000 compounded annually for a. 10 years at 6 percent. b. 10 years at 12 percent. c. 20 years at 6 percent. d. Why is the interest earned in part (c) not twice the amount...

See Answer

Q: Josipovich, Inc., is obligated to pay its creditors $11

Josipovich, Inc., is obligated to pay its creditors $11,300 very soon. a. What is the market value of the shareholders’ equity if assets have a market value of $12,400? b. What if assets equal $9,600?...

See Answer

Q: The DuPont identity presented in the chapter is commonly referred to as

The DuPont identity presented in the chapter is commonly referred to as the three-factor DuPont identity. Another common way that the DuPont identity is expressed is the five-factor model, which is: D...

See Answer

Q: First National Bank charges 10.3 percent compounded monthly on its

First National Bank charges 10.3 percent compounded monthly on its business loans. First United Bank charges 10.5 percent compounded semiannually. As a potential borrower, to which bank would you go f...

See Answer

Q: Corporation Growth has $82,500 in taxable income, and

Corporation Growth has $82,500 in taxable income, and Corporation Income has $8,250,000 in taxable income. a. What is the tax bill for each firm? b. Suppose both firms have identified a new project t...

See Answer

Q: In addition to common- size financial statements, common–base

In addition to common- size financial statements, common–base year financial statements are often used. Common–base year financial statements are constructed by div...

See Answer

Q: Well-known financial writer Andrew Tobias argues that he can earn

Well-known financial writer Andrew Tobias argues that he can earn 177 percent per year buying wine by the case. Specifically, he assumes that he will consume one $10 bottle of fine Bordeaux per week f...

See Answer

Q: During 2015, Rainbow Umbrella Corp. had sales of $590

During 2015, Rainbow Umbrella Corp. had sales of $590,000. Cost of goods sold, administrative and selling expenses, and depreciation expenses were $455,000, $85,000, and $125,000, respectively. In add...

See Answer

Q: The discussion of EFN in the chapter implicitly assumed that the company

The discussion of EFN in the chapter implicitly assumed that the company was operating at full capacity. Often, this is not the case. For example, assume that Rosengarten was operating at 90 percent c...

See Answer

Q: One of your customers is delinquent on his accounts payable balance.

One of your customers is delinquent on his accounts payable balance. You’ve mutually agreed to a repayment schedule of $500 per month. You will charge 1.1 percent per month interest on the overdue bal...

See Answer

Q: In Problem 18, suppose Rainbow Umbrella Corp. paid out $

In Problem 18, suppose Rainbow Umbrella Corp. paid out $34,000 in cash dividends. Is this possible? If spending on net fixed assets and net working capital was zero, and if no new stock was issued dur...

See Answer

Q: Shelton, Inc., has sales of $435,000,

Shelton, Inc., has sales of $435,000, costs of $216,000, depreciation expense of $40,000, interest expense of $21,000, and a tax rate of 35 percent. What is the net income for the firm? Suppose the co...

See Answer

Q: The discussion of EFN in the chapter implicitly assumed that the company

The discussion of EFN in the chapter implicitly assumed that the company was operating at full capacity. Often, this is not the case. For example, assume that Rosengarten was operating at 90 percent c...

See Answer

Q: Cusic Industries had the following operating results for 2015: sales 5

Cusic Industries had the following operating results for 2015: sales 5 $20,300; cost of goods sold 5 $14,500; depreciation expense 5 $2,900; interest expense 5 $690; dividends paid 5 $660. At the begi...

See Answer

Q: The most recent financial statements for Moose Tours, Inc., appear

The most recent financial statements for Moose Tours, Inc., appear below. Sales for 2016 are projected to grow by 20 percent. Interest expense will remain constant; the tax rate and the dividend payou...

See Answer

Q: What is the future value in six years of $1,

What is the future value in six years of $1,000 invested in an account with an APR of 7.5 percent, a. Compounded annually? b. Compounded semiannually? c. Compounded monthly? d. Compounded continuously...

See Answer

Q: Consider the following abbreviated financial statements for Weston Enterprises: /

Consider the following abbreviated financial statements for Weston Enterprises: a. What is owners’ equity for 2014 and 2015? b. What is the change in net working capital for 2015? c...

See Answer

Q: The discussion of EFN in the chapter implicitly assumed that the company

The discussion of EFN in the chapter implicitly assumed that the company was operating at full capacity. Often, this is not the case. For example, assume that Rosengarten was operating at 90 percent c...

See Answer

Q: First Simple Bank pays 4.1 percent simple interest on its

First Simple Bank pays 4.1 percent simple interest on its investment accounts. If First Complex Bank pays interest on its accounts compounded annually, what rate should the bank set if it wants to mat...

See Answer

Q: Draw up an income statement and balance sheet for this company for

Draw up an income statement and balance sheet for this company for 2014 and 2015.

See Answer

Q: In Problem 21, suppose the firm wishes to keep its debt

In Problem 21, suppose the firm wishes to keep its debt–equity ratio constant. What is EFN now?

See Answer

Q: You are planning to save for retirement over the next 30 years

You are planning to save for retirement over the next 30 years. To do this, you will invest $750 per month in a stock account and $250 per month in a bond account. The return of the stock account is e...

See Answer

Q: Barrett, Inc., has sales of $19,800,

Barrett, Inc., has sales of $19,800, costs of $10,900, depreciation expense of $2,100, and interest expense of $1,250. If the tax rate is 40 percent, what is the operating cash flow, or OCF?

See Answer

Q: Gordon Driving School’s 2014 balance sheet showed net fixed assets of $

Gordon Driving School’s 2014 balance sheet showed net fixed assets of $1.32 million, and the 2015 balance sheet showed net fixed assets of $1.51 million. The company’s 2015 income statement showed a d...

See Answer

Q: Imprudential, Inc., has an unfunded pension liability of $550

Imprudential, Inc., has an unfunded pension liability of $550 million that must be paid in 20 years. To assess the value of the firm’s stock, financial analysts want to discount this liability back to...

See Answer

Q: On the balance sheet, the net fixed assets (NFA)

On the balance sheet, the net fixed assets (NFA) account is equal to the gross fixed assets (FA) account, which records the acquisition cost of fixed assets, minus the accumulated depreciation (AD) ac...

See Answer

Q: Risks In broad terms, why is some risk diversifiable? Why

Risks In broad terms, why is some risk diversifiable? Why are some risks nondiversifiable? Does it follow that an investor can control the level of unsystematic risk in a portfolio, but not the level...

See Answer

Q: If a portfolio has a positive investment in every asset, can

If a portfolio has a positive investment in every asset, can the expected return on the portfolio be greater than that on every asset in the portfolio? Can it be less than that on every asset in the p...

See Answer

Q: Suppose a financial manager is quoted as saying, “Our firm

Suppose a financial manager is quoted as saying, “Our firm uses the stand-alone principle. Because we treat projects like mini firms in our evaluation process, we include financing costs because they...

See Answer

Q: The shareholders of the Stackhouse Company need to elect seven new directors

The shareholders of the Stackhouse Company need to elect seven new directors. There are 960,000 shares outstanding currently trading at $48 per share. You would like to serve on the board of directors...

See Answer

Q: Why does traditional NPV analysis tend to underestimate the true value of

Why does traditional NPV analysis tend to underestimate the true value of a capital budgeting project?

See Answer

Q: Are there any circumstances under which an investor might be more concerned

Are there any circumstances under which an investor might be more concerned about the nominal return on an investment than the real return?

See Answer

Q: Is it possible for the risk premium to be negative before an

Is it possible for the risk premium to be negative before an investment is undertaken? Can the risk premium be negative after the fact? Explain.

See Answer

Q: Is it possible that a risky asset could have a beta of

Is it possible that a risky asset could have a beta of zero? Explain. Based on the CAPM, what is the expected return on such an asset? Is it possible that a risky asset could have a negative beta? Wha...

See Answer

Q: In contrast to the CAPM, the APT does not indicate which

In contrast to the CAPM, the APT does not indicate which factors are expected to determine the risk premium of an asset. How can we determine which factors should be included? For example, one risk fa...

See Answer

Q: Suppose Tom O’Bedlam, president of Bedlam Products, Inc., has

Suppose Tom O’Bedlam, president of Bedlam Products, Inc., has hired you to determine the firm’s cost of debt and cost of equity capital. a. The stock currently sells for $50 per share, and the dividen...

See Answer

Q: A company is contemplating a long-term bond issue. It

A company is contemplating a long-term bond issue. It is debating whether to include a call provision. What are the benefits to the company from including a call provision? What are the costs? How do...

See Answer

Q: How would you answer in the following debate? Q:

How would you answer in the following debate? Q: Isn’t it true that the riskiness of a firm’s equity will rise if the firm increases its use of debt financing? A: Yes, that’s the essence of MM Proposi...

See Answer

Q: If you use the stock beta and the security market line to

If you use the stock beta and the security market line to compute the discount rate for a project, what assumptions are you implicitly making?

See Answer

Q: What are the sources of agency costs of equity?

What are the sources of agency costs of equity?

See Answer

Q: Criteria Discuss the IRS criteria for determining whether a lease is tax

Criteria Discuss the IRS criteria for determining whether a lease is tax deductible. In each case give a rationale for the criterion.

See Answer

Q: Describe the difference between systematic risk and unsystematic risk.

Describe the difference between systematic risk and unsystematic risk.

See Answer

Q: Music City, Inc., has no debt outstanding and a total

Music City, Inc., has no debt outstanding and a total market value of $295,000. Earnings before interest and taxes, EBIT, are projected to be $23,000 if economic conditions are normal. If there is str...

See Answer

Q: Star Mining buys a gold mine, but the cost of extraction

Star Mining buys a gold mine, but the cost of extraction is currently too high to make the mine profitable. In option terminology, what type of option(s) does the company have on this mine?

See Answer

Q: What are some of the difficulties that might come up in actual

What are some of the difficulties that might come up in actual applications of the various criteria we discussed in this chapter? Which one would be the easiest to implement in actual applications? Th...

See Answer

Q: Suppose you know that a company’s stock currently sells for $67

Suppose you know that a company’s stock currently sells for $67 per share and the required return on the stock is 10.8 percent. You also know that the total return on the stock is evenly divided betwe...

See Answer

Q: Your firm is contemplating the purchase of a new $530,

Your firm is contemplating the purchase of a new $530,000 computer-based order entry system. The system will be depreciated straight-line to zero over its five-year life. It will be worth $50,000 at t...

See Answer

Q: Ang Electronics, Inc., has developed a new DVDR. If

Ang Electronics, Inc., has developed a new DVDR. If the DVDR is successful, the present value of the payoff (when the product is brought to market) is $27 million. If the DVDR fails, the present value...

See Answer

Q: A Japanese company has a bond outstanding that sells for 106 percent

A Japanese company has a bond outstanding that sells for 106 percent of its ¥100,000 par value. The bond has a coupon rate of 2.8 percent paid annually and matures in 21 years. What is the yield to ma...

See Answer

Q: What is the main difference between the FTE approach and the other

What is the main difference between the FTE approach and the other two approaches?

See Answer

Q: Based on the following information, calculate the expected return and standard

Based on the following information, calculate the expected return and standard deviation:

See Answer

Q: The following three stocks are available in the market:

The following three stocks are available in the market: Assume the market model is valid. a. Write the market model equation for each stock. b. What is the return on a portfolio with weights of 30 p...

See Answer

Q: Frusciante, Inc., has 290,000 bonds outstanding. The

Frusciante, Inc., has 290,000 bonds outstanding. The bonds have a par value of $1,000, a coupon rate of 7 percent paid semiannually, and 8 years to maturity. The current YTM on the bonds is 7.5 percen...

See Answer

Q: Kolby Corp. is comparing two different capital structures. Plan I

Kolby Corp. is comparing two different capital structures. Plan I would result in 1,300 shares of stock and $80,640 in debt. Plan II would result in 2,900 shares of stock and $19,200 in debt. The inte...

See Answer

Q: Janetta Corp. has EBIT of $850,000 per year

Janetta Corp. has EBIT of $850,000 per year that is expected to continue in perpetuity. The unlevered cost of equity for the company is 14 percent, and the corporate tax rate is 35 percent. The compan...

See Answer

Q: Steinberg Corporation and Dietrich Corporation are identical firms except that Dietrich is

Steinberg Corporation and Dietrich Corporation are identical firms except that Dietrich is more levered. Both companies will remain in business for one more year. The companies’ economists agree that...

See Answer

Q: Daniel Kaffe, CFO of Kendrick Enterprises, is evaluating a 10

Daniel Kaffe, CFO of Kendrick Enterprises, is evaluating a 10-year, 7.5 percent loan with gross proceeds of $4,450,000. The interest payments on the loan will be made annually. Flotation costs are est...

See Answer

Q: In the previous problem, suppose the company has announced it is

In the previous problem, suppose the company has announced it is going to repurchase $22,400 worth of stock. What effect will this transaction have on the equity of the company? How many shares will b...

See Answer

Q: In the previous problem, if the SEC filing fee and associated

In the previous problem, if the SEC filing fee and associated administrative expenses of the offering are $1,900,000, how many shares need to be sold?

See Answer

Q: You work for a nuclear research laboratory that is contemplating leasing a

You work for a nuclear research laboratory that is contemplating leasing a diagnostic scanner (leasing is a common practice with expensive, high-tech equipment). The scanner costs $5,800,000, and it w...

See Answer

Q: You work for a nuclear research laboratory that is contemplating leasing a

You work for a nuclear research laboratory that is contemplating leasing a diagnostic scanner (leasing is a common practice with expensive, high-tech equipment). The scanner costs $5,800,000, and it w...

See Answer

Q: Sardano and Sons is a large, publicly held company that is

Sardano and Sons is a large, publicly held company that is considering leasing a warehouse. One of the company’s divisions specializes in manufacturing steel, and this particular warehouse is the only...

See Answer

Q: An analyst has recently informed you that at the issuance of a

An analyst has recently informed you that at the issuance of a company’s convertible bonds, one of the two following sets of relationships existed: Assume the bonds are available f...

See Answer

Q: Compute the internal rate of return for the cash flows of the

Compute the internal rate of return for the cash flows of the following two projects:

See Answer

Q: If increases in dividends tend to be followed by (immediate)

If increases in dividends tend to be followed by (immediate) increases in share prices, how can it be said that dividend policy is irrelevant?

See Answer

Q: Zipcar, the car sharing company, went public in April 2011

Zipcar, the car sharing company, went public in April 2011. Assisted by the investment bank Goldman Sachs, Zipcar sold 9.68 million shares at $18 each, thereby raising a total of $174.24 million. By t...

See Answer

Q: Zoso is a rental car company that is trying to determine whether

Zoso is a rental car company that is trying to determine whether to add 25 cars to its fleet. The company fully depreciates all its rental cars over five years using the straight-line method. The new...

See Answer

Q: If the risk of a stock increases, what is likely to

If the risk of a stock increases, what is likely to happen to the price of call options on the stock? To the price of put options? Why?

See Answer

Q: What is wrong with the simple view that it is cheaper to

What is wrong with the simple view that it is cheaper to issue a bond with a warrant or a convertible feature because the required coupon is lower?

See Answer

Q: If a market is semistrong form efficient, is it also weak

If a market is semistrong form efficient, is it also weak form efficient? Explain.

See Answer

Q: Based on the dividend growth model, what are the two components

Based on the dividend growth model, what are the two components of the total return on a share of stock? Which do you think is typically larger?

See Answer

Q: Why do noninvestment-grade bonds have much higher direct costs than

Why do noninvestment-grade bonds have much higher direct costs than investment-grade issues?

See Answer

Q: When is EAC analysis appropriate for comparing two or more projects?

When is EAC analysis appropriate for comparing two or more projects? Why is this method used? Are there any implicit assumptions required by this method that you find troubling? Explain.

See Answer

Q: The Mango Republic has just liberalized its markets and is now permitting

The Mango Republic has just liberalized its markets and is now permitting foreign investors. Tesla Manufacturing has analyzed starting a project in the country and has determined that the project has...

See Answer

Q: Two years ago, General Materials’ and Standard Fixtures’ stock prices were

Two years ago, General Materials’ and Standard Fixtures’ stock prices were the same. During the first year, General Materials’ stock price increased by 10 percent while Standard Fixtures’ stock price...

See Answer

Q: Briefly explain why the covariance of a security with the rest of

Briefly explain why the covariance of a security with the rest of a well-diversified portfolio is a more appropriate measure of the risk of the security than the security’s variance.

See Answer

Q: Both Dow Chemical Company, a large natural gas user, and

Both Dow Chemical Company, a large natural gas user, and Superior Oil, a major natural gas producer, are thinking of investing in natural gas wells near Houston. Both are all-equity financed companies...

See Answer

Q: Is there an easily identifiable debt–equity ratio that will maximize

Is there an easily identifiable debt–equity ratio that will maximize the value of a firm? Why or why not?

See Answer

Q: Chanelle, Inc., is proposing a rights offering. Presently,

Chanelle, Inc., is proposing a rights offering. Presently, there are 625,000 shares outstanding at $87 each. There will be 85,000 new shares offered at $78 each. a. What is the new market value of the...

See Answer

Q: Refer to the observed capital structures given in Table 17.3

Refer to the observed capital structures given in Table 17.3 of the text. What do you notice about the types of industries with respect to their average debt–equity ratios? Are certa...

See Answer

Q: What is meant by the term off–balance sheet financing?

What is meant by the term off–balance sheet financing? When do leases provide such financing, and what are the accounting and economic consequences of such activity?

See Answer

Q: You are discussing real options with a colleague. During the discussion

You are discussing real options with a colleague. During the discussion, the colleague states, “Real option analysis makes no sense because it says that a real option on a risky venture is worth more...

See Answer

Q: Why does a strict NPV calculation typically understate the value of a

Why does a strict NPV calculation typically understate the value of a company or project?

See Answer

Q: Why do firms issue convertible bonds and bonds with warrants?

Why do firms issue convertible bonds and bonds with warrants?

See Answer

Q: Watson, Inc., is an all-equity firm. The

Watson, Inc., is an all-equity firm. The cost of the company’s equity is currently 11.9 percent, and the risk-free rate is 3.5 percent. The company is currently considering a project that will cost $1...

See Answer

Q: The market value balance sheet for Outbox Manufacturing is shown here.

The market value balance sheet for Outbox Manufacturing is shown here. Outbox has declared a stock dividend of 25 percent. The stock goes ex dividend tomorrow (the chronology for a stock dividend is s...

See Answer

Q: The Green Hills Co. has just gone public. Under a

The Green Hills Co. has just gone public. Under a firm commitment agreement, Green Hills received $29.96 for each of the 7.5 million shares sold. The initial offering price was $32 per share, and the...

See Answer

Q: Super Sonics Entertainment is considering buying a machine that costs $480

Super Sonics Entertainment is considering buying a machine that costs $480,000. The machine will be depreciated over five years by the straight-line method and will be worthless at that time. The comp...

See Answer

Q: Wet for the Summer, Inc., manufactures filters for swimming pools

Wet for the Summer, Inc., manufactures filters for swimming pools. The company is deciding whether to implement a new technology in its pool filters. One year from now the company will know whether th...

See Answer

Q: Sportime Fitness Center, Inc., issued convertible bonds with a conversion

Sportime Fitness Center, Inc., issued convertible bonds with a conversion price of $49. The bonds are available for immediate conversion. The current price of the company’s common stock is $43 per sha...

See Answer

Q: Maxwell Software, Inc., has the following mutually exclusive projects.

Maxwell Software, Inc., has the following mutually exclusive projects. a. Suppose the company’s payback period cutoff is two years. Which of these two projects should be chosen? b....

See Answer

Q: Dog Up! Franks is looking at a new sausage system with

Dog Up! Franks is looking at a new sausage system with an installed cost of $345,000. This cost will be depreciated straight-line to zero over the project’s five-year life, at the end of which the sau...

See Answer

Q: The manager for a growing firm is considering the launch of a

The manager for a growing firm is considering the launch of a new product. If the product goes directly to market, there is a 50 percent chance of success. For $125,000 the manager can conduct a focus...

See Answer

Q: Suppose you are evaluating a callable, convertible bond. If the

Suppose you are evaluating a callable, convertible bond. If the stock price volatility increases, how will this affect the price of the bond?

See Answer

Q: You find a zero coupon bond with a par value of $

You find a zero coupon bond with a par value of $10,000 and 17 years to maturity. If the yield to maturity on this bond is 4.9 percent, what is the dollar price of the bond? Assume semiannual compound...

See Answer

Q: Using the following returns, calculate the average returns, the variances

Using the following returns, calculate the average returns, the variances, and the standard deviations for X and Y:

See Answer

Q: A portfolio is invested 20 percent in Stock G, 55 percent

A portfolio is invested 20 percent in Stock G, 55 percent in Stock J, and 25 percent in Stock K. The expected returns on these stocks are 9 percent, 11 percent, and 14 percent, respectively. What is t...

See Answer

Q: You are forming an equally weighted portfolio of stocks. Many stocks

You are forming an equally weighted portfolio of stocks. Many stocks have the same beta of .84 for Factor 1 and the same beta of 1.69 for Factor 2. All stocks also have the same expected return of 11...

See Answer

Q: Fama’s Llamas has a weighted average cost of capital of 9.

Fama’s Llamas has a weighted average cost of capital of 9.8 percent. The company’s cost of equity is 13 percent, and its cost of debt is 6.5 percent. The tax rate is 35 percent. What is Fama’s debt–eq...

See Answer

Q: Harrison, Inc., has the following book value balance sheet:

Harrison, Inc., has the following book value balance sheet: a. What is the debt–equity ratio based on book values? b. Suppose the market value of the company’s de...

See Answer

Q: Ignoring taxes in Problem 6, what is the price per share

Ignoring taxes in Problem 6, what is the price per share of equity under Plan I? Plan II? What principle is illustrated by your answers?

See Answer

Q: Agency Costs Fountain Corporation’s economists estimate that a good business environment and

Agency Costs Fountain Corporation’s economists estimate that a good business environment and a bad business environment are equally likely for the coming year. The managers of the co...

See Answer

Q: What is forecasting risk? In general, would the degree of

What is forecasting risk? In general, would the degree of forecasting risk be greater for a new product or a cost-cutting proposal? Why?

See Answer

Q: Bill plans to open a self-serve grooming center in a

Bill plans to open a self-serve grooming center in a storefront. The grooming equipment will cost $265,000, to be paid immediately. Bill expects aftertax cash inflows of $59,000 annually for seven yea...

See Answer

Q: A coworker claims that looking at so much marginal this and incremental

A coworker claims that looking at so much marginal this and incremental that is just a bunch of nonsense, and states, “Listen, if our average revenue doesn’t exceed our average cost, then we will have...

See Answer

Q: What is the relationship between the one-factor model and the

What is the relationship between the one-factor model and the CAPM?

See Answer

Q: What are the implications of the efficient market hypothesis for investors who

What are the implications of the efficient market hypothesis for investors who buy and sell stocks in an attempt to “beat the market”?

See Answer

Q: What is a proxy?

What is a proxy?

See Answer

Q: Last month, Central Virginia Power Company, which had been having

Last month, Central Virginia Power Company, which had been having trouble with cost overruns on a nuclear power plant that it had been building, announced that it was “temporarily suspending dividend...

See Answer

Q: Zipcar, the car sharing company, went public in April 2011

Zipcar, the car sharing company, went public in April 2011. Assisted by the investment bank Goldman Sachs, Zipcar sold 9.68 million shares at $18 each, thereby raising a total of $174.24 million. By t...

See Answer

Q: True or false: The unsystematic risk of a share of stock

True or false: The unsystematic risk of a share of stock is irrelevant for valuing the stock because it can be diversified away; therefore, it is also irrelevant for valuing a call option on the stock...

See Answer

Q: Companies pay rating agencies such as Moody’s and S&P to

Companies pay rating agencies such as Moody’s and S&P to rate their bonds, and the costs can be substantial. However, companies are not required to have their bonds rated in the first place; doing so...

See Answer

Q: In the context of the dividend growth model, is it true

In the context of the dividend growth model, is it true that the growth rate in dividends and the growth rate in the price of the stock are identical?

See Answer

Q: Are the capital budgeting criteria we discussed applicable to not-for

Are the capital budgeting criteria we discussed applicable to not-for-profit corporations? How should such entities make capital budgeting decisions? What about the U.S. government? Should it evaluate...

See Answer

Q: Is it true that a U.S. Treasury security is

Is it true that a U.S. Treasury security is risk-free?

See Answer

Q: Referring to the previous questions, under what circumstances might a company

Referring to the previous questions, under what circumstances might a company choose not to pay dividends?

See Answer

Q: “When evaluating projects, we’re only concerned with the relevant incremental

“When evaluating projects, we’re only concerned with the relevant incremental aftertax cash flows. Therefore, because depreciation is a noncash expense, we should ignore its effects when evaluating pr...

See Answer

Q: U.S. Treasury bonds are not rated. Why?

U.S. Treasury bonds are not rated. Why? Often, junk bonds are not rated. Why?

See Answer

Q: Two years ago, the Lake Minerals and Small Town Furniture stock

Two years ago, the Lake Minerals and Small Town Furniture stock prices were the same. The average annual return for both stocks over the past two years was 10 percent. Lake Minerals’ stock price incre...

See Answer

Q: Consider the following quotation from a leading investment manager: “The

Consider the following quotation from a leading investment manager: “The shares of Southern Co. have traded close to $12 for most of the past three years. Since Southern’s stock has demonstrated very...

See Answer

Q: Under what circumstances would it be appropriate for a firm to use

Under what circumstances would it be appropriate for a firm to use different costs of capital for its different operating divisions? If the overall firm WACC was used as the hurdle rate for all divisi...

See Answer

Q: As mentioned in the text, some firms have filed for bankruptcy

As mentioned in the text, some firms have filed for bankruptcy because of actual or likely litigation-related losses. Is this a proper use of the bankruptcy process?

See Answer

Q: The DRK Corporation has recently developed a dividend reinvestment plan, or

The DRK Corporation has recently developed a dividend reinvestment plan, or DRIP. The plan allows investors to reinvest cash dividends automatically in DRK in exchange for new shares of stock. Over ti...

See Answer

Q: Why might a firm choose to engage in a sale and leaseback

Why might a firm choose to engage in a sale and leaseback transaction? Give two reasons.

See Answer

Q: Ayden, Inc., has an issue of preferred stock outstanding that

Ayden, Inc., has an issue of preferred stock outstanding that pays a $4.50 dividend every year, in perpetuity. If this issue currently sells for $87 per share, what is the required return?

See Answer

Q: National Electric Company (NEC) is considering a $68 million

National Electric Company (NEC) is considering a $68 million project in its power systems division. Tom Edison, the company’s chief financial officer, has evaluated the project and determined that the...

See Answer

Q: Given the choice, would a firm prefer to use MACRS depreciation

Given the choice, would a firm prefer to use MACRS depreciation or straight-line depreciation? Why?

See Answer

Q: Given that RadNet was up by about 411 per cent for 2014

Given that RadNet was up by about 411 per cent for 2014, why didn’t all investors hold RadNet?

See Answer

Q: The company with the common equity accounts shown here has declared a

The company with the common equity accounts shown here has declared a stock dividend of 15 percent when the market value of its stock is $57 per share. What effects on the equity accounts will the dis...

See Answer

Q: Raggio, Inc., has 145,000 shares of stock outstanding

Raggio, Inc., has 145,000 shares of stock outstanding. Each share is worth $75, so the company’s market value of equity is $10,875,000. Suppose the firm issues 30,000 new shares at the following price...

See Answer

Q: Quartz Corporation is a relatively new firm. Quartz has experienced enough

Quartz Corporation is a relatively new firm. Quartz has experienced enough losses during its early years to provide it with at least eight years of tax loss carryforwards. Thus, Quartz’s effective tax...

See Answer

Q: There is a European put option on a stock that expires in

There is a European put option on a stock that expires in two months. The stock price is $82, and the standard deviation of the stock returns is 70 percent. The option has a strike price of $90, and t...

See Answer

Q: An asset used in a four-year project falls in the

An asset used in a four-year project falls in the five-year MACRS class for tax purposes. The asset has an acquisition cost of $8,300,000 and will be sold for $1,700,000 at the end of the project. If...

See Answer

Q: B&B has a new baby powder ready to market.

B&B has a new baby powder ready to market. If the firm goes directly to the market with the product, there is only a 55 percent chance of success. However, the firm can conduct customer segment resear...

See Answer

Q: Refer to Table 10.1(given below) in the

Refer to Table 10.1(given below) in the text and look at the period from 1973 through 1978. a. Calculate the arithmetic average returns for large-company stocks and T-bills over this period. b. Calc...

See Answer

Q: Consider the following information: / a.

Consider the following information: a. What is the expected return on an equally weighted portfolio of these three stocks? b. What is the variance of a portfolio invested 20 percent each in A and B,...

See Answer

Q: There are two stock markets, each driven by the same common

There are two stock markets, each driven by the same common force, F, with an expected value of zero and standard deviation of 10 percent. There are many securities in each market; thus, you can inves...

See Answer

Q: The Starr Co. just paid a dividend of $1.

The Starr Co. just paid a dividend of $1.95 per share on its stock. The dividends are expected to grow at a constant rate of 4.5 percent per year, indefinitely. If investors require a return of 11 per...

See Answer

Q: You own stock in the Lewis-Striden Drug Company. Suppose

You own stock in the Lewis-Striden Drug Company. Suppose you had expected the following events to occur last month: a. The government would announce that real GNP had grown 1.2 percent during the prev...

See Answer

Q: Filer Manufacturing has 8.3 million shares of common stock outstanding

Filer Manufacturing has 8.3 million shares of common stock outstanding. The current share price is $53, and the book value per share is $4. The company also has two bond issues outstanding. The first...

See Answer

Q: If you can borrow all the money you need for a project

If you can borrow all the money you need for a project at 6 percent, doesn’t it follow that 6 percent is your cost of capital for the project?

See Answer

Q: KIC, Inc., plans to issue $5 million of bonds

KIC, Inc., plans to issue $5 million of bonds with a coupon rate of 8 percent and 30 years to maturity. The current market interest rates on these bonds are 7 percent. In one year, the interest rate o...

See Answer

Q: Star, Inc., a prominent consumer products firm, is debating

Star, Inc., a prominent consumer products firm, is debating whether or not to convert its all-equity capital structure to one that is 35 percent debt. Currently there are 6,000 shares outstanding and...

See Answer

Q: Good Time Company is a regional chain department store. It will

Good Time Company is a regional chain department store. It will remain in business for one more year. The probability of a boom year is 60 percent and the probability of a recession is 40 percent. It...

See Answer

Q: Suppose the following two independent investment opportunities are available to Relax,

Suppose the following two independent investment opportunities are available to Relax, Inc. The appropriate discount rate is 8.5 percent. a. Compute the profitability index for each of the two proje...

See Answer

Q: How can the return on a portfolio be expressed in terms of

How can the return on a portfolio be expressed in terms of a factor model?

See Answer

Q: Critically evaluate the following statements: Playing the stock market is like

Critically evaluate the following statements: Playing the stock market is like gambling. Such speculative investing has no social value other than the pleasure people get from this form of gambling.

See Answer

Q: Do you think preferred stock is more like debt or equity?

Do you think preferred stock is more like debt or equity? Why?

See Answer

Q: Why is the use of debt financing referred to as financial “

Why is the use of debt financing referred to as financial “leverage”?

See Answer

Q: What are the advantages of using the SML approach to finding the

What are the advantages of using the SML approach to finding the cost of equity capital? What are the disadvantages? What are the specific pieces of information needed to use this method? Are all of t...

See Answer

Q: Ren-Stimpy International is planning to raise fresh equity capital by

Ren-Stimpy International is planning to raise fresh equity capital by selling a large new issue of common stock. Ren-Stimpy is currently a publicly traded corporation, and it is trying to choose betwe...

See Answer

Q: Suppose a certain stock currently sells for $30 per share.

Suppose a certain stock currently sells for $30 per share. If a put option and a call option are available with $30 exercise prices, which do you think will sell for more? Explain.

See Answer

Q: How is the APV of a project calculated?

How is the APV of a project calculated?

See Answer

Q: Your company currently uses traditional capital budgeting techniques, including net present

Your company currently uses traditional capital budgeting techniques, including net present value. After hearing about the use of real option analysis, your boss decides that your company should use r...

See Answer

Q: Why will convertible bonds not be voluntarily converted to stock before expiration

Why will convertible bonds not be voluntarily converted to stock before expiration?

See Answer

Q: How does sensitivity analysis interact with break-even analysis?

How does sensitivity analysis interact with break-even analysis?

See Answer

Q: You own a callable, convertible bond with a conversion ratio of

You own a callable, convertible bond with a conversion ratio of 25.18. The stock is currently selling for $47 per share. The issuer of the bond has announced a call at a call price of 110. What are yo...

See Answer

Q: What are the three factors that determine a company’s price−earnings

What are the three factors that determine a company’s price−earnings ratio?

See Answer

Q: The investment in Project A is $1 million, and the

The investment in Project A is $1 million, and the investment in Project B is $2 million. Both projects have a unique internal rate of return of 20 percent. Is the following statement true or false? F...

See Answer

Q: A major college textbook publisher has an existing finance textbook. The

A major college textbook publisher has an existing finance textbook. The publisher is debating whether to produce an “essentialized” version, meaning a shorter (and lower-priced) book. What are some o...

See Answer

Q: An option can often have more than one source of value.

An option can often have more than one source of value. Consider a logging company. The company can log the timber today or wait another year (or more) to log the timber. What advantages would waiting...

See Answer

Q: What is the difference between arithmetic and geometric returns? Suppose you

What is the difference between arithmetic and geometric returns? Suppose you have invested in a stock for the last 10 years. Which number is more important to you, the arithmetic or geometric return?...

See Answer

Q: A broker has advised you not to invest in oil industry stocks

A broker has advised you not to invest in oil industry stocks because they have high standard deviations. Is the broker’s advice sound for a risk-averse investor like yourself? Why or why not?

See Answer

Q: What rule should a firm follow when making financing decisions? How

What rule should a firm follow when making financing decisions? How can firms create valuable financing opportunities?

See Answer

Q: How is it possible that dividends are so important, but at

How is it possible that dividends are so important, but at the same time dividend policy is irrelevant?

See Answer

Q: What is data mining? Why might it overstate the relation between

What is data mining? Why might it overstate the relation between some stock attribute and returns?

See Answer

Q: Consider a levered firm’s projects that have similar risks to the firm

Consider a levered firm’s projects that have similar risks to the firm as a whole. Is the discount rate for the projects higher or lower than the rate computed using the security market line? Why?

See Answer

Q: Several celebrated investors and stock pickers frequently mentioned in the financial press

Several celebrated investors and stock pickers frequently mentioned in the financial press have recorded huge returns on their investments over the past two decades. Does the success of these particul...

See Answer

Q: As was mentioned in the chapter, new equity issues are generally

As was mentioned in the chapter, new equity issues are generally only a small portion of all new issues. At the same time, companies continue to issue new debt. Why do companies tend to issue little n...

See Answer

Q: For initial public offerings of common stock, 2007 was a relatively

For initial public offerings of common stock, 2007 was a relatively slow year, with only about $35.6 billion raised by the process. Relatively few of the 159 firms involved paid cash dividends. Why do...

See Answer

Q: The yields on nonconvertible preferred stock are lower than the yields on

The yields on nonconvertible preferred stock are lower than the yields on corporate bonds. Why is there a difference? Which investors are the primary holders of preferred stock? Why?

See Answer

Q: When should a firm force conversion of convertibles? Why?

When should a firm force conversion of convertibles? Why?

See Answer

Q: The newspaper reported last week that Bennington Enterprises earned $29 million

The newspaper reported last week that Bennington Enterprises earned $29 million this year. The report also stated that the firm’s return on equity is 17 percent. Bennington retains 80 percent of its e...

See Answer

Q: The Wildcat Oil Company is trying to decide whether to lease or

The Wildcat Oil Company is trying to decide whether to lease or buy a new computer-assisted drilling system for its oil exploration business. Management has decided that it must use the system to stay...

See Answer

Q: In the previous problem, assume that the exercise style on the

In the previous problem, assume that the exercise style on the option is American rather than European. What is the price of the option now? (Hint: How will you find the value of the option if it can...

See Answer

Q: General Modems has five-year warrants that currently trade in the

General Modems has five-year warrants that currently trade in the open market. Each warrant gives its owner the right to purchase one share of common stock for an exercise price of $55. a. Suppose the...

See Answer

Q: Why do companies issue options to executives if they cost the company

Why do companies issue options to executives if they cost the company more than they are worth to the executive? Why not just give cash and split the difference? Wouldn’t that make both the company an...

See Answer

Q: Howell Petroleum is considering a new project that complements its existing business

Howell Petroleum is considering a new project that complements its existing business. The machine required for the project costs $3.9 million. The marketing department predicts that sales related to t...

See Answer

Q: You are considering investing in a company that cultivates abalone for sale

You are considering investing in a company that cultivates abalone for sale to local restaurants. Use the following information: The discount rate for the company is 15 percent, the initial investme...

See Answer

Q: You’ve observed the following returns on SkyNet Data Corporation’s stock over the

You’ve observed the following returns on SkyNet Data Corporation’s stock over the past five years: 21 percent, 17 percent, 26 percent, 27 percent, and 4 percent. a. What was the arithmetic average ret...

See Answer

Q: Consider the following information: / a.

Consider the following information: a. Your portfolio is invested 30 percent each in A and C, and 40 percent in B. What is the expected return of the portfolio? b. What is the variance of this portf...

See Answer

Q: T-bills currently yield 3.9 percent. Stock in

T-bills currently yield 3.9 percent. Stock in Nina Manufacturing is currently selling for $63 per share. There is no possibility that the stock will be worth less than $61 per share in one year. a. Wh...

See Answer

Q: Assume that the following market model adequately describes the return-generating

Assume that the following market model adequately describes the return-generating behavior of risky assets: Here: Rit = The return on the ith asset at Time t. RMt = The return on a portfolio containin...

See Answer

Q: In the previous problem, suppose the company’s stock has a beta

In the previous problem, suppose the company’s stock has a beta of 1.15. The risk-free rate is 3.7 percent, and the market risk premium is 7 percent. Assume that the overall cost of debt is the weight...

See Answer

Q: ABC Co. and XYZ Co. are identical firms in all

ABC Co. and XYZ Co. are identical firms in all respects except for their capital structure. ABC is all equity financed with $640,000 in stock. XYZ uses both stock and perpetual debt; its stock is wort...

See Answer

Q: When personal taxes on interest income and bankruptcy costs are considered,

When personal taxes on interest income and bankruptcy costs are considered, the general expression for the value of a levered firm in a world in which the tax rate on equity distributions equals zero...

See Answer

Q: Bolero, Inc., has compiled the following information on its financing

Bolero, Inc., has compiled the following information on its financing costs: The company is in the 35 percent tax bracket and has a target debt–equity ratio of 60 percent. The targ...

See Answer

Q: In the previous problem, suppose the company instead decides on a

In the previous problem, suppose the company instead decides on a five-for-one stock split. The firm’s 45 cent per share cash dividend on the new (postsplit) shares represents an increase of 10 percen...

See Answer

Q: What is the primary difference between a warrant and a traded call

What is the primary difference between a warrant and a traded call option?

See Answer

Q: The Newton Company has 50,000 shares of stock that each

The Newton Company has 50,000 shares of stock that each sell for $40. Suppose the company issues 9,000 shares of new stock at the following prices: $40, $20, and $10. What is the effect of each of the...

See Answer

Q: A project has an initial cost of I, has a required

A project has an initial cost of I, has a required return of R, and pays C annually for N years. a. Find C in terms of I and N such that the project has a payback period just equal to its life. b. Fin...

See Answer

Q: What is homemade leverage?

What is homemade leverage?

See Answer

Q: What steps can stockholders take to reduce the costs of debt?

What steps can stockholders take to reduce the costs of debt?

See Answer

Q: Firms sometimes use the threat of a bankruptcy filing to force creditors

Firms sometimes use the threat of a bankruptcy filing to force creditors to renegotiate terms. Critics argue that in such cases the firm is using bankruptcy laws “as a sword rather than a shield.” Is...

See Answer

Q: In 1980, a certain assistant professor of finance bought 12 initial

In 1980, a certain assistant professor of finance bought 12 initial public offerings of common stock. He held each of these for approximately one month and then sold them. The investment rule he follo...

See Answer

Q: Explain why the aftertax borrowing rate is the appropriate discount rate to

Explain why the aftertax borrowing rate is the appropriate discount rate to use in lease evaluation.

See Answer

Q: Suppose the interest rate on T-bills suddenly and unexpectedly rises

Suppose the interest rate on T-bills suddenly and unexpectedly rises. All other things being the same, what is the impact on call option values? On put option values?

See Answer

Q: Insurance, whether purchased by a corporation or an individual, is

Insurance, whether purchased by a corporation or an individual, is in essence an option. What type of option is an insurance policy?

See Answer

Q: What is the difference between the term structure of interest rates and

What is the difference between the term structure of interest rates and the yield curve?

See Answer

Q: Is it unfair or unethical for corporations to create classes of stock

Is it unfair or unethical for corporations to create classes of stock with unequal voting rights?

See Answer

Q: In the context of capital budgeting, what is an opportunity cost

In the context of capital budgeting, what is an opportunity cost?

See Answer

Q: Consider the following two mutually exclusive projects available to Global Investments,

Consider the following two mutually exclusive projects available to Global Investments, Inc.: The appropriate discount rate for the projects is 10 percent. Global Investments chose to undertake Proj...

See Answer

Q: You are discussing a project analysis with a coworker. The project

You are discussing a project analysis with a coworker. The project involves real options, such as expanding the project if successful, or abandoning the project if it fails. Your coworker makes the fo...

See Answer

Q: Comment on the following remarks: a. Leasing reduces risk

Comment on the following remarks: a. Leasing reduces risk and can reduce a firm’s cost of capital. b. Leasing provides 100 percent financing. c. If the tax advantages of leasing were eliminated, leasi...

See Answer

Q: The historical asset class returns presented in the chapter are not adjusted

The historical asset class returns presented in the chapter are not adjusted for inflation. What would happen to the estimated risk premium if we did account for inflation? The returns are also not ad...

See Answer

Q: Is the following statement true or false? A risky security cannot

Is the following statement true or false? A risky security cannot have an expected return that is less than the risk-free rate because no risk-averse investor would be willing to hold this asset in eq...

See Answer

Q: What factors determine the beta of a stock? Define and describe

What factors determine the beta of a stock? Define and describe each.

See Answer

Q: For each of the following scenarios, discuss whether profit opportunities exist

For each of the following scenarios, discuss whether profit opportunities exist from trading in the stock of the firm under the conditions that (1) the market is not weak form efficient, (2) the mark...

See Answer

Q: What is the difference between internal financing and external financing?

What is the difference between internal financing and external financing?

See Answer

Q: The Phew Charitable Trust pays no taxes on its capital gains or

The Phew Charitable Trust pays no taxes on its capital gains or on its dividend income or interest income. Would it be irrational for it to have low-dividend, high-growth stocks in its portfolio? Woul...

See Answer

Q: In April 2014, International Lease Finance Corporation (ILFC) announced

In April 2014, International Lease Finance Corporation (ILFC) announced a deal to purchase eight Airbus A330-200 and A350-900 passenger aircraft. ILFC then signed a long-term lease contract on the pla...

See Answer

Q: A convertible bond with a par value of $1,000

A convertible bond with a par value of $1,000 has a conversion ratio of 19.2. What is the conversion price?

See Answer

Q: When you take out an ordinary student loan, it is usually

When you take out an ordinary student loan, it is usually the case that whoever holds that loan is given a guarantee by the U.S. government, meaning that the government will make up any payments you s...

See Answer

Q: Utility companies often face a decision to build new plants that burn

Utility companies often face a decision to build new plants that burn coal, oil, or both. If the prices of both coal and gas are highly volatile, how valuable is the decision to build a plant that can...

See Answer

Q: How would the analysis of real options change if a company has

How would the analysis of real options change if a company has competitors?

See Answer

Q: A warrant with six months until expiration entitles its owner to buy

A warrant with six months until expiration entitles its owner to buy 10 shares of the issuing firm’s common stock for an exercise price of $31 per share. If the current market price of the stock is $1...

See Answer

Q: The Spring Flower Co. has earnings of $2.35

The Spring Flower Co. has earnings of $2.35 per share. The benchmark PE for the company is 18. What stock price would you consider appropriate? What if the benchmark PE were 21?

See Answer

Q: You are in discussions to purchase an option on an office building

You are in discussions to purchase an option on an office building with a strike price of $63 million. The building is currently valued at $60 million. The option will allow you to purchase the buildi...

See Answer

Q: Vital Silence Corp. has just issued a 30-year callable

Vital Silence Corp. has just issued a 30-year callable, convertible bond with an annual coupon rate of 6 percent. The bond has a conversion price of $93. The company’s stock is selling for $28 per sha...

See Answer

Q: You are evaluating two different silicon wafer milling machines. The Techron

You are evaluating two different silicon wafer milling machines. The Techron I costs $245,000, has a three-year life, and has pretax operating costs of $39,000 per year. The Techron II costs $315,000,...

See Answer

Q: Niko has purchased a brand new machine to produce its High Flight

Niko has purchased a brand new machine to produce its High Flight line of shoes. The machine has an economic life of five years. The depreciation schedule for the machine is straight-line with no salv...

See Answer

Q: If Treasury bills are currently paying 3.9 percent and the

If Treasury bills are currently paying 3.9 percent and the inflation rate is 2.1 percent, what is the approximate real rate of interest? The exact real rate?

See Answer

Q: In Problem 9, suppose the average inflation rate over this period

In Problem 9, suppose the average inflation rate over this period was 4.2 percent, and the average T-bill rate over the period was 5.1 percent. a. What was the average real return on the company’s sto...

See Answer

Q: Why does the value of a share of stock depend on dividends

Why does the value of a share of stock depend on dividends?

See Answer

Q: What happens to the price of a convertible bond if interest rates

What happens to the price of a convertible bond if interest rates increase?

See Answer

Q: You own a stock portfolio invested 15 percent in Stock Q,

You own a stock portfolio invested 15 percent in Stock Q, 35 percent in Stock R, 30 percent in Stock S, and 20 percent in Stock T. The betas for these four stocks are .75, 1.90, 1.38, and 1.16, respec...

See Answer

Q: Assume that the returns on individual securities are generated by the following

Assume that the returns on individual securities are generated by the following two-factor model: Here: Rα is the return on Security i at Time t. F1t and F2t are market factors with zero...

See Answer

Q: Kose, Inc., has a target debt–equity ratio of

Kose, Inc., has a target debt–equity ratio of .45. Its WACC is 9.8 percent, and the tax rate is 35 percent. a. If Kose’s cost of equity is 13 percent, what is its pretax cost of debt? b. If instead yo...

See Answer

Q: Overnight Publishing Company (OPC) has $2.5 million

Overnight Publishing Company (OPC) has $2.5 million in excess cash. The firm plans to use this cash either to retire all of its outstanding debt or to repurchase equity. The firm’s debt is held by one...

See Answer

Q: Triad Corporation has established a joint venture with Tobacco Road Construction,

Triad Corporation has established a joint venture with Tobacco Road Construction, Inc., to build a toll road in North Carolina. The initial investment in paving equipment is $93 million. The equipment...

See Answer

Q: The Mann Company belongs to a risk class for which the appropriate

The Mann Company belongs to a risk class for which the appropriate discount rate is 10 percent. Mann currently has 240,000 outstanding shares selling at $105 each. The firm is contemplating the declar...

See Answer

Q: Teardrop, Inc., wishes to expand its facilities. The company

Teardrop, Inc., wishes to expand its facilities. The company currently has 6.8 million shares outstanding and no debt. The stock sells for $65 per share, but the book value per share is $20. Net incom...

See Answer

Q: The Wildcat Oil Company is trying to decide whether to lease or

The Wildcat Oil Company is trying to decide whether to lease or buy a new computer-assisted drilling system for its oil exploration business. Management has decided that it must use the system to stay...

See Answer

Q: What are the prices of a call option and a put option

What are the prices of a call option and a put option with the following characteristics?

See Answer

Q: Suppose you are offered $9,400 today but must make

Suppose you are offered $9,400 today but must make the following payments: a. What is the IRR of this offer? b. If the appropriate discount rate is 10 percent, should you accept this offer? c. If th...

See Answer

Q: A project has perpetual cash flows of C per period, a

A project has perpetual cash flows of C per period, a cost of I, and a required return of R. What is the relationship between the project’s payback and its IRR? What implications does your answer have...

See Answer

Q: If a project with conventional cash flows has a payback period less

If a project with conventional cash flows has a payback period less than the project’s life, can you definitively state the algebraic sign of the NPV? Why or why not? If you know that the discounted p...

See Answer

Q: Looking back at the crossover bonds we discussed in the chapter,

Looking back at the crossover bonds we discussed in the chapter, why do you think split ratings such as these occur?

See Answer

Q: What is wrong with measuring the performance of a U.S

What is wrong with measuring the performance of a U.S. growth stock manager against a benchmark composed of British stocks?

See Answer

Q: What is the basic goal of financial management with regard to capital

What is the basic goal of financial management with regard to capital structure?

See Answer

Q: The following material represents the cover page and summary of the prospectus

The following material represents the cover page and summary of the prospectus for the initial public offering of the Pest Investigation Control Corporation (PICC), which is going public tomorrow with...

See Answer

Q: In 2003, Porsche unveiled its new sports utility vehicle (SUV

In 2003, Porsche unveiled its new sports utility vehicle (SUV), the Cayenne. With a price tag of over $40,000, the Cayenne goes from zero to 62 mph in 8.5 seconds. Porsche’s decision to enter the SUV...

See Answer

Q: Evaluate the following statement: Managers should not focus on the current

Evaluate the following statement: Managers should not focus on the current stock value because doing so will lead to an overemphasis on short-term profits at the expense of long-term profits.

See Answer

Q: Projects A and B have the following cash flows:

Projects A and B have the following cash flows: a. If the cash flows from the projects are identical, which of the two projects would have a higher IRR? Why? b. If C1B 5 2C1A, C2B 5 2C2A, and C3B 5...

See Answer

Q: Why is it that municipal bonds are not taxed at the federal

Why is it that municipal bonds are not taxed at the federal level, but are taxable across state lines? Why is it that U.S. Treasury bonds are not taxable at the state level? (You may need to dust off...

See Answer

Q: What are the comparative advantages of a competitive offer and a negotiated

What are the comparative advantages of a competitive offer and a negotiated offer, respectively?

See Answer

Q: The shareholders of Bryant Power Corp. need to elect three new

The shareholders of Bryant Power Corp. need to elect three new directors to the board. There are 16,500,000 shares of common stock outstanding, and the current share price is $13.75. If the company us...

See Answer

Q: If Wild Widgets, Inc., were an all-equity company

If Wild Widgets, Inc., were an all-equity company, it would have a beta of .95. The company has a target debt–equity ratio of .40. The expected return on the market portfolio is 11 percent, and Treasu...

See Answer

Q: What is the impact of lengthening the time to expiration on an

What is the impact of lengthening the time to expiration on an option’s value? Explain.

See Answer

Q: Which of the following should be treated as an incremental cash flow

Which of the following should be treated as an incremental cash flow when computing the NPV of an investment? a. A reduction in the sales of a company’s other products caused by the investment. b. An...

See Answer

Q: Universal Laser, Inc., just paid a dividend of $2

Universal Laser, Inc., just paid a dividend of $2.90 on its stock. The growth rate in dividends is expected to be a constant 6 percent per year, indefinitely. Investors require a 15 percent return on...

See Answer

Q: Given the following information for Huntington Power Co., find the WACC

Given the following information for Huntington Power Co., find the WACC. Assume the company’s tax rate is 35 percent. Debt: 10,000 5.6 percent coupon bonds outstanding, $1,000 par value, 25 years to m...

See Answer

Q: Williams Industries has decided to borrow money by issuing perpetual bonds with

Williams Industries has decided to borrow money by issuing perpetual bonds with a coupon rate of 6.5 percent, payable annually. The one-year interest rate is 6.5 percent. Next year, there is a 35 perc...

See Answer

Q: In the previous question, suppose the corporate tax rate is 35

In the previous question, suppose the corporate tax rate is 35 percent. What is EBIT in this case? What is the WACC? Explain.

See Answer

Q: For the company in the previous problem, what is the value

For the company in the previous problem, what is the value of being able to issue subsidized debt instead of having to issue debt at the terms it would normally receive? Assume the face amount and mat...

See Answer

Q: You own 1,000 shares of stock in Avondale Corporation.

You own 1,000 shares of stock in Avondale Corporation. You will receive a dividend of $2.60 per share in one year. In two years, Avondale will pay a liquidating dividend of $53 per share. The required...

See Answer

Q: The all-equity firm Metallica Heavy Metal Mining (MHMM)

The all-equity firm Metallica Heavy Metal Mining (MHMM) Corporation wants to diversify its operations. Some recent financial information for the company is shown here: MHMM is considering an investm...

See Answer

Q: The Wildcat Oil Company is trying to decide whether to lease or

The Wildcat Oil Company is trying to decide whether to lease or buy a new computer-assisted drilling system for its oil exploration business. Management has decided that it must use the system to stay...

See Answer

Q: Roll Corporation (RC) currently has 465,000 shares of

Roll Corporation (RC) currently has 465,000 shares of stock outstanding that sell for $73 per share. Assuming no market imperfections or tax effects exist, what will the share price be after: a. RC ha...

See Answer

Q: What are the prices of a call option and a put option

What are the prices of a call option and a put option with the following characteristics?

See Answer

Q: Rob Stevens is the chief executive officer of Isner Construction, Inc

Rob Stevens is the chief executive officer of Isner Construction, Inc., and owns 850,000 shares of stock. The company currently has 5.1 million shares of stock and convertible bonds with a face value...

See Answer

Q: Define the three forms of market efficiency.

Define the three forms of market efficiency.

See Answer

Q: Massey Machine Shop is considering a four-year project to improve

Massey Machine Shop is considering a four-year project to improve its production efficiency. Buying a new machine press for $730,000 is estimated to result in $270,000 in annual pretax cost savings. T...

See Answer

Q: Consider a project with a required return of R percent that costs

Consider a project with a required return of R percent that costs $I and will last for N years. The project uses straight-line depreciation to zero over the N-year life; there are neither salvage valu...

See Answer

Q: Given the information in Problem 10, what was the average real

Given the information in Problem 10, what was the average real risk-free rate over this time period? What was the average real risk premium?

See Answer

Q: You own a portfolio equally invested in a risk-free asset

You own a portfolio equally invested in a risk-free asset and two stocks. If one of the stocks has a beta of 1.73 and the total portfolio is equally as risky as the market, what must the beta be for t...

See Answer

Q: Consider the following cash flows on two mutually exclusive projects for the

Consider the following cash flows on two mutually exclusive projects for the Bahamas Recreation Corporation (BRC). Both projects require an annual return of 14 percent. As a financial analyst for BR...

See Answer

Q: Technical analysis is a controversial investment practice. Technical analysis covers a

Technical analysis is a controversial investment practice. Technical analysis covers a wide array of techniques, which are all used in an attempt to predict the direction of a particular stock or the...

See Answer

Q: What factors influence a firm’s choice of external versus internal equity financing

What factors influence a firm’s choice of external versus internal equity financing?

See Answer

Q: The Woods Co. and the Garcia Co. have both announced

The Woods Co. and the Garcia Co. have both announced IPOs at $40 per share. One of these is undervalued by $11, and the other is overvalued by $3, but you have no way of knowing which is which. You pl...

See Answer

Q: Historically, the U.S. tax code treated dividend payments

Historically, the U.S. tax code treated dividend payments made to shareholders as ordinary income. Thus, dividends were taxed at the investor’s marginal tax rate, which was as high as 38.6 percent in...

See Answer

Q: In April 2014, International Lease Finance Corporation (ILFC) announced

In April 2014, International Lease Finance Corporation (ILFC) announced a deal to purchase eight Airbus A330-200 and A350-900 passenger aircraft. ILFC then signed a long-term lease contract on the pla...

See Answer

Q: In 2003, Porsche unveiled its new sports utility vehicle (SUV

In 2003, Porsche unveiled its new sports utility vehicle (SUV), the Cayenne. With a price tag of over $40,000, the Cayenne goes from zero to 62 mph in 8.5 seconds. Porsche’s decision to enter the SUV...

See Answer

Q: What are the differences between preferred stock and debt?

What are the differences between preferred stock and debt?

See Answer

Q: You are evaluating Project A and Project B. Project A has

You are evaluating Project A and Project B. Project A has a short period of future cash flows, while Project B has relatively long future cash flows. Which project will be more sensitive to changes in...

See Answer

Q: Technical analysis is a controversial investment practice. Technical analysis covers a

Technical analysis is a controversial investment practice. Technical analysis covers a wide array of techniques, which are all used in an attempt to predict the direction of a particular stock or the...

See Answer

Q: Several publicly traded companies have issued more than one class of stock

Several publicly traded companies have issued more than one class of stock. Why might a company issue more than one class of stock?

See Answer

Q: What are the possible reasons why the stock price typically drops on

What are the possible reasons why the stock price typically drops on the announcement of a seasoned new equity issue?

See Answer

Q: Metallica Bearings, Inc., is a young start-up company

Metallica Bearings, Inc., is a young start-up company. No dividends will be paid on the stock over the next nine years, because the firm needs to plow back its earnings to fuel growth. The company wil...

See Answer

Q: The following Treasury bond quote appeared in The Wall Street Journal on

The following Treasury bond quote appeared in The Wall Street Journal on May 11, 2004: Why would anyone buy this Treasury bond with a negative yield to maturity? How is this possible?

See Answer

Q: You work for a nuclear research laboratory that is contemplating leasing a

You work for a nuclear research laboratory that is contemplating leasing a diagnostic scanner (leasing is a common practice with expensive, high-tech equipment). The scanner costs $5,800,000, and it w...

See Answer

Q: Weston Industries has a debt–equity ratio of 1.5

Weston Industries has a debt–equity ratio of 1.5. Its WACC is 10.5 percent, and its cost of debt is 6 percent. The corporate tax rate is 35 percent. a. What is the company’s cost of equity capital? b....

See Answer

Q: MVP, Inc., has produced rodeo supplies for over 20 years

MVP, Inc., has produced rodeo supplies for over 20 years. The company currently has a debt–equity ratio of 50 percent and is in the 40 percent tax bracket. The required return on the...

See Answer

Q: In the previous problem, suppose you want only $500 total

In the previous problem, suppose you want only $500 total in dividends the first year. What will your homemade dividend be in two years?

See Answer

Q: In the previous problem, what would the ROE on the investment

In the previous problem, what would the ROE on the investment have to be if we wanted the price after the offering to be $75 per share? (Assume the PE ratio remains constant.) What is the NPV of this...

See Answer

Q: What are the main features of a corporate bond that would be

What are the main features of a corporate bond that would be listed in the indenture?

See Answer

Q: In a world with no taxes, no transaction costs, and

In a world with no taxes, no transaction costs, and no costs of financial distress, is the following statement true, false, or uncertain? If a firm issues equity to repurchase some of its debt, the pr...

See Answer

Q: Monster Magnet Manufacturing is considering leasing some equipment. The annual lease

Monster Magnet Manufacturing is considering leasing some equipment. The annual lease payment would be $295,000 per year for six years. The appropriate interest rate is 7 percent and the company is in...

See Answer

Q: What are the deltas of a call option and a put option

What are the deltas of a call option and a put option with the following characteristics? What does the delta of the option tell you?

See Answer

Q: Bauble, Inc., an all-equity firm, has eight

Bauble, Inc., an all-equity firm, has eight shares of stock outstanding. Yesterday, the firm’s assets consisted of nine ounces of platinum, currently worth $1,650 per ounce. Today, the company issued...

See Answer

Q: Hagar Industrial Systems Company (HISC) is trying to decide between

Hagar Industrial Systems Company (HISC) is trying to decide between two different conveyor belt systems. System A costs $290,000, has a four-year life, and requires $89,000 in pretax annual operating...

See Answer

Q: The price of Ervin Corp. stock will be either $53

The price of Ervin Corp. stock will be either $53 or $67 at the end of the year. Call options are available with one year to expiration. T-bills currently yield 5 percent. a. Suppose the current price...

See Answer

Q: Consider a four-year project with the following information: Initial

Consider a four-year project with the following information: Initial fixed asset investment 5 $410,000; straight-line depreciation to zero over the four-year life; zero salvage value; price 5 $35; var...

See Answer

Q: A stock has had returns of 14.38 percent, 8

A stock has had returns of 14.38 percent, 8.43 percent, 11.97 percent, 25.83 percent, and −9.17 percent over the past five years, respectively. What was the holding period return for the stock?

See Answer

Q: Titan Mining Corporation has 8.7 million shares of common stock

Titan Mining Corporation has 8.7 million shares of common stock outstanding and 230,000 6.4 percent semiannual bonds outstanding, par value $1,000each. The common stock currently sells for $37 per sha...

See Answer

Q: The Cori’s Sausage Corporation is trying to choose between the following two

The Cori’s Sausage Corporation is trying to choose between the following two mutually exclusive design projects: a. If the required return is 10 percent and the company applies the...

See Answer

Q: What are the implications for bond investors of the lack of transparency

What are the implications for bond investors of the lack of transparency in the bond market?

See Answer

Q: Historically, the U.S. tax code treated dividend payments

Historically, the U.S. tax code treated dividend payments made to shareholders as ordinary income. Thus, dividends were taxed at the investor’s marginal tax rate, which was as high as 38.6 percent in...

See Answer

Q: Do you agree or disagree with the following statement? A firm’s

Do you agree or disagree with the following statement? A firm’s stockholders will never want the firm to invest in projects with negative net present values. Why?

See Answer

Q: In April 2014, International Lease Finance Corporation (ILFC) announced

In April 2014, International Lease Finance Corporation (ILFC) announced a deal to purchase eight Airbus A330-200 and A350-900 passenger aircraft. ILFC then signed a long-term lease contract on the pla...

See Answer

Q: What is the impact of an increase in the volatility of the

What is the impact of an increase in the volatility of the underlying stock’s return on an option’s value? Explain.

See Answer

Q: In 2003, Porsche unveiled its new sports utility vehicle (SUV

In 2003, Porsche unveiled its new sports utility vehicle (SUV), the Cayenne. With a price tag of over $40,000, the Cayenne goes from zero to 62 mph in 8.5 seconds. Porsche’s decision to enter the SUV...

See Answer

Q: Gary Levin is the chief executive officer of Mountain Brook Trading Company

Gary Levin is the chief executive officer of Mountain Brook Trading Company. The board of directors has just granted Mr. Levin 25,000 at-the-money European call options on the company’s stock, which i...

See Answer

Q: One of the less flattering interpretations of the acronym MIRR is “

One of the less flattering interpretations of the acronym MIRR is “meaningless internal rate of return.” Why do you think this term is applied to MIRR?

See Answer

Q: In the middle to late 1990s, the performance of the pros

In the middle to late 1990s, the performance of the pros was unusually poor—on the order of 90 percent of all equity mutual funds underperformed a passively managed index fund. How does this bear on t...

See Answer

Q: Do you agree or disagree with the following statement: In an

Do you agree or disagree with the following statement: In an efficient market, callable and noncallable bonds will be priced in such a way that there will be no advantage or disadvantage to the call p...

See Answer

Q: The growing perpetuity model expresses the value of a share of stock

The growing perpetuity model expresses the value of a share of stock as the present value of the expected dividends from that stock. How can you conclude that dividend policy is irrelevant when this m...

See Answer

Q: Megabucks Industries is planning to raise fresh equity capital by selling a

Megabucks Industries is planning to raise fresh equity capital by selling a large new issue of common stock. Megabucks, a publicly traded corporation, is trying to choose between an underwritten cash...

See Answer

Q: An insurance policy is considered analogous to an option. From the

An insurance policy is considered analogous to an option. From the policyholder’s point of view, what type of option is an insurance policy? Why?

See Answer

Q: Bucksnort, Inc., has an odd dividend policy. The company

Bucksnort, Inc., has an odd dividend policy. The company has just paid a dividend of $9 per share and has announced that it will increase the dividend by $4 per share for each of the next five years,...

See Answer

Q: Why are the costs of selling equity so much larger than the

Why are the costs of selling equity so much larger than the costs of selling debt?

See Answer

Q: Neon Corporation’s stock returns have a covariance with the market portfolio of

Neon Corporation’s stock returns have a covariance with the market portfolio of .0415. The standard deviation of the returns on the market portfolio is 20 percent, and the expected market risk premium...

See Answer

Q: Flychucker Corporation is evaluating an extra dividend versus a share repurchase.

Flychucker Corporation is evaluating an extra dividend versus a share repurchase. In either case $6,300 would be spent. Current earnings are $2.60 per share, and the stock currently sells for $51 per...

See Answer

Q: The Webber Company is an international conglomerate with a real estate division

The Webber Company is an international conglomerate with a real estate division that owns the right to erect an office building on a parcel of land in downtown Sacramento over the next year. This buil...

See Answer

Q: A company’s stock currently sells for $73 per share. Last

A company’s stock currently sells for $73 per share. Last week the firm issued rights to raise new equity. To purchase a new share, a stockholder must remit $14 and three rights. a. What is the ex-rig...

See Answer

Q: An asset costs $720,000 and will be depreciated in

An asset costs $720,000 and will be depreciated in a straight-line manner over its three-year life. It will have no salvage value. The corporate tax rate is 34 percent, and the appropriate interest ra...

See Answer

Q: You own a lot in Key West, Florida, that is

You own a lot in Key West, Florida, that is currently unused. Similar lots have recently sold for $1.3 million. Over the past five years, the price of land in the area has increased 12 percent per yea...

See Answer

Q: The capital structure of Ricketti Enterprises, Inc., consists of 25

The capital structure of Ricketti Enterprises, Inc., consists of 25 million shares of common stock and 1.5 million warrants. Each warrant gives its owner the right to purchase one share of common stoc...

See Answer

Q: Suppose in the previous problem that HISC always needs a conveyor belt

Suppose in the previous problem that HISC always needs a conveyor belt system; when one wears out, it must be replaced. Which system should the firm choose now?

See Answer

Q: You are considering a new product launch. The project will cost

You are considering a new product launch. The project will cost $760,000, have a four-year life, and have no salvage value; depreciation is straight-line to zero. Sales are projected at 420 units per...

See Answer

Q: You purchased a zero coupon bond one year ago for $160

You purchased a zero coupon bond one year ago for $160.53. The market interest rate is now 7.5 percent. If the bond had 25 years to maturity when you originally purchased it, what was your total retur...

See Answer

Q: A stock has an expected return of 13.4 percent,

A stock has an expected return of 13.4 percent, the risk-free rate is 3.8 percent, and the market risk premium is 7 percent. What must the beta of this stock be?

See Answer

Q: Complete the following sentence for each of these investors: a

Complete the following sentence for each of these investors: a. A buyer of call options. b. A buyer of put options. c. A seller (writer) of call options. d. A seller (writer) of put options. “The (buy...

See Answer

Q: An all-equity firm is considering the following projects:

An all-equity firm is considering the following projects: The T-bill rate is 3.5 percent, and the expected return on the market is 11 percent. a. Which projects have a higher expected return than th...

See Answer

Q: Hannon Home Products, Inc., recently issued $2 million worth

Hannon Home Products, Inc., recently issued $2 million worth of 3 percent convertible debentures. Each convertible bond has a face value of $1,000. Each convertible bond can be converted into 23.50 sh...

See Answer

Q: Shadow Corp. has no debt but can borrow at 6.

Shadow Corp. has no debt but can borrow at 6.5 percent. The firm’s WACC is currently 9.8 percent, and the tax rate is 35 percent. a. What is the company’s cost of equity? b. If the company converts to...

See Answer

Q: Cutler Petroleum, Inc., is trying to evaluate a generation project

Cutler Petroleum, Inc., is trying to evaluate a generation project with the following cash flows: a. If the company requires a 10 percent return on its investments, should it accept this project? Wh...

See Answer

Q: Take a look back at Figure 8.4(given below

Take a look back at Figure 8.4(given below). Notice the wide range of coupon rates. Why are they so different?

See Answer

Q: It is sometimes stated that “the net present value approach assumes

It is sometimes stated that “the net present value approach assumes reinvestment of the intermediate cash flows at the required return.” Is this claim correct? To answer, suppose you calculate the NPV...

See Answer

Q: A hundred years ago or so, companies did not compile annual

A hundred years ago or so, companies did not compile annual reports. Even if you owned stock in a particular company, you were unlikely to be allowed to see the balance sheet and income statement for...

See Answer

Q: If interest rates fall, will the price of noncallable bonds move

If interest rates fall, will the price of noncallable bonds move up higher than that of callable bonds? Why or why not?

See Answer

Q: It is said that the equity holders of a levered firm can

It is said that the equity holders of a levered firm can be thought of as holding a call option on the firm’s assets. Explain what is meant by this statement.

See Answer

Q: Lohn Corporation is expected to pay the following dividends over the next

Lohn Corporation is expected to pay the following dividends over the next four years: $13, $8, $6.50, and $2.40. Afterwards, the company pledges to maintain a constant 4.5 percent growth rate in divid...

See Answer

Q: Suppose your company needs $35 million to build a new assembly

Suppose your company needs $35 million to build a new assembly line. Your target debt–equity ratio is .75. The flotation cost for new equity is 6 percent, but the flotation cost for debt is only 2 per...

See Answer

Q: Explain the following limits on the prices of warrants: a

Explain the following limits on the prices of warrants: a. If the stock price is below the exercise price of the warrant, the lower bound on the price of a warrant is zero. b. If the stock price is ab...

See Answer

Q: In the previous problem, suppose the project requires an initial investment

In the previous problem, suppose the project requires an initial investment in net working capital of $285,000 and the fixed asset will have a market value of $225,000 at the end of the project. What...

See Answer

Q: Dorman Industries has a new project available that requires an initial investment

Dorman Industries has a new project available that requires an initial investment of $4.3 million. The project will provide unlevered cash flows of $710,000 per year for the next 20 years. The company...

See Answer

Q: The net income of Novis Corporation is $85,000.

The net income of Novis Corporation is $85,000. The company has 25,000 outstanding shares and a 100 percent payout policy. The expected value of the firm one year from now is $1,725,000. The appropria...

See Answer

Q: Valley Corp.’s stock is currently selling at $37 per

Valley Corp.’s stock is currently selling at $37 per share. There are 1 million shares outstanding. The firm is planning to raise $2.5 million to finance a new project. What are the ex-rights stock pr...

See Answer

Q: Wolfson Corporation has decided to purchase a new machine that costs $

Wolfson Corporation has decided to purchase a new machine that costs $2.8 million. The machine will be depreciated on a straight-line basis and will be worthless after four years. The corporate tax ra...

See Answer

Q: In the previous problem, suppose you wanted the option to sell

In the previous problem, suppose you wanted the option to sell the land to the buyer in one year. Assuming all the facts are the same, describe the transaction that would occur today. What is the pric...

See Answer

Q: You have been hired to value a new 20-year callable

You have been hired to value a new 20-year callable, convertible bond. The bond has a 5.8 percent coupon rate, payable annually. The conversion price is $150, and the stock currently sells for $32.20....

See Answer

Q: Vandalay Industries is considering the purchase of a new machine for the

Vandalay Industries is considering the purchase of a new machine for the production of latex. Machine A costs $3,100,000 and will last for six years. Variable costs are 35 percent of sales, and fixed...

See Answer

Q: McGilla Golf has decided to sell a new line of golf clubs

McGilla Golf has decided to sell a new line of golf clubs. The clubs will sell for $850 per set and have a variable cost of $430 per set. The company has spent $150,000 for a marketing study that dete...

See Answer

Q: Locate the Treasury bond in Figure 8.4(given below

Locate the Treasury bond in Figure 8.4(given below) maturing in February 2037. What is its coupon rate? What is its bid price? What was the previous day’s asked price? Assume a par v...

See Answer

Q: You bought a share of 3.5 percent preferred stock for

You bought a share of 3.5 percent preferred stock for $92.07 last year. The market price for your stock is now $96.12. What was your total return for last year?

See Answer

Q: L.J.’s Toys, Inc., just purchased a

L.J.’s Toys, Inc., just purchased a $375,000 machine to produce toy cars. The machine will be fully depreciated by the straight-line method over its five-year economic life. Each toy sells for $21. Th...

See Answer

Q: The next dividend payment by ECY, Inc., will be $

The next dividend payment by ECY, Inc., will be $2.90 per share. The dividends are anticipated to maintain a growth rate of 5.5 percent, forever. If the stock currently sells for $53.10 per share, wha...

See Answer

Q: Wii Brothers, a game manufacturer, has a new idea for

Wii Brothers, a game manufacturer, has a new idea for an adventure game. It can market the game either as a traditional board game or as an interactive DVD, but not both. Consider the following cash f...

See Answer

Q: The bird-in-the-hand argument, which states

The bird-in-the-hand argument, which states that a dividend today is safer than the uncertain prospect of a capital gain tomorrow, is often used to justify high dividend payout ratios. Explain the fal...

See Answer

Q: Explain why shelf registration has been used by many firms instead of

Explain why shelf registration has been used by many firms instead of syndication.

See Answer

Q: A controversy erupted regarding bond-rating agencies when some agencies began

A controversy erupted regarding bond-rating agencies when some agencies began to provide unsolicited bond ratings. Why do you think this is controversial?

See Answer

Q: It is sometimes stated that “the internal rate of return approach

It is sometimes stated that “the internal rate of return approach assumes reinvestment of the intermediate cash flows at the internal rate of return.” Is this claim correct? To answer, suppose you cal...

See Answer

Q: Aerotech, an aerospace technology research firm, announced this morning that

Aerotech, an aerospace technology research firm, announced this morning that it has hired the world’s most knowledgeable and prolific space researchers. Before today, Aerotech’s stock had been selling...

See Answer

Q: Sinking funds have both positive and negative characteristics for bondholders. Why

Sinking funds have both positive and negative characteristics for bondholders. Why?

See Answer

Q: The desire for current income is not a valid explanation of preference

The desire for current income is not a valid explanation of preference for high current dividend policy because investors can always create homemade dividends by selling a portion of their stocks. Is...

See Answer

Q: Every IPO is unique, but what are the basic empirical regularities

Every IPO is unique, but what are the basic empirical regularities in IPOs?

See Answer

Q: Suppose the company in Problem 1 has a market-to-

Suppose the company in Problem 1 has a market-to-book ratio of 1.0. a. Calculate return on equity, ROE, under each of the three economic scenarios before any debt is issued. Also calculate the percent...

See Answer

Q: Rhiannon Corporation has bonds on the market with 11.5 years

Rhiannon Corporation has bonds on the market with 11.5 years to maturity, a YTM of 7.3 percent, and a current price of $1,080. The bonds make semiannual payments. What must the coupon rate be on these...

See Answer

Q: You are the CEO of Titan Industries and have just been awarded

You are the CEO of Titan Industries and have just been awarded a large number of employee stock options. The company has two mutually exclusive projects available. The first project has a large NPV an...

See Answer

Q: Tom Scott is the owner, president, and primary salesperson for

Tom Scott is the owner, president, and primary salesperson for Scott Manufacturing. Because of this, the company’s profits are driven by the amount of work Tom does. If he works 40 hours each week, th...

See Answer

Q: Phillips Co. is growing quickly. Dividends are expected to grow

Phillips Co. is growing quickly. Dividends are expected to grow at a rate of 25 percent for the next three years, with the growth rate falling off to a constant 5 percent thereafter. If the required r...

See Answer

Q: Newkirk, Inc., is an unlevered firm with expected annual earnings

Newkirk, Inc., is an unlevered firm with expected annual earnings before taxes of $21 million in perpetuity. The current required return on the firm’s equity is 16 percent, and the firm distributes al...

See Answer

Q: Gibson Co. has a current period cash flow of $1

Gibson Co. has a current period cash flow of $1.3 million and pays no dividends. The present value of the company’s future cash flows is $18 million. The company is entirely financed with equity and h...

See Answer

Q: Hoobastink Mfg. is considering a rights offer. The company has

Hoobastink Mfg. is considering a rights offer. The company has determined that the ex-rights price will be $61. The current price is $68 per share, and there are 10 million shares outstanding. The rig...

See Answer

Q: An asset costs $590,000 and will be depreciated in

An asset costs $590,000 and will be depreciated in a straight-line manner over its three-year life. It will have no salvage value. The lessor can borrow at 7 percent and the lessee can borrow at 9 per...

See Answer

Q: You are given the following information concerning options on a particular stock

You are given the following information concerning options on a particular stock: a. What is the intrinsic value of the call option? Of the put option? b. What is the time value of the call option?...

See Answer

Q: Superior Clamps, Inc., has a capital structure consisting of 7

Superior Clamps, Inc., has a capital structure consisting of 7 million shares of common stock and 900,000 warrants. Each warrant gives its owner the right to purchase one share of newly issued common...

See Answer

Q: Consider the following cash flows on two mutually exclusive projects:

Consider the following cash flows on two mutually exclusive projects: The cash flows of Project A are expressed in real terms, whereas those of Project B are expressed in nominal terms. The appropri...

See Answer

Q: Suppose you bought a bond with a 5.8 percent coupon

Suppose you bought a bond with a 5.8 percent coupon rate one year ago for $1,030. The bond sells for $1,059 today. a. Assuming a $1,000 face value, what was your total dollar return on this investment...

See Answer

Q: In the previous problem, you feel that the values are accurate

In the previous problem, you feel that the values are accurate to within only 610 percent. What are the best-case and worst-case NPVs? (Hint: The price and variable costs for the two existing sets of...

See Answer

Q: Locate the Treasury bond in Figure 8.4(given below

Locate the Treasury bond in Figure 8.4(given below) maturing in November 2039. Is this a premium or a discount bond? What is its current yield? What is its yield to maturity? What is the bid-ask sprea...

See Answer

Q: Gemini, Inc., an all-equity firm, is considering

Gemini, Inc., an all-equity firm, is considering an investment of $1.4 million that will be depreciated according to the straight-line method over its four-year life. The project is expected to genera...

See Answer

Q: You bought a stock three months ago for $62.18

You bought a stock three months ago for $62.18 per share. The stock paid no dividends. The current share price is $65.37. What is the APR of your investment? The EAR?

See Answer

Q: Southern Alliance Company needs to raise $55 million to start a

Southern Alliance Company needs to raise $55 million to start a new project and will raise the money by selling new bonds. The company will generate no internal equity for the foreseeable future. The...

See Answer

Q: In Problem 14, what is the cost of equity after recapitalization

In Problem 14, what is the cost of equity after recapitalization? What is the WACC? What are the implications for the firm’s capital structure decision? Problem 14: Bruce & Co. expects its EBIT to be...

See Answer

Q: Hanmi Group, a consumer electronics conglomerate, is reviewing its annual

Hanmi Group, a consumer electronics conglomerate, is reviewing its annual budget in wireless technology. It is considering investments in three different technologies to develop wireless communication...

See Answer

Q: The 100-year bonds we discussed in the chapter have something

The 100-year bonds we discussed in the chapter have something in common with junk bonds. Critics charge that, in both cases, the issuers are really selling equity in disguise. What are the issues here...

See Answer

Q: When the 56-year-old founder of Gulf & Western

When the 56-year-old founder of Gulf & Western, Inc., died of a heart attack, the stock price immediately jumped from $18.00 a share to $20.25, a 12.5 percent increase. This is evidence of market inef...

See Answer

Q: Cap Henderson owns Neotech stock because its price has been steadily rising

Cap Henderson owns Neotech stock because its price has been steadily rising over the past few years and he expects this performance to continue. Cap is trying to convince Sarah Jones to purchase some...

See Answer

Q: You have $10,000 to invest in a stock portfolio

You have $10,000 to invest in a stock portfolio. Your choices are Stock X with an expected return of 13 percent and Stock Y with an expected return of 8.5 percent. If your goal is to create a portfoli...

See Answer

Q: You find a put and a call with the same exercise price

You find a put and a call with the same exercise price and maturity. What do you know about the relative prices of the put and call? Prove your answer and provide an intuitive explanation.

See Answer

Q: Synovec Corp. is experiencing rapid growth. Dividends are expected to

Synovec Corp. is experiencing rapid growth. Dividends are expected to grow at 30 percent per year during the next three years, 18 percent over the following year, and then 8 percent per year indefinit...

See Answer

Q: A stock has a beta of 1.13 and an expected

A stock has a beta of 1.13 and an expected return of 12.1 percent. A risk-free asset currently earns 3.6 percent. a. What is the expected return on a portfolio that is equally invested in the two asse...

See Answer

Q: The owners’ equity accounts for Hexagon International are shown here:

The owners’ equity accounts for Hexagon International are shown here: a. If the company’s stock currently sells for $39 per share and a 10 percent stock dividend...

See Answer

Q: Och, Inc., is considering a project that will result in

Och, Inc., is considering a project that will result in initial aftertax cash savings of $2.9 million at the end of the first year, and these savings will grow at a rate of 4 percent per year indefini...

See Answer

Q: Levered, Inc., and Unlevered, Inc., are identical in

Levered, Inc., and Unlevered, Inc., are identical in every way except their capital structures. Each company expects to earn $23 million before interest per year in perpetuity, with each company distr...

See Answer

Q: Mojito Mint Company has a debt–equity ratio of .35

Mojito Mint Company has a debt–equity ratio of .35. The required return on the company’s unlevered equity is 12.8 percent, and the pretax cost of the firm’s debt is 6.5 percent. Sales revenue for the...

See Answer

Q: The Sharpe Co. just paid a dividend of $1.

The Sharpe Co. just paid a dividend of $1.60 per share of stock. Its target payout ratio is 40 percent. The company expects to have earnings per share of $5.10 one year from now. a. If the adjustment...

See Answer

Q: Show that the value of a right can be written as:

Show that the value of a right can be written as: where PRO, PS, and PX stand for the “rights-on” price, the subscription price, and the ex-rights price, respecti...

See Answer

Q: Automobiles are often leased, and there are several terms unique to

Automobiles are often leased, and there are several terms unique to auto leases. Suppose you are considering leasing a car. The price you and the dealer agree on for the car is $32,000. This is the ba...

See Answer

Q: Suppose stock returns can be explained by the following three-factor

Suppose stock returns can be explained by the following three-factor model: Assume there is no firm-specific risk. The information for each stock is presented here: The risk premiums for the factor...

See Answer

Q: A stock is currently priced at $84. The stock will

A stock is currently priced at $84. The stock will either increase or decrease by 17 percent over the next year. There is a call option on the stock with a strike price of $80 and one year until expir...

See Answer

Q: Omega Airline’s capital structure consists of 2.7 million shares of

Omega Airline’s capital structure consists of 2.7 million shares of common stock and zero coupon bonds with a face value of $18 million that mature in six months. The firm just announced that it will...

See Answer

Q: Sparkling Water, Inc., expects to sell 2.7 million

Sparkling Water, Inc., expects to sell 2.7 million bottles of drinking water each year in perpetuity. This year each bottle will sell for $1.35 in real terms and will cost $.85 in real terms. Sales in...

See Answer

Q: McGilla Golf would like to know the sensitivity of NPV to changes

McGilla Golf would like to know the sensitivity of NPV to changes in the price of the new clubs and the quantity of new clubs sold. What is the sensitivity of the NPV to each of these variables?

See Answer

Q: The Clifford Corporation has announced a rights offer to raise $26 

The Clifford Corporation has announced a rights offer to raise $26 million for a new journal, the Journal of Financial Excess. This journal will review potential articles after the author pays a nonre...

See Answer

Q: You buy a zero coupon bond at the beginning of the year

You buy a zero coupon bond at the beginning of the year that has a face value of $1,000, a YTM of 6.3 percent, and 25 years to maturity. If you hold the bond for the entire year, how much in interest...

See Answer

Q: Refer to Table 10.1. What was the average real

Refer to Table 10.1. What was the average real return for Treasury bills from 1926 through 1932?

See Answer

Q: Consider the following cash flows of two mutually exclusive projects for AZ

Consider the following cash flows of two mutually exclusive projects for AZ-Motorcars. Assume the discount rate for both projects is 10 percent. a. Based on the payback period, which project should...

See Answer

Q: a. What is the relationship between the price of a bond

a. What is the relationship between the price of a bond and its YTM? b. Explain why some bonds sell at a premium over par value while other bonds sell at a discount. What do you know about the relatio...

See Answer

Q: Today, the following announcement was made: “Early today the

Today, the following announcement was made: “Early today the Justice Department reached a decision in the Universal Product Care (UPC) case. UPC has been found guilty of discriminatory practices in hi...

See Answer

Q: For the firm in the previous problem, suppose the book value

For the firm in the previous problem, suppose the book value of the debt issue is $35 million. In addition, the company has a second debt issue on the market, a zero coupon bond with 12 years left to...

See Answer

Q: A put and a call have the same maturity and strike price

A put and a call have the same maturity and strike price. If they have the same price, which one is in the money? Prove your answer and provide an intuitive explanation.

See Answer

Q: Antiques R Us is a mature manufacturing firm. The company just

Antiques R Us is a mature manufacturing firm. The company just paid a dividend of $13, but management expects to reduce the payout by 4 percent per year, indefinitely. If you require a return of 10 pe...

See Answer

Q: Refer back to Table 10.2. What range of returns

Refer back to Table 10.2. What range of returns would you expect to see 68 percent of the time for long-term corporate bonds? What about 95 percent of the time?

See Answer

Q: Asset W has an expected return of 11.9 percent and

Asset W has an expected return of 11.9 percent and a beta of 1.2. If the risk-free rate is 4 percent, complete the following table for portfolios of Asset W and a risk-free asset. Illustrate the relat...

See Answer

Q: The Saunders Investment Bank has the following financing outstanding. What is

The Saunders Investment Bank has the following financing outstanding. What is the WACC for the company? Debt: 50,000 bonds with a coupon rate of 5.7 percent and a current price quote of 106.5; the bon...

See Answer

Q: List the three assumptions that lie behind the Modigliani–Miller theory

List the three assumptions that lie behind the Modigliani–Miller theory in a world without taxes. Are these assumptions reasonable in the real world? Explain.

See Answer

Q: Use the option quote information shown here to answer the questions that

Use the option quote information shown here to answer the questions that follow. The stock is currently selling for $83. a. Are the call options in the money? What is the intrinsic value of an RWJ C...

See Answer

Q: Tool Manufacturing has an expected EBIT of $67,000 in

Tool Manufacturing has an expected EBIT of $67,000 in perpetuity and a tax rate of 35 percent. The firm has $130,000 in outstanding debt at an interest rate of 8 percent, and its unlevered cost of cap...

See Answer

Q: Bruin Industries just issued $265,000 of perpetual 8 percent

Bruin Industries just issued $265,000 of perpetual 8 percent debt and used the proceeds to repurchase stock. The company expects to generate $123,000 of earnings before interest and taxes in perpetuit...

See Answer

Q: The Gecko Company and the Gordon Company are two firms whose business

The Gecko Company and the Gordon Company are two firms whose business risk is the same but that have different dividend policies. Gecko pays no dividend, whereas Gordon has an expected dividend yield...

See Answer

Q: Beasley, Inc. is going to elect nine board members next

Beasley, Inc. is going to elect nine board members next month. Betty Brown owns 12.4 percent of the total shares outstanding. How confident can she be of having one of her candidate friends elected un...

See Answer

Q: Wuttke Corp. wants to raise $5,375,000

Wuttke Corp. wants to raise $5,375,000 via a rights offering. The company currently has 950,000 shares of common stock outstanding that sell for $55 per share. Its underwriter has set a subscription p...

See Answer

Q: Return to the case of the diagnostic scanner discussed in Problems 1

Return to the case of the diagnostic scanner discussed in Problems 1 through 6. Suppose the entire $5,800,000 purchase price of the scanner is borrowed. The rate on the loan is 8 percent, and the loan...

See Answer

Q: In the previous problem, assume the risk-free rate is

In the previous problem, assume the risk-free rate is only 5 percent. What is the risk-neutral value of the option now? What happens to the risk-neutral probabilities of a stock price increase and a s...

See Answer

Q: Etonic, Inc., is considering an investment of $395,

Etonic, Inc., is considering an investment of $395,000 in an asset with an economic life of five years. The firm estimates that the nominal annual cash revenues and expenses at the end of the first ye...

See Answer

Q: We are examining a new project. We expect to sell 7

We are examining a new project. We expect to sell 7,000 units per year at $38 net cash flow apiece for the next 10 years. In other words, the annual operating cash flow is projected to be $38 3 7,000...

See Answer

Q: Miller Corporation has a premium bond making semiannual payments. The bond

Miller Corporation has a premium bond making semiannual payments. The bond pays a coupon of 8.5 percent, has a YTM of 7 percent, and has 13 years to maturity. The Modigliani Company has a discount bon...

See Answer

Q: The treasurer of Amaro Canned Fruits, Inc., has projected the

The treasurer of Amaro Canned Fruits, Inc., has projected the cash flows of Projects A, B, and C as follows: Suppose the relevant discount rate is 12 percent per year. a. Compute the profitability i...

See Answer

Q: Jared Lazarus has just been named the new chief executive officer of

Jared Lazarus has just been named the new chief executive officer of BluBell Fitness Centers, Inc. In addition to an annual salary of $475,000, his three-year contract states that his compensation wil...

See Answer

Q: Your aunt is in a high tax bracket and would like to

Your aunt is in a high tax bracket and would like to minimize the tax burden of her investment portfolio. She is willing to buy and sell to maximize her aftertax returns, and she has asked for your ad...

See Answer

Q: All else being the same, which has more interest rate risk

All else being the same, which has more interest rate risk, a long-term bond or a short-term bond? What about a low coupon bond compared to a high coupon bond? What about a long-term, high coupon bond...

See Answer

Q: Flatte Restaurant is considering the purchase of a $7,500

Flatte Restaurant is considering the purchase of a $7,500 soufflé maker. The soufflé maker has an economic life of five years and will be fully depreciated by the straight-line method. The machine wil...

See Answer

Q: If the market places the same value on $1 of dividends

If the market places the same value on $1 of dividends as on $1 of capital gains, then firms with different payout ratios will appeal to different clienteles of investors. One clientele is as good as...

See Answer

Q: Mau Corporation stock currently sells for $64.87 per share

Mau Corporation stock currently sells for $64.87 per share. The market requires a return of 10.5 percent on the firm’s stock. If the company maintains a constant 5 percent growth rate in dividends, wh...

See Answer

Q: Laurel, Inc., and Hardy Corp. both have 6.

Laurel, Inc., and Hardy Corp. both have 6.5 percent coupon bonds outstanding, with semiannual interest payments, and both are priced at par value. The Laurel, Inc., bond has 3 years to maturity, where...

See Answer

Q: Refer back to Table 10.2(given below). What

Refer back to Table 10.2(given below). What range of returns would you expect to see 68 percent of the time for large-company stocks? What about 95 percent of the time?

See Answer

Q: Stock Y has a beta of 1.20 and an expected

Stock Y has a beta of 1.20 and an expected return of 12.7 percent. Stock Z has a beta of .90 and an expected return of 11.1 percent. If the risk-free rate is 4.5 percent and the market risk premium is...

See Answer

Q: Goodbye, Inc., recently issued new securities to finance a new

Goodbye, Inc., recently issued new securities to finance a new TV show. The project cost $19 million, and the company paid $1,150,000 in flotation costs. In addition, the equity issued had a flotation...

See Answer

Q: Cavo Corporation expects an EBIT of $26,850 every year

Cavo Corporation expects an EBIT of $26,850 every year forever. The company currently has no debt, and its cost of equity is 14 percent. The tax rate is 35 percent. a. What is the current value of the...

See Answer

Q: Blue Angel, Inc., a private firm in the holiday gift

Blue Angel, Inc., a private firm in the holiday gift industry, is considering a new project. The company currently has a target debt–equity ratio of .40, but the industry target debt–equity ratio is ....

See Answer

Q: The Best Manufacturing Company is considering a new investment. Financial projections

The Best Manufacturing Company is considering a new investment. Financial projections for the investment are tabulated here. The corporate tax rate is 34 percent. Assume all sales revenue is received...

See Answer

Q: As discussed in the text, in the absence of market imperfections

As discussed in the text, in the absence of market imperfections and tax effects, we would expect the share price to decline by the amount of the dividend payment when the stock goes ex dividend. Once...

See Answer

Q: Franklin Corporation is comparing two different capital structures, an all-

Franklin Corporation is comparing two different capital structures, an all-equity plan (Plan I) and a levered plan (Plan II). Under Plan I, the company would have 315,000 shares of stock outstanding....

See Answer

Q: Mitsi Inventory Systems, Inc., has announced a rights offer.

Mitsi Inventory Systems, Inc., has announced a rights offer. The company has announced that it will take four rights to buy a new share in the offering at a subscription price of $30. At the close of...

See Answer

Q: High electricity costs have made Farmer Corporation’s chicken-plucking machine economically

High electricity costs have made Farmer Corporation’s chicken-plucking machine economically worthless. Only two machines are available to replace it. The International Plucking Machine (IPM) model is...

See Answer

Q: Phillips Industries runs a small manufacturing operation. For this fiscal year

Phillips Industries runs a small manufacturing operation. For this fiscal year, it expects real net cash flows of $235,000. The company is an ongoing operation, but it expects competitive pressures to...

See Answer

Q: In the previous problem, suppose you think it is likely that

In the previous problem, suppose you think it is likely that expected sales will be revised upward to 9,500 units if the first year is a success and revised downward to 3,800 units if the first year i...

See Answer

Q: Consider the following cash flows of two mutually exclusive projects for Tokyo

Consider the following cash flows of two mutually exclusive projects for Tokyo Rubber Company. Assume the discount rate for both projects is 8 percent. a. Based on the payback period, which project...

See Answer

Q: A call option matures in six months. The underlying stock price

A call option matures in six months. The underlying stock price is $75, and the stock’s return has a standard deviation of 30 percent per year. The risk-free rate is 4 percent per year, compounded con...

See Answer

Q: One thing put–call parity tells us is that given any

One thing put–call parity tells us is that given any three of a stock, a call, a put, and a T-bill, the fourth can be synthesized or replicated using the other three. For example, how can we replicate...

See Answer

Q: Newtech Corp. is going to adopt a new chip-testing

Newtech Corp. is going to adopt a new chip-testing device that can greatly improve its production efficiency. Do you think the lead engineer can profit from purchasing the firm’s stock before the news...

See Answer

Q: In spite of the theoretical argument that dividend policy should be irrelevant

In spite of the theoretical argument that dividend policy should be irrelevant, the fact remains that many investors like high dividends. If this preference exists, a firm can boost its share price by...

See Answer

Q: In the previous problem, suppose the projections given for price,

In the previous problem, suppose the projections given for price, quantity, variable costs, and fixed costs are all accurate to within 610 percent. Calculate the best-case and worst-case NPV figures....

See Answer

Q: Suppose the president of the company in the previous problem stated that

Suppose the president of the company in the previous problem stated that the company should increase the amount of debt in its capital structure because of the tax-advantaged status of its interest pa...

See Answer

Q: Fifth National Bank just issued some new preferred stock. The issue

Fifth National Bank just issued some new preferred stock. The issue will pay an annual dividend of $5 in perpetuity, beginning five years from now. If the market requires a return of 4.7 percent on th...

See Answer

Q: The Faulk Corp. has a 6 percent coupon bond outstanding.

The Faulk Corp. has a 6 percent coupon bond outstanding. The Gonas Company has a 14 percent bond outstanding. Both bonds have 12 years to maturity, make semiannual payments, and have a YTM of 10 perce...

See Answer

Q: You find a certain stock that had returns of 12 percent,

You find a certain stock that had returns of 12 percent, 215 percent, 13 percent, and 27 percent for four of the last five years. If the average return of the stock over this period was 10.5 percent,...

See Answer

Q: In the previous problem, what would the risk-free rate

In the previous problem, what would the risk-free rate have to be for the two stocks to be correctly priced?

See Answer

Q: Floyd Industries stock has a beta of 1.15. The

Floyd Industries stock has a beta of 1.15. The company just paid a dividend of $.85, and the dividends are expected to grow at 4.5 percent per year. The expected return on the market is 11 percent, an...

See Answer

Q: The Dart Company is financed entirely with equity. The company is

The Dart Company is financed entirely with equity. The company is considering a loan of $2.6 million. The loan will be repaid in equal installments over the next two years, and it has an interest rate...

See Answer

Q: National Business Machine Co. (NBM) has $4.

National Business Machine Co. (NBM) has $4.5 million of extra cash after taxes have been paid. NBM has two choices to make use of this cash. One alternative is to invest the cash in financial assets....

See Answer

Q: Bridgton Golf Academy is evaluating new golf practice equipment. The “

Bridgton Golf Academy is evaluating new golf practice equipment. The “Dimple-Max” equipment costs $64,000, has a three-year life, and costs $7,500 per year to operate. The relevant discount rate is 12...

See Answer

Q: In the previous problem, suppose the scale of the project can

In the previous problem, suppose the scale of the project can be doubled in one year in the sense that twice as many units can be produced and sold. Naturally, expansion would be desirable only if the...

See Answer

Q: Consider two mutually exclusive new product launch projects that Nagano Golf is

Consider two mutually exclusive new product launch projects that Nagano Golf is considering. Assume the discount rate for both products is 15 percent. Project A: Nagano NP-30. Professional clubs that...

See Answer

Q: An investment project costs $17,500 and has annual cash

An investment project costs $17,500 and has annual cash flows of $4,300 for six years. What is the discounted payback period if the discount rate is 0 percent? What if the discount rate is 10 percent?...

See Answer

Q: Microhard has issued a bond with the following characteristics: Par

Microhard has issued a bond with the following characteristics: Par: $1,000 Time to maturity: 20 years Coupon rate: 7 percent Semiannual payments Calculate the price of this bond if the YTM is: a. 7 p...

See Answer

Q: TransTrust Corp. has changed how it accounts for inventory. Taxes

TransTrust Corp. has changed how it accounts for inventory. Taxes are unaffected, although the resulting earnings report released this quarter is 20 percent higher than what it would have been under t...

See Answer

Q: You have found the following stock quote for RJW Enterprises, Inc

You have found the following stock quote for RJW Enterprises, Inc., in the financial pages of today’s newspaper. What is the annual dividend? What was the closing price for this stoc...

See Answer

Q: Your buddy comes to you with a sure-fire way to

Your buddy comes to you with a sure-fire way to make some quick money and help pay off your student loans. His idea is to sell T-shirts with the words “I get” on them. “You get it?” He says, “You see...

See Answer

Q: Hacker Software has 6.2 percent coupon bonds on the market

Hacker Software has 6.2 percent coupon bonds on the market with 9 years to maturity. The bonds make semiannual payments and currently sell for 104 percent of par. What is the current yield on the bond...

See Answer

Q: A stock has had returns of 24 percent, 12 percent,

A stock has had returns of 24 percent, 12 percent, 38 percent, 22 percent, 21 percent, and 216 percent over the last six years. What are the arithmetic and geometric returns for the stock?

See Answer

Q: Schultz Industries is considering the purchase of Arras Manufacturing. Arras is

Schultz Industries is considering the purchase of Arras Manufacturing. Arras is currently a supplier for Schultz, and the acquisition would allow Schultz to better control its material supply. The cur...

See Answer

Q: Alpha Corporation and Beta Corporation are identical in every way except their

Alpha Corporation and Beta Corporation are identical in every way except their capital structures. Alpha Corporation, an all-equity firm, has 18,000 shares of stock outstanding, currently worth $35 pe...

See Answer

Q: After completing its capital spending for the year, Carlson Manufacturing has

After completing its capital spending for the year, Carlson Manufacturing has $1,000 extra cash. Carlson’s managers must choose between investing the cash in Treasury bonds that yield 8 percent or pay...

See Answer

Q: RightPrice Investors, Inc., is considering the purchase of a $

RightPrice Investors, Inc., is considering the purchase of a $415,000 computer with an economic life of five years. The computer will be fully depreciated over five years using the straight-line metho...

See Answer

Q: Dream, Inc., has debt outstanding with a face value of

Dream, Inc., has debt outstanding with a face value of $5 million. The value of the firm if it were entirely financed by equity would be $18.65 million. The company also has 360,000 shares of stock ou...

See Answer

Q: Treasury bid and ask quotes are sometimes given in terms of yields

Treasury bid and ask quotes are sometimes given in terms of yields, so there would be a bid yield and an ask yield. Which do you think would be larger? Explain.

See Answer

Q: You are evaluating a project that costs $75,000 today

You are evaluating a project that costs $75,000 today. The project has an inflow of $155,000 in one year and an outflow of $65,000 in two years. What are the IRRs for the project? What discount rate r...

See Answer

Q: In Problem 1, what was the dividend yield? The capital

In Problem 1, what was the dividend yield? The capital gains yield?

See Answer

Q: A stock is currently priced at $35. A call option

A stock is currently priced at $35. A call option with an expiration of one year has an exercise price of $50. The risk-free rate is 7 percent per year, compounded continuously, and the standard devia...

See Answer

Q: Empirical research has found that there have been significant increases in stock

Empirical research has found that there have been significant increases in stock price on the day an initial dividend (i.e., the first time a firm pays a cash dividend) is announced. What does this fi...

See Answer

Q: The Durkin Investing Agency has been the best stock picker in the

The Durkin Investing Agency has been the best stock picker in the country for the past two years. Before this rise to fame occurred, the Durkin newsletter had 200 subscribers. Those subscribers beat t...

See Answer

Q: A firm is considering an investment in a new machine with a

A firm is considering an investment in a new machine with a price of $15.6 million to replace its existing machine. The current machine has a book value of $5.4 million and a market value of $4.1 mill...

See Answer

Q: Young screenwriter Carl Draper has just finished his first script. It

Young screenwriter Carl Draper has just finished his first script. It has action, drama, and humor, and he thinks it will be a blockbuster. He takes the script to every motion picture studio in town a...

See Answer

Q: RAK Co. wants to issue new 20-year bonds for

RAK Co. wants to issue new 20-year bonds for some much-needed expansion projects. The company currently has 6.4 percent coupon bonds on the market that sell for $1,063, make semiannual payments, and m...

See Answer

Q: A stock has had the following year-end prices and dividends

A stock has had the following year-end prices and dividends: What are the arithmetic and geometric returns for the stock?

See Answer

Q: Using the CAPM, show that the ratio of the risk premiums

Using the CAPM, show that the ratio of the risk premiums on two assets is equal to the ratio of their betas.

See Answer

Q: Critically evaluate the following statement: Playing the stock market is like

Critically evaluate the following statement: Playing the stock market is like gambling. Such speculative investing has no social value, other than the pleasure people get from this form of gambling.

See Answer

Q: Happy Times, Inc., wants to expand its party stores into

Happy Times, Inc., wants to expand its party stores into the Southeast. In order to establish an immediate presence in the area, the company is considering the purchase of the privately held Joe’s Par...

See Answer

Q: Acetate, Inc., has equity with a market value of $

Acetate, Inc., has equity with a market value of $29.5 million and debt with a market value of $8 million. Treasury bills that mature in one year yield 5 percent per year, and the expected return on t...

See Answer

Q: You own a portfolio that has $2,700 invested in

You own a portfolio that has $2,700 invested in Stock A and $3,800 invested in Stock B. If the expected returns on these stocks are 9.5 percent and 14 percent, respectively, what is the expected retur...

See Answer

Q: Sunburn Sunscreen has a zero coupon bond issue outstanding with a $

Sunburn Sunscreen has a zero coupon bond issue outstanding with a $20,000 face value that matures in one year. The current market value of the firm’s assets is $21,700. The standard deviation of the r...

See Answer

Q: An investment under consideration has a payback of six years and a

An investment under consideration has a payback of six years and a cost of $573,000. If the required return is 12 percent, what is the worst-case NPV? The best-case NPV? Explain. Assume the cash flows...

See Answer

Q: Your broker commented that well-managed firms are better investments than

Your broker commented that well-managed firms are better investments than poorly managed firms. As evidence your broker cited a recent study examining 100 small manufacturing firms that eight years ea...

See Answer

Q: Pasqually Mineral Water, Inc., will pay a quarterly dividend per

Pasqually Mineral Water, Inc., will pay a quarterly dividend per share of $.90 at the end of each of the next 12 quarters. Thereafter, the dividend will grow at a quarterly rate of 1 percent, forever....

See Answer

Q: Sanders Enterprises, Inc., has been considering the purchase of a

Sanders Enterprises, Inc., has been considering the purchase of a new manufacturing facility for $750,000. The facility is to be fully depreciated on a straight-line basis over seven years. It is expe...

See Answer

Q: Hickock Mining is evaluating when to open a gold mine. The

Hickock Mining is evaluating when to open a gold mine. The mine has 44,000 ounces of gold left that can be mined, and mining operations will produce 5,500 ounces per year. The required return on the g...

See Answer

Q: You purchase a bond with an invoice price of $950.

You purchase a bond with an invoice price of $950. The bond has a coupon rate of 5.2 percent, and there are 2 months to the next semiannual coupon date. What is the clean price of the bond?

See Answer

Q: True or false: The most important characteristic in determining the expected

True or false: The most important characteristic in determining the expected return of a well-diversified portfolio is the variances of the individual assets in the portfolio. Explain.

See Answer

Q: Refer to Table 10.1 in the text and look at

Refer to Table 10.1 in the text and look at the period from 1973 through 1980. a. Calculate the average return for Treasury bills and the average annual inflation rate (consumer price index) for this...

See Answer

Q: Consider the following information about three stocks: /

Consider the following information about three stocks: a. If your portfolio is invested 40 percent each in A and B and 20 percent in C, what is the portfolio expected return? The variance? The stand...

See Answer

Q: Photochronograph Corporation (PC) manufactures time series photographic equipment. It

Photochronograph Corporation (PC) manufactures time series photographic equipment. It is currently at its target debt–equity ratio of .55. It’s considering building a new $50 million manufacturing fac...

See Answer

Q: Suppose a three-factor model is appropriate to describe the returns

Suppose a three-factor model is appropriate to describe the returns of a stock. Information about those three factors is presented in the following chart: a. What is the systematic risk of the stock...

See Answer

Q: The Veblen Company and the Knight Company are identical in every respect

The Veblen Company and the Knight Company are identical in every respect except that Veblen is not levered. The market value of Knight Company’s 6 percent bonds is $1.4 million. Fina...

See Answer

Q: Suppose the firm in the previous problem is considering two mutually exclusive

Suppose the firm in the previous problem is considering two mutually exclusive investments. Project A has an NPV of $1,200, and Project B has an NPV of $1,600. As a result of taking Project A, the sta...

See Answer

Q: This problem is useful for testing the ability of financial calculators and

This problem is useful for testing the ability of financial calculators and computer software. Consider the following cash flows. How many different IRRs are there? (Hint: Search between 20 percent an...

See Answer

Q: A famous economist just announced in The Wall Street Journal his findings

A famous economist just announced in The Wall Street Journal his findings that the recession is over and the economy is again entering an expansion. Assume market efficiency. Can you profit from inves...

See Answer

Q: Briley, Inc., is expected to pay equal dividends at the

Briley, Inc., is expected to pay equal dividends at the end of each of the next two years. Thereafter, the dividend will grow at a constant annual rate of 4 percent, forever. The current stock price i...

See Answer

Q: Juggernaut Satellite Corporation earned $23 million for the fiscal year ending

Juggernaut Satellite Corporation earned $23 million for the fiscal year ending yesterday. The firm also paid out 30 percent of its earnings as dividends yesterday. The firm will continue to pay out 30...

See Answer

Q: What is the quirk in the tax code that makes a levered

What is the quirk in the tax code that makes a levered firm more valuable than an otherwise identical unlevered firm?

See Answer

Q: Allied Products, Inc., is considering a new product launch.

Allied Products, Inc., is considering a new product launch. The firm expects to have an annual operating cash flow of $13.5 million for the next 10 years. Allied Products uses a discount rate of 13 pe...

See Answer

Q: You purchase a bond with a coupon rate of 5.9

You purchase a bond with a coupon rate of 5.9 percent and a clean price of $984. If the next semiannual coupon payment is due in four months, what is the invoice price?

See Answer

Q: You bought one of Bergen Manufacturing Co.’s 6.4

You bought one of Bergen Manufacturing Co.’s 6.4 percent coupon bonds one year ago for $1,032.50. These bonds make annual payments and mature six years from now. Suppose you decide to sell your bonds...

See Answer

Q: You want to create a portfolio equally as risky as the market

You want to create a portfolio equally as risky as the market, and you have $1,000,000 to invest. Given this information, fill in the rest of the following table:

See Answer

Q: Advance, Inc., is trying to determine its cost of debt

Advance, Inc., is trying to determine its cost of debt. The firm has a debt issue outstanding with 13 years to maturity that is quoted at 95 percent of face value. The issue makes semiannual payments...

See Answer

Q: Trower Corp. has a debt–equity ratio of .85

Trower Corp. has a debt–equity ratio of .85. The company is considering a new plant that will cost $145 million to build. When the company issues new equity, it incurs a flotation cost of 8 percent. T...

See Answer

Q: Locomotive Corporation is planning to repurchase part of its common stock by

Locomotive Corporation is planning to repurchase part of its common stock by issuing corporate debt. As a result, the firm’s debt–equity ratio is expected to rise from 35 percent to 50 percent. The fi...

See Answer

Q: Frostbite Thermalwear has a zero coupon bond issue outstanding with a face

Frostbite Thermalwear has a zero coupon bond issue outstanding with a face value of $25,000 that matures in one year. The current market value of the firm’s assets is $27,200. The standard deviation o...

See Answer

Q: With the growing popularity of casual surf print clothing, two recent

With the growing popularity of casual surf print clothing, two recent MBA graduates decided to broaden this casual surf concept to encompass a “surf lifestyle for the home.” With limited capital, they...

See Answer

Q: The Yurdone Corporation wants to set up a private cemetery business.

The Yurdone Corporation wants to set up a private cemetery business. According to the CFO, Barry M. Deep, business is “looking up.” As a result, the cemetery project will provide a net cash inflow of...

See Answer

Q: You are determining whether your company should undertake a new project and

You are determining whether your company should undertake a new project and have calculated the NPV of the project using the WACC method when the CFO, a former accountant, notices that you did not use...

See Answer

Q: Suppose the market is semistrong form efficient. Can you expect to

Suppose the market is semistrong form efficient. Can you expect to earn excess returns if you make trades based on: a. Your broker’s information about record earnings for a stock? b. Rumors about a me...

See Answer

Q: Four years ago, Bling Diamond, Inc., paid a dividend

Four years ago, Bling Diamond, Inc., paid a dividend of $1.51 per share. The company paid a dividend of $1.87 per share yesterday. Dividends will grow over the next five years at the same rate they gr...

See Answer

Q: Applied Nanotech is thinking about introducing a new surface cleaning machine.

Applied Nanotech is thinking about introducing a new surface cleaning machine. The marketing department has come up with the estimate that Applied Nanotech can sell 15 units per year at $275,000 net c...

See Answer

Q: Erna Corp. has 9 percent coupon bonds making annual payments with

Erna Corp. has 9 percent coupon bonds making annual payments with a YTM of 7.81 percent. The current yield on these bonds is 8.42 percent. How many years do these bonds have left until they mature?

See Answer

Q: Suppose the returns on long-term government bonds are normally distributed

Suppose the returns on long-term government bonds are normally distributed. Based on the historical record, what is the approximate probability that your return on these bonds will be less than 23.7 p...

See Answer

Q: An election is being held to fill three seats on the board

An election is being held to fill three seats on the board of directors of a firm in which you hold stock. The company has 17,400 shares outstanding. If the election is conducted under cumulative voti...

See Answer

Q: You have $100,000 to invest in a portfolio containing

You have $100,000 to invest in a portfolio containing Stock X and Stock Y. Your goal is to create a portfolio that has an expected return of 12.9 percent. If Stock X has an expected return of 11.2 per...

See Answer

Q: This is a comprehensive project evaluation problem bringing together much of what

This is a comprehensive project evaluation problem bringing together much of what you have learned in this and previous chapters. Suppose you have been hired as a financial consultant to Defense Elec...

See Answer

Q: Green Manufacturing, Inc., plans to announce that it will issue

Green Manufacturing, Inc., plans to announce that it will issue $1.8 million of perpetual debt and use the proceeds to repurchase common stock. The bonds will sell at par with a coupon rate of 6 perce...

See Answer

Q: Suppose Sunburn Sunscreen and Frostbite Thermalwear in the previous problems have decided

Suppose Sunburn Sunscreen and Frostbite Thermalwear in the previous problems have decided to merge. Because the two companies have seasonal sales, the combined firm’s return on assets will have a stan...

See Answer

Q: On Tuesday, December 8, Hometown Power Co.’s board

On Tuesday, December 8, Hometown Power Co.’s board of directors declares a dividend of 75 cents per share payable on Wednesday, January 17, to shareholders of record as of Wednesday, January 3. When i...

See Answer

Q: You have been hired as a consultant for Pristine Urban-Tech

You have been hired as a consultant for Pristine Urban-Tech Zither, Inc. (PUTZ), manufacturers of fine zithers. The market for zithers is growing quickly. The company bought some land three years ago...

See Answer

Q: The Utah Mining Corporation is set to open a gold mine near

The Utah Mining Corporation is set to open a gold mine near Provo, Utah. According to the treasurer, Monty Goldstein, “This is a golden opportunity.” The mine will cost $2,700,000 to open and will hav...

See Answer

Q: Imagine that a particular macroeconomic variable that influences your firm’s net earnings

Imagine that a particular macroeconomic variable that influences your firm’s net earnings is positively serially correlated. Assume market efficiency. Would you expect price changes in your stock to b...

See Answer

Q: Pilot Plus Pens is deciding when to replace its old machine.

Pilot Plus Pens is deciding when to replace its old machine. The machine’s current salvage value is $2.8 million. Its current book value is $1.6 million. If not sold, the old machine will require main...

See Answer

Q: Suppose the following bond quote for IOU Corporation appears in the financial

Suppose the following bond quote for IOU Corporation appears in the financial page of today’s newspaper. Assume the bond has a face value of $1,000, semiannual coupon payments, and t...

See Answer

Q: What are the direct and indirect costs of bankruptcy? Briefly explain

What are the direct and indirect costs of bankruptcy? Briefly explain each.

See Answer

Q: Repeat parts (a) and (b) in Problem 1

Repeat parts (a) and (b) in Problem 1 assuming the company has a tax rate of 35 percent. Problem 1: Music City, Inc., has no debt outstanding and a total market value of $295,000. Earnings before int...

See Answer

Q: Assuming that the returns from holding small-company stocks are normally

Assuming that the returns from holding small-company stocks are normally distributed, what is the approximate probability that your money will double in value in a single year? Triple in value?

See Answer

Q: Based on the following information, calculate the expected return and standard

Based on the following information, calculate the expected return and standard deviation of each of the following stocks. Assume each state of the economy is equally likely to happen. What are the cov...

See Answer

Q: Why is underpricing not a great concern with bond offerings?

Why is underpricing not a great concern with bond offerings?

See Answer

Q: Williamson, Inc., has a debt–equity ratio of 2

Williamson, Inc., has a debt–equity ratio of 2.3. The firm’s weighted average cost of capital is 10 percent, and its pretax cost of debt is 6 percent. The tax rate is 35 percent. a. What is the compan...

See Answer

Q: A company has a single zero coupon bond outstanding that matures in

A company has a single zero coupon bond outstanding that matures in 10 years with a face value of $10 million. The current value of the company’s assets is $9.05 million, and the standard deviation of...

See Answer

Q: Butler International Limited is evaluating a project in Erewhon. The project

Butler International Limited is evaluating a project in Erewhon. The project will create the following cash flows: All cash flows will occur in Erewhon and are expressed in dollars. In an attempt to...

See Answer

Q: The efficient market hypothesis implies that all mutual funds should obtain the

The efficient market hypothesis implies that all mutual funds should obtain the same expected risk-adjusted returns. Therefore, we can simply pick mutual funds at random. Is this statement true or fal...

See Answer

Q: Consider Pacific Energy Company and U.S. Bluechips, Inc

Consider Pacific Energy Company and U.S. Bluechips, Inc., both of which reported earnings of $630,000. Without new projects, both firms will continue to generate earnings of $630,000 in perpetuity. As...

See Answer

Q: Office Automation, Inc., must choose between two copiers, the

Office Automation, Inc., must choose between two copiers, the XX40 or the RH45. The XX40 costs $1,499 and will last for three years. The copier will require a real aftertax cost of $120 per year after...

See Answer

Q: Consider a project to supply Detroit with 25,000 tons of

Consider a project to supply Detroit with 25,000 tons of machine screws annually for automobile production. You will need an initial $2,700,000 investment in threading equipment to get the project sta...

See Answer

Q: : You’ve just found a 10 percent coupon bond on the market

You’ve just found a 10 percent coupon bond on the market that sells for par value. What is the maturity on this bond?

See Answer

Q: An investment project provides cash inflows of $790 per year for

An investment project provides cash inflows of $790 per year for eight years. What is the project payback period if the initial cost is $3,200? What if the initial cost is $4,800? What if it is $7,300...

See Answer

Q: Based on the following information, calculate the expected return and standard

Based on the following information, calculate the expected return and standard deviation for each of the following stocks. What are the covariance and correlation between the returns of the two stocks...

See Answer

Q: We are evaluating a project that costs $588,000,

We are evaluating a project that costs $588,000, has an eight-year life, and has no salvage value. Assume that depreciation is straight-line to zero over the life of the project. Sales are projected a...

See Answer

Q: Cost of Capital In a world of corporate taxes only, show

Cost of Capital In a world of corporate taxes only, show that the R WACC can be written as R WACC = R0 x [1 - tC (B/V )].

See Answer

Q: Pricing Model Ken is interested in buying a European call option written

Pricing Model Ken is interested in buying a European call option written on Southeastern Airlines, Inc., a non-dividend-paying common stock, with a strike price of $65 and one year until expiration. C...

See Answer

Q: Consider two streams of cash flows, A and B. Stream

Consider two streams of cash flows, A and B. Stream A’s first cash flow is $11,600 and is received three years from today. Future cash flows in Stream A grow by 4 percent in perpetuity. Stream B’s fir...

See Answer

Q: In the previous problem, what is the probability that the return

In the previous problem, what is the probability that the return is less than 2100 percent? (Think.) What are the implications for the distribution of returns?

See Answer

Q: Assume that markets are efficient. During a trading day American Golf

Assume that markets are efficient. During a trading day American Golf, Inc., announces that it has lost a contract for a large golfing project that, prior to the news, it was widely believed to have s...

See Answer

Q: Ramsay Corp. currently has an EPS of $3.10

Ramsay Corp. currently has an EPS of $3.10, and the bench-mark PE for the company is 21. Earnings are expected to grow at 6 percent per year. a. What is your estimate of the current stock price? b. W...

See Answer

Q: FFDP Corp. has yearly sales of $42 million and costs

FFDP Corp. has yearly sales of $42 million and costs of $13 million. The company’s balance sheet shows debt of $64 million and cash of $21 million. There are 1,750,000 shares outstanding and the indus...

See Answer

Q: In Problem 26, suppose you’re confident about your own projections,

In Problem 26, suppose you’re confident about your own projections, but you’re a little unsure about Detroit’s actual machine screw requirements. What is the sensitivity of the project OCF to changes...

See Answer

Q: Tesla Corporation needs to raise funds to finance a plant expansion,

Tesla Corporation needs to raise funds to finance a plant expansion, and it has decided to issue 25-year zero coupon bonds to raise the money. The required return on the bonds will be 5.4 percent. a....

See Answer

Q: Which has greater interest rate risk, a 30-year Treasury

Which has greater interest rate risk, a 30-year Treasury bond or a 30-year BB corporate bond?

See Answer

Q: What is the intrinsic value of a call option? Of a

What is the intrinsic value of a call option? Of a put option? How do we interpret these values?

See Answer

Q: Security F has an expected return of 10 percent and a standard

Security F has an expected return of 10 percent and a standard deviation of 49 percent per year. Security G has an expected return of 14 percent and a standard deviation of 73 percent per year. a. Wha...

See Answer

Q: Assuming a world of corporate taxes only, show that the cost

Assuming a world of corporate taxes only, show that the cost of equity, RS , is as given in the chapter by MM Proposition II with corporate taxes.

See Answer

Q: Pricing Model Rob wishes to buy a European put option on BioLabs

Pricing Model Rob wishes to buy a European put option on BioLabs, Inc., a non-dividend-paying common stock, with a strike price of $50 and six months until expiration. The company’s common stock is cu...

See Answer

Q: Dickinson Brothers, Inc., is considering investing in a machine to

Dickinson Brothers, Inc., is considering investing in a machine to produce computer keyboards. The price of the machine will be $975,000, and its economic life is five years. The machine will be fully...

See Answer

Q: Darin Clay, the CFO of MakeMoney.com, has to

Darin Clay, the CFO of MakeMoney.com, has to decide between the following two projects: The expected rate of return for either of the two projects is 12 percent. What is the range of initial investm...

See Answer

Q: Prospectors, Inc., is a publicly traded gold prospecting company in

Prospectors, Inc., is a publicly traded gold prospecting company in Alaska. Although the firm’s searches for gold usually fail, the prospectors occasionally find a rich vein of ore. What pattern would...

See Answer

Q: Aday Acoustics, Inc., projects unit sales for a new seven

Aday Acoustics, Inc., projects unit sales for a new seven-octave voice emulation implant as follows: Production of the implants will require $1,500,000 in net working capital to start and additional...

See Answer

Q: Consider the following project for Hand Clapper, Inc. The company

Consider the following project for Hand Clapper, Inc. The company is considering a four-year project to manufacture clap-command garage door openers. This project requires an initial investment of $15...

See Answer

Q: Suppose your company needs to raise $50 million and you want

Suppose your company needs to raise $50 million and you want to issue 30-year bonds for this purpose. Assume the required return on your bond issue will be 6 percent, and you’re evaluating two issue a...

See Answer

Q: Suppose the returns on long-term corporate bonds and T-

Suppose the returns on long-term corporate bonds and T-bills are normally distributed. Based on the historical record, use the NORMDIST function in Excel ® to answer the following questions: a. What i...

See Answer

Q: As a shareholder of a firm that is contemplating a new project

As a shareholder of a firm that is contemplating a new project, would you be more concerned with the accounting break-even point, the cash break-even point (the point at which operating cash flow is z...

See Answer

Q: Given that Transocean was down by about 63 percent for 2014,

Given that Transocean was down by about 63 percent for 2014, why did some investors hold the stock? Why didn’t they sell out before the price declined so sharply?

See Answer

Q: Suppose the expected returns and standard deviations of Stocks A and B

Suppose the expected returns and standard deviations of Stocks A and B are E(RA) = .11, E(RB) = .13, α A = .39, and αB = .76. a. Calculate the expected return and standard deviation of a portfolio tha...

See Answer

Q: Assume a firm’s debt is risk-free, so that the

Assume a firm’s debt is risk-free, so that the cost of debt equals the risk-free rate, Rf . Define βA as the firm’s asset beta—that is, the systematic risk of the firm’s assets. Define βS to be the be...

See Answer

Q: Pricing Model Maverick Manufacturing, Inc., must purchase gold in three

Pricing Model Maverick Manufacturing, Inc., must purchase gold in three months for use in its operations. Maverick’s management has estimated that if the price of gold were to rise above $1,380 per ou...

See Answer

Q: McKeekin Corp. has a project with the following cash flows:

McKeekin Corp. has a project with the following cash flows: What is the IRR of the project? What is happening here?

See Answer

Q: Some people argue that the efficient market hypothesis cannot explain the 1987

Some people argue that the efficient market hypothesis cannot explain the 1987 market crash or the high price-to-earnings ratios of Internet stocks during the late 1990s. What alternative hypothesis i...

See Answer

Q: Fincher Manufacturing has projected sales of $135 million next year.

Fincher Manufacturing has projected sales of $135 million next year. Costs are expected to be $76 million and net investment is expected to be $15 million. Each of these values is expected to grow at...

See Answer

Q: M.V.P. Games, Inc., has hired

M.V.P. Games, Inc., has hired you to perform a feasibility study of a new video game that requires an initial investment of $8 million. M.V.P. expects a total annual operating cash flow of $1.5 millio...

See Answer

Q: Bond P is a premium bond with a coupon of 8.

Bond P is a premium bond with a coupon of 8.5 percent. Bond D has a coupon of 5.5 percent and is selling at a discount. Both bonds make annual payments, have a YTM of 7 percent, and have 10 years to m...

See Answer

Q: You have been provided the following data about the securities of three

You have been provided the following data about the securities of three firms, the market portfolio, and the risk-free asset: a. Fill in the missing values in the table. b. Is the stock of Firm A co...

See Answer

Q: Milano Pizza Club owns three identical restaurants popular for their specialty pizzas

Milano Pizza Club owns three identical restaurants popular for their specialty pizzas. Each restaurant has a debt–equity ratio of 40 percent and makes interest payments of $41,000 at the end of each y...

See Answer

Q: Under what two assumptions can we use the dividend growth model presented

Under what two assumptions can we use the dividend growth model presented in the chapter to determine the value of a share of stock? Comment on the reasonableness of these assumptions.

See Answer

Q: Suppose a firm’s business operations mirror movements in the economy as a

Suppose a firm’s business operations mirror movements in the economy as a whole very closely—that is, the firm’s asset beta is 1.0. Use the result of the previous problem to find the equity beta for t...

See Answer

Q: Classify the following events as mostly systematic or mostly unsystematic. Is

Classify the following events as mostly systematic or mostly unsystematic. Is the distinction clear in every case? a. Short-term interest rates increase unexpectedly. b. The interest rate a company pa...

See Answer

Q: An investor is said to take a position in a “collar

An investor is said to take a position in a “collar” if she buys the asset, buys an out-of-the-money put option on the asset, and sells an out-of-the-money call option on the asset. The two options sh...

See Answer

Q: A proposed cost-saving device has an installed cost of $

A proposed cost-saving device has an installed cost of $710,000. The device will be used in a five-year project but is classified as three-year MACRS property for tax purposes. The required initial ne...

See Answer

Q: Consider four different stocks, all of which have a required return

Consider four different stocks, all of which have a required return of 14 percent and a most recent dividend of $3.50 per share. Stocks W, X, and Y are expected to maintain constant growth rates in di...

See Answer

Q: The Cornchopper Company is considering the purchase of a new harvester.

The Cornchopper Company is considering the purchase of a new harvester. Cornchopper has hired you to determine the break-even purchase price in terms of present value of the harvester. This break-even...

See Answer

Q: The YTM on a bond is the interest rate you earn on

The YTM on a bond is the interest rate you earn on your investment if interest rates don’t change. If you actually sell the bond before it matures, your realized return is known as the holding period...

See Answer

Q: The market portfolio has an expected return of 11 percent and a

The market portfolio has an expected return of 11 percent and a standard deviation of 19 percent. The risk-free rate is 4.3 percent. a. What is the expected return on a well-diversified portfolio with...

See Answer

Q: Beginning with the cost of capital equation—that is:

Beginning with the cost of capital equation—that is: show that the cost of equity capital for a levered firm can be written as follows:

See Answer

Q: McLemore Industries has a zero coupon bond issue that matures in two

McLemore Industries has a zero coupon bond issue that matures in two years with a face value of $75,000. The current value of the company’s assets is $46,000, and the standard deviation of the return...

See Answer

Q: In our capital budgeting examples, we assumed that a firm would

In our capital budgeting examples, we assumed that a firm would recover all of the working capital it invested in a project. Is this a reasonable assumption? When might it not be valid?

See Answer

Q: Another utilization of cash flow analysis is setting the bid price on

Another utilization of cash flow analysis is setting the bid price on a project. To calculate the bid price, we set the project NPV equal to zero and find the required price. Thus the bid price repres...

See Answer

Q: Most corporations pay quarterly dividends on their common stock rather than annual

Most corporations pay quarterly dividends on their common stock rather than annual dividends. Barring any unusual circumstances during the year, the board raises, lowers, or maintains the current divi...

See Answer

Q: Consider the following statement: For the APT to be useful,

Consider the following statement: For the APT to be useful, the number of systematic risk factors must be small. Do you agree or disagree with this statement? Why?

See Answer

Q: The Frush Corporation has two different bonds currently outstanding. Bond M

The Frush Corporation has two different bonds currently outstanding. Bond M has a face value of $30,000 and matures in 20 years. The bond makes no payments for the first six years, then pays $800 ever...

See Answer

Q: A portfolio that combines the risk-free asset and the market

A portfolio that combines the risk-free asset and the market portfolio has an expected return of 8 percent and a standard deviation of 17 percent. The risk-free rate is 4.3 percent, and the expected r...

See Answer

Q: Brozik Corp. has a zero coupon bond that matures in five

Brozik Corp. has a zero coupon bond that matures in five years with a face value of $40,000. The current value of the company’s assets is $38,000, and the standard deviation of its return on assets is...

See Answer

Q: The technique for calculating a bid price can be extended to many

The technique for calculating a bid price can be extended to many other types of problems. Answer the following questions using the same technique as setting a bid price; that is, set the project NPV...

See Answer

Q: Storico Co. just paid a dividend of $3.40

Storico Co. just paid a dividend of $3.40 per share. The company will increase its dividend by 20 percent next year and will then reduce its dividend growth rate by 5 percentage points per year until...

See Answer

Q: The following Treasury bond quote appeared in The Wall Street Journal on

The following Treasury bond quote appeared in The Wall Street Journal on May 11, 2004: Why would anyone buy this Treasury bond with a negative yield to maturity? How is this possible?

See Answer

Q: Suppose the risk-free rate is 4.7 percent and

Suppose the risk-free rate is 4.7 percent and the market portfolio has an expected return of 11.2 percent. The market portfolio has a variance of .0382. Portfolio Z has a correlation coefficient with...

See Answer

Q: Assume a firm is considering a new typical project that requires an

Assume a firm is considering a new typical project that requires an initial investment with sales, variable costs and fixed costs over its life. Will the project usually reach the accounting, cash, or...

See Answer

Q: Strudler Real Estate, Inc., a construction firm financed by both

Strudler Real Estate, Inc., a construction firm financed by both debt and equity, is undertaking a new project. If the project is successful, the value of the firm in one year will be $213 million, bu...

See Answer

Q: Your company has been approached to bid on a contract to sell

Your company has been approached to bid on a contract to sell 15,000 voice recognition (VR) computer keyboards a year for four years. Due to technological improvements, beyond that time they will be o...

See Answer

Q: This one’s a little harder. Suppose the current share price for

This one’s a little harder. Suppose the current share price for the firm in the previous problem is $62.40 and all the dividend information remains the same. What required return must investors be dem...

See Answer

Q: Why do we use an aftertax figure for cost of debt but

Why do we use an aftertax figure for cost of debt but not for cost of equity?

See Answer

Q: When Marilyn Monroe died, ex-husband Joe DiMaggio vowed to

When Marilyn Monroe died, ex-husband Joe DiMaggio vowed to place fresh flowers on her grave every Sunday as long as he lived. The week after she died in 1962, a bunch of fresh flowers that the former...

See Answer

Q: Consider the following information about Stocks I and II:

Consider the following information about Stocks I and II: The market risk premium is 7.5 percent, and the risk-free rate is 4 percent. Which stock has the most systematic risk? Which one has the mos...

See Answer

Q: In addition to the five factors discussed in the chapter, dividends

In addition to the five factors discussed in the chapter, dividends also affect the price of an option. The Black–Scholes option pricing model with dividends is: All of the variabl...

See Answer

Q: Suppose we are thinking about replacing an old computer with a new

Suppose we are thinking about replacing an old computer with a new one. The old one cost us $450,000; the new one will cost $580,000. The new machine will be depreciated straight-line to zero over its...

See Answer

Q: The Stambaugh Corporation currently has earnings per share of $8.

The Stambaugh Corporation currently has earnings per share of $8.20. The company has no growth and pays out all earnings as dividends. It has a new project that will require an investment of $1.95 per...

See Answer

Q: You are planning to save for retirement over the next 30 years

You are planning to save for retirement over the next 30 years. To save for retirement, you will invest $900 per month in a stock account in real dollars and $300 per month in a bond account in real d...

See Answer

Q: How does a bond issuer decide on the appropriate coupon rate to

How does a bond issuer decide on the appropriate coupon rate to set on its bonds? Explain the difference between the coupon rate and the required return on a bond.

See Answer

Q: Suppose you observe the following situation: /

Suppose you observe the following situation: Assume these securities are correctly priced. Based on the CAPM, what is the expected return on the market? What is the risk-free rate?

See Answer

Q: The put–call parity condition is altered when dividends are paid

The put–call parity condition is altered when dividends are paid. The dividend-adjusted put–call parity formula is: where d is again the continuously compounded d...

See Answer

Q: Hardwick Enterprises is evaluating alternative uses for a three-story manufacturing

Hardwick Enterprises is evaluating alternative uses for a three-story manufacturing and warehousing building that it has purchased for $1,250,000. The company can continue to rent the building to the...

See Answer

Q: Burklin, Inc., has earnings of $21 million and is

Burklin, Inc., has earnings of $21 million and is projected to grow at a constant rate of 5 percent forever because of the benefits gained from the learning curve. Currently, all earnings are paid out...

See Answer

Q: What is the main difference between the WACC and APV methods?

What is the main difference between the WACC and APV methods?

See Answer

Q: Paul Adams owns a health club in downtown Los Angeles. He

Paul Adams owns a health club in downtown Los Angeles. He charges his customers an annual fee of $400 and has an existing customer base of 700. Paul plans to raise the annual fee by 6 percent every ye...

See Answer

Q: There are three securities in the market. The following chart shows

There are three securities in the market. The following chart shows their possible payoffs: a. What are the expected return and standard deviation of each security? b. What are the covariances and c...

See Answer

Q: In the chapter we noted that the delta for a put option

In the chapter we noted that the delta for a put option is N(d1) − 1. Is this the same thing as −N(−d1)? (Hint: Yes, but why?)

See Answer

Q: The Biological Insect Control Corporation (BICC) has hired you as

The Biological Insect Control Corporation (BICC) has hired you as a consultant to evaluate the NPV of its proposed toad ranch. The company plans to breed toads and sell them as ecologically desirable...

See Answer

Q: Suppose you observe the following situation: /

Suppose you observe the following situation: a. Calculate the expected return on each stock. b. Assuming the capital asset pricing model holds and Stock A’s beta is greater than St...

See Answer

Q: How do you determine the appropriate cost of debt for a company

How do you determine the appropriate cost of debt for a company? Does it make a difference if the company’s debt is privately placed as opposed to being publicly traded? How would you estimate the cos...

See Answer

Q: Put Pricing Model Use the Black–Scholes model for pricing a

Put Pricing Model Use the Black–Scholes model for pricing a call, put–call parity, and the previous question to show that the Black–Scholes model...

See Answer

Q: O’Bannon Electronics has an investment opportunity to produce a new HDTV.

O’Bannon Electronics has an investment opportunity to produce a new HDTV. The required investment on January 1 of this year is $145 million. The firm will depreciate the investment t...

See Answer

Q: There are two stocks in the market, Stock A and Stock

There are two stocks in the market, Stock A and Stock B. The price of Stock A today is $75. The price of Stock A next year will be $64 if the economy is in a recession, $87 if the economy is normal, a...

See Answer

Q: A stock is currently priced at $50. The stock will

A stock is currently priced at $50. The stock will never pay a dividend. The risk-free rate is 12 percent per year, compounded continuously, and the standard deviation of the stock’s return is 60 perc...

See Answer

Q: After extensive medical and marketing research, Pill, Inc., believes

After extensive medical and marketing research, Pill, Inc., believes it can penetrate the pain reliever market. It is considering two alternative products. The first is a medication for headache pain....

See Answer

Q: What is the impact of a stock repurchase on a company’s debt

What is the impact of a stock repurchase on a company’s debt ratio? Does this suggest another use for excess cash?

See Answer

Q: Assume Stocks A and B have the following characteristics:

Assume Stocks A and B have the following characteristics: The covariance between the returns on the two stocks is .001. a. Suppose an investor holds a portfolio consisting of only Stock A and Stock...

See Answer

Q: You purchase one call and sell one put with the same strike

You purchase one call and sell one put with the same strike price and expiration date. What is the delta of your portfolio? Why?

See Answer

Q: J. Smythe, Inc., manufactures fine furniture. The company

J. Smythe, Inc., manufactures fine furniture. The company is deciding whether to introduce a new mahogany dining room table set. The set will sell for $6,100, including a set of eight chairs. The comp...

See Answer

Q: Your financial planner offers you two different investment plans. Plan X

Your financial planner offers you two different investment plans. Plan X is a $20,000 annual perpetuity. Plan Y is a 10-year, $34,000 annual annuity. Both plans will make their first payment one year...

See Answer

Q: What are the main differences between corporate debt and equity? Why

What are the main differences between corporate debt and equity? Why do some firms try to issue equity in the guise of debt?

See Answer

Q: What is the value of an investment that pays $50,

What is the value of an investment that pays $50,000 every other year forever, if the first payment occurs one year from today and the discount rate is 13 percent compounded daily? What is the value t...

See Answer

Q: As discussed in the text, an annuity due is identical to

As discussed in the text, an annuity due is identical to an ordinary annuity except that the periodic payments occur at the beginning of each period and not at the end of the period. Show that the rel...

See Answer

Q: A check-cashing store is in the business of making personal

A check-cashing store is in the business of making personal loans to walk-up customers. The store makes only one-week loans at 6.5 percent interest per week. a. What APR must the store report to its c...

See Answer

Q: What is the equation for the present value of a growing perpetuity

What is the equation for the present value of a growing perpetuity with a payment of C one period from today if the payments grow by C each period?

See Answer

Q: A useful rule of thumb for the time it takes an investment

A useful rule of thumb for the time it takes an investment to double with discrete compounding is the “Rule of 72.” To use the Rule of 72, you simply divide 72 by the interest rate to determine the nu...

See Answer

Q: A corollary to the Rule of 72 is the Rule of 69

A corollary to the Rule of 72 is the Rule of 69.3. The Rule of 69.3 is exactly correct except for rounding when interest rates are compounded continuously. Prove the Rule of 69.3 for continuously comp...

See Answer

Q: Taxes are an important consideration in the leasing decision. Which is

Taxes are an important consideration in the leasing decision. Which is more likely to lease: A profitable corporation in a high tax bracket or a less profitable one in a low tax bracket? Why?

See Answer

Q: In the aggregate, debt offerings are much more common than equity

In the aggregate, debt offerings are much more common than equity offerings and typically much larger as well. Why?

See Answer

Q: What are the two options that many businesses have?

What are the two options that many businesses have?

See Answer

Q: What is the essential difference between sensitivity analysis and scenario analysis?

What is the essential difference between sensitivity analysis and scenario analysis?

See Answer

Q: Explain what is meant by business and financial risk. Suppose Firm

Explain what is meant by business and financial risk. Suppose Firm A has greater business risk than Firm B. Is it true that Firm A also has a higher cost of equity capital? Explain.

See Answer

Q: A convertible bond with a par value of $1,000

A convertible bond with a par value of $1,000 has a conversion price of $72.45. What is the conversion ratio of the bond?

See Answer

Q: You work for a nuclear research laboratory that is contemplating leasing a

You work for a nuclear research laboratory that is contemplating leasing a diagnostic scanner (leasing is a common practice with expensive, high-tech equipment). The scanner costs $5,800,000, and it w...

See Answer

Q: A substantial percentage of the companies listed on the NYSE and the

A substantial percentage of the companies listed on the NYSE and the NASDAQ don’t pay dividends, but investors are nonetheless willing to buy shares in them. How is this possible given your answer to...

See Answer

Q: Suppose a project has conventional cash flows and a positive NPV.

Suppose a project has conventional cash flows and a positive NPV. What do you know about its payback? Its discounted payback? Its profitability index? It’s IRR? Explain.

See Answer

Q: Your company currently produces and sells steel shaft golf clubs. The

Your company currently produces and sells steel shaft golf clubs. The board of directors wants you to consider the introduction of a new line of titanium bubble woods with graphite shafts. Which of th...

See Answer

Q: David McClemore, the CFO of Ultra Bread, has decided to

David McClemore, the CFO of Ultra Bread, has decided to use an APT model to estimate the required return on the company’s stock. The risk factors he plans to use are the risk premium on the stock mark...

See Answer

Q: Which of the following statements are true about the efficient market hypothesis

Which of the following statements are true about the efficient market hypothesis? a. It implies perfect forecasting ability. b. It implies that prices reflect all available information. c. It implies...

See Answer

Q: Preferred stock doesn’t offer a corporate tax shield on the dividends paid

Preferred stock doesn’t offer a corporate tax shield on the dividends paid. Why do we still observe some firms issuing preferred stock?

See Answer

Q: What are the key differences between leasing and borrowing? Are they

What are the key differences between leasing and borrowing? Are they perfect substitutes?

See Answer

Q: In a world with no taxes, no transaction costs, and

In a world with no taxes, no transaction costs, and no costs of financial distress, is the following statement true, false, or uncertain? Moderate borrowing will not increase the required return on a...

See Answer

Q: How does the existence of financial distress costs and agency costs affect

How does the existence of financial distress costs and agency costs affect Modigliani and Miller’s theory in a world where corporations pay taxes?

See Answer

Q: Due to large losses incurred in the past several years, a

Due to large losses incurred in the past several years, a firm has $2 billion in tax loss carryforwards. This means that the next $2 billion of the firm’s income will be free from corporate income tax...

See Answer

Q: It is sometimes suggested that firms should follow a “residual”

It is sometimes suggested that firms should follow a “residual” dividend policy. With such a policy, the main idea is that a firm should focus on meeting its investment needs and maintaining its desir...

See Answer

Q: What are some of the potential problems with looking at IRRs when

What are some of the potential problems with looking at IRRs when evaluating a leasing decision?

See Answer

Q: What is the difference between an American option and a European option

What is the difference between an American option and a European option?

See Answer

Q: Define each of the following investment rules and discuss any potential shortcomings

Define each of the following investment rules and discuss any potential shortcomings of each. In your definition, state the criterion for accepting or rejecting independent projects under each rule. a...

See Answer

Q: For the company in the previous problem, what is the dividend

For the company in the previous problem, what is the dividend yield? What is the expected capital gains yield?

See Answer

Q: For the company in Problem 2, show how the equity accounts

For the company in Problem 2, show how the equity accounts will change if: a. The company declares a four-for-one stock split. How many shares are outstanding now? What is the new par value per share?...

See Answer

Q: Stone Shoe Co. has concluded that additional equity financing will be

Stone Shoe Co. has concluded that additional equity financing will be needed to expand operations and that the needed funds will be best obtained through a rights offering. It has correctly determined...

See Answer

Q: You work for a nuclear research laboratory that is contemplating leasing a

You work for a nuclear research laboratory that is contemplating leasing a diagnostic scanner (leasing is a common practice with expensive, high-tech equipment). The scanner costs $5,800,000, and it w...

See Answer

Q: What is a call option? A put option? Under what

What is a call option? A put option? Under what circumstances might you want to buy each? Which one has greater potential profit? Why?

See Answer

Q: What is the price of a 15-year, zero coupon

What is the price of a 15-year, zero coupon bond paying $1,000 at maturity, assuming semiannual compounding, if the YTM is: a. 6 percent? b. 8 percent? c. 10 percent?

See Answer

Q: Use the option quote information shown here to answer the questions that

Use the option quote information shown here to answer the questions that follow. The stock is currently selling for $114. a. Suppose you buy 10 contracts of the February 110 call option. How much wi...

See Answer

Q: Gasworks, Inc., has been approached to sell up to 5

Gasworks, Inc., has been approached to sell up to 5 million gallons of gasoline in three months at a price of $2.65 per gallon. Gasoline is currently selling on the wholesale market at $2.34 per gallo...

See Answer

Q: Eckely, Inc., recently issued bonds with a conversion ratio of

Eckely, Inc., recently issued bonds with a conversion ratio of 14.5. If the stock price at the time of the bond issue was $53.16, what was the conversion premium?

See Answer

Q: Down Under Boomerang, Inc., is considering a new three-

Down Under Boomerang, Inc., is considering a new three- year expansion project that requires an initial fixed asset investment of $1.65 million. The fixed asset will be depreciated straight-line to ze...

See Answer

Q: In each of the following cases, find the unknown variable.

In each of the following cases, find the unknown variable. Ignore taxes.

See Answer

Q: Watters Umbrella Corp. issued 15-year bonds 2 years ago

Watters Umbrella Corp. issued 15-year bonds 2 years ago at a coupon rate of 5.9 percent. The bonds make semiannual payments. If these bonds currently sell for 105 percent of par value, what is the YTM...

See Answer

Q: Rework Problems 1 and 2 assuming the ending share price is $

Rework Problems 1 and 2 assuming the ending share price is $57.

See Answer

Q: Suppose a factor model is appropriate to describe the returns on a

Suppose a factor model is appropriate to describe the returns on a stock. The current expected return on the stock is 10.5 percent. Information about those factors is presented in the following chart:...

See Answer

Q: Shanken Corp. issued a 30-year, 5.9

Shanken Corp. issued a 30-year, 5.9 percent semiannual bond 6 years ago. The bond currently sells for 108 percent of its face value. The company’s tax rate is 35 percent. a. What is the pretax cost of...

See Answer

Q: The following figures present the results of four cumulative abnormal returns (

The following figures present the results of four cumulative abnormal returns (CAR) studies. Indicate whether the results of each study support, reject, or are inconclusive about the semistrong form o...

See Answer

Q: What are the two types of risk that are measured by a

What are the two types of risk that are measured by a levered beta?

See Answer

Q: An investment project has annual cash inflows of $5,000

An investment project has annual cash inflows of $5,000, $5,500, $6,000, and $7,000, and a discount rate of 12 percent. What is the discounted payback period for these cash flows if the initial cost i...

See Answer

Q: Discuss the accounting criteria for determining whether a lease must be reported

Discuss the accounting criteria for determining whether a lease must be reported on the balance sheet. In each case give a rationale for the criterion.

See Answer

Q: What is dilution, and why does it occur when warrants are

What is dilution, and why does it occur when warrants are exercised?

See Answer

Q: The St. Anger Corporation needs to raise $55 million to

The St. Anger Corporation needs to raise $55 million to finance its expansion into new markets. The company will sell new shares of equity via a general cash offering to raise the needed funds. If the...

See Answer

Q: You work for a nuclear research laboratory that is contemplating leasing a

You work for a nuclear research laboratory that is contemplating leasing a diagnostic scanner (leasing is a common practice with expensive, high-tech equipment). The scanner costs $5,800,000, and it w...

See Answer

Q: The price of Tara, Inc., stock will be either $

The price of Tara, Inc., stock will be either $50 or $70 at the end of the year. Call options are available with one year to expiration. T-bills currently yield 5 percent. a. Suppose the current price...

See Answer

Q: Jet Black is an international conglomerate with a petroleum division and is

Jet Black is an international conglomerate with a petroleum division and is currently competing in an auction to win the right to drill for crude oil on a large piece of land in one year. The current...

See Answer

Q: A warrant gives its owner the right to purchase three shares of

A warrant gives its owner the right to purchase three shares of common stock at an exercise price of $64 per share. The current market price of the stock is $68. What is the minimum value of the warra...

See Answer

Q: In the previous problem, suppose the fixed asset actually falls into

In the previous problem, suppose the fixed asset actually falls into the three-year MACRS class. All the other facts are the same. What is the project’s Year 1 net cash flow now? Year 2? Year 3? What...

See Answer